McKinney Maternal Test Bank Chap 1-30

Ace your homework & exams now with Quizwiz!

1. The nurse is preparing a dose of naloxone for a newborn who weighs 6.9 pounds. How much naloxone does the nurse administer? ______ mg

0.31 mg The dose is 0.1 mg/kg for this 3.1-kg baby. PTS: 1 DIF: Cognitive Level: Application/Applying REF: p. 642 | Drug Guide OBJ: Nursing Process: Implementation MSC: Client Needs: Physiologic Integrity

1. A newborn infant weighing 8 lb needs naloxone (Narcan). This infant should receive approximately _____ mg.

0.36 The dose of naloxone is 0.1 mg/kg. This baby weighs 3.6 kg, so 0.1 × 3.6 = 0.36 mg. PTS: 1 DIF: Cognitive Level: Application/Applying REF: Table 18.1 OBJ: Nursing Process: Implementation MSC: Client Needs: Physiologic Integrity Chapter 19: Nursing Care during Obstetric Procedures McKinney: Evolve Resources for Maternal-Child Nursing, 5th Edition MULTIPLE CHOICE

1. An infant was born weighing 7.2 pounds. Calculate this infant's oral intake needs. _______

223.2-327 mL/day This infant weighs 3.27 kg. The range of intake for the first 3 days is 60 to 100 mL/kg/day. This infant needs 223.2 to 327 mL/day of fluid intake. PTS: 1 DIF: Cognitive Level: Application/Applying REF: p. 434 | Box 21.2 OBJ: Nursing Process: Implementation MSC: Client Needs: Physiologic Integrity Chapter 22: The Normal Newborn: Nursing Care McKinney: Evolve Resources for Maternal-Child Nursing, 5th Edition MULTIPLE CHOICE

1. A pregnant woman has the following assessments determined from a biophysical profile: reactive nonstress test, 3 fetal breathing movements within 30 minutes, 1 trunk movement in 30 minutes, opened and closed hand twice in 30 minutes, largest amniotic pocket of 1 cm. Calculate this woman's score. This woman's score is _____.

8 The scoring is as follows for each criteria: 2-2-1-2-1 = 8. PTS: 1 DIF: Cognitive Level: Analysis/Analyzing REF: Table 15.1 OBJ: Nursing Process: Assessment MSC: Client Needs: Physiologic Integrity Chapter 16: Giving Birth McKinney: Evolve Resources for Maternal-Child Nursing, 5th Edition MULTIPLE CHOICE

2. One of the assessments performed in the delivery room is checking the umbilical cord for blood vessels. Which finding is considered within normal limits? a. Two arteries and one vein b. Two arteries and two veins c. Two veins and one artery d. One artery and one vein

A The umbilical cord contains two arteries and one vein to transport blood between the fetus and the placenta. Any option other than two arteries and one vein is considered abnormal and requires further assessment. PTS: 1 DIF: Cognitive Level: Knowledge/Remembering REF: p. 206 OBJ: Nursing Process: Assessment MSC: Client Needs: Physiologic Integrity

7. What is most likely to be a concern for the older mother? a. The importance of having enough rest and sleep b. Information about effective contraceptive methods c. Nutrition and diet planning d. Information about exercise and fitness

A The woman who delays childbearing may have unique concerns, one of which is having less energy than younger mothers. The older mother usually has more financial means to search out effective contraceptive methods. The older mother often is better off financially and can afford better nutrition. Information about exercise and fitness is readily available. PTS: 1 DIF: Cognitive Level: Comprehension/Understanding REF: p. 506 OBJ: Nursing Process: Assessment MSC: Client Needs: Psychosocial Integrity

4. The nurse is performing a routine assessment on a 14-month-old infant and notes that the anterior fontanel is closed. This should be interpreted as a(n) a. normal finding—nurse should document finding in chart. b. questionable finding—infant should be rechecked in 1 month. c. abnormal finding—indicates need for immediate referral to practitioner. d. abnormal finding—indicates need for developmental assessment.

A This is a normal finding. The anterior fontanel closes between ages 12 and 18 months. The posterior fontanel closes between 2 and 3 months of age. There is no need for a recheck, a referral, or a developmental assessment. PTS: 1 DIF: Cognitive Level: Analysis/Analyzing REF: p. 93 | Table 6.1 OBJ: Nursing Process: Assessment MSC: Client Needs: Health Promotion and Maintenance

2. In order to help patients manage discomfort and pain during labor, nurses should be aware that a. the predominant pain of the first stage of labor is the visceral pain located in the lower portion of the abdomen. b. somatic pain is the extreme discomfort between contractions. c. the somatic pain of the second stage of labor is more generalized and related to fatigue. d. pain during the third stage is a somewhat milder version of the second stage.

A This pain comes from cervical changes, distention of the lower uterine segment, and uterine ischemia. Somatic pain is a faster, sharp pain. Somatic pain is most prominent during late first-stage labor and during second-stage labor as the descending fetus puts direct pressure on maternal tissues. Second-stage labor pain is intense, sharp, burning, and localized. Third-stage labor pain is similar to that of the first stage. PTS: 1 DIF: Cognitive Level: Knowledge/Remembering REF: p. 355 OBJ: Nursing Process: Assessment MSC: Client Needs: Health Promotion and Maintenance

1. The nurse sees this pattern on the fetal monitor. What action by the nurse is most appropriate? a. Apply oxygen via face mask, and position the woman on her left side. b. Document the findings in the chart along with maternal vital signs. c. Prepare to start an infusion of oxytocin per unit protocol. d. Decrease the rate of the woman's IV maintenance fluids.

A This tracing shows a late deceleration. The mother should be given oxygen and positioned on her left side. The findings should be documented, but only after interventions have occurred. Oxytocin would increase uterine activity (and increase stress on the fetus) so should not be started, or if already running, discontinued. IV fluids should be increased. PTS: 1 DIF: Cognitive Level: Analysis REF: Figure 17.10 OBJ: Nursing Process: Implementation MSC: Client Needs: Physiologic Integrity

2. Which maternal condition is considered a contraindication for the application of internal monitoring devices? a. Unruptured membranes b. Cervix is dilated to 4 cm c. External monitors are currently being used d. Fetus has a known heart defect

A To apply internal monitoring devices, the membranes must be ruptured. Cervical dilation of 4 cm permits the insertion of fetal scalp electrodes and intrauterine catheter. The external monitor can be discontinued after the internal ones are applied. A compromised fetus should be monitored with the most accurate monitoring devices. PTS: 1 DIF: Cognitive Level: Comprehension/Understanding REF: p. 338 OBJ: Nursing Process: Planning MSC: Client Needs: Physiologic Integrity

2. The perinatal nurse is caring for a woman in the immediate postbirth period. Assessment reveals that the woman is experiencing profuse bleeding. The most likely etiology for the bleeding is a. uterine atony. b. uterine inversion. c. vaginal hematoma. d. vaginal laceration.

A Uterine atony is marked hypotonia of the uterus. It is the leading cause of postpartum hemorrhage. The other situations can cause bleeding but are not the most common cause. PTS: 1 DIF: Cognitive Level: Knowledge/Remembering REF: p. 599 OBJ: Nursing Process: Assessment MSC: Client Needs: Physiologic Integrity

4. The nursing faculty explains that the fetus can survive in a low-oxygen environment due to which of the following? (Select all that apply.) a. Fetal hemoglobin carries more oxygen than an adult's. b. The fetus has higher average hemoglobin and hematocrit. c. Hemoglobin carries more oxygen at low partial pressures of carbon dioxide. d. Fetal blood is more acidic than the maternal blood. e. The fetus does not need gas exchange while in utero.

A, B, C The fetus can survive in low oxygen environments due to its hemoglobin being able to carry more oxygen that the mom, having a higher level of hemoglobin and hematocrit, and the fact that hemoglobin can carry more oxygen at low partial pressures of carbon dioxide. Fetal blood is alkaline. The fetus does need gas exchange in utero. PTS: 1 DIF: Cognitive Level: Knowledge/Remembering REF: p. 208 OBJ: Integrated Process: Teaching-Learning MSC: Client Needs: Physiologic Integrity Chapter 13: Adaptations to Pregnancy McKinney: Evolve Resources for Maternal-Child Nursing, 5th Edition MULTIPLE CHOICE

1. Nurses use many different nonpharmacologic methods of pain management. Examples of nonpharmacologic pain management techniques include which of the following? (Select all that apply.) a. Swaddling b. Nonnutritive sucking (pacifier) c. Skin-to-skin contact with the mother d. Sucrose e. Acetaminophen

A, B, C, D These interventions are all appropriate nonpharmacologic techniques used to manage pain in neonates. Other interventions include soothing music, dim lighting and speaking to the infant in a quiet voice. Acetaminophen is a pharmacologic method of treating pain. PTS: 1 DIF: Cognitive Level: Knowledge/Remembering REF: p. 471 OBJ: Nursing Process: Implementation MSC: Client Needs: Physiologic Integrity

8. The nurse is discussing contraceptive choices with an adolescent girl who wants to become sexually active. Which factors are important to consider? (Select all that apply.) a. Motivation b. Cognitive development c. Chronological age d. Parental opinions e. Frequency of intercourse

A, B, E Motivation, cognitive development, and planned frequency of intercourse are some of the factors to consider when counseling an adolescent about birth control choices. Chronological age is not as important as developmental state. Parents generally do not need to give consent or be informed when a teen seeks contraception. PTS: 1 DIF: Cognitive Level: Comprehension/Understanding REF: Nursing Quality Alert Box | Box 9.4 OBJ: Nursing Process: Assessment MSC: Client Needs: Health Promotion and Maintenance Chapter 10: Hereditary and Environmental Influences on Development McKinney: Evolve Resources for Maternal-Child Nursing, 5th Edition MULTIPLE CHOICE

2. The labor and delivery nurse learns that recommendations from ACOG related to VBAC risks include which of the following? (Select all that apply.) a. Immediate availability of the obstetric provider b. Delivery at a tertiary care center c. Availability of anesthesia personnel d. Personnel who can assist with the cesarean birth e. Use of misoprostol for cervical ripening

A, C, D A VBAC delivery should only be attempted with the obstetric provider in house and anesthesia along with operative personnel readily available to perform a cesarean birth. VBAC deliveries may be done in community hospitals if appropriate policies and guidelines for care are in place. Misoprostol administration is contraindicated in a patient with a previous uterine scar. PTS: 1 DIF: Cognitive Level: Knowledge/Remembering REF: p. 388 | Box 19.1 OBJ: Nursing Process: Planning MSC: Client Needs: Health Promotion and Maintenance COMPLETION

2. Which interventions should the nurse teach that are appropriate for preventing childhood obesity? (Select all that apply.) a. Establish consistent times for meals and snacks. b. Sign your child up for sports teams. c. Teach the family and child how to prepare foods in a healthy manner. d. Show the family how to read food labels. e. Limit computer and television time.

A, C, D, E Preventing obesity includes encouraging families to establish consistent times for meals and snacks, teaching them how to select and prepare healthful foods, and limiting computer and television time. Participating in sports is a great activity, but parents should not sign their kids up for teams without consulting them first. PTS: 1 DIF: Cognitive Level: Comprehension/Understanding REF: p. 144 OBJ: Integrated Process: Teaching-Learning MSC: Client Needs: Health Promotion and Maintenance

3. The generalist nurse working with child-bearing families understands that his or her practice related to genetics includes which of the following? (Select all that apply.) a. Identifying families at risk and providing referrals b. Interpreting genetic test results for the family c. Assessing the couple's concern about genetic alterations d. Helping create a family tree or pedigree e. Providing support in all phases of genetic counseling

A, C, D, E The nurse who works with women and families in the childbearing years is in a wonderful position to help identify families at risk and provide referrals, assess concerns, create pedigrees, and provide support. Interpreting genetic test results is provided by those who have advanced training and education in that area and would not be expected of the generalist nurse. PTS: 1 DIF: Cognitive Level: Application/Applying REF: p. 180 OBJ: Nursing Process: Implementation MSC: Client Needs: Health Promotion and Maintenance

1. A nurse has completed a teaching session for parents about "baby-proofing" the home. Which statements made by the parents indicate an understanding of the teaching? (Select all that apply.) a. "We will put plastic fillers in all electrical plugs." b. "We will place poisonous substances in a high cupboard." c. "We will place a gate at the top and bottom of stairways." d. "We will keep our household hot water heater at 130 degrees." e. "We will remove front knobs from the stove."

A, C, E By the time babies reach 6 months of age, they begin to become much more active, curious, and mobile. Putting plastic fillers on all electrical plugs can prevent an electrical shock. Putting gates at the top and bottom of stairways will prevent falls. Removing front knobs from the stove can prevent burns. Poisonous substances should be stored in a locked cabinet not in a cabinet that children can reach when they begin to climb. The household hot water heater should be turned down to 120 degrees or less. PTS: 1 DIF: Cognitive Level: Evaluation/Evaluating REF: p. 97 OBJ: Nursing Process: Evaluation MSC: Client Needs: Health Promotion and Maintenance

2. The nurse teaches a student that indications for percutaneous umbilical cord sampling (PUBS) include which of the following? (Select all that apply.) a. Rh disease b. Fetal well-being c. Infection d. Lung maturity e. Karyotyping

A, C, E Rh disease, infection, and, infrequently, for karyotyping are all indications for PUBS. NST or BPP are used to determination fetal well-being. An amniocentesis is done in order to determine lung maturity. PTS: 1 DIF: Cognitive Level: Comprehension/Understanding REF: p. 281 OBJ: Integrated Process: Teaching-Learning MSC: Client Needs: Physiologic Integrity COMPLETION

3. A lactating woman tells the nurse she is glad to no longer have to follow specific dietary recommendations, now that her baby has been born. The nurse responds by teaching her that lactating women have an even greater need for which nutrients? (Select all that apply.) a. Vitamin A b. Vitamin D c. Folic acid d. Iron e. Iodine

A, E The lactating woman needs more vitamin A and iodine than the pregnant woman. The requirements for vitamin D are the same in both groups. The lactating woman needs less folic acid and iron than a pregnant woman. PTS: 1 DIF: Cognitive Level: Comprehension/Understanding REF: Table 14.3 OBJ: Integrated Process: Teaching-Learning MSC: Client Needs: Health Promotion and Maintenance

5. The primary reason for evaluating alpha-fetoprotein (AFP) levels in maternal serum is to determine if the fetus has a. hemophilia. b. a neural tube defect. c. sickle cell anemia. d. a normal lecithin/sphingomyelin (L/S) ratio.

B An open neural tube allows a high level of AFP to seep into the amniotic fluid and enter the maternal serum. Hemophilia is a genetic defect and is best detected with chromosomal studies such as chorionic villus sampling or amniocentesis. Sickle cell is a genetic defect and is best detected with chromosomal studies such as chorionic villus sampling or amniocentesis. L/S ratios are determined with an amniocentesis, which is usually done in the third trimester. PTS: 1 DIF: Cognitive Level: Knowledge/Remembering REF: p. 277 OBJ: Nursing Process: Assessment MSC: Client Needs: Physiologic Integrity

9. A woman with severe preeclampsia is being treated with bed rest and intravenous magnesium sulfate. The drug classification of this medication is a. tocolytic. b. anticonvulsant. c. antihypertensive. d. diuretic.

B Anticonvulsant drugs act by blocking neuromuscular transmission and depress the central nervous system to control seizure activity. A tocolytic drug does slow the frequency and intensity of uterine contractions, but it is not used for that purpose in this scenario. Decreased peripheral blood pressure is a therapeutic response (side effect) of the anticonvulsant magnesium sulfate. Diuresis is a therapeutic response to magnesium sulfate. PTS: 1 DIF: Cognitive Level: Knowledge/Remembering REF: p. 539 OBJ: Nursing Process: Assessment MSC: Client Needs: Physiologic Integrity 10. What is the only known cure for preeclampsia? a. Magnesium sulfate b. Antihypertensive medications c. Delivery of the fetus d. Administration of acetylsalicylic acid (ASA) every day of the pregnancy ANS: C If the fetus is viable and near term, delivery is the only known definitive treatment for preeclampsia. Magnesium sulfate is one of the medications used to treat but not to cure preeclampsia. Antihypertensive medications are used to lower the dangerously elevated blood pressures in preeclampsia and eclampsia. Low doses of ASA (81 mg) have been administered to women at high risk for developing preeclampsia. PTS: 1 DIF: Cognitive Level: Knowledge REF: p. 539 OBJ: Nursing Process: Planning MSC: Client Needs: Physiologic Integrity 11. Which clinical sign is not included in the symptoms of preeclampsia? a. Hypertension b. Edema c. Proteinuria d. Glycosuria ANS: D Spilling glucose into the urine is not one of the three classic symptoms of preeclampsia. Hypertension is usually the first sign noted. Edema occurs but is considered a non-specific sign. Edema can lead to rapid weight gain. Proteinuria should be assessed through a 24- hour UA. PTS: 1 DIF: Cognitive Level: Knowledge/Remembering REF: p. 537 OBJ: Nursing Process: Assessment MSC: Client Needs: Physiologic Integrity 12. A nurse is assessing a woman receiving magnesium sulfate. The nurse assesses her deep tendon reflexes at 0 and 1+. What action by the nurse is best? a. Hold the magnesium sulfate. b. Ask the provider to order a 24-hour UA. c. Assess the woman's temperature. d. Take the woman's blood pressure. ANS: A Absent or hypoactive deep tendon reflexes are indicative of magnesium sulfate toxicity. The nurse should hold the magnesium and notify the provider. There is no need for a 24- hour UA at this point. Temperature changes are not related to magnesium. Blood pressure can be assessed, but that is not the priority. PTS: 1 DIF: Cognitive Level: Application/Applying REF: p. 542 OBJ: Nursing Process: Assessment MSC: Client Needs: Safe and Effective Care Environment 13. The labor of a pregnant woman with preeclampsia is going to be induced. The nurse reviews the woman's latest laboratory test findings, which reveal a low platelet count, an elevated aspartate transaminase (AST) level, and a falling hematocrit. What action by the nurse is most important? a. Palpate the woman's abdomen for tenderness. b. Document findings and begin the Pitocin infusion. c. Instruct the woman to ask for help getting out of bed. d. Assess the woman's drinking history. ANS: C This woman has HELLP syndrome, with is characterized by low platelet counts and hepatic dysfunction. She is at risk for bleeding, so the nurse instructs her to call for assistance in getting in and out of bed. The nurse does not palpate the abdomen even though the woman may complain of abdominal pain because of possible rupture of a subcapsular hematoma. The findings should be documented but the nurse should intervene based on the abnormal findings. The liver enzymes are not elevated because of alcohol intake. PTS: 1 DIF: Cognitive Level: Application/Applying REF: p. 544 OBJ: Nursing Process: Implementation MSC: Client Needs: Safe and Effective Care Environment 14. The nurse is explaining how to assess edema to the nursing students working on the antepartum unit. Which score indicates edema of lower extremities, face, hands, and sacral area? a. +1 edema b. +2 edema c. +3 edema d. +4 edema ANS: C Edema of the extremities, face, and sacral area is classified as +3 edema. Edema classified as +1 indicates minimal edema of the lower extremities. Marked edema of the lower extremities is termed +2 edema. Generalized massive edema (+4) includes accumulation of fluid in the peritoneal cavity. PTS: 1 DIF: Cognitive Level: Comprehension/Understanding REF: p. 537 | Table 25.2 OBJ: Nursing Process: Assessment MSC: Client Needs: Physiologic Integrity 15. The prenatal clinic nurse monitored women for preeclampsia. If all four women were in the clinic at the same time, which one should the nurse see first? a. Blood pressure increase to 138/86 mm Hg b. Weight gain of 0.5 kg during the past 2 weeks c. A dipstick value of 3+ for protein in her urine d. Pitting pedal edema at the end of the day ANS: C Proteinuria is defined as a concentration of 1+ or greater via dipstick measurement. A dipstick value of 3+ is indicative of severe preeclampsia and should alert the nurse that additional testing or assessment should be made. Generally, hypertension is defined as a BP of 140/90 or higher. Preeclampsia may be manifested as a rapid weight gain. Gaining 0.5 kg during the past 2 weeks does not qualify as rapid. Edema occurs in many normal pregnancies as well as in women with preeclampsia. Therefore, the presence of edema is no longer considered diagnostic of preeclampsia. PTS: 1 DIF: Cognitive Level: Analysis/Analyzing REF: p. 538 | Table 25.3 OBJ: Nursing Process: Assessment MSC: Client Needs: Safe and Effective Care Environment 16. A patient with pregnancy-induced hypertension is admitted complaining of pounding headache, visual changes, and epigastric pain. Nursing care is based on the knowledge that these signs indicate a. Anxiety due to hospitalization b. Worsening disease and impending seizure c. Effects of magnesium sulfate d. Gastrointestinal upset ANS: B Headache and visual disturbances are due to increased cerebral edema. Epigastric pain indicates distention of the hepatic capsules and often warns that a seizure is imminent. These sign are not due to anxiety or magnesium sulfate or related to gastrointestinal upset. PTS: 1 DIF: Cognitive Level: Knowledge/Remembering REF: p. 538 | Table 25.3 OBJ: Nursing Process: Assessment MSC: Client Needs: Physiologic Integrity 17. Rh incompatibility can occur if the woman is Rh negative and her a. fetus is Rh positive. b. husband is Rh positive. c. fetus is Rh negative. d. husband and fetus are both Rh negative. ANS: A For Rh incompatibility to occur, the mother must be Rh negative and her fetus Rh positive. The husband's Rh factor is a concern only as it relates to the possible Rh factor of the fetus. If the fetus is Rh negative, the blood types are compatible and no problems should occur. If the fetus is Rh negative, the blood type with the mother is compatible. The husband's blood type does not enter into the problem. PTS: 1 DIF: Cognitive Level: Knowledge/Remembering REF: p. 545 OBJ: Nursing Process: Assessment MSC: Client Needs: Physiologic Integrity 18. In which situation is a dilation and curettage (D&C) indicated? a. Complete abortion at 8 weeks b. Incomplete abortion at 16 weeks c. Threatened abortion at 6 weeks d. Incomplete abortion at 10 weeks ANS: D D&C is used to remove the products of conception from the uterus and can be used safely until week 14 of gestation. After that there is a greater risk of excessive bleeding, and this procedure may not be used. If all the products of conception have been passed (complete abortion), a D&C is not used. If the pregnancy is still viable (threatened abortion), a D&C is not used. PTS: 1 DIF: Cognitive Level: Knowledge/Remembering REF: p. 525 OBJ: Nursing Process: Planning MSC: Client Needs: Physiologic Integrity 19. What order should the nurse expect for a patient admitted with a threatened abortion? a. Abstinence from sexual activity b. Pitocin IV c. NPO d. Narcotic analgesia every 3 hours, prn ANS: A The woman may be counseled to avoid sexual activity with a threatened abortion. Activity restrictions were once recommended, but they have not shown effectiveness as treatment. Pitocin would be contraindicated. There is no reason for the woman to be NPO. In fact, hydration is important. Narcotic analgesia is not indicated. PTS: 1 DIF: Cognitive Level: Comprehension/Understanding REF: p. 524 OBJ: Nursing Process: Planning MSC: Client Needs: Health Promotion and Maintenance 20. What data on a patient's health history places her at risk for an ectopic pregnancy? a. Use of oral contraceptives for 5 years b. Recurrent pelvic infections c. Ovarian cyst 2 years ago d. Heavy menstrual flow of 4 days' duration ANS: B Infection and subsequent scarring of the fallopian tubes prevents normal movement of the fertilized ovum into the uterus for implantation. Oral contraceptives, ovarian cysts, and heavy menstrual flows do not increase risk. PTS: 1 DIF: Cognitive Level: Knowledge/Remembering REF: p. 526 | Box 25.1 OBJ: Nursing Process: Assessment MSC: Client Needs: Physiologic Integrity 21. What finding on a prenatal visit at 10 weeks might suggest a hydatidiform mole? a. Complaint of frequent mild nausea b. Blood pressure of 120/80 mm Hg c. Fundal height measurement of 18 cm d. History of bright red spotting for 1 day, weeks ago ANS: C The uterus in a hydatidiform molar pregnancy is often larger than would be expected on the basis of the duration of the pregnancy. Many women have nausea in the first trimester. A woman with a molar pregnancy may have early-onset pregnancy-induced hypertension. The history of bleeding is normally described as being brownish. PTS: 1 DIF: Cognitive Level: Knowledge/Remembering REF: p. 528 OBJ: Nursing Process: Assessment MSC: Client Needs: Health Promotion and Maintenance 22. What routine nursing assessment is contraindicated in the patient admitted with suspected placenta previa? a. Monitoring FHR and maternal vital signs b. Observing vaginal bleeding or leakage of amniotic fluid c. Determining frequency, duration, and intensity of contractions d. Determining cervical dilation and effacement ANS: D Vaginal examination of the cervix may result in perforation of the placenta and subsequent hemorrhage and is therefore contraindicated. Monitoring FHR and maternal vital signs is a necessary part of the assessment for this woman. Monitoring for bleeding and rupture of membranes is not contraindicated in this woman. Monitoring contractions is not contraindicated in this woman. PTS: 1 DIF: Cognitive Level: Knowledge/Remembering REF: p. 530 OBJ: Nursing Process: Assessment MSC: Client Needs: Physiologic Integrity 23. The primary symptom present in abruptio placentae that distinguishes it from placenta previa is a. vaginal bleeding. b. rupture of membranes. c. presence of abdominal pain. d. changes in maternal vital signs. ANS: C Pain in abruptio placentae occurs in response to increased pressure behind the placenta and within the uterus. Placenta previa manifests with painless vaginal bleeding, but both may have vaginal bleeding. Rupture of membranes may occur with both conditions. Maternal vital signs may change with both if bleeding is pronounced. PTS: 1 DIF: Cognitive Level: Knowledge/Remembering REF: p. 529 | p. 532 OBJ: Nursing Process: Assessment MSC: Client Needs: Physiologic Integrity 24. Which laboratory marker is indicative of disseminated intravascular coagulation (DIC)? a. Positive KB test b. Presence of fibrin split products c. Thrombocytopenia d. Positive drug screen ANS: B Degradation of fibrin leads to the accumulation of multiple fibrin clots throughout the body's vasculature. The other lab tests are not indicative of DIC. PTS: 1 DIF: Cognitive Level: Knowledge/Remembering REF: p. 525 OBJ: Nursing Process: Assessment MSC: Client Needs: Physiologic Integrity 25. A woman taking magnesium sulfate has respiratory rate of 10 breaths/min. In addition to discontinuing the medication, the nurse should a. vigorously stimulate the woman. b. instruct her to take deep breaths. c. administer calcium gluconate. d. increase her IV fluids. ANS: C Calcium gluconate reverses the effects of magnesium sulfate. Stimulation, instruction on taking deep breaths, and increasing her fluid rate will not increase the respirations. PTS: 1 DIF: Cognitive Level: Application/Applying REF: p. 539 | Drug Guide Box OBJ: Nursing Process: Implementation MSC: Client Needs: Physiologic Integrity 26. A 32-year-old primigravida is admitted with a diagnosis of ectopic pregnancy. Nursing care is based on the knowledge that a. bed rest and analgesics are the recommended treatment. b. she will be unable to conceive in the future. c. a D&C will be performed to remove the products of conception. d. hemorrhage is the major concern. ANS: D Severe bleeding occurs if the fallopian tube ruptures. The recommended treatment is to remove the pregnancy before hemorrhaging. If the tube must be removed, her fertility will decrease but she will not be infertile. A D&C is done on the inside of the uterine cavity. The ectopic pregnancy is located within the tubes. PTS: 1 DIF: Cognitive Level: Comprehension/Understanding REF: p. 527 OBJ: Nursing Process: Planning MSC: Client Needs: Physiologic Integrity 27. The nurse learns that which is the most common cause of spontaneous abortion? a. Chromosomal abnormalities b. Infections c. Endocrine imbalance d. Immunologic factors ANS: A Around 60% of pregnancy losses from spontaneous abortion in the first trimester result from chromosomal abnormalities that are incompatible with life. Maternal infection, endocrine imbalances, and immunologic factors may also be causes of early miscarriage. PTS: 1 DIF: Cognitive Level: Knowledge/Remembering REF: p. 523 OBJ: Nursing Process: Assessment MSC: Client Needs: Health Promotion and Maintenance 28. Methotrexate is recommended as part of the treatment plan for which obstetric complication? a. Complete hydatidiform mole b. Missed abortion c. Unruptured ectopic pregnancy d. Abruptio placentae ANS: C Methotrexate is an effective, nonsurgical treatment option for a hemodynamically stable woman whose ectopic pregnancy is unruptured and less than 3.5 cm in diameter. Methotrexate is not indicated or recommended as a treatment option for a complete hydatidiform mole, a missed abortion, or abruptio placentae. PTS: 1 DIF: Cognitive Level: Knowledge/Remembering REF: p. 527 OBJ: Nursing Process: Planning MSC: Client Needs: Physiologic Integrity 29. The nurse caring for a woman hospitalized for hyperemesis gravidarum should expect that initial treatment involves a. corticosteroids to reduce inflammation. b. IV therapy to correct fluid and electrolyte imbalances. c. an antiemetic, such as pyridoxine, to control nausea and vomiting. d. enteral nutrition to correct nutritional deficits. ANS: B Initially, the woman who is unable to down clear liquids by mouth requires IV therapy for correction of fluid and electrolyte imbalances. Corticosteroids are not the expected treatment for this disorder. Pyridoxine is vitamin B6, not an antiemetic. Promethazine, a common antiemetic, may be prescribed. In severe cases of hyperemesis gravidarum, enteral nutrition via a feeding tube may be necessary to correct maternal nutritional deprivation. This is not an initial treatment for this patient. PTS: 1 DIF: Cognitive Level: Comprehension/Understanding REF: p. 535 OBJ: Nursing Process: Implementation MSC: Client Needs: Physiologic Integrity 30. A woman with preeclampsia has a seizure. What action by the nurse takes priority? a. Insert an oral airway. b. Suction the mouth to prevent aspiration. c. Administer oxygen by mask. d. Stay with the patient and call for help. ANS: D If a patient seizes, the nurse should stay with her and call for help. Nursing actions during a seizure are directed toward ensuring a patent airway and patient safety. Insertion of an oral airway during seizure activity is no longer the standard of care. The nurse should attempt to keep the airway patent by turning the patient's head to the side to prevent aspiration. Once the seizure has ended, it may be necessary to suction the patient's mouth. Oxygen may or may not be needed after the seizure has ended. PTS: 1 DIF: Cognitive Level: Application/Applying REF: p. 544 OBJ: Nursing Process: Implementation MSC: Client Needs: Safe and Effective Care Environment 31. A woman is in the emergency department with severe abdominal pain. When her pregnancy test comes back positive, she yells "I can't be pregnant! I had a tubal ligation two months ago!" What action by the nurse is the priority? a. Provide emotional support to the woman. b. Facilitate an ultrasound examination. c. Call the lab to have them repeat the test. d. Administer an opioid pain medication. ANS: B A failed tubal ligation is a risk factor for ectopic pregnancy. After a blood pregnancy test, a transvaginal ultrasound is needed to look for a gestational sac within the uterus. Of course the nurse provides emotional support, but that is not the priority. There is no need to repeat the test. Pain medications may be contraindicated if surgery is needed and consents have not yet been signed. PTS: 1 DIF: Cognitive Level: Application/Applying REF: pp. 526-527 | Box 25.1 OBJ: Nursing Process: Implementation MSC: Client Needs: Safe and Effective Care Environment 32. A woman who is 8 months pregnant is brought to the emergency department after a serious motor vehicle crash. Although she has no apparent injuries, she is admitted to the hospital. Her partner is upset and wants to know why she just can't come home. What response by the nurse is best? a. "This is standard procedure for all pregnant crash victims." b. "She needs to be monitored for some potential complications." c. "We may have to deliver the baby at any time now." d. "We are giving her medicine to keep her from laboring." ANS: B After serious trauma, a woman may be admitted and observed because an abruptio placentae may take up to 24 hours to become apparent. Not all motor vehicle crash patients will need to be admitted. The baby may or may not need to be delivered at any time, but this statement will frighten the partner. There is no indication the patient is in labor. PTS: 1 DIF: Cognitive Level: Comprehension/Understanding REF: p. 532 | Safety Alert Box OBJ: Nursing Process: Implementation MSC: Client Needs: Physiologic Integrity MULTIPLE RESPONSE

2. After a forceps-assisted birth, the mother is observed to have continuous bright red lochia but a firm fundus. What other finding is important to correlate with these data? a. Mild, intermittent perineal pain b. Edema and discoloration of the labia and perineum c. Lack of an episiotomy d. Lack of pain in the perineal area

B Edema and discoloration of the labia and perineum along with continuous bright red lochia and firm fundus are correlated with a vaginal wall hematoma. Perineal pain, lack of episiotomy, and lack of pain are not correlated with a hematoma. PTS: 1 DIF: Cognitive Level: Comprehension/Understanding REF: p. 386 OBJ: Nursing Process: Assessment MSC: Client Needs: Physiologic Integrity

6. A woman delivered a 9-lb, 10-oz baby 1 hour ago. When you arrive to perform her 15-minute assessment, she tells you that she "feels all wet underneath." You discover that both pads are completely saturated and that she is lying in a 6-inch-diameter puddle of blood. What is your first action? a. Call for help. b. Assess the fundus for firmness. c. Take her blood pressure. d. Check the perineum for lacerations.

B Firmness of the uterus is necessary to control bleeding from the placental site. The nurse should first assess for firmness and massage the fundus as indicated. Calling for help is not needed unless corrective action does not improve the situation. Another nurse can take the blood pressure or the original nurse can do so after assessing the fundus and massaging it if needed. Checking the perineum for lacerations would be appropriate if the fundus was firm. PTS: 1 DIF: Cognitive Level: Application/Applying REF: pp. 599-600 OBJ: Nursing Process: Implementation MSC: Client Needs: Physiologic Integrity

9. A nurse is taking vital signs on a pregnant woman. Preconception pulse was 76 beats/minute. Today the pulse is 97 beats/minute. What action by the nurse is best? a. Inform the provider immediately. b. Document findings in the chart. c. Prepare to start an IV infusion. d. Retake the pulse in 15 minutes.

B The pulse of a pregnant woman increases about 15 to 20 beats/minute throughout the pregnancy. The nurse should document the findings, but no other actions are needed as this is a normal finding. PTS: 1 DIF: Cognitive Level: Application/Applying REF: p. 216 OBJ: Nursing Process: Implementation MSC: Client Needs: Physiologic Integrity 10. Physiologic anemia often occurs during pregnancy as a result of a. inadequate intake of iron. b. dilution of hemoglobin concentration. c. the fetus establishing iron stores. d. decreased production of erythrocytes. ANS: B When blood volume expansion is more pronounced and occurs earlier than the increase in red blood cells, the woman will have physiologic anemia, which is the result of dilution of hemoglobin concentration rather than inadequate hemoglobin. Inadequate intake of iron may lead to true anemia. If the woman does not take an adequate amount of iron, true anemia may occur when the fetus pulls stored iron from the maternal system. There is an increased production of erythrocytes during pregnancy. PTS: 1 DIF: Cognitive Level: Knowledge/Remembering REF: p. 216 OBJ: Nursing Process: Assessment MSC: Client Needs: Physiologic Integrity 11. While assessing her patient, what does the nurse interpret as a positive sign of pregnancy? a. Fetal movement felt by the woman b. Amenorrhea c. Breast changes d. Visualization of fetus by ultrasound ANS: D The only positive signs of pregnancy are auscultation of fetal heart tones, visualization of the fetus by ultrasound, and fetal movement felt by the examiner. Fetal movement felt by the woman, amenorrhea, and breast changes are all presumptive signs. PTS: 1 DIF: Cognitive Level: Knowledge/Comprehension REF: p. 224 OBJ: Nursing Process: Assessment MSC: Client Needs: Health Promotion and Maintenance 12. A woman is currently pregnant; she has a 5-year-old son and a 3-year-old daughter born at full term. She had one other pregnancy that terminated at 8 weeks. Her gravida and para are a. gravida 3 para 2. b. gravida 4 para 3. c. gravida 4 para 2. d. gravida 3 para 3. ANS: C She has had four pregnancies, including the current one (gravida 4). She had two pregnancies that terminated after 20 weeks (para 2). The pregnancy that terminated at 8 weeks is classified as an abortion, which is not included in the gravida-para classification. PTS: 1 DIF: Cognitive Level: Comprehension/Understanding REF: p. 225 OBJ: Nursing Process: Assessment MSC: Client Needs: Health Promotion and Maintenance 13. A woman's last menstrual period was June 10. The nurse estimates the date of delivery (EDD) to be a. April 7. b. March 17. c. March 27. d. April 17. ANS: B To determine the EDD, the nurse uses the first day of the last menstrual period (June 10), subtracts 3 months (March 10), and adds 7 days (March 17). The year is corrected if needed. April 7 would be subtracting 2 months instead of 3 months and then subtracting 3 days instead of adding 7 days. March is the correct month, but instead of adding 7 days, 17 days were added to get March 27. April 17 is subtracting 2 months instead of 3 months. PTS: 1 DIF: Cognitive Level: Application/Applying REF: p. 225 OBJ: Nursing Process: Assessment MSC: Client Needs: Health Promotion and Maintenance 14. A nurse sees a woman in her first trimester of pregnancy. The nurse explains that the woman can expect to visit her physician every 4 weeks so that a. she develops trust in the health care team. b. her questions about labor can be answered. c. the condition of the mother and fetus can be monitored. d. problems can be eliminated. ANS: C This routine allows monitoring of maternal health and fetal growth and ensures that problems will be identified early. If the woman begins prenatal care in the first trimester, every 4 weeks is the recommended schedule for visits. Developing a trusting relationship should be established during these visits, but that is not the primary reason. Most women do not have questions concerning labor until the last trimester of the pregnancy. All problems cannot be eliminated because of prenatal visits, but they can be identified. PTS: 1 DIF: Cognitive Level: Comprehension/Understanding REF: p. 228 OBJ: Nursing Process: Assessment MSC: Client Needs: Health Promotion and Maintenance 15. A patient in her first trimester complains of nausea and vomiting. She asks, "Why does this happen?" The nurse's best response is a. "It is due to an increase in gastric motility." b. "It may be due to changes in hormones." c. "It is related to an increase in glucose levels." d. "It is caused by a decrease in gastric secretions." ANS: B Nausea and vomiting are believed to be caused by increased levels of hormones and decreased gastric motility. Glucose levels decrease in the first trimester. Hypoglycemia, if experienced, can also lead to nausea. Gastric secretions do decrease, but this is not the main cause of nausea and vomiting. PTS: 1 DIF: Cognitive Level: Comprehension/Understanding REF: p. 231 OBJ: Integrated Process: Teaching-Learning MSC: Client Needs: Physiologic Integrity 16. The nurse teaches a pregnant woman that one of the most effective methods for preventing venous stasis is to a. wear elastic stockings in the afternoons. b. sleep with the foot of the bed elevated. c. rest often with the feet elevated. d. sit with the legs crossed. ANS: C Elevating the feet and legs improves venous return and prevents venous stasis. Elastic stockings should be applied before lowering the legs in the morning. Elevating the legs at night may cause pressure on the diaphragm and increase breathing problems. Sitting with the legs crossed will decrease circulation in the legs and increase venous stasis. PTS: 1 DIF: Cognitive Level: Comprehension/Understanding REF: p. 230 OBJ: Integrated Process: Teaching-Learning MSC: Client Needs: Physiologic Integrity 17. A patient notices that the doctor writes "positive Chadwick's sign" on her chart. She asks the nurse what this means. The nurse's best response is a. "It refers to the bluish color of the cervix in pregnancy." b. "It means the cervix is softening." c. "The doctor was able to flex the uterus against the cervix." d. "That refers to a positive sign of pregnancy." ANS: A Increased vascularity of the pelvic organs during pregnancy results in the bluish color of the cervix, vagina, and labia, called Chadwick's sign. The nurse should also know that this is a presumptive, not positive, sign of pregnancy. Softening of the cervix is Goodell's sign. The softening of the lower segment of the uterus (Hegar's sign) can allow the uterus to be flexed against the cervix. PTS: 1 DIF: Cognitive Level: Comprehension/Understanding REF: p. 215 OBJ: Integrated Process: Teaching-Learning MSC: Client Needs: Physiologic Integrity 18. A woman is at her first prenatal visit and is distressed at needing an HIV test. What response by the nurse is best? a. "We ask all women to be tested for HIV during their pregnancy." b. "This test is required by law for pregnant women." c. "Infection with HIV will make your pregnancy very high risk." d. "You could have been exposed and not know it." ANS: A A voluntary HIV test should be conducted on all women, regardless of risk factors. This explanation is accurate and helps lessen the woman's feeling of stigma. It also lets the woman know it is voluntary. The test is not required by law. Although an HIV infection will increase the risk of complications, this explanation is too limited to be a good answer. It is true the woman may have been exposed, but that comment is demeaning and could be offensive. PTS: 1 DIF: Cognitive Level: Comprehension/Understanding REF: Table 13.3 OBJ: Integrated Process: Teaching-Learning MSC: Client Needs: Health Promotion and Maintenance 19. To relieve a leg cramp, the patient should be instructed to a. massage the affected muscle. b. stretch and point the toe. c. dorsiflex the foot. d. apply a warm pack. ANS: C Dorsiflexion of the foot stretches the leg muscle and relieves the painful muscle contraction. Since she is prone to blood clots in the legs, massaging the affected leg muscle is contraindicated. Pointing the toes will contract the muscle and not relieve the pain. Warm packs can be used to relax the muscle, but more immediate relief is necessary, such as dorsiflexion of the foot. PTS: 1 DIF: Cognitive Level: Comprehension/Understanding REF: p. 231 OBJ: Integrated Process: Teaching-Learning MSC: Client Needs: Physiologic Integrity 20. The multiple marker screen is used to assess the fetus for which condition? a. Down syndrome b. Diaphragmatic hernia c. Congenital cardiac abnormality d. Anencephaly ANS: A The maternal serum level of alpha-fetoprotein is used to screen for trisomy 18 or 21 and neural tube defects. The quadruple marker test does not detect hernias. Additional testing, such as ultrasonography, would be required to diagnose diaphragmatic hernia. Congenital cardiac abnormality would most likely be identified during an ultrasound examination. PTS: 1 DIF: Cognitive Level: Knowledge/Remembering REF: Table 13.3 OBJ: Nursing Process: Assessment MSC: Client Needs: Health Promotion and Maintenance 21. A nurse is caring for patients in the prenatal clinical who are all 35 weeks along. Which patient should the nurse see first? a. Shortness of breath when climbing stairs b. Abdominal pain c. Ankle edema in the afternoon d. Backache with prolonged standing ANS: B Abdominal pain may indicate preterm labor or placental abruption so this patient should be seen first. Shortness of breath climbing stairs, afternoon ankle edema, and backache are all normal findings at this stage of pregnancy. PTS: 1 DIF: Cognitive Level: Application/Applying REF: p. 236 OBJ: Nursing Process: Assessment MSC: Client Needs: Safe and Effective Care Environment 22. A patient at 32 weeks of gestation reports that she has severe lower back pain. The nurse's assessment should include a. observation of posture and body mechanics. b. palpation of the lumbar spine. c. exercise pattern and duration. d. ability to sleep for at least 6 hours uninterrupted. ANS: A Correct posture and body mechanics can reduce lower back pain caused by increasing lordosis. Pregnancy should not cause alterations in the spine. Any assessment for malformation should be done early in the pregnancy. Exercise and sleep are not as important to assess as are posture and body mechanics, which can contribute to the pain. PTS: 1 DIF: Cognitive Level: Application/Applying REF: p. 230 OBJ: Nursing Process: Assessment MSC: Client Needs: Health Promotion and Maintenance 23. A pregnant couple has formulated a birth plan and is reviewing it with the nurse at an expectant parent's class. Which aspect of their birth plan would require further discussion with the nurse? a. "My husband and I have agreed that my sister will be my coach." b. "We plan to use Lamaze to reduce the pain during labor." c. "We want the labor and birth to take place in a birthing room with our son present. d. "We will not use the fetal monitor during labor." ANS: D A birth plan consists of what the woman and partner wish to have happen during labor and delivery. Intermittent or continuous fetal monitoring is one aspect of care for consideration; however, it is unrealistic to state that monitoring will not be used. The nurse should explain the purpose to ensure the couple is making an informed decision. The woman can refuse this procedure but would need to understand how this might negatively impact her child. The couple also need to understand that the entire plan is tentative depending on what events actually occur. The other statements are appropriate for a birth plan. PTS: 1 DIF: Cognitive Level: Application/Applying REF: Box 13.2 OBJ: Nursing Process: Implementation MSC: Client Needs: Health Promotion and Maintenance 24. A couple ask the prenatal nurse to explain centering pregnancy. Which statement accurately applies to this model of care? a. A way to control labor pain and remain centered during the process b. A philosophy of making the pregnancy the center of the family's life c. Education and support sessions are provided to small cohorts of women d. Labor practice where the woman is surrounded by an extensive network of people ANS: C This method involves ten 1.5- to 2-hour sessions with small groups of women and health care providers beginning at 12 to 16 weeks of pregnancy and ending in early postpartum. Sessions include assessment, education, and social support. It is not a way to control labor pain, a philosophy of making the pregnancy the center of the family's life, or the use of a large network of people during labor. PTS: 1 DIF: Cognitive Level: Application REF: p. 228 OBJ: Nursing Process: Implementation MSC: Client Needs: Health Promotion and Maintenance 25. Which comment by a woman in her first trimester indicates ambivalent feelings? a. "I wanted to become pregnant, but I'm scared about being a mother." b. "I haven't felt well since this pregnancy began." c. "I'm concerned about the amount of weight I've gained." d. "My body is changing so quickly." ANS: A Ambivalence refers to conflicting feelings. This woman is demonstrating this conflict. The other statements do not indicate ambivalence. PTS: 1 DIF: Cognitive Level: Analysis/Analyzing REF: p. 236 OBJ: Nursing Process: Assessment MSC: Client Needs: Psychosocial Integrity 26. A patient who is 7 months pregnant states, "I'm worried that something will happen to my baby." The nurse's best response is a. "There is nothing to worry about." b. "The doctor is taking good care of you and your baby." c. "Tell me about your concerns." d. "Your baby is doing fine." ANS: C Encouraging the client to discuss her feelings is the best approach. Women during their third trimester need reassurance that such fears are not unusual in pregnancy. An open-ended request to share information will encourage the patient to explain concerns further. The other statements belittle the patient's concerns and provide false hope. PTS: 1 DIF: Cognitive Level: Application/Applying REF: p. 238 OBJ: Integrated Process: Communication and Documentation MSC: Client Needs: Psychosocial Integrity 27. Which of the following behaviors by a pregnant woman would be an example of mimicry? a. Babysitting for a neighbor's children b. Wearing maternity clothes before they are needed c. Daydreaming about the newborn d. Imagining oneself as a good mother ANS: B Mimicry refers to observing and copying the behaviors of others, in this case, other pregnant women. Wearing maternity clothes before they are needed helps the expectant mother "feel" what it's like to be obviously pregnant. Babysitting other children is a form of role playing where the woman practices the expected role of motherhood. Daydreaming is a type of fantasy where the woman "tries on" a variety of behaviors in preparation for motherhood. Imagining herself as a good mother is the woman's effort to look for a good role fit. She observes behavior of other mothers and compares them with her own expectations. PTS: 1 DIF: Cognitive Level: Comprehension/Understanding REF: p. 239 OBJ: Nursing Process: Assessment MSC: Client Needs: Psychosocial Integrity 28. A step in maternal role attainment that relates to the woman giving up certain aspects of her previous life is termed a. looking for a fit. b. role playing. c. fantasy. d. grief work. ANS: D The woman experiences sadness as she realizes that she must give up certain aspects of her previous self and that she can never go back. Looking for a fit is when the woman observes the behaviors of mothers and compares them with her own expectations. Role playing involves searching for opportunities to provide care for infants in the presence of another person. Fantasies allow the woman to try on a variety of behaviors. This usually deals with how the child will look and the characteristics of the child. PTS: 1 DIF: Cognitive Level: Knowledge/Remembering REF: p. 240 OBJ: Nursing Process: Assessment MSC: Client Needs: Psychosocial Integrity 29. The maternal task that begins in the first trimester and continues throughout the neonatal period is called a. seeking safe passage for herself and her baby. b. securing acceptance of the baby by others. c. learning to give of herself. d. developing attachment with the baby. ANS: D Developing attachment (strong ties of affection) to the unborn baby begins in early pregnancy when the woman accepts that she is pregnant. By the second trimester, the baby becomes real, and feelings of love and attachment surge. Seeing safe passage is a task that ends with delivery. During this task the woman seeks health care and cultural practices. Securing acceptance continues throughout pregnancy as the woman reworks relationships. Learning to give of herself occurs during pregnancy and is sometimes noticed as the woman gives to others in the form of food or presents. PTS: 1 DIF: Cognitive Level: Knowledge/Remembering REF: p. 240 OBJ: Nursing Process: Assessment MSC: Client Needs: Health Promotion and Maintenance 30. Which situation best describes a man "trying on" fathering behaviors? a. Spending more time with his siblings b. Taking a nephew to the park to play c. Reading books on newborn care d. Exhibiting physical symptoms related to pregnancy ANS: B Interacting with children and assuming the behavior and role of a father best describe a man "trying on" being a father. The man normally will seek closer ties with his father during this time, not his siblings. While some fathers do everything they can to learn about infant care, others are not ready to learn when the information is presented, so the nurse should provide the information again after the baby is born and it is more relevant. Exhibiting symptoms related to pregnancy is called couvade. PTS: 1 DIF: Cognitive Level: Comprehension/Understanding REF: p. 241 OBJ: Nursing Process: Assessment MSC: Client Needs: Psychosocial Integrity 31. A 36-year-old divorcee with a successful modeling career finds out that her 18-year-old married daughter is expecting her first child. What is a major factor in determining how the woman will respond to becoming a grandmother? a. Her career b. Being divorced c. Her age d. Age of the daughter ANS: C Age is a major factor in determining the emotional response of prospective grandparents. Young grandparents may not be happy with the stereotype of grandparents as being old. Career responsibilities may have demands that make the grandparents not as accessible, but it is not a major factor in determining the woman's response to becoming a grandmother. Being divorced is not a major factor that determines adaptation of grandparents. The age of the daughter is not a major factor that determines adaptation of grandparents. The age of the grandparent is a major factor. PTS: 1 DIF: Cognitive Level: Comprehension REF: p. 242 OBJ: Nursing Process: Assessment MSC: Client Needs: Psychosocial Integrity 32. The nurse who practices in a prenatal clinic understands that a major concern of lower socioeconomic groups is to a. maintain group health insurance on their families. b. meet health needs as they occur. c. practice preventive health care. d. maintain an optimistic view of life. ANS: B Because of economic uncertainty, lower socioeconomic groups place more emphasis on meeting the needs of the present rather than on future goals. Lower socioeconomic groups usually do not have group health insurance. They may value health care but cannot afford preventive health care. They may struggle for basic needs and often do not see a way to improve their situation. It is difficult to maintain optimism. PTS: 1 DIF: Cognitive Level: Comprehension/Understanding REF: Table 13.6 OBJ: Nursing Process: Assessment MSC: Client Needs: Health Promotion and Maintenance 33. What comment by a new mother exhibits understanding of her toddler's response to a new sibling? a. "I can't believe he is sucking his thumb again." b. "He is being difficult, and I don't have time to deal with him." c. "My husband will stay with the baby so I can take our son to the park." d. "When we brought the baby home, we made our son stop sleeping in the crib." ANS: C It is important for a mother to seek time alone with her toddler to reassure him that he is loved. Toddlers can feel jealous and resentful having to share the mother's attention. It is normal for a child to regress when a new sibling is introduced into the home. As difficult as it is, the mother must make time to spend with the toddler. Changes in sleeping arrangements should be made several weeks before the birth so that the child does not feel displaced by the new baby. PTS: 1 DIF: Cognitive Level: Evaluation/Evaluating REF: p. 242 OBJ: Nursing Process: Evaluation MSC: Client Needs: Health Promotion and Maintenance 34. A nurse in labor and delivery is caring for a Muslim woman during the active phase of labor. You note that when you touch her, she quickly draws away. Which response by the nurse is best? a. Continue to touch her as much as you need to while providing care. b. Assume that she doesn't like you and decrease your time with her. c. Limit touching to a minimum, as this may not be acceptable in her culture. d. Ask the charge nurse to reassign you to another patient. ANS: C Touching is an important component of communication in various cultures, but if the patient appears to find it offensive, the nurse should respect her cultural beliefs and limit touching her. By continuing to touch her, the nurse is showing disrespect for her cultural beliefs. A cultural response to touch does not reflect like or dislike. Being assigned to another patient is inappropriate; all nurses must be able to provide culturally appropriate care. PTS: 1 DIF: Cognitive Level: Application/Applying REF: p. 246 OBJ: Integrated Process: Culture and Spirituality MSC: Client Needs: Psychosocial Integrity 35. A nurse is encouraging a patient to attend an early pregnancy class for the second trimester. What topic would be inconsistent with the nurse's knowledge of topics presented in this class? a. Fetal development b. Body mechanics c. Childbirth choices d. Managing morning sickness ANS: D Managing morning sickness would be taught in a first trimester early pregnancy class. The other topics are appropriate for second trimester classes. PTS: 1 DIF: Cognitive Level: Knowledge/Remembering REF: p. 231 OBJ: Integrated Process: Teaching-Learning MSC: Client Needs: Health Promotion and Maintenance 36. A pregnant patient of 28 weeks' gestation complains of pain in the right inguinal area. What action by the nurse is best? a. Assess the woman for early labor. b. Position the woman on the right side. c. Palpate the woman's abdomen. d. Document the findings in the chart. ANS: B Pain in the right inguinal area is most likely due to the round ligament. The nurse can position the woman on her right side to see if that relieves the pain. Heat can also help. There is no need to assess for labor or palpate the abdomen. The findings should be documented after the nurse responds. PTS: 1 DIF: Cognitive Level: Application/Applying REF: p. 230 OBJ: Nursing Process: Implementation MSC: Client Needs: Physiologic Integrity 37. A student nurse is teaching a pregnant woman ways to manage constipation. Which instruction by the student causes the nurse to provide a correction? a. Drink at least 8 glasses of liquids a day. b. The fat in cheese helps lubricate the bowels. c. You do need to continue your iron pills. d. Add extra fiber, which can be found in fruit. ANS: B Cheese tends to cause constipation, so this statement by the student needs correction by the nurse. The other statements are all correct. PTS: 1 DIF: Cognitive Level: Comprehension/Understanding REF: p. 231 OBJ: Integrated Process: Teaching-Learning MSC: Client Needs: Physiologic Integrity 38. The nurse is caring for a patient whose English is limited. When the nurse provides information, the patient smiles and nods her head. What action by the patient indicates that the goal for a primary nursing diagnosis for this patient has been met? a. Keeps next appointment and brings a translator with her. b. Gains an appropriate amount of weight at next visit. c. Husband accompanies patient to appointments. d. Continues to eat culturally appropriate foods. ANS: A The primary goal for this situation is Impaired Verbal Communication due to lack of English proficiency. If the patient is able to understand and keep her next appointment and brings a translator with her to help facilitate communication that shows that the goal of adequate communication has been met. The other actions do not address communication. PTS: 1 DIF: Cognitive Level: Evaluation/Evaluating REF: p. 246 OBJ: Nursing Process: Evaluation MSC: Client Needs: Psychosocial Integrity 39. The nurse in the OB triage area has four patients to see. Which patient should the nurse see first? a. First trimester, vomiting for an hour b. Second trimester, fingers swollen c. Third trimester, painful urination d. Third trimester, painful vaginal bleeding ANS: D This patient may have a placenta previa or abruptio placentae or might be having a spontaneous abortion. The nurse needs to see this patient first. The other patients may have normal vomiting of the first trimester. Swollen fingers indicate edema that needs to be investigated. Painful urination probably indicates a urinary tract infection. The priority patient is the one with bleeding. PTS: 1 DIF: Cognitive Level: Analysis/Analyzing REF: Safety Alert Box OBJ: Nursing Process: Assessment MSC: Client Needs: Safe and Effective Care Environment MULTIPLE RESPONSE

9. A nurse is caring for a child who is a Christian Scientist. What intervention should the nurse include in the care plan for this child? a. Offer iced tea to the child who is experiencing deficient fluid volume. b. Offer to inform a Christian Science practitioner of the child's admission. c. Allow parents to sign a form opting out of routine immunizations. d. Ask parents whether the child has been baptized.

B When a Christian Science believer is hospitalized, a parent or patient may request that a Christian Science practitioner be notified as opposed to the hospital-assigned clergy. Coffee and tea are declined as a drink. Christian Science believers seek exemption from immunizations but obey legal requirements. Baptism is not a ceremony for the Christian Science religion. PTS: 1 DIF: Cognitive Level: Application/Applying REF: Table 3.1 OBJ: Integrated Process: Culture and Spirituality MSC: Client Needs: Psychosocial Integrity 10. To resolve family conflict, it is necessary to have open communication, accurate perception of the problem, and a(n) a. intact family structure. b. arbitrator. c. willingness to consider the view of others. d. balance in personality types. ANS: C Without constructive efforts to resolve the conflict, such as the willingness of the members of a group to consider the views of others, conflict resolution cannot take place. The structure of a family may affect family dynamics, but it is still possible to resolve conflict without an intact family structure if all of the ingredients of conflict resolution are present. Conflicts can be resolved without the assistance of an arbitrator. Most families have diverse personality types among their members. This diversity may make conflict resolution more difficult but should not impede it as long as the ingredients of conflict resolution are present. PTS: 1 DIF: Cognitive Level: Comprehension/Understanding REF: p. 37 OBJ: Nursing Process: Assessment MSC: Client Needs: Psychosocial Integrity 11. Which statement is true about the characteristics of a healthy family? a. The parents and children have rigid assignments for all the family tasks. b. Young families assume the total responsibility for the parenting tasks, refusing any assistance. c. The family is overwhelmed by the significant changes that occur as a result of childbirth. d. Adults agree on the majority of basic parenting principles. ANS: D Adults in a healthy family communicate with each other so that minimal discord occurs in parenting principles, such as discipline and sleep schedules. Healthy families remain flexible in their role assignments. Members of a healthy family accept assistance without feeling guilty. Healthy families can adapt to the significant changes that are common during the months after childbirth. PTS: 1 DIF: Cognitive Level: Knowledge/Remembering REF: p. 37 OBJ: Nursing Process: Assessment MSC: Client Needs: Psychosocial Integrity 12. A nurse observes that parents discuss rules with their children when the children do not agree with the rules. Which style of parenting is being displayed? a. Autocratic b. Authoritative c. Permissive d. Disciplinarian ANS: B A parent who discusses the rules with which children do not agree is using an authoritative parenting style. A parent who expects children to follow rules without questioning is using an authoritarian parenting style. A parent who does not consistently enforce rules and allows the child to decide whether he or she wishes to follow rules is using a permissive parenting style. A disciplinarian style would be similar to the authoritarian style. PTS: 1 DIF: Cognitive Level: Knowledge/Remembering REF: p. 43 OBJ: Nursing Process: Assessment MSC: Client Needs: Psychosocial Integrity 13. What should the nurse expect to be problematic for a family whose religious affiliation is Jehovah's Witness? a. Birth control b. Autopsy c. Plasma expanders d. Blood transfusion ANS: D Jehovah's Witnesses do not accept blood transfusions but may accept alternatives such as plasma expanders. Birth control and autopsy are also allowed. PTS: 1 DIF: Cognitive Level: Knowledge/Remembering REF: Table 3.1 OBJ: Integrated Process: Culture and Spirituality MSC: Client Needs: Psychosocial Integrity 14. A traditional family structure in which married male and female partners and their children live as an independent unit is known as a(n) _____ family. a. extended b. binuclear c. nuclear d. blended ANS: C A nuclear family is one in which two opposite-sex parents and their children live together. This is also known as a traditional family. Extended or multigenerational families include other blood relatives in addition to the parents. Binuclear is not a listed family type according to U.S. Census Bureau data but would include two nuclear families living together. A blended family is reconstructed after divorce and involves the merger of two families. PTS: 1 DIF: Cognitive Level: Knowledge/Remembering REF: p. 34 OBJ: Nursing Process: Assessment MSC: Client Needs: Psychosocial Integrity 15. A pictorial tool that can assist the nurse in assessing aspects of family life related to health care is the a. genogram. b. ecomap. c. life cycle model. d. human development wheel. ANS: A A genogram (also known as a pedigree) is a diagram that depicts the relationships and health issues of family members over generations, usually three. An ecomap is a pictorial representation of the family structures and their relationships with the external environment. The life cycle model in no way illustrates a family genogram. This model focuses on stages that a person reaches throughout his or her life. The human development wheel describes various stages of growth and development rather than a family's relationships to each other. PTS: 1 DIF: Cognitive Level: Knowledge/Remembering REF: p. 45 OBJ: Nursing Process: Assessment MSC: Client Needs: Psychosocial Integrity 16. According to Friedman's classifications, providing such physical necessities as food, clothing, and shelter is the __________ family function. a. economic b. socialization c. reproductive d. health care ANS: D Physical necessities such as food, clothing, and shelter are considered part of health care. The economic function provides resources but is not concerned with health care and other basic necessities. The socialization function teaches the child cultural values. The reproductive function is concerned with ensuring family continuity. PTS: 1 DIF: Cognitive Level: Knowledge/Remembering REF: p. 45 OBJ: Nursing Process: Assessment MSC: Client Needs: Physiologic Integrity 17. The nurse is in a unique position to assess children for symptoms of neglect. Which high-risk family situation places the child at the greatest risk for being neglected? a. Marital conflict and divorce b. Adolescent parenting c. Substance abuse d. A child with special needs ANS: C Parents who abuse drugs or alcohol may neglect their children because obtaining and using the substance(s) may have a stronger pull on the parents than the care of their children. Although divorce is traumatic to children, research has shown that living in a home filled with conflict is also detrimental. In this situation conflict may arise and young children may be unable to verbalize their distress; however, the child is not likely to be neglected. Teenage parenting often has a negative effect on the health and social outcomes of the entire family. Adolescent girls are at risk for a number of pregnancy complications, are unlikely to attain a high level of education, and are more likely to be poor. But this does not equate with a higher risk of neglect. When a child is born with a birth defect or has an illness that requires special care, the family is under additional stress. These families often suffer financial hardship as health insurance benefits quickly reach their maximum. But again, this does not lead to neglect as a frequent problem. PTS: 1 DIF: Cognitive Level: Knowledge/Remembering REF: p. 37 OBJ: Nursing Process: Assessment MSC: Client Needs: Health Promotion and Maintenance 18. A nurse is caring for the seriously ill child of a single parent. The parent reports being overwhelmed with the situation and not being able to make decisions. What action by the nurse is best? a. Refer the patient to the hospital's social work department. b. Call the chaplain service and ask for a chaplain visit. c. Ask the parent if any other family member can come and assist. d. Have the parent describe coping methods used for past crises. ANS: D Helping the patient to marshal internal and external resources is vital to promoting coping. The nurse should ask about previous coping methods used and help the parent adapt them to the current situation. Referring the parent to social work does not allow the nurse to be of assistance and the parent may not want to have a visit from a clergy member. Both of those options are dismissive. Other family members may or may not be able to come to assist, but this closed-ended question will not elicit much information. PTS: 1 DIF: Cognitive Level: Application/Applying REF: p. 38 OBJ: Nursing Process: Assessment MSC: Client Needs: Psychosocial Integrity 19. The nurse is caring for a patient from a different culture and is frustrated by what appears to be a lack of cooperation on the patient's part. A colleague states that the patient is "in America and should do what everyone else does." This is an example of what trait? a. Ethnocentrism b. Cultural congruency c. Rudeness d. Ignorance ANS: A Ethnocentrism is the belief that one's culture is superior to any others. The nurse stating that all patients should follow common American behaviors is demonstrating this behavior. This does not demonstrate cultural congruency. Although the colleague may be rude or ignorant, the more specific description of this behavior is ethnocentrism. PTS: 1 DIF: Cognitive Level: Comprehension/Understanding REF: p. 38 OBJ: Integrated Process: Culture and Spirituality MSC: Client Needs: Psychosocial Integrity MULTIPLE RESPONSE

7. To initiate the milk ejection reflex, the mother should a. wear a firm-fitting bra. b. drink plenty of fluids. c. place the infant to the breast d. apply cool packs to her breast.

C Oxytocin, which causes the milk let-down reflex, increases in response to nipple stimulation. A firm bra is important to support the breast but will not initiate the let-down reflex. Drinking plenty of fluids is necessary for adequate milk production but will not initiate the let-down reflex. Cool packs to the breast will decrease the let-down reflex. PTS: 1 DIF: Cognitive Level: Knowledge/Remembering REF: p. 484 OBJ: Nursing Process: Implementation MSC: Client Needs: Health Promotion and Maintenance

8. The greatest risk to the newborn after an elective cesarean birth is a. trauma due to manipulation during delivery. b. tachypnea due to maternal anesthesia. c. prematurity due to miscalculation of gestation. d. tachycardia due to maternal narcotics.

C Regardless of the many criteria used to determine gestational age, inadvertent preterm birth still occurs. There is reduced trauma with a cesarean birth. Maternal anesthesia may cause respiratory distress. Maternal narcotics may cause respiratory distress. PTS: 1 DIF: Cognitive Level: Knowledge/Remembering REF: p. 389 OBJ: Nursing Process: Assessment MSC: Client Needs: Physiologic Integrity

9. While assessing the newborn, the nurse should be aware that the average expected apical pulse range of a full-term, quiet, alert newborn is ________ beats/min. a. 80 to 100 b. 100 to 120 c. 120 to 160 d. 150 to 180

C The average infant heart rate while awake is 120 to 160 beats/min. PTS: 1 DIF: Cognitive Level: Knowledge/Remembering REF: Box 21.3 OBJ: Nursing Process: Assessment MSC: Client Needs: Physiologic Integrity 10. What is a result of hypothermia in the newborn? a. Shivering to generate heat b. Decreased oxygen demands c. Increased glucose demands d. Decreased metabolic rate ANS: C In hypothermia, the basal metabolic rate (BMR) is increased in an attempt to compensate, thus requiring more glucose. Shivering is not an effective method of heat production for newborns. Oxygen demands increase with hypothermia. The metabolic rate increases with hypothermia. PTS: 1 DIF: Cognitive Level: Knowledge/Remembering REF: p. 427 OBJ: Nursing Process: Assessment MSC: Client Needs: Physiologic Integrity 11. The nurse needs to assess infants for the development of high levels of bilirubin. Which baby can the nurse check last? a. Was bruised during a difficult delivery b. Developed a cephalhematoma c. Was born prematurely d. Breastfeeds during the first hour of life ANS: D The infant who is fed early will be less likely to retain meconium and reabsorb bilirubin from the intestines back into the circulation. Bruising, cephalhematomas, and prematurity increase the baby's risk of high bilirubin. PTS: 1 DIF: Cognitive Level: Application/Applying REF: p. 432 OBJ: Nursing Process: Assessment MSC: Client Needs: Physiologic Integrity 12. A nurse is administering vitamin K to an infant shortly after birth. The parents ask why their baby needs a shot. The nurse explains that vitamin K is a. important in the production of red blood cells. b. necessary in the production of platelets. c. not initially synthesized because of a sterile bowel at birth. d. responsible for the breakdown of bilirubin and prevention of jaundice. ANS: C The bowel is initially sterile in the newborn, and vitamin K cannot be synthesized until food is introduced into the bowel. Vitamin K is vital for clotting, so without it the infant is at increased risk of bleeding problems. It is not needed to produce red blood cells, platelets, or break down bilirubin. PTS: 1 DIF: Cognitive Level: Comprehension/Understanding REF: p. 430 OBJ: Integrated Process: Teaching-Learning MSC: Client Needs: Physiologic Integrity 13. A meconium stool can be differentiated from a transitional stool in the newborn because the meconium stool is a. seen at age 3 days. b. the residue of a milk curd. c. passed in the first 12 hours of life. d. lighter in color and looser in consistency. ANS: C Meconium stool is usually passed in the first 12 hours of life, and 99% of newborns have their first stool within 48 hours. If meconium is not passed by 48 hours, obstruction is suspected. It is dark in color and sticky and develops from matter in the intestines during intrauterine life. PTS: 1 DIF: Cognitive Level: Knowledge/Remembering REF: p. 431 OBJ: Nursing Process: Assessment MSC: Client Needs: Physiologic Integrity 14. When the newborn infant is fed, the most likely cause of regurgitation is a. placing the infant in a prone position after a feeding. b. the gastrocolic reflex. c. an underdeveloped pyloric sphincter. d. a relaxed cardiac sphincter. ANS: D The underlying cause of newborn regurgitation is a relaxed cardiac sphincter. PTS: 1 DIF: Cognitive Level: Knowledge/Remembering REF: p. 430 OBJ: Nursing Process: Implementation MSC: Client Needs: Health Promotion and Maintenance 15. The student nurse learns that the process in which bilirubin is changed from a fat-soluble product to a water-soluble product is known as a. enterohepatic circuit. b. conjugation of bilirubin. c. unconjugation of bilirubin. d. albumin binding. ANS: B Conjugation of bilirubin is the process of changing the bilirubin from a fat-soluble to a water-soluble product. The enterohepatic circuit is the route by which part of the bile produced by the liver enters the intestine, is reabsorbed by the liver, and then is recycled into the intestine. Unconjugated bilirubin is fat soluble. Albumin binding is to attach something to a protein molecule. PTS: 1 DIF: Cognitive Level: Knowledge/Remembering REF: p. 431 OBJ: Integrated Process: Teaching-Learning MSC: Client Needs: Physiologic Integrity 16. Which statement is correct regarding the fluid balance in a newborn versus that in an adult? a. The infant has a smaller percentage of surface area to body mass. b. The infant has a smaller percentage of water to body mass. c. The infant has a greater percentage of insensible water loss. d. The infant has a 50% more effective glomerular filtration rate. ANS: C Insensible water loss is greater in the infant due to the newborn's large body surface area and rapid respiratory rate. The infant's surface area is large compared to an adult's. Infants have a larger percentage of water to body mass. The filtration rate is less than in adults because the kidneys are immature in a newborn. PTS: 1 DIF: Cognitive Level: Knowledge/Remembering REF: p. 434 OBJ: Nursing Process: Assessment MSC: Client Needs: Physiologic Integrity 17. An infant has an elevated immunoglobulin M (IgM) level. What action by the nurse is most appropriate? a. Encourage the mother to breastfeed the baby. b. Document the findings in the infant's chart. c. Assess the infant for other signs of allergy. d. Take a set of vital signs on the infant, and then notify the provider. ANS: D An elevated level of IgM is associated with exposure to infection in utero because IgM does not cross the placenta. The nurse should take a set of vital signs and notify the provider so further investigation can occur. It is not related to breastfeeding or allergies. The information should be documented, but this is not the most important action. PTS: 1 DIF: Cognitive Level: Application/Applying REF: p. 434 OBJ: Nursing Process: Assessment MSC: Client Needs: Physiologic Integrity 18. In which infant behavioral state is bonding most likely to occur? a. Drowsy b. Active alert c. Quiet alert d. Crying ANS: C In the quiet alert state, the infant is interested in his or her surroundings and will often gaze at the mother or father or both. In the drowsy state the eyes may remain closed. If open they are unfocused. The infant is not interested in the environment at this time. In the active alert state infants are often fussy, restless, and not focused. During the crying state the infant does not respond to stimulation and cannot focus on parents. PTS: 1 DIF: Cognitive Level: Knowledge/Remembering REF: p. 435 OBJ: Nursing Process: Assessment MSC: Client Needs: Health Promotion and Maintenance 19. To prevent heat loss from convection in a newborn, which action by the nurse is best? a. Place the baby in a warmer. b. Dry the baby after a bath. c. Move infant away from blowing fan. d. Wrap the baby in warmed blankets. ANS: C Convection occurs when infants are exposed to cold air currents. Moving the baby out of the fan's air currents will reduce this loss. The warmer prevents heat loss from radiant heat loss. Drying the baby prevents evaporative heat loss. Warm blankets prevent conductive heat loss. PTS: 1 DIF: Cognitive Level: Application/Applying REF: p. 427 OBJ: Nursing Process: Implementation MSC: Client Needs: Physiologic Integrity 20. The hips of a newborn are examined for developmental dysplasia. Which sign indicates an incomplete development of the acetabulum? a. Negative Ortolani's sign b. Asymmetric thigh and gluteal creases c. Negative Barlow test d. Equal knee heights ANS: B Asymmetric thigh and gluteal creases may indicate potential dislocation of the hip. Positive Ortolani's sign yields a "clunking" sensation and indicates a dislocated femoral head moving into the acetabulum. During a positive Barlow test, the examiner can feel the femoral head move out of the acetabulum. If the hip is dislocated, the knee on the affected side will be lower. PTS: 1 DIF: Cognitive Level: Knowledge/Remembering REF: p. 444 OBJ: Nursing Process: Assessment MSC: Client Needs: Health Promotion and Maintenance 21. Which newborn reflex is elicited by stroking the lateral sole of the infant's foot from the heel to the ball of the foot? a. Babinski b. Tonic neck c. Stepping d. Plantar grasp ANS: A The Babinski reflex causes the toes to flare outward and the big toe to dorsiflex. The tonic neck reflex (also called the fencing reflex) refers to the posture assumed by newborns when in a supine position. The stepping reflex occurs when infants are held upright with their heel touching a solid surface and the infant appears to be walking. The plantar grasp reflex is similar to the palmar grasp reflex: when the area below the toes are touched, the infant's toes curl over the nurse's finger. PTS: 1 DIF: Cognitive Level: Knowledge/Remembering REF: p. 448 | Table 21.3 OBJ: Nursing Process: Assessment MSC: Client Needs: Health Promotion and Maintenance 22. Infants in whom cephalhematomas develop are at increased risk for a. infection. b. jaundice. c. caput succedaneum. d. erythema toxicum. ANS: B Cephalhematomas are characterized by bleeding between the bone and its covering, the periosteum. Because of the breakdown of the red blood cells within a hematoma, the infants are at greater risk for jaundice. Cephalhematomas do not increase the risk for infection, caput, or erythema toxicum. PTS: 1 DIF: Cognitive Level: Knowledge/Remembering REF: p. 451 | Box 21.5 OBJ: Nursing Process: Assessment MSC: Client Needs: Physiologic Integrity 23. A maculopapular rash with a red base and a small white papule in the center is a. milia. b. mongolian spots. c. erythema toxicum. d. café au lait spots. ANS: C This is a description of erythema toxicum, a normal rash in the newborn. Milia are minute epidermal cysts on the face of the newborn. Mongolian spots are bluish-black discolorations found on dark-skinned newborns, usually on the sacrum. Café au lait spots are pale tan (the color of coffee with milk) macules. Occasional spots occur normally in newborns, but they can indicate a genetic disorder. PTS: 1 DIF: Cognitive Level: Knowledge/Remembering REF: p. 435 OBJ: Nursing Process: Assessment MSC: Client Needs: Health Promotion and Maintenance 24. Plantar creases should be evaluated within a few hours of birth because a. the newborn has to be footprinted. b. as the skin dries, the creases will become more prominent. c. heel sticks may be required. d. creases will be less prominent after 24 hours. ANS: B As the infant's skin begins to dry, the creases will appear more prominent, and the infant's gestation could be misinterpreted. Footprinting will not interfere with the creases. Heel sticks will not interfere with the creases. The creases will appear more prominent after 24 hours. PTS: 1 DIF: Cognitive Level: Knowledge/Remembering REF: p. 454 OBJ: Nursing Process: Assessment MSC: Client Needs: Health Promotion and Maintenance 25. A newborn who is large for gestational age (LGA) is _________ percentile for weight. a. below the 90th b. less than the 10th c. greater than the 90th d. between the 10th and 90th ANS: C The LGA rating is based on weight and is defined as greater than the 90th percentile in weight. An infant between the 10th and 90th percentiles is average for gestational age. An infant in less than the 10th percentile is small for gestational age. PTS: 1 DIF: Cognitive Level: Knowledge/Remembering REF: p. 458 OBJ: Nursing Process: Assessment MSC: Client Needs: Health Promotion and Maintenance 26. A new mother asks, "Why are you doing a gestational age assessment on my baby? I delivered on time." The nurse's best response is a. "This must be done to meet insurance requirements." b. "It helps us identify infants who are at risk for any problems." c. "The gestational age determines how long the infant will be hospitalized." d. "It was ordered by your doctor." ANS: B The nurse should provide the mother with accurate information about various procedures performed on the newborn. A gestational age assessment helps identify at-risk infants. It is not done for insurance requirements or to determine hospital days. Assessing gestational age is a nursing assessment and does not have to be ordered. PTS: 1 DIF: Cognitive Level: Comprehension/Understanding REF: p. 454 OBJ: Integrated Process: Teaching-Learning MSC: Client Needs: Health Promotion and Maintenance 27. Which nursing action is designed to avoid unnecessary heat loss in the newborn? a. Place a blanket over the scale before weighing the infant. b. Maintain room temperature at 70° F. c. Undress the infant completely for assessments so they can be finished quickly. d. Take the rectal temperature every hour to detect early changes. ANS: A Padding the scale prevents heat loss from the infant to a cold surface by conduction. Room temperature should be appropriate to prevent heat loss from convection. Also, if the room is warm enough, radiation will assist in maintaining body heat. Undressing the infant completely will expose the child to cooler room temperatures and cause a drop in body temperature due to convection. Hourly assessments are not necessary for a normal newborn with a stable temperature. Rectal temperatures are usually not done on the newborn. PTS: 1 DIF: Cognitive Level: Application/Applying REF: p. 428 | Figure 21.2 OBJ: Nursing Process: Implementation MSC: Client Needs: Physiologic Integrity 28. What characteristic shows the greatest gestational maturity? a. Few rugae on the scrotum and testes high in the scrotum b. Infant's arms and legs extended c. Some peeling and cracking of the skin d. The arm can be positioned with the elbow beyond the midline of the chest ANS: C Peeling, cracking, dryness, and a few visible veins in the skin are signs of maturity in the newborn. The other signs are indicative of a younger gestational age. PTS: 1 DIF: Cognitive Level: Knowledge/Remembering REF: p. 437 | Table 21.2 OBJ: Nursing Process: Assessment MSC: Client Needs: Health Promotion and Maintenance 29. The nurse is concerned about an infection in a newborn. What finding does the nurse assess for? a. More than two soft stools per day b. Leukocytosis with a left shift c. Poor feeding behaviors d. An axillary temperature greater than 37.5° C ANS: D Due to their immature immune system, newborns often do not have fever and leukocytosis with infection. Signs of infection are subtler and include changes in activity, tone, feeding, and color. More than two stools is an expected finding. PTS: 1 DIF: Cognitive Level: Application/Applying REF: p. 434 OBJ: Nursing Process: Assessment MSC: Client Needs: Physiologic Integrity 30. An African-American woman noticed some bruises on her newborn girl's buttocks. She asks the nurse who spanked her daughter. The nurse explains that these marks are called a. lanugo. b. vascular nevi. c. nevus flammeus. d. mongolian spots. ANS: D A mongolian spot is a bluish black area of pigmentation that may appear over any part of the exterior surface of the body. It is more commonly noted on the back and buttocks and most frequently is seen on infants whose ethnic origins are Mediterranean, Latin American, Asian, or African. Lanugo is the fine, downy hair seen on a term newborn. A vascular nevus, commonly called a strawberry mark, is a type of capillary hemangioma. A nevus flammeus, commonly called a port-wine stain, is most frequently found on the face. PTS: 1 DIF: Cognitive Level: Comprehension/Understanding REF: p. 435 OBJ: Nursing Process: Diagnosis MSC: Client Needs: Health Promotion and Maintenance 31. What is the quickest and most common method to obtain neonatal blood for glucose screening 1 hour after birth? a. Puncture the lateral pad of the heel. b. Obtain a sample from the umbilical cord. c. Puncture a fingertip. d. Obtain a laboratory chemical determination. ANS: A A drop of blood obtained by heel stick is the quickest method of glucose screening. The calcaneus bone should be avoided as osteomyelitis may result from injury to the foot. Most umbilical cords are clamped in the delivery room and are not available for routine testing. A neonate's fingertips are too fragile to use for this purpose. Laboratory chemical determination is the most accurate but the lengthiest method. PTS: 1 DIF: Cognitive Level: Knowledge/Remembering REF: p. 449 OBJ: Nursing Process: Implementation MSC: Client Needs: Health Promotion and Maintenance 32. A new mother states that her infant must be cold because the baby's hands and feet are blue. The nurse explains that this is a common and temporary condition called a. acrocyanosis. b. erythema neonatorum. c. harlequin color. d. vernix caseosa. ANS: A Acrocyanosis, or the appearance of slightly cyanotic hands and feet, is caused by vasomotor instability, capillary stasis, and a high hemoglobin level. Acrocyanosis is normal and appears intermittently over the first 7 to 10 days. Erythema toxicum (also called erythema neonatorum) is a transient newborn rash that resembles flea bites. The harlequin sign is a benign, transient color change in newborns. Half of the body is pale, and the other half is ruddy or bluish red with a line of demarcation. Vernix caseosa is a cheeselike, whitish substance that serves as a protective covering. PTS: 1 DIF: Cognitive Level: Comprehension/Understanding REF: p. 428 | Figure 21.2 OBJ: Integrated Process: Teaching-Learning MSC: Client Needs: Health Promotion and Maintenance 33. The parents of a newborn ask the nurse how much the newborn can see. The parents specifically want to know what types of visual stimuli they should provide for their newborn. The nurse responds to the parents by telling them a. "Infants can see very little until about 3 months of age." b. "Infants can track their parent's eyes and prefer complex patterns." c. "The infant's eyes must be protected. Infants enjoy looking at bright stripes." d. "It's important to shield the newborn's eyes. Overhead lights help them see better." ANS: B Infants can track their parents' faces, including eyes and prefer to look at complex patterns. Newborns seem to have clearest visual acuity at about 19 cm. Infants prefer complex patterns, regardless of color and also prefer low lighting. PTS: 1 DIF: Cognitive Level: Comprehension/Understanding REF: p. 445 | p. 448 OBJ: Nursing Process: Planning MSC: Client Needs: Health Promotion and Maintenance 34. A first-time father is changing the diaper of his 1-day-old daughter. He asks the nurse, "What is this black, sticky stuff in her diaper?" The nurse's best response is a. "That's meconium, which is your baby's first stool. It's normal." b. "That's transitional stool." c. "That means your baby is bleeding internally." d. "Oh, don't worry about that. It's okay." ANS: A This describes a meconium stool, which the nurse should educate the father about. It is not a transitional stool nor does it indicate bleeding. Telling the father not to worry about it is belittling and does not provide information. PTS: 1 DIF: Cognitive Level: Comprehension/Understanding REF: p. 451 | Box 21.5 OBJ: Integrated Process: Teaching-Learning MSC: Client Needs: Health Promotion and Maintenance 35. A nurse assesses a newborn's lab values and notes a WBC of 31,000 mm3. What action by the nurse is best? a. Take a set of vital signs and notify the provider. b. Document the findings in the infant's chart. c. Follow unit protocol to initiate a sepsis workup. d. Perform a heel stick for a bedside blood glucose reading. ANS: B The leukocyte (white blood cell [WBC]) count at birth is 9100 to 34,000/mm3. This is a normal finding so the only action required is to document these results. PTS: 1 DIF: Cognitive Level: Application/Applying REF: p. 430 OBJ: Integrated Process: Communication and Documentation MSC: Client Needs: Health Promotion and Maintenance 36. What information does the student learn about the newborn's developing cardiovascular system? a. The heart rate of a crying infant may rise to 120 beats/min. b. Heart murmurs heard after the first few hours are cause for concern. c. The point of maximal impulse (PMI) is on the third or fourth left intercostal space. d. Persistent bradycardia may indicate respiratory distress syndrome (RDS). ANS: C The newborns' PMI is found in the left third to fourth intercostal space. The normal heart rate for infants who are not sleeping is 120 to 160 beats/min. However, a crying infant temporarily could have a heart rate of 180 beats/min. Heart murmurs during the first few days of life have no pathologic significance; an irregular heart rate past the first few hours should be evaluated further. Persistent tachycardia may indicate RDS; bradycardia may be a sign of congenital heart blockage. PTS: 1 DIF: Cognitive Level: Comprehension/Understanding REF: p. 436 | Table 21.2 OBJ: Nursing Process: Assessment MSC: Client Needs: Health Promotion and Maintenance 37. The cheeselike, whitish substance that fuses with the epidermis and serves as a protective coating is called a. vernix caseosa b. surfactant c. caput succedaneum d. acrocyanosis ANS: A Vernix caseosa is a cheeselike substance on the skin. This protection is needed because the infant's skin is so thin. Surfactant is a protein that lines the alveoli of the infant's lungs. Caput succedaneum is the swelling of the tissue over the presenting part of the fetal head. Acrocyanosis is cyanosis of the hands and feet, resulting in a blue coloring. PTS: 1 DIF: Cognitive Level: Knowledge/Remembering REF: p. 452 OBJ: Nursing Process: Assessment MSC: Client Needs: Health Promotion and Maintenance 38. A nurse receives handoff report. Which newborn should the nurse assess first? a. Temperature 97.7° F (36.5° C) b. Pulse 144 beats/minute c. Respiratory rate 78 breaths/minute d. Glucose reading 58 mg/dL ANS: C A newborn's respiratory rate should be 30 to 60 breaths/minute, so the nurse needs to assess the infant with the high respiratory rate first. The other values are within normal limits. PTS: 1 DIF: Cognitive Level: Analysis/Analyzing REF: p. 435 OBJ: Nursing Process: Assessment MSC: Client Needs: Safe and Effective Care Environment 39. A nurse assesses a newborn and finds him to be jittery with a poor suck reflex. What action by the nurse takes priority? a. Ensure the warmer is set to the correct temperature. b. Obtain a heel stick for bedside glucose reading. c. Listen to the newborn's heart and lungs. d. Perform a gestational age assessment. ANS: B These are signs of possible hypoglycemia. The nurse should obtain blood for a glucose determination. Ensuring the warmer is set correctly and further assessing the baby are appropriate but not related to these findings. There would be no need to repeat the gestational age assessment. PTS: 1 DIF: Cognitive Level: Application/Applying REF: p. 449 | Safety Alert Box OBJ: Nursing Process: Assessment MSC: Client Needs: Physiologic Integrity 40. An infant has been exposed to cold stress. After taking measures to warm the infant, what action does the nurse perform next? a. Obtain a blood glucose reading. b. Listen to the infant's lungs. c. Document the warming interventions. d. Determine how the baby got cold. ANS: A In trying to maintain temperature, the infant expends a lot of energy, using glucose. The infant is at risk of hypoglycemia, so a glucose reading should be obtained. Documenting and investigating the incident are important but need to wait until the glucose is obtained and actions taken if needed. Listening to the lungs is not specifically needed in this case since there is no indication that the infant has respiratory distress. This action can occur later as well. PTS: 1 DIF: Cognitive Level: Application/Applying REF: p. 429 | Box 21.1 OBJ: Nursing Process: Assessment MSC: Client Needs: Safe and Effective Care Environment 41. A nurse is supervising a student nurse who is assessing an infant's rooting reflex. Which action by the student warrants further instruction by the nurse? a. Tells parents this reflex will disappear within 4 months b. Strokes face from side of mouth to cheek c. Notes normal findings when infant turns head toward touch d. Performs assessment on infant while sleeping ANS: D This reflex is difficult to assess on an infant just after feeding or when asleep. The other actions by the student are correct. PTS: 1 DIF: Cognitive Level: Application/Applying REF: p. 448 | Table 21.3 OBJ: Nursing Process: Assessment MSC: Client Needs: Health Promotion and Maintenance MULTIPLE RESPONSE

2. The labor nurse is evaluating the patient's most recent 10-minute segment on the monitor strip and notes a late deceleration. This is likely to be caused by which physiologic alterations? (Select all that apply.) a. Spontaneous fetal movement b. Compression of the fetal head c. Placental abruption d. Cord around the baby's neck e. Maternal supine hypotension

C, E Late decelerations are almost always caused by uteroplacental insufficiency. Insufficiency is caused by uterine tachysystole, maternal hypotension, epidural or spinal anesthesia, IUGR, intraamniotic infection, or placental abruption. Spontaneous fetal movement, vaginal examination, fetal scalp stimulation, fetal reaction to external sounds, uterine contractions, fundal pressure, and abdominal palpation are all likely to cause accelerations of the FHR. Early decelerations are most often the result of fetal head compression and may be caused by uterine contractions, fundal pressure, vaginal examination and placement of an internal electrode. A variable deceleration is likely caused by umbilical cord compression. This may happen when the cord is around the baby's neck, arm, leg or other body part, a short cord, a knot in the cord, or a prolapsed cord. PTS: 1 DIF: Cognitive Level: Knowledge/Remembering REF: Table 17.1 OBJ: Nursing Process: Assessment MSC: Client Needs: Health Promotion and Maintenance

9. In providing anticipatory guidance to parents, which parental behavior does the nurse teach as most important in fostering moral development? a. Telling the child what is right and wrong b. Vigilantly monitoring the child and her peers c. Weekly family meetings to discuss behavior d. Living as the parents say they believe

D Parents living what they believe gives non-ambivalent messages and fosters the child's moral development and reasoning. Telling the child what is right and wrong is not effective unless the child has experienced what she hears. Parents need to live according to the values they are teaching to their children. Vigilant monitoring of the child and her peers is an inappropriate action for the parent to initiate. It does not foster moral development and reasoning in the child. Weekly family meetings to discuss behaviors may or may not be helpful in the development of moral reasoning. PTS: 1 DIF: Cognitive Level: Application/Applying REF: p. 135 OBJ: Integrated Process: Teaching-Learning MSC: Client Needs: Health Promotion and Maintenance 10. The nurse is providing anticipatory guidance for parents of a school-age child. Which behavior does the nurse suggest to best assist the child in negotiating the developmental task of industry? a. Identifying failures immediately and asking the child's peers for feedback b. Structuring the environment so that the child can master tasks c. Completing homework for children who are having difficulty with them d. Decreasing expectations to eliminate potential failures ANS: B The task of the caring teacher or parent is to identify areas in which a child is competent and to build on successful experiences to foster feelings of mastery and success. Structuring the environment to enhance self-confidence and to provide the opportunity to solve increasingly more complex problems will promote a sense of mastery. Asking peers for feedback reinforces the child's feelings of failure. When parents complete children's homework for them, it sends the message that they do not trust their child to do a good job. Providing assistance and suggestions and praising their best efforts are more appropriate. Decreasing expectations to eliminate failures will not promote a sense of achievement or mastery. PTS: 1 DIF: Cognitive Level: Application/Applying REF: p. 141 OBJ: Integrated Process: Teaching-Learning MSC: Client Needs: Health Promotion and Maintenance 11. A nurse is assessing an older school-age child recently admitted to the hospital. Which assessment does the nurse perform to determine if child is in an appropriate stage of cognitive development? a. Give the child a collection of similar objects, and ask him or her to organize them. b. Ask the child to perform a series of math problems using subtraction. c. Determine the child's vocabulary and reading comprehension. d. Find out what play activities the child enjoys engaging in. ANS: B The ability to classify things from simple to complex and the ability to identify differences and similarities are cognitive skills of the older school-age child; this demonstrates use of classification and logical thought processes. The emergency of this ability explains why children of this age enjoy collecting things. Subtraction and addition are appropriate cognitive activities for the young school-age child. Vocabulary is not as valid an assessment of cognitive ability as is the child's ability to classify. Play activity is not as valid an assessment of cognitive function as is the child's ability to classify. PTS: 1 DIF: Cognitive Level: Application/Applying REF: p. 134 OBJ: Nursing Process: Assessment MSC: Client Needs: Health Promotion and Maintenance 12. Which does the nurse teach as an appropriate disciplinary intervention for the school-age child? a. Time-out periods b. Consequences that are consistent with the behavior c. Physical punishment d. Lectures about inappropriate behavior ANS: B A consequence that is related to the inappropriate behavior is the recommended discipline. Responsibility can be developed in children through the use of natural and logical consequences related to actions. Time-out periods are more appropriate for younger children. Physical intervention is an inappropriate form of discipline. It does not connect the discipline with the child's inappropriate behavior. Lengthy discussions typically are not helpful. PTS: 1 DIF: Cognitive Level: Comprehension/Understanding REF: p. 139 OBJ: Integrated Process: Teaching-Learning MSC: Client Needs: Health Promotion and Maintenance 13. A parent of a chubby 8-year-old wants to know how to keep the child from gaining more weight. What response by the nurse is best? a. Do not allow your child to snack. b. Make a school lunch every day. c. Model the behaviors you'd like to see. d. Place your child on a restricted diet. ANS: C One good option for obesity prevention is to model the behaviors the parents want the child to emulate. The parents should set good examples with eating health and engaging in regular exercise. Snacks, if healthy, can be an important part of a nutritious day. Even if the parent makes a lunch for school each day, there is no guarantee the child will eat it. Children will likely rebel against a strict diet. PTS: 1 DIF: Cognitive Level: Comprehension/Understanding REF: p. 144 OBJ: Integrated Process: Teaching-Learning MSC: Client Needs: Physiologic Integrity 14. An 8-year-old girl tells the nurse that she has cancer because God is punishing her for "being bad." She shares her concern that if she dies, she will go to hell. What action by the nurse is most appropriate? a. Reassure the child that she is not being punished. b. Share concerns about development with the parents. c. Request a child-life specialist to intervene. d. Have the chaplain console the child. ANS: A Children at this age may view illness or injury as a punishment for a real or imagined transgression. The nurse should reassure the child that she is not being punished. Since this is a common belief at this age, there are no concerns to share with parents. A child-life specialist or chaplain visit may be appropriate, but the nurse needs to respond to this statement him- or herself. PTS: 1 DIF: Cognitive Level: Application/Applying REF: p. 136 OBJ: Nursing Process: Implementation MSC: Client Needs: Psychosocial Integrity 15. A group of boys ages 9 and 10 years have formed a "boys-only" club that is open to neighborhood and school friends who have skateboards. This should be interpreted as a. behavior that encourages bullying and sexism. b. behavior that reinforces poor peer relationships. c. characteristic of social development of this age. d. characteristic of children who are at risk for membership in gangs. ANS: C One of the outstanding characteristics of middle childhood is the creation of formalized groups or clubs. Peer-group identification and association are essential to a child's socialization. Poor relationships with peers and a lack of group identification can contribute to bullying. A boys-only club does not have a direct correlation with later gang activity. PTS: 1 DIF: Cognitive Level: Application REF: p. 134 OBJ: Nursing Process: Assessment MSC: Client Needs: Health Promotion and Maintenance 16. A 9-year-old girl often comes to the school nurse complaining of stomach pains. Her teacher says she is completing her schoolwork satisfactorily, but lately she has been somewhat aggressive and stubborn in the classroom. What action by the school nurse is most appropriate? a. Assess the child for unusual stress. b. Perform a detailed physical exam. c. Call the parents in for a conference. d. Screen the child for developmental delay. ANS: A Signs of stress include stomach pains or headache, sleep problems, bedwetting, changes in eating habits, aggressive or stubborn behavior, reluctance to participate, or regression to early behaviors. The nurse should assess the child for stress. The other actions are not warranted although the nurse may want to have a conference with parents after screening the child. PTS: 1 DIF: Cognitive Level: Application/Applying REF: Box 8.2 OBJ: Nursing Process: Assessment MSC: Client Needs: Psychological Integrity 17. The school nurse has been asked to begin teaching sex education in the 5th grade. The nurse should recognize that a. children in 5th grade are too young for sex education. b. children should be discouraged from asking too many questions. c. correct terminology should be reserved for children who are older. d. sex can be presented as a normal part of growth and development. ANS: D When sexual information is presented to school-age children, sex should be treated as a normal part of growth and development. Fifth graders are usually 10 to 11 years old. This age is not too young to speak about physiologic changes in their bodies. The students should be encouraged to ask questions. Preadolescents need precise and concrete information. PTS: 1 DIF: Cognitive Level: Comprehension/Understanding REF: p. 132 OBJ: Nursing Process: Implementation MSC: Client Needs: Health Promotion and Maintenance 18. A nurse is interested in preventing injuries to children while they play. What action by the nurse would most likely lead to the biggest impact? a. Volunteering for an organization that gives away bicycle helmets. b. Providing education on the need for knee pads when skating. c. Teaching parents that children too big for child care seats should sit in the front seat. d. Encouraging children to play only on formal, constructed playgrounds. ANS: A Head injuries from bicycles are a large part of serious injury to children in this age group. They need to be taught to only ride a bike while wearing a helmet. The nurse's best option is to volunteer for an organization that gives away helmets. Knee pads when skating is also a good idea, but that won't have the impact of helmets. Once a child is too big for a child care seat and the seat belt fits appropriately, the child should sit in the back seat. Playing on constructed playgrounds only will not prevent injuries and is unrealistic. PTS: 1 DIF: Cognitive Level: Application/Applying REF: p. 139 OBJ: Nursing Process: Implementation MSC: Client Needs: Health Promotion and Maintenance 19. A parent reports getting annoyed with a 6-year-old child who seems to always get cranky and irritable when playing with friends. What suggestion by the nurse is best? a. Maybe he should not play with those friends anymore. b. The parents should monitor the children's play more closely. c. When the child gets cranky he should be told to rest. d. The parents should assess the child's diet for protein. ANS: C Children often do not recognize that they are becoming fatigued. Six-year-olds in particular are quite bad about this. Signs of fatigue include being cranky. The parent should have the child rest at this point. Forbidding the child's friends, monitoring play more closely, and assessing the diet for protein are not needed for this problem. PTS: 1 DIF: Cognitive Level: Application/Applying REF: p. 133 OBJ: Integrated Process: Teaching-Learning MSC: Client Needs: Physiologic Integrity 20. A school nurse reports to the parents that their child is complaining of frequent headaches. What suggestion does the nurse offer to the parents? a. A complete neurologic workup b. A vision screening exam c. Decreased amount of household stress d. Assessment for seasonal allergies ANS: B Children often manifest visual problems during the school-age period. These children may squint, move closer to the television or to the front of the class if possible, or complain of headaches. The parents should obtain a visual screening exam for their child. None of the other options is needed at this point. PTS: 1 DIF: Cognitive Level: Application/Applying REF: p. 134 OBJ: Nursing Process: Implementation MSC: Client Needs: Physiologic Integrity 21. A school-age child got a hand-knitted sweater from a relative as a gift. The child refuses to wear it, and it is causing a great deal of conflict in the family as the relative wants to see the child in it. What information can the nurse provide the family about this issue? a. This is a time when strict discipline is needed and should be enforced. b. It's best to choose your battles carefully or you'll fight over everything. c. Teach the child a polite way of expressing dislike for the sweater. d. Children this age find it painful to be different from their peers. ANS: D Children at this age do find it very painful to be different in any way from their peers. The sweater may be very different from anything the peers are wearing, which makes the child reluctant to wear it. The nurse can provide this information to the family so they have information they can use in working out a solution to this problem. Strict discipline is not needed. Telling parents to choose their battles does not help them solve this situation. Children should be taught polite ways in which to express themselves, but this also does not help to solve the family conflict. PTS: 1 DIF: Cognitive Level: Application/Applying REF: p. 136 OBJ: Integrated Process: Teaching-Learning MSC: Client Needs: Health Promotion and Maintenance 22. A parent is worried that a child is not eating well. What does the nurse teach the parent to address this problem? a. Limit sports and team events that occur over the dinner hour. b. Pack a nutritious lunch to take to school every day. c. Teach about healthy snacks available at school. d. Ensure the child gets 2 cups of milk products a day. ANS: A Sports and team schedules often disrupt mealtime, especially dinner, and families often find themselves eating fast food on the way to practices and games. The family's best option is to limit activities that occur during this time. The child may not eat a packed lunch and may choose unhealthy foods from the schools' vending machines. Children in this age group need 3 cups of milk and dairy products per day. PTS: 1 DIF: Cognitive Level: Application/Applying REF: p. 145 OBJ: Nursing Process: Implementation MSC: Client Needs: Health Promotion and Maintenance MULTIPLE RESPONSE

9. A new mother is distresses over the "white substance" covering her infant because it "looks ugly." What action by the nurse is most appropriate? a. Scrub the substance off of the baby. b. Reassure the mom that it will go away. c. Report the findings to the provider. d. Explain that the vernix caseosa protects fetal skin from amniotic fluid.

D Prolonged exposure to amniotic fluid during the fetal period could result in breakdown of the skin without the protection of the vernix caseosa. This can be washed off gently, when the baby gets the first bath. Although it will not remain, this statement does not explain the purpose of the substance. This does not need to be reported. PTS: 1 DIF: Cognitive Level: Comprehension/Understanding REF: p. 204 OBJ: Integrated Process: Teaching-Learning MSC: Client Needs: Health Promotion and Maintenance 10. A woman who is 16 weeks pregnant asks the nurse, "Is it possible to tell by ultrasound if the baby is a boy or girl yet?" The best answer is a. "A baby's sex is determined as soon as conception occurs, and the differences are apparent." b. "The baby has developed enough that we can determine the sex by examining the genitals through ultrasound." c. "Boys and girls look alike until approximately 20 weeks after conception, and then they begin to look different." d. "It might be possible to determine your baby's sex, but the external organs look very similar right now." ANS: B Although gender is determined at conception, the external genitalia of males and females look similar through the 9th week. By the 12th week, the external genitalia are distinguishable as male or female. PTS: 1 DIF: Cognitive Level: Comprehension/Understanding REF: p. 204 OBJ: Integrated Process: Teaching-Learning MSC: Client Needs: Physiologic Integrity 11. The placenta allows exchange of oxygen, nutrients, and waste products between the mother and fetus by a. contact between maternal blood and fetal capillaries within the chorionic villi. b. interaction of maternal and fetal pH levels within the endometrial vessels. c. a mixture of maternal and fetal blood within the intervillous spaces. d. passive diffusion of maternal carbon dioxide and oxygen into the fetal capillaries. ANS: A Fetal capillaries within the chorionic villi are bathed with oxygen- and nutrient-rich maternal blood within the intervillous spaces. The endometrial vessels are part of the uterus. There is no interaction with the fetal blood at this point. Maternal and fetal blood do not normally mix. Maternal carbon dioxide does not enter into the fetal circulation. PTS: 1 DIF: Cognitive Level: Knowledge/Understanding REF: p. 206 OBJ: Integrated Process: Teaching-Learning MSC: Client Needs: Physiologic Integrity 12. A patient is sent from the physician's office for assessment of oligohydramnios. The nurse is aware that this condition can result in a. excessive fetal urine secretion. b. newborn respiratory distress. c. central nervous system abnormality. d. gastrointestinal blockage. ANS: B Because an abnormally small amount of amniotic fluid restricts normal lung development, the infant may have inadequate respiratory function after birth, when the placenta no longer performs respiratory function. Oligohydramnios may be caused by a decrease in urine secretion. Excessive amniotic fluid production may occur when the fetus has a central nervous system abnormality. Excessive amniotic fluid production may occur when the gastrointestinal tract prevents normal ingestion of amniotic fluid. PTS: 1 DIF: Cognitive Level: Knowledge/Remembering REF: p. 209 OBJ: Nursing Process: Assessment MSC: Client Needs: Physiologic Integrity 13. When explaining twin conception, the nurse points out that dizygotic twins develop from a. a single fertilized ovum and are always of the same sex. b. a single fertilized ovum and may be the same sex or different sexes. c. two fertilized ova and are the same sex. d. two fertilized ova and may be the same sex or different sexes. ANS: D Dizygotic twins are two different zygotes, each conceived from a single ovum and a single sperm. They may be both male, both female, or one male and one female. A single fertilized ovum that produces twins is called monozygotic. Monozygotic twins are always the same sex. Dizygotic twins are from two fertilized ova and may or may not be the same sex. PTS: 1 DIF: Cognitive Level: Knowledge/Remembering REF: p. 213 OBJ: Integrated Process: Teaching-Learning MSC: Client Needs: Physiologic Integrity 14. Which statement related to oogenesis is correct? a. Two million primary oocytes will mature. b. At birth, all ova are contained in the female's ovaries. c. The oocytes complete their division during fetal life. d. Monthly, at least two oocytes mature. ANS: B All of the cells that may undergo meiosis in a woman's lifetime are contained in the ovaries at birth. Only 400 to 500 ova will mature during the approximately 35 years of a woman's reproductive life. The primary oocytes begin their first meiotic division during fetal life but remain suspended until puberty. Every month, one primary oocyte matures and completes meiotic division yielding two unequal cells. PTS: 1 DIF: Cognitive Level: Knowledge/Remembering REF: p. 195 OBJ: Integrated Process: Teaching-Learning MSC: Client Needs: Physiologic Integrity 15. After implantation, tiny projections develop out of the trophoblast and extend into the endometrium. These projections are referred to as a. decidua basalis. b. decidua capsularis. c. decidua vera. d. chorionic villi. ANS: D These villi are vascular processes that obtain oxygen and nutrients from the maternal bloodstream and dispose of carbon dioxide and waste products into the maternal blood. The deciduas basalis is the portion of the endometrium where the chorionic villi tap into the maternal blood vessels. The deciduas capsularis is the portion of the endometrium that covers the blastocyst. The portion of the endometrium that lines the rest of the uterus is called decidua vera. PTS: 1 DIF: Cognitive Level: Knowledge/Remembering REF: p. 199 OBJ: Integrated Process: Teaching-Learning MSC: Client Needs: Physiologic Integrity 16. A nurse is teaching a prenatal class. The nurse teaches that during weeks 25 to 28, which fetal development occurs? a. Eyes reopen. b. Vernix caseosa covers the skin. c. Lanugo may develop. d. Brown fat is deposited. ANS: A During this time frame the eyes reopen, and the fetus becomes plumper with smoother skin. The other changes occur during weeks 17 to 20. PTS: 1 DIF: Cognitive Level: Comprehension/Understanding REF: p. 206 OBJ: Integrated Process: Teaching-Learning MSC: Client Needs: Physiologic Integrity 17. A young patient comes in for her first prenatal examination. This is her first child. She asks "How does my baby get air inside my uterus?" The correct response is a. "The baby's lungs work in the uterus to exchange oxygen and carbon dioxide." b. "The baby absorbs oxygen from your blood system." c. "The placenta provides oxygen to the baby and excretes carbon dioxide into your bloodstream." d. "The placenta delivers oxygen-rich blood through the umbilical artery to the baby's abdomen." ANS: C The placenta functions by supplying oxygen and excreting carbon dioxide to the maternal bloodstream. The fetal lungs do not function for respiratory gas exchange in utero. The baby does not simply absorb oxygen from a woman's blood system. Blood and gas transport occurs through the placenta. The placenta delivers oxygen-rich blood through the umbilical vein, not artery. PTS: 1 DIF: Cognitive Level: Comprehension/Understanding REF: p. 206 OBJ: Integrated Process: Teaching-Learning MSC: Client Needs: Physiologic Integrity 18. The most basic information a maternity nurse should have concerning conception is a. ova are considered fertile 48 to 72 hours after ovulation. b. sperm remain viable in the woman's reproductive system for an average of 12 to 24 hours. c. conception is achieved when a sperm successfully penetrates the membrane surrounding the ovum. d. implantation in the endometrium occurs 6 to 10 days after conception. ANS: D Implantation occurs 6 to 10 days after conception and is complete after 10 days. Ova are considered fertile for approximately 24 hours after ovulation. Sperm remain viable in the woman's reproductive system for an average of 2 to 3 days. Penetration of the ovum by the sperm is called fertilization. Conception occurs when the zygote, the first cell of the new individual, is formed. PTS: 1 DIF: Cognitive Level: Knowledge/Remembering REF: p. 198 OBJ: Integrated Process: Teaching-Learning MSC: Client Needs: Physiologic Integrity 19. With regard to the structure and function of the placenta, the maternity nurse should be aware that a. as the placenta widens, it gradually thins to allow easier passage of air and nutrients. b. as one of its early functions, the placenta acts as an endocrine gland. c. the placenta is able to keep out most potentially toxic substances, such as cigarette smoke, to which the mother is exposed. d. optimal blood circulation is achieved through the placenta when the woman is lying on her back or standing. ANS: B The placenta produces four hormones necessary to maintain the pregnancy. The placenta widens until week 20 and continues to grow thicker. Toxic substances such as nicotine and carbon monoxide readily cross the placenta into the fetus. Optimal circulation occurs when the woman is lying on her side. PTS: 1 DIF: Cognitive Level: Knowledge/Remembering REF: p. 206 OBJ: Integrated Process: Teaching-Learning MSC: Client Needs: Physiologic Integrity 20. Which statement is accurate about the development of fetal organs and systems? a. The cardiovascular system is the first organ system to function in the developing human. b. Hematopoiesis originating in the yolk sac begins in the liver at 10 weeks. c. The body changes from straight to C-shaped at 8 weeks. d. The gastrointestinal system is mature at 32 weeks. ANS: A The heart is developmentally complete by the end of the embryonic stage. Hematopoiesis begins in the liver during the 6th week. The body becomes C-shaped at 21 weeks. The gastrointestinal system is complete at 36 weeks. PTS: 1 DIF: Cognitive Level: Knowledge/Remembering REF: p. 204 OBJ: Integrated Process: Teaching-Learning MSC: Client Needs: Physiologic Integrity 21. What does the student learn about recent trends in multiple births? a. The rate of twin births has declined. b. The rate of higher order pregnancies has increased. c. Higher order pregnancies are now very rare. d. Twinning is the most common form of multiple pregnancy. ANS: D Twinning is the most common form of multiple pregnancy, and the rate has been increasing, not declining. Higher order births increased for a time but have now decreased, although they are not rare. PTS: 1 DIF: Cognitive Level: Knowledge/Remembering REF: p. 212 OBJ: Integrated Process: Teaching-Learning MSC: Client Needs: Health Promotion and Maintenance MULTIPLE RESPONSE

9. A 17-month-old child is expected to be in what stage according to Piaget? a. Trust b. Preoperations c. Secondary circular reaction d. Sensorimotor period

D The 17-month-old is in the fifth stage of the sensorimotor phase, tertiary circular reactions. Learning in this stage occurs mainly by trial and error. Trust is Erikson's first stage. Preoperation is the stage of cognitive development usually present in older toddlers and preschoolers. Secondary circular reactions last from approximately ages 4 to 8 months. PTS: 1 DIF: Cognitive Level: Knowledge/Remembering REF: p. 109 OBJ: Nursing Process: Assessment MSC: Client Needs: Health Promotion and Maintenance 10. Which statement is correct about toilet training? a. Bladder training is usually accomplished before bowel training. b. Wanting to please the parent helps motivate the child to use the toilet. c. Watching older siblings use the toilet confuses the child. d. Children should be forced to sit on the toilet when first learning. ANS: B Voluntary control of the anal and urethral sphincters is achieved some time after the child is walking. The child must be able to recognize the urge to let go and to hold on. The child must want to please the parent by holding on rather than pleasing himself or herself by letting go. Bowel training precedes bladder training. Watching older siblings provides role modeling and facilitates imitation for the toddler. The child should be introduced to the potty chair or toilet in a nonthreatening manner. PTS: 1 DIF: Cognitive Level: Knowledge/Remembering REF: p. 124 OBJ: Integrated Process: Teaching-Learning MSC: Client Needs: Health Promotion and Maintenance 11. What should the nurse teach a parent who is concerned about preventing sleep problems in a 2-year-old child? a. Have the child always sleep in a quiet, darkened room. b. Provide high-carbohydrate snacks before bedtime. c. Have the child's daytime caretaker eliminate naps. d. Use a nightlight in the child's room. ANS: D The boundaries between reality and fantasy are not well defined for children of this age, so monsters and scary creatures that lurk in the preschooler's imagination become real to the child after the light is turned off. A nightlight may help ease the child's fears. A dark room may be scary to a preschooler. High-carbohydrate snacks increase energy and do not promote relaxation. Most 2-year-olds take one nap each day. Many give up the habit by age 3 years. Insufficient rest during the day can lead to irritability and difficulty sleeping at night. PTS: 1 DIF: Cognitive Level: Application/Applying REF: p. 118 OBJ: Integrated Process: Teaching-Learning MSC: Client Needs: Health Promotion and Maintenance 12. Which statement, made by a nursing student to the father of a 4-year-old child, warrants correction by the nurse? a. "Because the 'baby teeth' are not permanent, they are not important to the child." b. "Encourage your child to practice brushing his teeth after you have thoroughly cleaned them." c. "Your child's 'permanent teeth' will begin to come in around 6 years of age." d. "Fluoride supplements are needed if you do not have fluoridated water." ANS: A Deciduous teeth are important because they maintain spacing and play an important role in the growth and development of the jaws and face and in speech development. Toddlers and preschoolers lack the manual dexterity to remove plaque adequately, so parents must assume this responsibility. But encouraging the child to practice will aid in increasing his or her abilities. Secondary teeth erupt at approximately 6 years of age. If the family does not have fluoridated water, the child will need fluoride treatments. PTS: 1 DIF: Cognitive Level: Application/Applying REF: p. 118 OBJ: Nursing Process: Evaluation MSC: Client Needs: Physiologic Integrity 13. What do parents of preschool children need to understand about discipline? a. Both parents and the child should agree on the method of discipline. b. Discipline should involve some physical restriction. c. The method of discipline should be consistent with that of the child's peers. d. Discipline should include positive reinforcement of desired behaviors. ANS: D Effective discipline strategies should involve a comprehensive approach that includes consideration of the parent-child relationship, reinforcement of desired behaviors, and consequences for negative behaviors. Discipline does not need to be agreed on by the child. Preschoolers feel secure with limits and appropriate, consistent discipline. Both parents should be in agreement so that the discipline is consistently applied. Discipline does not necessarily need to include physical restriction. Discipline does not need to be consistent with that of the child's peers. PTS: 1 DIF: Cognitive Level: Knowledge/Remembering REF: p. 119 OBJ: Integrated Process: Teaching-Learning MSC: Client Needs: Health Promotion and Maintenance 14. In providing anticipatory guidance to parents whose child will soon be entering kindergarten, which is a critical factor to include in this teaching? a. The child needs to be able to sit still. b. The child should be able to count to 25. c. The parent should have interaction and be responsive to the child. d. The child should attend a preschool program first. ANS: C The earliest interactions between parent and infant lay the foundation for school readiness. Probably the most important factor in the development of academic competency is the relationship between parent and child. Sitting still and counting are important skills but are not as vital as parental involvement and responsiveness. Preschool is a helpful experience but not required to enter kindergarten. PTS: 1 DIF: Cognitive Level: Application/Applying REF: p. 127 OBJ: Integrated Process: Teaching-Learning MSC: Client Needs: Health Promotion and Maintenance 15. The parents of a newborn say that their toddler "hates the baby.... He suggested that we put him in the trash can so the trash truck could take him away." The nurse's best action is to a. assess the older child for signs of child abuse. b. refer the family for psychological counseling. c. assist the family to deal with this response. d. encourage the family to give the toddler extra attention. ANS: C The arrival of a new infant represents a crisis for even the best prepared toddler. Toddlers have their entire schedule and routines disrupted because of the new family member. This is a normal response. The nurse should work with parents on ways to involve the toddler in the newborn's care and to help focus attention on the toddler. There is no need to assess for child abuse or to refer the family for counseling. Giving the toddler some extra attention and "special time" will probably help, but this is too narrow in scope to be the best answer. The nurse should help brainstorm several different strategies. PTS: 1 DIF: Cognitive Level: Application/Applying REF: p. 125 OBJ: Nursing Process: Implementation MSC: Client Needs: Health Promotion and Maintenance 16. A parent is very frustrated by the amount of time a toddle says "no" and asks the nurse about effective strategies to manage this negativism. The most appropriate recommendation is to a. punish the child for the behavior. b. provide more attention to the child. c. ask the child to not always say "no." d. reduce the opportunities for a "no" answer. ANS: D The nurse should suggest that the parent phrase questions or directives with restrictive choices rather than yes or no answers. This provides a sense of control for the toddler and reduces the opportunity for negativism. Negativism is not an indication of stubbornness or insolence and should not be punished. The negativism is not a function of attention; the child is testing limits to gain an understanding of the world. The toddler is too young for this approach. PTS: 1 DIF: Cognitive Level: Application/Applying REF: p. 113 OBJ: Integrated Process: Teaching-Learning MSC: Client Needs: Health Promotion and Maintenance 17. A father tells the nurse that his toddler wants the same plate and cup used at every meal, even if they go to a restaurant. The nurse should suggest that the family do which of the following? a. Do not take the child to restaurants until this behavior has stopped. b. Take the child but do not give in to this demand. c. Explain to the child that restaurants have their own dishes. d. Suggest the family take the dishes and use them at the restaurant. ANS: D The child is exhibiting the ritualism that is characteristic at this age. Ritualism is the need to maintain the sameness and reliability. It provides a sense of comfort to the toddler. It will dictate certain principles in feeding practices, including rejecting a favorite food because it is served in a different container. The family can take the dishes and serve the toddler's food and drink with them. Not taking the child out sometimes deprives him or her of a social experience. Not giving in sets the stage for temper tantrums. This child is too young to understand an explanation. PTS: 1 DIF: Cognitive Level: Application/Applying REF: p. 113 OBJ: Integrated Process: Teaching-Learning MSC: Client Needs: Health Promotion and Maintenance 18. Parents tell the nurse that their preschool-age child seems to have an imaginary friend named Bob. Whenever their child is scolded or disciplined, the child in turn scolds Bob. What response by the nurse is most appropriate? a. Ask the child to introduce Bob when the parents are not present. b. Inform the parents that this is normal behavior in this age group. c. Suggest the parents discuss the situation with the provider. d. Refer the child for hearing and vision screening. ANS: B In the early preschool years, boundaries between reality and fantasy blur. Children at the age may develop imaginary friends who can keep them company or take the blame when the child misbehaves. The nurse informs the parents that this is normal behavior. The child likely will not "introduce" Bob to a stranger. The nurse him- or herself needs to provide this anticipatory guidance and not just suggest the parents talk to the provider. There is no reason for sensory screening. PTS: 1 DIF: Cognitive Level: Application/Applying REF: p. 115 OBJ: Integrated Process: Teaching-Learning MSC: Client Needs: Health Promotion and Maintenance 19. The nursing student has planned teaching for a toddler parent group on poison prevention in the home. In reviewing the presentation with the nurse, what information requires the nurse to provide more instruction to the student? a. Lock all medications away securely. b. Place cleaning supplies in a top cabinet. c. Try not to let your child watch you take pills. d. Call Poison Control right away for an exposure. ANS: B Anything potentially poisonous including things like medication, cleaning supplies, or personal care items must be stored in places completely inaccessible to children. Toddlers view climbing as a challenge, so a top cabinet is not inaccessible. The other instructions are appropriate. PTS: 1 DIF: Cognitive Level: Application/Applying REF: p. 109 OBJ: Integrated Process: Teaching-Learning MSC: Client Needs: Health Promotion and Maintenance 20. The nurse is presenting information on burn safety to a toddler and preschool parenting group at a local community center. To avoid the most common cause of fire death in children this age, what information does the nurse provide? a. Practice family fire drills often. b. Cover outlets with plastic covers. c. Turn the water heater temperature to 110° F (43.3° C). d. Keep children out of the kitchen when cooking. ANS: A Children younger than 5 years are at the greatest risk for burn deaths in a house fire. They often panic and hide in closets or under beds rather than escape safely. Parents need to practice fire drills with their children to teach them what to do in the event of a house fire. Covering outlets, turning the water heater down, and keeping children out of the kitchen when cooking are more appropriate for younger children. PTS: 1 DIF: Cognitive Level: Comprehension REF: p. 133 OBJ: Nursing Process: Planning MSC: Client Needs: Health Promotion and Maintenance MULTIPLE RESPONSE

2. An immunocompromised child is in the clinic for immunizations. Which vaccine prescriptions should the nurse question? (Select all that apply.) a. DTaP b. HepA c. IPV d. Varicella e. MMR

D, E Children who are immunologically compromised should not receive live viral vaccines. Varicella is a live vaccine and should not be given except in special circumstances. MMR is a live vaccine and should not be given to immunologically compromised children. DTaP, HepA, and IPV can be given safely. PTS: 1 DIF: Cognitive Level: Knowledge/Remembering REF: p. 77 OBJ: Nursing Process: Planning MSC: Client Needs: Physiologic Integrity

1. The provider orders an infusion of magnesium sulfate to run at 4 g/hour. The pharmacy delivers a bag of 4 g magnesium sulfate in 250 mL. At what rate does the nurse set the pump? ___________________

250 mL/hour 4 g/250 mL = 0.16 mg/mL 4/0.016 - 250 mL/hour PTS: 1 DIF: Cognitive Level: Application/Applying REF: Table 27.3 OBJ: Nursing Process: Implementation MSC: Client Needs: Physiologic Integrity Chapter 28: The Woman with a Postpartum Complication McKinney: Evolve Resources for Maternal-Child Nursing, 5th Edition MULTIPLE CHOICE

1. A newborn weighs 8.7 pounds. How many kilocalories does this breastfed term infant require each day? _________

336 to 395 The breastfed baby needs 85 to 100 kcal/kg/day. This baby weighs 8.7 pounds (3.95 kg) so 85 × 3.95 = 85. 3.95 × 100 = 395. PTS: 1 DIF: Cognitive Level: Application/Applying REF: p. 481 | Box 23.1 OBJ: Nursing Process: Implementation MSC: Client Needs: Physiologic Integrity Chapter 24: The Childbearing Family with Special Needs McKinney: Evolve Resources for Maternal-Child Nursing, 5th Edition MULTIPLE CHOICE

1. A nurse assesses a woman and gathers the following data: Dilation: 4 cm Effacement: 60% Fetal station: 0 Cervical consistency: medium Cervical position: middle Calculate this woman's Bishop score ____________

8 This scoring system evaluates the woman's cervix and how easily labor can be induced. The individual components are: 2-2-2-1-1 = 8. PTS: 1 DIF: Cognitive Level: Application/Applying REF: Table 19.1 OBJ: Nursing Process: Assessment MSC: Client Needs: Health Promotion and Maintenance Chapter 20: Postpartum Adaptations McKinney: Evolve Resources for Maternal-Child Nursing, 5th Edition MULTIPLE CHOICE

1. A nurse assessing a 2-month-old infant notes that the child can briefly hold the head erect when held against the shoulder. What action by the nurse is best? a. Document the findings in the child's chart. b. Notify the provider immediately. c. Conduct a lead-exposure assessment. d. Prepare the parents for genetic testing.

A A 2-month-old infant is able to briefly hold the head erect. If a parent were holding the infant against the parent's shoulder, the infant would be able to lift his or her head briefly. Since this is normal behavior, all that is required of the nurse is documentation. There is no need to notify the provider immediately, conduct a lead-exposure assessment, or prepare the parents for genetic testing. PTS: 1 DIF: Cognitive Level: Application/Applying REF: Table 6.1 OBJ: Nursing Process: Implementation MSC: Client Needs: Health Promotion and Maintenance

3. The nurse is assessing an infant's growth and development. The parents want education on how to stimulate this process. What action suggested by the nurse is inconsistent with knowledge of this topic? a. Have the family draw a three-generation family pedigree. b. Show the family how to coo and babble with their child. c. Encourage the parents to buy interactive toys for the child. d. Involve the child in activities that are outside the home.

A A family pedigree can help show relationships and health care problems but will not stimulate growth and development. Activities that are stimulating for a child include the consistent use of language by the parents, allowing play time with interactive toys (toys that make noises or do something in response to the baby's actions), and exposing the child to new sights and sounds. PTS: 1 DIF: Cognitive Level: Application/Applying REF: p. 66 OBJ: Integrated Process: Teaching-Learning MSC: Client Needs: Health Promotion and Maintenance

8. Which factor is most likely to result in fetal hypoxia during a dysfunctional labor? a. Incomplete uterine relaxation b. Maternal fatigue and exhaustion c. Maternal sedation with narcotics d. Administration of tocolytic drugs

A A high uterine resting tone, with inadequate relaxation between contractions, reduces maternal blood flow to the placenta and decreases fetal oxygen supply. Maternal fatigue or sedation does not decrease uterine blood flow. Tocolytic drugs decrease contractions. This will increase uterine blood flow. PTS: 1 DIF: Cognitive Level: Knowledge/Remembering REF: p. 579 OBJ: Nursing Process: Assessment MSC: Client Needs: Physiologic Integrity

4. A nurse manager plans staffing for the Labor and Delivery unit. How does the available staff influence the selection of either continuous electronic or intermittent auscultation as the fetal monitoring method? a. There must be a 1:1 nurse-to-patient ratio regardless of the method used. b. Staffing patterns do not influence fetal monitoring choices. c. Use of intermittent auscultation requires a lower nurse-to-patient ratio. d. More nurses are needed when electronic fetal monitoring is used.

A A one-to-one ratio is needed during the second stage of labor or if a high-risk condition exists, regardless of the monitoring method used. Intermittent auscultation is more staff-intensive. Less nursing time is needed with electronic monitoring, giving the nurse more time for teaching and supporting the laboring woman. PTS: 1 DIF: Cognitive Level: Comprehension/Understanding REF: p. 336 OBJ: Nursing Process: Planning MSC: Client Needs: Safe and Effective Care Environment

3. Which of these is a secondary sexual characteristic? a. Female breast development b. Production of sperm c. Maturation of ova d. Secretion of gonadotropin-releasing hormone

A A secondary sexual characteristic is one not directly related to reproduction, such as development of the characteristic female body form. Production of sperm, maturation of ova, and secretion of hormones are all directly related to reproduction and not secondary sexual characteristics. PTS: 1 DIF: Cognitive Level: Knowledge/Remembering REF: p. 183 OBJ: Nursing Process: Assessment MSC: Client Needs: Physiologic Integrity

5. In counseling a patient who has decided to relinquish her baby for adoption, the nurse should do which of the following? a. Affirm her decision while acknowledging her maturity in making it. b. Question her about her feelings regarding adoption. c. Tell her she can always change her mind about adoption. d. Ask her if anyone is coercing her into the decision to relinquish her baby.

A A supportive, affirming approach by the nurse will strengthen the patient's resolve and help her to appreciate the significance of the event. The teen needs help in coping with her feelings about this decision. It is important for the nurse to support and affirm the decision the patient has made. This will strengthen the patient's resolve to follow through. Later the patient should be given an opportunity to express her feelings. Telling her she can always change her mind should not be an option after the baby is born and placed with the adoptive parents. It is important that the teenager is treated as an adult, with the assumption that she is capable of making an important decision on her own. PTS: 1 DIF: Cognitive Level: Application/Applying REF: p. 501 OBJ: Nursing Process: Implementation MSC: Client Needs: Psychosocial Integrity

4. Which activity does the nurse recommend to help develop fine motor skills in the school-age child? a. Drawing b. Singing c. Soccer d. Swimming

A Activities such as drawing, building models, and playing a musical instrument increase the school-age child's fine motor skills. Activities such as soccer or swimming help develop gross motor skills. Singing does not increase motor skills. PTS: 1 DIF: Cognitive Level: Comprehension/Understanding REF: p. 133 OBJ: Nursing Process: Evaluation MSC: Client Needs: Health Promotion and Maintenance

3. The nursing student learns that spontaneous termination of a pregnancy is considered to be an abortion if a. the pregnancy is less than 20 weeks. b. the fetus weighs less than 1000 g. c. the products of conception are passed intact. d. no evidence exists of intrauterine infection.

A An abortion is the termination of pregnancy before the age of viability (20 weeks). The weight of a fetus is not considered because some fetuses of an older age may have a low birth weight. A spontaneous abortion may be complete or incomplete. A spontaneous abortion may be caused by many problems, one being intrauterine infection. PTS: 1 DIF: Cognitive Level: Knowledge/Remembering REF: p. 523 OBJ: Integrated Process: Teaching-Learning MSC: Client Needs: Health Promotion and Maintenance

8. The nurse providing newborn stabilization must be aware that the primary side effect of maternal narcotic analgesia in the newborn is a. respiratory depression. b. bradycardia. c. acrocyanosis. d. tachypnea.

A An infant delivered within 5 hours of maternal analgesic administration (timing depends on drug used) is at risk for respiratory depression from the sedative effects of the opioid. Bradycardia, acrocyanosis, and tachypnea are not anticipated side effects of maternal analgesics. PTS: 1 DIF: Cognitive Level: Knowledge/Remembering REF: p. 365 | Table 18.1 OBJ: Nursing Process: Assessment MSC: Client Needs: Physiologic Integrity

8. The nurse should alert the provider when a. the infant is dusky and turns cyanotic when crying. b. acrocyanosis is present at age 1 hour. c. the infant's blood glucose is 45 mg/dL. d. the infant goes into a deep sleep at age 1 hour.

A An infant who is dusky and becomes cyanotic when crying is showing poor adaptation to extrauterine life. The nurse needs to notify the provider. Acrocyanosis is an expected finding during the early neonatal life. A blood glucose of 45 mg/dL is within normal range for a newborn. Infants enter the period of deep sleep when they are about 1 hour old. PTS: 1 DIF: Cognitive Level: Application/Applying REF: p. 452 OBJ: Nursing Process: Implementation MSC: Client Needs: Physiologic Integrity

4. Excessive anxiety in labor heightens the woman's sensitivity to pain by increasing a. muscle tension. b. blood flow to the uterus. c. the pain threshold. d. rest time between contractions.

A Anxiety and fear increase muscle tension, diverting oxygenated blood to the woman's brain and skeletal muscles. Prolonged tension results in general fatigue, increased pain perception, and reduced ability to use coping skills. It can also decrease blood flow to the uterus, the pain threshold, and the amount of rest the mother gets between contractions. PTS: 1 DIF: Cognitive Level: Knowledge REF: p. 356 OBJ: Nursing Process: Assessment MSC: Client Needs: Psychosocial Integrity

7. The priority nursing intervention when admitting a pregnant woman who has experienced a bleeding episode in late pregnancy is to a. assess fetal heart rate (FHR) and maternal vital signs. b. perform a venipuncture for hemoglobin and hematocrit levels. c. place clean disposable pads to collect any drainage. d. monitor uterine contractions.

A Assessment of the FHR and maternal vital signs will assist the nurse in determining the degree of the blood loss and its effect on the mother and fetus. The blood levels can be obtained later. It is important to assess future bleeding and provide for comfort, but the top priority is mother/fetal well-being. Monitoring uterine contractions is important but not the top priority. PTS: 1 DIF: Cognitive Level: Application/Applying REF: p. 530 OBJ: Nursing Process: Implementation MSC: Client Needs: Health Promotion and Maintenance

1. A pregnant woman's mother is worried that her daughter is not "big enough" at 20 weeks. The nurse palpates and measures the fundal height at 20 cm, which is even with the woman's umbilicus. What should the nurse report to the woman and her mother? a. "The body of the uterus is at the belly button level, just where it should be at this time." b. "You're right. We'll inform the practitioner immediately." c. "When you come for next month's appointment, we'll check you again to make sure that the baby is growing." d. "Lightening has occurred, so the fundal height is lower than expected."

A At 20 weeks, the fundus is usually located at the umbilical level. Because the uterus grows in a predictable pattern, obstetric nurses should know that the uterus of 20 weeks of gestation is located at the level of the umbilicus. There is no need to inform the practitioner. The nurse should reassure both mother and patient that the findings are normal. The descent of the fetal head (lightening) occurs in late pregnancy. PTS: 1 DIF: Cognitive Level: Application/Applying REF: p. 214 | p. 229 OBJ: Nursing Process: Implementation MSC: Client Needs: Health Promotion and Maintenance

3. Nurses can prevent evaporative heat loss in the newborn by a. drying the baby after birth and wrapping the baby in a dry blanket. b. keeping the baby out of drafts and away from air conditioners. c. placing the baby away from the outside wall and the windows. d. warming the stethoscope and nurse's hands before touching the baby.

A Because the infant is wet with amniotic fluid and blood, heat loss by evaporation occurs quickly. Heat loss by convection occurs when drafts come from open doors and air currents created by people moving around. If the heat loss is caused by placing the baby near cold surfaces or equipment, it is termed a radiation heat loss. Conduction heat loss occurs when the baby comes in contact with cold objects or surfaces. PTS: 1 DIF: Cognitive Level: Knowledge/Remembering REF: p. 427 OBJ: Nursing Process: Implementation MSC: Client Needs: Physiologic Integrity

5. The nurse advises the mother of a 3-month-old exclusively breastfed infant to a. start giving the infant a vitamin D supplement. b. start using an infant feeder and add rice cereal to the formula. c. start feeding the infant rice cereal with a spoon at the evening feeding. d. continue breastfeeding without any supplements.

A Breast milk does not provide an adequate amount of dietary vitamin D. Infants who are exclusively breastfed need vitamin D supplements to prevent rickets. An infant feeder is an inappropriate method of providing the infant with caloric intake. Solid foods are not recommended for a 3-month-old infant. Rice cereal and other solid foods are contraindicated in a 3-month-old infant. Solid feedings do not typically begin before 4 to 6 months of age. PTS: 1 DIF: Cognitive Level: Comprehension/Understanding REF: p. 93 | Table 6.1 OBJ: Integrated Process: Teaching-Learning MSC: Client Needs: Health Promotion and Maintenance

2. A pregnant woman wants to breastfeed her infant; however, her husband is not convinced that there are any scientific reasons to do so. Which statement by the nurse is true? Bottle-feeding using commercially prepared infant formulas a. increases the risk that the infant will develop allergies. b. helps the infant sleep through the night. c. ensures that the infant is getting iron in a form that is easily absorbed. d. requires that multivitamin supplements be given to the infant.

A Breastfeeding is less likely to cause allergies. Newborns should be fed through the night regardless of feeding method. Iron is better absorbed from breast milk than from formula. Commercial formulas are designed to meet the nutritional needs of the infant and to resemble breast milk. No supplements are necessary. PTS: 1 DIF: Cognitive Level: Comprehension/Understanding REF: p. 482 | Box 23.2 OBJ: Integrated Process: Teaching-Learning MSC: Client Needs: Physiologic Integrity; Basic Care and Comfort

3. The ability to mentally understand that 1 + 3 = 4 and 4 - 1 = 3 occurs in which stage of cognitive development? a. Concrete operations b. Formal operations c. Intuitive thought d. Preoperations

A By 7 to 8 years of age, the child is able to retrace a process (reversibility) and has the skills necessary for solving mathematical problems. This stage is called concrete operations. The formal operations stage deals with abstract reasoning and does not occur until adolescence. Thinking in the intuitive stage is based on immediate perceptions. A child in this stage often solves problems by random guessing. PTS: 1 DIF: Cognitive Level: Knowledge/Remembering REF: p. 133 OBJ: Nursing Process: Assessment MSC: Client Needs: Health Promotion and Maintenance

5. A nurse determines that a child consistently displays predictable behavior and is regular in performing daily habits. Which temperament is the child displaying? a. Easy b. Slow-to-warm-up c. Difficult d. Shy

A Children with an easy temperament are even tempered, predictable, and regular in their habits. They react positively to new stimuli. The slow-to-warm-up temperament type prefers to be inactive and moody. A high activity level and adapting slowly to new stimuli are characteristics of a difficult temperament. Shyness is a personality type and not a characteristic of temperament. PTS: 1 DIF: Cognitive Level: Knowledge/Remembering REF: p. 43 OBJ: Nursing Process: Assessment MSC: Client Needs: Psychosocial Integrity

5. A nursing faculty member explains to students that which patient status is an acceptable indication for serial oxytocin induction of labor? a. Past 42 weeks' gestation b. Multiple fetuses c. Polyhydramnios d. History of long labors

A Continuing a pregnancy past the normal gestational period is likely to be detrimental to fetal health. Multiple fetuses and polyhydramnios overdistend the uterus, making induction of labor high risk. History of rapid labors is a reason for induction of labor because of the possibility that the baby would otherwise be born in uncontrolled circumstances. PTS: 1 DIF: Cognitive Level: Comprehension/Understanding REF: p. 378 OBJ: Integrated Process: Teaching-Learning MSC: Client Needs: Health Promotion and Maintenance

4. Birth for the nulliparous woman with a fetus in a breech presentation is usually by a. cesarean delivery. b. vaginal delivery. c. forceps-assisted delivery. d. vacuum extraction.

A Delivery for the nulliparous woman with a fetus in breech presentation is almost always cesarean section. The greatest fetal risk in the vaginal delivery of breech presentation is that the head (largest part of the fetus) is the last to be delivered. The delivery of the rest of the baby must be quick so that the infant can breathe. PTS: 1 DIF: Cognitive Level: Knowledge/Remembering REF: p. 576 OBJ: Nursing Process: Planning MSC: Client Needs: Physiologic Integrity

1. Which statement best describes development in infants and children? a. Development, a predictable and orderly process, occurs at varying rates within normal limits. b. Development is primarily related to the growth in the number and size of cells. c. Development occurs in a proximodistal direction with fine muscle development occurring first. d. Development is more easily and accurately measured than growth.

A Development, a continuous and orderly process, provides the basis for increases in the child's function and complexity of behavior. The increases in rate of function and complexity can vary normally within limits for each child. An increase in the number and size of cells is a definition for growth. Development proceeds in a proximodistal direction with fine muscle organization occurring as a result of large muscle organization. Development is a more complex process that is affected by many factors; therefore, it is less easily and accurately measured. Growth is a predictable process with standard measurement methods. PTS: 1 DIF: Cognitive Level: Knowledge/Remembering REF: p. 62 OBJ: Nursing Process: Assessment MSC: Client Needs: Health Promotion and Maintenance

3. The nurse providing care for the pregnant woman understands that a factor indicating the need for fetal diagnostic procedures is a. maternal diabetes. b. maternal age older than 30 years. c. previous infant more than 3000 g at birth. d. weight gain of 25 pounds.

A Diabetes is a risk factor in pregnancy because of possible impairment of placental perfusion. Other indications for testing include a maternal age greater than 35 years, having had another infant weighing greater than 4000 g at birth, or excessive weight gain. A weight gain of 25 to 35 pounds is recommended for the woman who begins pregnancy at a normal weight. PTS: 1 DIF: Cognitive Level: Knowledge/Remembering REF: Box 15.1 OBJ: Nursing Process: Assessment MSC: Client Needs: Health Promotion and Maintenance

5. A school nurse is teaching a health class for 5th grade children. The nurse plans to include which statement to best describe growth in the early school-age period? a. Boys grow faster than girls. b. Puberty occurs earlier in boys than in girls. c. Puberty occurs at the same age for all races and ethnicities. d. It is a period of rapid physical growth.

A During the school-age developmental period, boys are approximately 1 inch taller and 2 pounds heavier than girls. Puberty occurs 1 1/2 to 2 years later in boys, which is developmentally later than puberty in girls (not unusual in 9- or 10-year-old girls). Puberty occurs approximately 1 year earlier in African-American girls than in white girls. Physical growth is slow and steady during the school-age years. PTS: 1 DIF: Cognitive Level: Comprehension/Understanding REF: p. 130 OBJ: Integrated Process: Teaching-Learning MSC: Client Needs: Health Promotion and Maintenance

6. Both members of an expectant couple are carriers for phenylketonuria (PKU), an autosomal recessive disorder. In counseling them about the risk to their unborn child, the nurse should tell them that a. the child has a 25% chance of being affected. b. the child will be a carrier, like the parents. c. the child has a 50% chance of being affected. d. one of four of their children will be affected.

A Each child born to a couple who carries an autosomal recessive trait has a 25% chance of having the disorder, because the child receives either a normal or an abnormal gene from each parent. If one member of the couple has the autosomal recessive disorder, all of their children will be carriers. If both parents are carriers, each child has a 50% chance of being a carrier. Each child has the identical odds of being affected. PTS: 1 DIF: Cognitive Level: Comprehension/Understanding REF: Box 10.1 OBJ: Integrated Process: Teaching-Learning MSC: Client Needs: Physiologic Integrity

3. Which mechanism of labor occurs when the largest diameter of the fetal presenting part passes the pelvic inlet? a. Engagement b. Extension c. Internal rotation d. External rotation

A Engagement occurs when the presenting part fully enters the pelvic inlet. Extension occurs when the fetal head meets resistance from the tissues of the pelvic floor and the fetal neck stops under the symphysis. This causes the fetal head to extend. Internal rotation occurs when the fetus enters the pelvic inlet. The rotation allows the longest fetal head diameter to conform to the longest diameter of the maternal pelvis. External rotation occurs after the birth of the head. The head then turns to the side so the shoulders can internally rotate and are positioned with their transverse diameter in the anteroposterior diameter of the pelvic outlet. PTS: 1 DIF: Cognitive Level: Knowledge/Remembering REF: p. 299 OBJ: Nursing Process: Assessment MSC: Client Needs: Health Promotion and Maintenance

5. The theorist who viewed developmental progression as a lifelong series of conflicts that need resolution is a. Erikson. b. Freud. c. Kohlberg. d. Piaget.

A Erik Erikson viewed development as a series of conflicts affected by social and cultural factors. Each conflict must be resolved for the child to progress emotionally, with unsuccessful resolution leaving the child emotionally disabled. Sigmund Freud proposed a psychosexual theory of development. He proposed that certain parts of the body assume psychological significance as foci of sexual energy. The foci shift as the individual moves through the different stages (oral, anal, phallic, latency, and genital) of development. Lawrence Kohlberg described moral development as having three levels (preconventional, conventional, and postconventional). His theory closely parallels Piaget's. PTS: 1 DIF: Cognitive Level: Knowledge/Remembering REF: p. 69 OBJ: Integrated Process: Teaching-Learning MSC: Client Needs: Health Promotion and Maintenance

3. An infant with severe meconium aspiration syndrome (MAS) is not responding to conventional treatment. Which treatment may be necessary for this infant? a. Extracorporeal membrane oxygenation b. Respiratory support with ventilator c. Insertion of laryngoscope and suctioning of the trachea d. Insertion of an endotracheal tube

A Extracorporeal membrane oxygenation is a highly technical method that oxygenates the blood while bypassing the lungs, allowing the infant's lungs to rest and recover. The infant is most likely intubated and on a ventilator already. Laryngoscope insertion and tracheal suctioning are performed after birth before the infant takes the first breath. PTS: 1 DIF: Cognitive Level: Knowledge/Remembering REF: p. 644 OBJ: Nursing Process: Planning MSC: Client Needs: Physiologic Integrity

4. Which response by the nurse to the woman's statement, "I'm afraid to have a cesarean birth," would be the most therapeutic? a. "What concerns you most about a cesarean birth?" b. "Everything will be OK." c. "Don't worry about it. It will be over soon." d. "The doctor will be in later, and you can talk to him."

A Focusing on what the woman is saying and asking for clarification are the most therapeutic responses. Stating that "everything will be ok" or "don't worry about it" belittles the woman's feelings and might be providing false hope. Telling the patient to talk to the doctor does not allow the woman to verbalize her feelings when she desires. PTS: 1 DIF: Cognitive Level: Application/Applying REF: Box 2.2 OBJ: Integrated Process: Communication and Documentation MSC: Client Needs: Psychosocial Integrity

1. Which man is most likely to have abnormal sperm formation resulting in infertility? a. A 20-year-old man with undescended testicles b. An uncircumcised 40-year-old man c. A 35-year-old man with previously treated sexually transmitted disease d. A 16-year-old adolescent who is experiencing nocturnal emissions

A For normal sperm formation, a man's testes must be cooler than his core body temperature. The cremaster muscle attached to each testicle causes the testes to rise closer to the body and become warmer or allow the testes to fall away from the body to become cooler. Circumcision does not prevent fertility. Scar tissue in the fallopian tubes as a result of a sexually transmitted disease can be a cause of infertility in women. Nocturnal emissions of seminal fluid are normal and expected in teenagers. PTS: 1 DIF: Cognitive Level: Knowledge/Remembering REF: p. 193 OBJ: Nursing Process: Assessment MSC: Client Needs: Physiologic Integrity

9. A nurse wants to work to increase the number of immunized children. What action by the nurse would best meet this goal? a. Present a workshop to the local home-schooling parent support group. b. Volunteer for a mass "back to school" immunization clinic. c. Prepare welcome and information packets to college freshmen. d. Work with the health department to bring immunizations to day cares.

A Home-schooled children are often overlooked when it comes to immunizations, because they are not in immunization-friendly systems such as day care, schools, and colleges where immunizations are required. The best way for the nurse to help increase the number of immunized children is to reach out to the home-schooled group. PTS: 1 DIF: Cognitive Level: Application/Applying REF: p. 75 OBJ: Nursing Process: Implementation MSC: Client Needs: Health Promotion and Maintenance 10. The parents of a preschool-aged child are in the clinic and report the child is seen playing with the genitals frequently. What response by the nurse is best? a. Reassure parents this is normal at this age. b. Teach parents about behavior modification. c. Refer parents and child to a psychologist. d. Ask the provider to speak to the parents. ANS: A Preschool children are in the Phallic or Oedipal/Electra Stage of Freud's theory during which the genitals become the focus of curiosity and interest. The nurse should explain that this behavior is normal at this stage. Teaching about disciplinary techniques and referrals to psychotherapy are inappropriate. The nurse may well want the provider to speak to the parents, but the nurse is responsible for patient/parent teaching and should provide education him- or herself. PTS: 1 DIF: Cognitive Level: Application/Applying REF: Table 5.2 OBJ: Integrated Process: Teaching-Learning MSC: Client Needs: Health Promotion and Maintenance 11. A nurse is teaching parents to avoid environmental injury to their 2-year-old child. What information does the nurse include in teaching? a. Avoiding sun exposure, secondhand smoke, and lead b. Living in a middle-class neighborhood c. Avoiding smoking and alcohol intake during pregnancy d. Limiting breastfeeding to avoid toxins being passed through breast milk ANS: A Lead can be present in the home and in toys made overseas. Environmental injury can also be the result of mercury, pesticides (flea and tick collars), radon, and exposure to the sun and secondhand smoke. It is important for the nurse to provide health teaching related to these factors. The nurse is unable to influence socioeconomic status, and the family may not want or be able to move. It is too late for the nurse to instruct the mother regarding smoking or alcohol intake during pregnancy. This should have been included in prenatal teaching. It is unlikely that a 2-year-old child will still be breastfeeding. PTS: 1 DIF: Cognitive Level: Comprehension/Understanding REF: p. 65 OBJ: Integrated Process: Teaching-Learning MSC: Client Needs: Health Promotion and Maintenance 12. Which immunizations should be used with caution in children with an allergy to eggs? a. HepB b. DTaP c. Hib d. MMR ANS: D Live measles vaccine is produced by using chick embryo cell culture, so there is a remote possibility of anaphylactic hypersensitivity in children with egg allergies. Most reactions are actually the result of other components in the vaccine. The other vaccines are safe for children with an egg allergy. PTS: 1 DIF: Cognitive Level: Knowledge/Remembering REF: p. 77 OBJ: Nursing Process: Planning MSC: Client Needs: Physiologic Integrity 13. When counseling parents and children about the importance of increased physical activity, the nurse will emphasize which of the following? a. Anaerobic exercise should comprise a major component of the child's daily exercise. b. All children should be physically active for at least 2 hours per day. c. It is not necessary to participate in physical education classes at school if a student is taking part in other activities. d. Make exercise a fun and habitual activity. ANS: D It is important to make exercise a fun and habitual activity. Encourage parents to investigate their community's different activity programs. This includes recreation centers, parks, and the YMCA. Aerobic exercise should comprise a major component of children's daily exercise; however, physical activity should also include muscle- and bone-strengthening activities. Children and adolescents should be physically active for at least 1 hour daily. Encourage all students to participate fully in any physical education classes. PTS: 1 DIF: Cognitive Level: Comprehension/Understanding REF: p. 80 OBJ: Integrated Process: Teaching-Learning MSC: Client Needs: Health Promotion and Maintenance 14. A student nurse is preparing to administer an Hib vaccination to an infant. What action by the student requires the registered nurse to intervene? a. Gives the vaccine information statement prior to administering the vaccine b. Wipes the dorsal gluteal area with alcohol prior to injection c. Obtains written informed consent before giving the vaccine d. Assesses the family's beliefs and values about vaccinations ANS: B The anterolateral thigh is the preferred site for intramuscular administration of vaccines for infants. When the student prepares the wrong site, the registered nurse should intervene. Federal law requires parents be given vaccine information statements and sign informed consent prior to the nurse's administering vaccinations. The nurse should also assess the family's beliefs and values related to vaccination, which can help dispel myths and guide teaching. PTS: 1 DIF: Cognitive Level: Application/Applying REF: p. 76 OBJ: Nursing Process: Implementation MSC: Client Needs: Safe and Effective Care Environment 15. A nurse is planning to teach about injury prevention to a group of parents. What action by the nurse would best ensure a successful event? a. Have handouts listing community resources. b. Provide free safety gear like bike helmets. c. Group parents by child's developmental stage. d. Present the material in an interactive way. ANS: C When providing anticipatory guidance to prevent injury, the most important thing for the nurse to know and understand is developmental levels of the children involved. Grouping parents by their child's developmental level allows the nurse to know this information about the group and to provide teaching specific to the group. The other options will help but are not as important as tailoring teaching to the specific needs of the children. PTS: 1 DIF: Cognitive Level: Application/Applying REF: p. 80 OBJ: Integrated Process: Teaching-Learning MSC: Client Needs: Health Promotion and Maintenance 16. A nurse is assessing a 1-year-old's food intake over the past 3 days. What information from the parent leads the nurse to provide education on nutrition? a. Child drinks 2 cups of 1% milk each day. b. Child loves to snack on fruit throughout the day. c. Child gets one 4-ounce cup of juice with breakfast. d. Parent allows child to regulate own portions at meals. ANS: A A child this age should not be drinking low-fat milk. Snacking on fruit, 4 ounces of juice, and not forcing the child to eat everything on the plate are appropriate activity and do not require education. PTS: 1 DIF: Cognitive Level: Comprehension/Understanding REF: Box 5.6 OBJ: Integrated Process: Teaching-Learning MSC: Client Needs: Health Promotion and Maintenance MULTIPLE RESPONSE

2. In assessing the knowledge of a pregestational woman with type 1 diabetes concerning changing insulin needs during pregnancy, the nurse recognizes that further teaching is warranted when the patient states a. "I will need to increase my insulin dosage during the first 3 months of pregnancy." b. "Insulin dosage will likely need to be increased during the second and third trimesters." c. "Episodes of hypoglycemia are more likely to occur during the first 3 months." d. "Insulin needs should return to normal within 7 to 10 days after birth if I am bottle feeding."

A Insulin needs are reduced in the first trimester due to increased insulin production by the pancreas and increased peripheral sensitivity to insulin. Also the woman may be experiencing nausea, vomiting, and anorexia that would decrease her insulin needs. The other statements show good understanding of this topic. PTS: 1 DIF: Cognitive Level: Evaluation/Evaluating REF: p. 553 OBJ: Nursing Process: Evaluation MSC: Client Needs: Physiologic Integrity

1. The nurse teaches parents that the formula used to guide time-out as a disciplinary method is a. 1 minute per each year of the child's age. b. to relate the length of the time-out to the severity of the behavior. c. never to use time-out for a child younger than 4 years. d. to follow the time-out with a treat.

A It is important to structure time-out in a time frame that allows the child to understand why he or she has been removed from the environment. The current guideline is 1 minute per age in years. Relating time to a behavior is subjective and is inappropriate when the child is very young. Time-out can be used with the toddler. Negative behavior should not be reinforced with a positive action. PTS: 1 DIF: Cognitive Level: Knowledge/Remembering REF: p. 44 OBJ: Integrated Process: Teaching-Learning MSC: Client Needs: Health Promotion and Maintenance

3. Which technique is least effective for the woman with persistent occiput posterior position? a. Lie supine and relax. b. Sit or kneel, leaning forward with support. c. Rock the pelvis back and forth while on hands and knees. d. Squat.

A Lying supine increases the discomfort of "back labor." A sitting or kneeling position may help the fetal head to rotate to occiput anterior. Rocking the pelvis encourages rotation from occiput posterior to occiput anterior. Squatting aids both rotation and fetal descent. PTS: 1 DIF: Cognitive Level: Knowledge/Remembering REF: p. 575 OBJ: Nursing Process: Implementation MSC: Client Needs: Health Promotion and Maintenance

2. Which analysis of maternal serum may predict chromosomal abnormalities in the fetus? a. Multiple-marker screening b. Lecithin/sphingomyelin (L/S) ratio c. Biophysical profile d. Type and crossmatch of maternal and fetal serum

A Maternal serum can be analyzed for abnormal levels of alpha-fetoprotein, human chorionic gonadotropin, and estriol. This multiple-marker screening may predict chromosomal defects in the fetus. The L/S ratio is used to determine fetal lung maturity. A biophysical profile is used for evaluating fetal status during the antepartum period. Five variables are used, but none is concerned with chromosomal problems. The blood type and crossmatch will not predict chromosomal defects in the fetus. PTS: 1 DIF: Cognitive Level: Knowledge/Remembering REF: p. 278 OBJ: Nursing Process: Assessment MSC: Client Needs: Physiologic Integrity

3. People who have two copies of the same abnormal autosomal dominant gene will usually be a. more severely affected by the disorder than will people with one copy of the gene. b. infertile and unable to transmit the gene. c. carriers of the trait but not affected with the disorder. d. mildly affected with the disorder.

A People who have two copies of an abnormal gene are usually more severely affected by the disorder because they have no normal gene to maintain normal function. Infertility may or may not be caused by chromosomal defects. A carrier of a trait has one recessive gene. Those mildly affected with the disorder will have only one copy of the abnormal gene. PTS: 1 DIF: Cognitive Level: Knowledge/Remembering REF: p. 173 OBJ: Nursing Process: Assessment MSC: Client Needs: Physiologic Integrity

3. Screening at 24 weeks of gestation reveals that a pregnant woman has gestational diabetes mellitus (GDM). In planning her care, the nurse and the woman mutually agree that an expected outcome is to prevent injury to the fetus as a result of GDM. The nurse identifies that the fetus is at greatest risk for a. macrosomia. b. congenital anomalies of the central nervous system. c. preterm birth. d. low birth weight.

A Poor glycemic control later in pregnancy increases the rate of fetal macrosomia. Poor glycemic control during the preconception time frame and into the early weeks of the pregnancy is associated with congenital anomalies. Preterm labor or birth is more likely to occur with severe diabetes and is the greatest risk in women with pregestational diabetes. Increased weight, or macrosomia, is the greatest risk factor for this woman. PTS: 1 DIF: Cognitive Level: Comprehension/Understanding REF: p. 554 OBJ: Nursing Process: Planning MSC: Client Needs: Physiologic Integrity

1. Which principle of teaching should the nurse use to ensure learning in a family situation? a. Motivate the family with praise and positive reinforcement. b. Present complex subject material first, while the family is alert and ready to learn. c. Families should be taught using medical jargon so they will be able to understand the technical language used by physicians. d. Learning is best accomplished using the lecture format.

A Praise and positive reinforcement are particularly important when a family is trying to master a frustrating task, such as breastfeeding. Learning is enhanced when the teaching is structured to present the simple tasks before the complex material. Even though a family may understand English fairly well, they may not understand the medical terminology or slang terms. A lively discussion stimulates more learning than a straight lecture, which tends to inhibit questions. PTS: 1 DIF: Cognitive Level: Knowledge/Remembering REF: p. 25 OBJ: Nursing Process: Planning MSC: Client Needs: Health Promotion and Maintenance

8. A woman tells the nurse at a prenatal interview that she has quit smoking, only has a glass of wine with dinner, and has cut down on coffee to four cups a day. What response by the nurse will be most helpful in promoting lifestyle changes? a. "You have made some great progress toward having a healthy baby. Let's talk about the changes you have made." b. "You need to do a lot better than that. You may still be hurting your baby right now." c. "Here are some pamphlets for you to study. They will help you find more ways to improve." d. "Those few things won't cause any trouble. Good for you."

A Praising her for making positive changes is an effective technique for motivating a patient. She still has to identify the risk factors to optimize the results so a discussion with the nurse can facilitate that. Telling her she has to do better is belittling to the patient. She will be less likely to confide in the nurse. The nurse is not acknowledging the efforts that the woman has already accomplished by simply giving her pamphlets. Those accomplishments need to be praised to motivate the woman to continue. Plus before giving written material, the nurse must assess the woman's literacy level. Alcohol and coffee consumption are still major risk factors and need to be addressed in a positive, nonjudgmental manner. PTS: 1 DIF: Cognitive Level: Application/Applying REF: p. 180 OBJ: Nursing Process: Implementation MSC: Client Needs: Health Promotion and Maintenance

8. The nurse is assessing a preschool aged child during a well-child checkup. This child has gained 2 pounds in 1 year. What action by the nurse is best? a. Ask the parent to provide a 3-day diet diary. b. Assess the child's teeth and gums. c. Plot the weight gain on the growth chart. d. Instruct the parent on today's needed vaccinations.

A Preschool children gain an average of 5 pounds a year. A gain of only 2 pounds is less than half of the expected weight gain and should be investigated. The other actions are part of a well-child checkup but are not related to the lack of weight gain. PTS: 1 DIF: Cognitive Level: Application/Applying REF: p. 126 OBJ: Nursing Process: Assessment MSC: Client Needs: Health Promotion and Maintenance

6. The student nurse learns that in fetal circulation, the pressure is greatest in the a. right atrium. b. left atrium. c. hepatic system. d. pulmonary veins.

A Pressure in fetal circulation is greatest in the right atrium, which allows a right-to-left shunting that aids in bypassing the lungs during intrauterine life. The pressure increases in the left atrium after birth and will close the foramen ovale. The liver does not filter the blood during fetal life until the end. It is functioning by birth. Blood bypasses the pulmonary vein during fetal life. PTS: 1 DIF: Cognitive Level: Comprehension/Understanding REF: p. 426 OBJ: Integrated Process: Teaching-Learning MSC: Client Needs: Physiologic Integrity

2. Which toy is the most developmentally appropriate for an 18- to 24-month-old child? a. A push-and-pull toy b. Nesting blocks c. A bicycle with training wheels d. A computer

A Push-and-pull toys encourage large muscle activity and are appropriate for toddlers. Nesting blocks are more appropriate for a 12- to 15-month-old child. This child is too young for bicycles or computers. PTS: 1 DIF: Cognitive Level: Knowledge/Remembering REF: Box 7.1 OBJ: Nursing Process: Assessment MSC: Client Needs: Health Promotion and Maintenance

2. A nurse is teaching a woman about spinnbarkeit. The student nurse asks why the woman would need this information. What response by the nurse is most appropriate? a. To assist in becoming pregnant or preventing pregnancy b. To determine if she can breastfeed c. To assess risk for genetic defects in the fetus d. To find out if her ova are suitable for fertilization

A Spinnbarkeit refers to the elasticity of cervical mucosa. The woman can assess this to avoid or promote pregnancy. It does not refer to breastfeeding, genetics, or her ova status. PTS: 1 DIF: Cognitive Level: Comprehension/Understanding REF: p. 192 OBJ: Integrated Process: Teaching-Learning MSC: Client Needs: Physiologic Integrity

7. A student nurse reads a patient's chart and sees the term "striae gravidarum," The student asks the registered nurse what this means. What response by the nurse is accurate? a. Stretch marks on the abdomen and breasts b. Dark pigmentation on the woman's face c. Bluish-purple discoloration of the vagina and labia d. Reddened bleeding gums in a pregnant woman

A Stretch marks occurring on the abdomen and/or breasts of a pregnant woman are called striae gravidarum. Dark pigmentation on the face is known as melisma, chloasma, or the mask of pregnancy. The bluish tint to the vagina and labia is known as Chadwick's sign. Reddened and bleeding gums are known as gingivitis in both pregnant and non-pregnant women. PTS: 1 DIF: Cognitive Level: Comprehension/Understanding REF: p. 219 OBJ: Integrated Process: Teaching-Learning MSC: Client Needs: Physiologic Integrity

2. Which woman is most likely to have severe afterbirth pains and request a narcotic analgesic? a. Gravida 5, para 5 b. Woman who is bottle-feeding her first child c. Primipara who delivered a 7-lb boy d. Woman who has started to breastfeed

A The discomfort of after pains is more acute for multiparas because repeated stretching of muscle fibers leads to loss of uterine muscle tone. After pains are particularly severe during breastfeeding, not bottle-feeding. The uterus of a primipara tends to remain contracted. The breastfeeding woman may have increased pain due to engorgement, but the multipara probably will have the most severe afterbirth pains. PTS: 1 DIF: Cognitive Level: Comprehension/Understanding REF: p. 396 OBJ: Nursing Process: Assessment MSC: Client Needs: Physiologic Integrity

4. Which finding 12 hours after birth requires further assessment? a. The fundus is palpable two fingerbreadths above the umbilicus. b. The fundus is palpable at the level of the umbilicus. c. The fundus is palpable one fingerbreadth below the umbilicus. d. The fundus is palpable two fingerbreadths below the umbilicus.

A The fundus rises to the umbilicus after delivery and remains there for about 24 hours. A fundus that is above the umbilicus may indicate uterine atony or urinary retention. The nurse needs to make further assessments. The other findings are within normal limits for the time period. PTS: 1 DIF: Cognitive Level: Knowledge/Remembering REF: p. 395 OBJ: Nursing Process: Assessment MSC: Client Needs: Physiologic Integrity

1. Which part of the mature sperm contains the male chromosomes? a. The head of the sperm b. The middle portion of the sperm c. X-bearing sperm d. The tail of the sperm

A The head of the sperm contains the male chromosomes that will join the chromosomes of the ovum. The middle portion of the sperm supplies energy for the tail's whip-like action. If an X-bearing sperm fertilizes the ovum, the baby will be female. The tail of the sperm helps propel the sperm toward the ovum. PTS: 1 DIF: Cognitive Level: Knowledge/Remembering REF: p. 197 OBJ: Integrated Process: Teaching-Learning MSC: Client Needs: Physiologic Integrity

7. The best time to teach nonpharmacologic pain control methods to an unprepared laboring woman is during which phase? a. Latent phase b. Active phase c. Transition phase d. Second stage

A The latent phase of labor is the best time for intrapartum teaching, because the woman is usually anxious enough to be attentive, yet comfortable enough to understand the teaching. During the active phase, the woman is focused internally and unable to concentrate on teaching. During transition, the woman is focused on keeping control; she is unable to focus on anyone else or learn at this time. During the second stage, the woman is focused on pushing. She normally handles the pain better at this point because she is active in doing something to hasten the delivery. PTS: 1 DIF: Cognitive Level: Knowledge/Remembering REF: p. 357 OBJ: Nursing Process: Planning MSC: Client Needs: Health Promotion and Maintenance

6. Which major neonatal complication is carefully monitored after the birth of the infant of a diabetic mother? a. Hypoglycemia b. Hypercalcemia c. Hypobilirubinemia d. Hypoinsulinemia

A The neonate is at highest risk for hypoglycemia because fetal insulin production is accelerated during pregnancy to metabolize excessive glucose from the mother. At birth, the maternal glucose supply stops and the neonatal insulin exceeds the available glucose, leading to hypoglycemia. Hypocalcemia is associated with preterm birth, birth trauma, and asphyxia, all common problems of the infant of a diabetic mother. Excess erythrocytes are broken down after birth, releasing large amounts of bilirubin into the neonate's circulation, which results in hyperbilirubinemia. Because fetal insulin production is accelerated during pregnancy, the neonate shows hyperinsulinemia. PTS: 1 DIF: Cognitive Level: Knowledge/Remembering REF: p. 552 OBJ: Nursing Process: Assessment MSC: Client Needs: Physiologic Integrity

5. Early postpartum hemorrhage is defined as signs and symptoms of hypovolemia with which of the following descriptions of blood loss? a. Cumulative blood loss >1000 mL in the first 24 hours after the birth process. b. 750 mL in the first 24 hours after vaginal delivery c. Cumulative blood loss >1000 mL in the first 48 hours after the birth process d. 1500 mL in the first 48 hours after cesarean delivery

A The newest definition of early postpoartum hemorrhage is cumulative blood loss >1000 mL with signs of hypovolemia within the first 24 hours after the birth process. Hemorrhage after 24 hours is considered late postpartum hemorrhage. PTS: 1 DIF: Cognitive Level: Knowledge/Remembering REF: p. 599 OBJ: Nursing Process: Assessment MSC: Client Needs: Physiologic Integrity

1. A pregnant woman's biophysical profile score is 8. She asks the nurse to explain the results. The nurse's best response is a. "The test results are within normal limits." b. "Immediate delivery by cesarean birth is needed." c. "Further tests are needed to determine the meaning of this score." d. "We will inform you of your options within the next week."

A The normal biophysical score ranges from 8 to 10 points if the amniotic fluid volume is adequate. A normal score allows conservative treatment of high-risk patients. Delivery can be delayed if fetal well-being is indicated. An immediate delivery is not needed. The results of the biophysical profile are usually available immediately after the procedure is performed. PTS: 1 DIF: Cognitive Level: Comprehension/Understanding REF: p. 283 | Table 15.1 OBJ: Integrated Process: Teaching-Learning MSC: Client Needs: Health Promotion and Maintenance

5. A student nurse is placing a tocotransducer on a woman for electronic fetal monitoring. What action by the student indicates to the registered nurse that the student understands the procedure? a. Places the tocotransducer over the uterine fundus b. Prepares sterile field for fetal scalp electrode placement c. Positions the tocotransducer on the woman's upper arm d. Attaches the tocotransducer to the woman's lower abdomen

A The tocotransducer monitors uterine activity and should be placed over the fundus, where the most intensive uterine contractions occur. No sterile field is needed. The tocotransducer is not placed on the upper arm or lower abdomen. PTS: 1 DIF: Cognitive Level: Evaluation/Evaluating REF: p. 338 OBJ: Nursing Process: Evaluation MSC: Client Needs: Health Promotion and Maintenance

8. A newborn has meconium aspiration at birth. The nurse notes increasing respiratory distress. What action takes priority? a. Obtain an oxygen saturation. b. Notify the provider at once. c. Stimulate the baby to increase respirations. d. Prepare to initiate ECMO.

A This baby has a risk for, and signs of, persistent pulmonary hypertension. The nurse first checks an oxygen saturation then notifies the provider, or alternatively, gets the reading (and other assessments) while another nurse does the notification. This baby most likely has tachypnea so stimulation to increase respirations is not needed. ECMO may or may not be needed depending on whether or not other treatments work. PTS: 1 DIF: Cognitive Level: Analysis/Analyzing REF: p. 645 OBJ: Nursing Process: Implementation MSC: Client Needs: Safe and Effective Care Environment

4. A baby is born with blood type AB. The father is type A, and the mother is type B. The father asks why the baby has a blood type different from those of his parents. The nurse's answer should be based on the knowledge that a. both A and B blood types are dominant. b. the baby has a mutation of the parents' blood types. c. type A is recessive and links more easily with type B. d. types A and B are recessive when linked together.

A Types A and B are equally dominant, and the baby can thus inherit one from each parent. The infant has inherited both blood types from the parents; it is not a mutation. PTS: 1 DIF: Cognitive Level: Comprehension/Understanding REF: p. 172 OBJ: Integrated Process: Teaching-Learning MSC: Client Needs: Physiologic Integrity

5. A nurse is caring for a late preterm infant. What action by the nurse is inconsistent with best practice to prevent cold stress? a. Wean the infant directly to an open crib. b. Check temperature every 3 to 4 hours. c. Encourage kangaroo care. d. Place infant on a radiant warmer.

A Weaning to an open crib takes many steps and is not done directly because of the risk of cold stress. The other actions help prevent cold stress. PTS: 1 DIF: Cognitive Level: Application/Applying REF: pp. 622-623 OBJ: Nursing Process: Implementation MSC: Client Needs: Physiologic Integrity

4. The faculty member teaches students that which of the following are examples of autosomal recessive disorders or traits? (Select all that apply.) a. Blood group O b. Tay-Sachs disease c. Huntington disease d. Neurofibromatosis e. Hemophilia A

A, B Autosomal recessive traits and disorders include blood group O, Tay-Sachs disease, and cystic fibrosis. Huntington disease and neurofibromatosis are examples of autosomal dominant disorders. Hemophilia A is an X-linked disorder. PTS: 1 DIF: Cognitive Level: Knowledge/Remembering REF: Box 10.1 OBJ: Integrated Process: Teaching-Learning MSC: Client Needs: Physiologic Integrity Chapter 11: Reproductive Anatomy and Physiology McKinney: Evolve Resources for Maternal-Child Nursing, 5th Edition MULTIPLE CHOICE

5. A home health care nurse is checking on a new mother with signs of obsessive-compulsive disorder. What assessment findings correlate with this condition? (Select all that apply.) a. Frequently checking on the baby b. Fear of being alone with the baby c. Woman states she feels worthless d. Woman has bought $5,000 worth of toys e. Mother states birth was very traumatic

A, B Postpartum OCD often manifests with women performing obsessive behaviors and voicing fear of being left alone with their baby. Feeling worthless is a sign of depression. A spending spree might be a sign of the manic phase of bipolar disease. Viewing the birth as traumatic may lead to PTSD. PTS: 1 DIF: Cognitive Level: Knowledge/Remembering REF: p. 615 OBJ: Nursing Process: Assessment MSC: Client Needs: Psychosocial Integrity Chapter 29: The High-Risk Newborn: Problems Related to Gestational Age and Development McKinney: Evolve Resources for Maternal-Child Nursing, 5th Edition MULTIPLE CHOICE

4. A woman's chart indicates she has a second-degree laceration. When assessing this patient, the nurse plans to observe which of the following structures? (Select all that apply.) a. Vaginal mucosa b. Perineal skin c. Peritoneal muscle d. Anus e. Rectum

A, B, C A second-degree perineal laceration includes vaginal mucosa, perineal skin, and peritoneal muscle. A third-degree laceration involves the anus, while a fourth-degree laceration includes the rectum. PTS: 1 DIF: Cognitive Level: Knowledge/Remembering REF: Box 20.1 OBJ: Nursing Process: Assessment MSC: Client Needs: Health Promotion and Maintenance Chapter 21: The Normal Newborn: Adaptation and Assessment McKinney: Evolve Resources for Maternal-Child Nursing, 5th Edition MULTIPLE CHOICE

2. A woman who is gravida 3 para 2 enters the intrapartum unit. Which nursing assessments take priority at this time? (Select all that apply.) a. Fetal heart rate b. Maternal vital signs c. The woman's nearness to birth d. Contraction patterns e. Last food and water intake

A, B, C All options describe relevant intrapartum nursing assessments, but the focused assessment has priority. If the maternal and fetal conditions are normal and birth is not imminent, other assessments can be performed in an unhurried manner. PTS: 1 DIF: Cognitive Level: Application/Applying REF: p. 305 OBJ: Nursing Process: Assessment MSC: Client Needs: Safe and Effective Care Environment Chapter 17: Intrapartum Fetal Surveillance McKinney: Evolve Resources for Maternal-Child Nursing, 5th Edition MULTIPLE CHOICE

2. Congenital anomalies can occur with the use of antiepileptic drugs, including (Select all that apply.) a. Craniofacial abnormalities b. Congenital heart disease c. Neural tube defects d. Gastroschisis e. Diaphragmatic hernia

A, B, C Congenital anomalies that can occur with antiepileptic drugs include craniofacial abnormalities, congenital heart disease, and neural tube defects. They are not known to cause gastroschisis or diaphragmatic hernias. PTS: 1 DIF: Cognitive Level: Knowledge/Remembering REF: p. 563 OBJ: Nursing Process: Assessment MSC: Client Needs: Physiologic Integrity

5. The nurse has taught a vegetarian pregnant woman foods that are high in iron. Which menu selections demonstrate good understanding of the material? (Select all that apply.) a. Cooked soybeans b. Canned stewed tomatoes c. Raisin bran cereal d. White bread e. Peaches

A, B, C Cooked soybeans, canned stewed tomatoes, and Raisin Bran cereal are all high in iron. White bread and peaches are not. PTS: 1 DIF: Cognitive Level: Evaluation/Evaluating REF: Table 14.4 OBJ: Nursing Process: Evaluation MSC: Client Needs: Physiologic Integrity Chapter 15: Prenatal Diagnostic Tests McKinney: Evolve Resources for Maternal-Child Nursing, 5th Edition MULTIPLE CHOICE

4. Which of the following factors place the intrapartum woman at risk for complications during labor and delivery? (Select all that apply.) a. Prolonged rupture of membranes b. Chorioamnionitis c. Fever d. History of stillbirth e. Drug use

A, B, C Prolonged rupture of membranes, Chorioamnionitis, and fever are specific to the intrapartum period. Stillbirths and drug use are problems found in the antepartum period. PTS: 1 DIF: Cognitive Level: Knowledge/Remembering REF: Box 17.1 OBJ: Nursing Process: Assessment MSC: Client Needs: Health Promotion and Maintenance Chapter 18: Pain Management for Childbirth McKinney: Evolve Resources for Maternal-Child Nursing, 5th Edition MULTIPLE CHOICE

4. The nurse teaches parents signs that a child might be being bullied or otherwise victimized. What signs does the nurse include in this teaching? (Select all that apply.) a. Spends an inordinate amount of time in the nurse's office b. Belongings frequently go missing or are damaged. c. The child wants to be driven to school. d. School performance improves. e. The child freely talks about his day.

A, B, C Signs that may indicate a child is being bullied are similar to signs of other types of stress and include nonspecific illness or complaints, withdrawal, depression, school refusal, and decreased school performance. Children express fear of going to school or riding the school bus. Very often, children will not talk about what is happening to them. Improving school performance and talking about the day are not indications of bullying. PTS: 1 DIF: Cognitive Level: Comprehension/Understanding REF: p. 147 OBJ: Integrated Process: Teaching-Learning MSC: Client Needs: Psychosocial Integrity

3. A nurse wishes to incorporate the American Nurses Association Code of Ethics for Nurses in daily practice. Which of the following actions best demonstrates successful integration of the code into daily routines? a. Strives to treat all patients equally and with caring kindness b. Calls the provider when the patient's pain is not controlled with prescribed medications c. Reads current literature related to practice area and brings ideas to unit management d. Routinely stays overtime in order to visit and bond with new families e. Decides to "play nicely" and not get involved in disputes about patient care

A, B, C The ANAs Code of Ethics includes statements about practicing with compassion and respect for the inherent dignity, worth, and unique attributes of every person, advocating for the patient, and advancing the profession through research and scholarly inquiry, professional standards development, and the generation of both nursing and health policy. Staying overtime may contribute to burn out and does not advance the Code of Ethics. Nurses are responsible for making decisions and taking action consistent with the obligation to promote health and to provide optimal care; not getting involved in patient care disputes does not uphold this standard. PTS: 1 DIF: Cognitive Level: Analysis/Analyzing REF: Box 2.1 OBJ: Integrated Process: Caring MSC: Client Needs: Safe and Effective Care Environment Chapter 03: The Childbearing and Child-Rearing Family McKinney: Evolve Resources for Maternal-Child Nursing, 5th Edition MULTIPLE CHOICE

3. The student learns about shunts that support fetal circulation. Which of the following are included in this support system? (Select all that apply.) a. Ductus venosus b. Foramen ovale c. Ductus arteriosus d. Foramen magnum e. Ductus deferens

A, B, C The ductus venosus, foramen ovale, and ductus arteriosus are part of fetal circulation. The foramen magnum is located at the base of the skull. The ductus (or vas) deferens is part of the male reproductive system. PTS: 1 DIF: Cognitive Level: Knowledge/Remembering REF: p. 210 OBJ: Integrated Process: Teaching-Learning MSC: Client Needs: Physiologic Integrity

1. A young female patient comes to the school nurse to discuss her irregular periods. In providing education regarding the female reproductive cycle, which phases of the ovarian cycle does the nurse include? (Select all that apply.) a. Follicular b. Ovulatory c. Luteal d. Proliferative e. Secretory

A, B, C The follicular phase is the period during which the ovum matures. It begins on day 1 and ends around day 14. The ovulatory phase occurs near the middle of the cycle, approximately 2 days before ovulation. After ovulation and under the influence of the luteinizing hormone, the luteal phase corresponds with the last 12 days of the menstrual cycle. The proliferative and secretory phases are part of the endometrial cycle. The proliferative phase takes place during the first half of the ovarian cycle when the ovum matures. The secretory phase occurs during the second half of the cycle when the uterus is prepared to accept the fertilized ovum. These are followed by the menstrual phase if fertilization does not occur. PTS: 1 DIF: Cognitive Level: Comprehension/Understanding REF: p. 190 OBJ: Integrated Process: Teaching-Learning MSC: Client Needs: Physiologic Integrity

2. When assessing cultural influences on a pregnant woman's diet, which actions by the nurse are best? (Select all that apply.) a. Learn about traditional foods in that culture. b. Ask the woman how she prepares food. c. Determine if there are specific "pregnancy" foods. d. Assess how traditional the woman is. e. Find out what support she has locally.

A, B, C The nurse should ask about traditional foods in her culture and how she (or others) prepare the food. In some cultures, specific foods are eaten during pregnancy, and the nurse should determine this as well. Assessing how traditional the woman is may or may not be helpful; some women who are not traditional at all return to their cultural practices during pregnancy for a number of reasons. Support systems are not related to food. PTS: 1 DIF: Cognitive Level: Application/Applying REF: p. 263 OBJ: Integrated Process: Culture and Spirituality MSC: Client Needs: Health Promotion and Maintenance

1. Which demonstrates the school-age child's developing logic in the stage of concrete operations? (Select all that apply.) a. Recognizes that 1 lb of feathers is equal to 1 lb of metal b. Recognizes that he can be a son, brother, or nephew at the same time c. Understands the principles of adding, subtracting, and reversibility d. Has thinking that is characterized by egocentrism, animism, and centration e. Often solves problems with random guessing instead of logic

A, B, C The school-age child understands that the properties of objects do not change when their order, form, or appearance does. Conservation occurs in the concrete operations stage. Comprehension of class inclusion occurs as the school-age child's logic increases. The child begins to understand that a person can be in more than one class at the same time. This is characteristic of concrete thinking and logical reasoning. The school-age child is able to understand principles of adding and subtracting, as well as the process of reversibility, which occurs in the stage of concrete operations. Egocentrism, animism, and centration occur in the intuitive thought stage, as does random guessing. PTS: 1 DIF: Cognitive Level: Comprehension/Understanding REF: pp. 133-134 OBJ: Nursing Process: Evaluation MSC: Client Needs: Health Promotion and Maintenance

2. Which are examples of appropriate techniques to wake a sleepy infant for breastfeeding? (Select all that apply.) a. Unwrap the infant. b. Change the diaper. c. Talk to the infant. d. Slap the infant's hands and feet. e. Apply a cold towel to the infant's abdomen.

A, B, C Unwrapping the infant, changing the diaper, or talking to the infant are appropriate ways of waking the sleeping baby. Slapping the hands and feet and applying cold towels are not appropriate methods. PTS: 1 DIF: Cognitive Level: Application/Applying REF: p. 493 OBJ: Nursing Process: Implementation MSC: Client Needs: Health Promotion and Maintenance

1. The nurse assesses pregnant women for exposure to human teratogens, including which of the following? (Select all that apply.) a. Infections b. Radiation c. Maternal conditions d. Drugs e. Chemicals

A, B, C, D Exposure to radiation and a number of infections may result in profound congenital deformities. These include varicella, rubella, syphilis, parvovirus, CMV, and toxoplasmosis. Certain maternal conditions such as diabetes and PKU may also affect organs and other parts of the embryo during this developmental period. Drugs such as antiseizure medication and some antibiotics, as well as chemicals including lead, mercury, tobacco, and alcohol, also may result in structural and functional abnormalities. Coffee is not considered a teratogen. PTS: 1 DIF: Cognitive Level: Knowledge/Remembering REF: Appendix A OBJ: Nursing Process: Assessment MSC: Client Needs: Physiologic Integrity

4. A 2-month-old child has not had any immunizations. Which ones should the nurse prepare to give? (Select all that apply.) a. Hib b. HepB c. MCV d. Varicella e. HPV

A, B, C, D Hib, HepB, MCV, and varicella are all appropriate vaccinations for this child. HPV is for adolescents. PTS: 1 DIF: Cognitive Level: Knowledge/Remembering REF: p. 75 OBJ: Nursing Process: Implementation MSC: Client Needs: Health Promotion and Maintenance Chapter 06: Health Promotion for the Infant McKinney: Evolve Resources for Maternal-Child Nursing, 5th Edition MULTIPLE CHOICE

1. Today's nurse often assumes the role of teacher or educator. Which strategies would be best to use for a nurse working with a new mother? (Select all that apply.) a. Computer-based learning b. Videos c. Printed material d. Group discussion e. Lecture

A, B, C, D To be effective as a teacher, the nurse must tailor teaching to specific needs and characteristics of the patient. Computer-based learning, videos, printed material, and group discussions have all be shown to be effective teaching strategies. Lecture is probably the least effective method as it does not allow for participation. PTS: 1 DIF: Cognitive Level: Knowledge/Remembering REF: p. 24 OBJ: Integrated Process: Teaching-Learning MSC: Client Needs: Health Promotion and Maintenance

3. Traditional ethnocultural beliefs related to the maintenance of health are likely to include which of the following? (Select all that apply.) a. Avoidance of natural events such as a solar eclipse b. Practicing silence, meditation, and prayer c. Protection of the soul by avoiding envy or jealousy d. Understanding that a hex, spell, or the evil eye may cause illness or injury e. Turning to Western medicine first before trying traditional practices

A, B, C, D Traditional ethnocultural beliefs related to health care can include avoiding some natural events; practicing silence, meditation, and prayer; protecting oneself against envy or jealousy on the part of others; and avoiding hexes, spells, and the evil eye. Usually people with these beliefs turn to their traditional practices prior to seeking Western medical care. PTS: 1 DIF: Cognitive Level: Knowledge/Remembering REF: p. 42 OBJ: Integrated Process: Culture and Spirituality MSC: Client Needs: Psychosocial Integrity

5. The school nurse is evaluating the school's athletic programs for safety. What factors should the nurse assess? (Select all that apply.) a. Students get adequate rest periods. b. Equipment is in good condition. c. Practices are appropriate for students. d. Post-game concussion assessment if needed e. Adequate fluids are available at all times.

A, B, C, E A safe athletic program has several features including adequate rest periods, good quality equipment, appropriate practice schedules and regimes, and adequate fluids. Concussion testing if warranted, should occur immediately as the student is withdrawn from the game, and not wait until after the game is over. PTS: 1 DIF: Cognitive Level: Evaluation/Evaluating REF: Safety Alert Box OBJ: Nursing Process: Evaluation MSC: Client Needs: Safe and Effective Care Environment

1. A patient is at 6 weeks' gestation and is having a transvaginal ultrasound. While preparing the patient for this procedure, she expresses concerns over the necessity for this test. The nurse explains that this diagnostic test may be necessary to determine which of the following? (Select all that apply.) a. Multifetal gestation b. Bicornuate uterus c. Presence and location of pregnancy (intrauterine or elsewhere) d. Amniotic fluid volume e. Presence of ovarian cysts

A, B, C, E A transvaginal ultrasound done in the first trimester can detect multifetal gestations, bicornuate uterus, presence and location of pregnancy, and presence of ovarian cysts. Amniotic fluid volume is assessed during the second and third trimesters. PTS: 1 DIF: Cognitive Level: Knowledge/Remembering REF: p. 275 OBJ: Integrated Process: Teaching-Learning MSC: Client Needs: Physiologic Integrity

4. The student nurse learns that breastfed babies are less likely to develop certain health conditions as adults. Which conditions does this include? (Select all that apply.) a. Diabetes b. Asthma c. Obesity d. Kidney failure e. Some cancers

A, B, C, E Breastfed infants are less likely to develop diabetes, asthma, obesity, and some cancers than bottle-fed infants. No difference is seen in the development of kidney failure. PTS: 1 DIF: Cognitive Level: Knowledge/Remembering REF: p. 482 | Box 23.2 OBJ: Integrated Process: Teaching-Learning MSC: Client Needs: Physiologic Integrity COMPLETION

1. Which play patterns does a 3-year-old child typically display? (Select all that apply.) a. Imaginary play b. Parallel play c. Cooperative play d. Structured play e. Associative play

A, B, C, E Children between ages 3 and 5 years enjoy parallel and associative play. Children learn to share and cooperate as they play in small groups. Play is often imitative, dramatic, and creative. Structured play is typical of school-age children. PTS: 1 DIF: Cognitive Level: Knowledge/Remembering REF: p. 122 OBJ: Nursing Process: Assessment MSC: Client Needs: Health Promotion and Maintenance

1. While developing an intrapartum care plan for the patient in early labor, it is important that the nurse recognize that psychosocial factors may influence a woman's experience of pain. These include (Select all that apply.) a. culture. b. anxiety and fear. c. previous experiences with pain. d. intervention of caregivers. e. support systems.

A, B, C, E Culture: a woman's sociocultural roots influence how she perceives, interprets, and responds to pain during childbirth. Some cultures encourage loud and vigorous expressions of pain, whereas others value self-control. The nurse should avoid praising some behaviors (stoicism) while belittling others (noisy expression). Anxiety and fear: extreme anxiety and fear magnify sensitivity to pain and impair a woman's ability to tolerate it. Anxiety and fear increase muscle tension in the pelvic area, which counters the expulsive forces of uterine contractions and pushing efforts. Previous experiences with pain: fear and withdrawal are a natural response to pain during labor. Learning about these normal sensations ahead of time helps a woman suppress her natural reactions of fear regarding the impending birth. If a woman previously had a long and difficult labor, she is likely to be anxious. She may also have learned ways to cope and may use these skills to adapt to the present labor experience. Support systems: an anxious partner is less able to provide help and support to a woman during labor. A woman's family and friends can be an important source of support if they convey realistic and positive information about labor and delivery. Although this may be necessary for the well-being of the woman and her fetus, some interventions add discomfort to the natural pain of labor (i.e., fetal monitor straps). PTS: 1 DIF: Cognitive Level: Knowledge/Remembering REF: p. 356 OBJ: Nursing Process: Planning MSC: Client Needs: Psychosocial Integrity

7. A nurse works for an organization that seeks to limit adolescent violence. In talking with donors, which risk factors for violence may lead to programming decisions? (Select all that apply.) a. Drug or alcohol use/abuse b. Poverty c. Hopelessness about the future d. Narcissism e. Lack of supervision

A, B, C, E Drug and alcohol use/abuse, poverty, hopelessness, and lack of supervision all are risk factors for violence. Narcissism is not. PTS: 1 DIF: Cognitive Level: Comprehension/Understanding REF: Box 9.5 OBJ: Integrated Process: Teaching-Learning MSC: Client Needs: Health Promotion and Maintenance

1. Many teens wait until the second or even third trimester to seek prenatal care. The nurse should understand that the reasons behind this delay include which of the following? (Select all that apply.) a. Lack of realization that they are pregnant b. Uncertainty as to where to go for care c. Continuing to deny the pregnancy d. A desire to gain control over their situation e. Wanting to hide the pregnancy as long as possible

A, B, C, E Not realizing they are pregnant, uncertainty over where to get care, denial, and wanting to hide the pregnancy are all reasons some teens delay prenatal care. Wanting to gain control over the situation does not lead to delaying care. PTS: 1 DIF: Cognitive Level: Knowledge/Remembering REF: p. 502 OBJ: Integrated Process: Teaching-Learning MSC: Client Needs: Psychosocial Integrity

2. What actions can the labor and delivery nurse take to decrease a woman's chance of contracting a puerperal infection? (Select all that apply.) a. Avoid straight catheterizing the woman unless she cannot void. b. Keep vaginal examinations to a minimum. c. Change wet peripads and linens frequently. d. Maintain the woman on bedrest while laboring. e. Use good hand hygiene before and after contact with the woman.

A, B, C, E Risk for infection increases with catheterization, vaginal examinations, exposure to wet linens and pads, and poor hand hygiene. Remaining on bedrest does not reduce the chance for infection. PTS: 1 DIF: Cognitive Level: Comprehension/Understanding REF: p. 609 | Table 28.2 OBJ: Nursing Process: Implementation MSC: Client Needs: Physiologic Integrity

3. A nurse is discussing the signs and symptoms of mastitis with a mother who is breastfeeding. What signs and symptoms should the nurse include in her discussion? (Select all that apply.) a. Breast tenderness b. Warmth in the breast c. An area of redness on the breast often resembling the shape of a pie wedge d. A small white blister on the tip of the nipple e. Fever and flulike symptoms

A, B, C, E Signs and symptoms of mastitis include breast tenderness and warmth, an area of redness on the breast, and fever or flulike symptoms. A small white blister on the tip of the nipple is generally associated with a plugged milk duct. PTS: 1 DIF: Cognitive Level: Knowledge/Remembering REF: p. 491 OBJ: Integrated Process: Teaching-Learning MSC: Client Needs: Physiologic Integrity

3. The nurse assesses a woman's episiotomy or perineal laceration using the acronym REEDA. What factors does this include? (Select all that apply.) a. Redness b. Edema c. Approximation d. Depth e. Discharge

A, B, C, E The acronym REEDA indicates redness, edema, ecchymosis or bruising, discharge, and approximation. Depth is not a consideration with this acronym. PTS: 1 DIF: Cognitive Level: Knowledge/Remembering REF: p. 402 OBJ: Nursing Process: Assessment MSC: Client Needs: Physiologic Integrity

4. The parents of a newborn are considering circumcision. What possible complications does the nurse teach them about? (Select all that apply.) a. Urinary retention b. Adhesions c. Necrosis of the site d. Kidney infection e. Unsatisfactory cosmetic result

A, B, C, E Urinary retention, adhesions, necrosis, and unsatisfactory cosmetic results are possible complications of this procedure. Kidney infection is not. PTS: 1 DIF: Cognitive Level: Knowledge/Remembering REF: p. 475 | Safety Alert Box OBJ: Integrated Process: Teaching-Learning MSC: Client Needs: Physiologic Integrity Chapter 23: Newborn Feeding McKinney: Evolve Resources for Maternal-Child Nursing, 5th Edition MULTIPLE CHOICE

3. While developing a care plan for a school-age child with a visual impairment, the nurse knows that which of the following actions are important in working with this special needs child? (Select all that apply.) a. Obtain a thorough assessment of the child's self-care abilities. b. Orient the child to various sounds in the environment. c. Tell the child's parents to stay continuously with their child during hospitalization. d. Allow the child to handle equipment as procedures are explained. e. Encourage the child to use a dry erase board to write his needs.

A, B, D Conducting a thorough assessment of the child's self-care abilities, orienting the child to various sounds in the environment, and allowing the child to handle equipment are all ways to enhance communication with a visually impaired child. Mandating that the child's parents stay continuously with their child may not be possible and is not usually necessary if the school-age child is at the expected level of growth and development. Encouraging a child to write his needs on a dry erase board would be an appropriate intervention for a child who is hearing impaired, not for a child with a visual deficit. PTS: 1 DIF: Cognitive Level: Knowledge/Remembering REF: p. 59 OBJ: Integrated Process: Communication and Documentation MSC: Client Needs: Psychosocial Integrity

2. A pregnant woman reports that she works in a long-term care setting and is concerned about the impending flu season. She asks about receiving the flu vaccine. Which vaccines could this patient receive? (Select all that apply.) a. Tetanus b. Hepatitis A and B c. Measles, mumps, rubella (MMR) d. Influenza e. Varicella

A, B, D Inactivated vaccines such as those for tetanus, hepatitis A, hepatitis B, and influenza are safe to administer for women who have a risk for contracting or developing the disease. Immunizations with live virus vaccines such as MMR, varicella (chickenpox), or smallpox are contraindicated during pregnancy because of the possible teratogenic effects on the fetus. PTS: 1 DIF: Cognitive Level: Knowledge/Remembering REF: p. 235 OBJ: Nursing Process: Implementation MSC: Client Needs: Health Promotion and Maintenance

1. The student nurse learns that maternal complications of diabetes include which of the following? (Select all that apply.) a. Atherosclerosis b. Retinopathy c. IUFD d. Nephropathy e. Caudal regression syndrome

A, B, D Maternal complications of diabetes include heart disease, retinopathy, nephropathy, and neuropathy. Stillbirth and caudal regression syndrome are fetal complications. PTS: 1 DIF: Cognitive Level: Knowledge/Remembering REF: p. 550 OBJ: Integrated Process: Teaching-Learning MSC: Client Needs: Physiologic Integrity

1. Medications used to manage postpartum hemorrhage include which of the following? (Select all that apply.) a. Oxytocin b. Methergine c. Terbutaline d. Hemabate e. Magnesium sulfate

A, B, D Pitocin, Methergine, and Hemabate are all used to manage PPH. Terbutaline and magnesium sulfate are tocolytics; relaxation of the uterus causes or worsens PPH. PTS: 1 DIF: Cognitive Level: Knowledge/Remembering REF: p. 600 OBJ: Nursing Process: Implementation MSC: Client Needs: Physiologic Integrity

4. The nurse is teaching a pregnant woman to reduce her intake of sodium. What products does the nurse teach the woman to avoid? (Select all that apply.) a. Products with the word "soda" in the ingredients b. Packaged gravy mixes c. Mayonnaise d. Cake mixes e. Fruit juices

A, B, D Products with the word "soda" in the ingredient list are high in sodium, as are packaged gravy mixes and cake mixes. Mayonnaise and fruit juice are lower in sodium and acceptable. PTS: 1 DIF: Cognitive Level: Comprehension/Understanding REF: Box 14.2 OBJ: Integrated Process: Teaching-Learning MSC: Client Needs: Physiologic Integrity

6. A nurse is assessing a child for toilet training readiness during a home visit. Which behaviors by the child are positive signs? (Select all that apply.) a. Removes own clothing b. Walks into bathroom on own c. Has been walking for 6 months d. Will give up toy when asked to e. Scratches as legs periodically

A, B, D Signs of readiness for toilet training include being able to remove own clothing, being willing to let go of a toy when asked, is able to sit, squat, and walk well, has been walking for 1 year, noticing if diaper is wet, pulls on diaper or exhibits other behavior indicating diaper needs to be changed, communicating the need to go to the bathroom or goes there by self and wanting to please parent by staying dry. PTS: 1 DIF: Cognitive Level: Knowledge/Remembering REF: Box 7.4 OBJ: Nursing Process: Assessment MSC: Client Needs: Health Promotion and Maintenance Chapter 08: Health Promotion for the School-Age Child McKinney: Evolve Resources for Maternal-Child Nursing, 5th Edition MULTIPLE CHOICE

1. When counseling the newly pregnant woman regarding the option of using a free-standing birth center for care, the nurse should be aware that this type of care setting includes which advantages? (Select all that apply.) a. Less expensive than acute-care hospitals b. Access to follow-up care for 6 weeks postpartum c. Equipped for obstetric emergencies d. Safe, home-like births in a familiar setting e. Staffing by lay midwives

A, B, D Women who are at low risk and desire a safe, home-like birth are very satisfied with this type of care setting. The new mother may return to the birth center for postpartum follow-up care, breastfeeding assistance, and family planning information for 6 weeks postpartum. Because birth centers do not incorporate advanced technologies into their services, costs are significantly less than those for a hospital setting. The major disadvantage of this care setting is that these facilities are not equipped to handle obstetric emergencies. Should unforeseen difficulties occur, the woman must be transported by ambulance to the nearest hospital. Birth centers are usually staffed by certified nurse-midwives (CNMs); however, in some states lay midwives may provide this service. PTS: 1 DIF: Cognitive Level: Comprehension/Understanding REF: p. 3 OBJ: Integrated Process: Teaching-Learning MSC: Client Needs: Safe and Effective Care Environment

2. A school nurse is working with unlicensed assistive personnel (UAPs). What aspects of delegation should the nurse incorporate into his or her practice in this setting? a. The registered nurse is always responsible for assessment. b. Uncomplicated medication administration can be performed by the UAP. c. The nurse does not need to supervise UAPs in this setting. d. The nurse must work within school district policies when delegating. e. Understanding the complexity of the child's needs is a consideration when delegating.

A, B, D, E Delegation to UAPs is very common in all health care settings, including schools. When delegating to a UAP in the school setting, factors for the nurse to consider include that the RN is always responsible for assessment, supervision is necessary, the complexity of the child's needs must be considered, and policies must be followed. Medication administration by the UAP may be allowed. PTS: 1 DIF: Cognitive Level: Knowledge/Remembering REF: p. 19 OBJ: Nursing Process: Implementation MSC: Client Needs: Safe and Effective Care Environment

1. The nurse who suspects that a patient has early signs of ectopic pregnancy should be observing her for which symptoms? (Select all that apply.) a. Pelvic pain b. Abdominal pain c. Unanticipated heavy bleeding d. Vaginal spotting or light bleeding e. Missed period

A, B, D, E Early signs of ectopic pregnancy include pelvic pain, abdominal pain, spotting or light bleeding, and a woman's report of a "missed period." Heavy bleeding is a later sign and occurs after the tube has ruptured. PTS: 1 DIF: Cognitive Level: Knowledge/Remembering REF: p. 526 OBJ: Nursing Process: Assessment MSC: Client Needs: Physiologic Integrity

1. The nurse who elects to practice in the area of obstetrics learns about the "four Ps." What are the "four Ps"? a. Powers b. Passage c. Position d. Passenger e. Psyche

A, B, D, E Powers: the two powers of labor are uterine contractions and pushing efforts. During the first stage of labor through full cervical dilation, uterine contractions are the primary force moving the fetus through the maternal pelvis. At some point after full dilation, the woman adds her voluntary pushing efforts to propel the fetus through the pelvis. Passage: the passage for birth of the fetus consists of the maternal pelvis and its soft tissues. The bony pelvis is more important to the successful outcome of labor, because bones and joints do not yield as readily to the forces of labor. Passenger: this is the fetus plus the membranes and placenta. Fetal lie, attitude, presentation, and position are all factors that affect the fetus as passenger. Psyche: the psyche is a crucial part of childbirth. Marked anxiety, fear, or fatigue decreases the woman's ability to cope. Position is not one of the "four Ps." PTS: 1 DIF: Cognitive Level: Knowledge/Remembering REF: p. 291 OBJ: Integrated Process: Teaching-Learning MSC: Client Needs: Health Promotion and Maintenance

3. The nurse is assessing parental knowledge of temper tantrums. Which are true statements about temper tantrums? (Select all that apply.) a. Temper tantrums are a common response to anger and frustration in toddlers. b. Temper tantrums often include screaming, kicking, throwing things, and head banging. c. Parents can effectively manage temper tantrums by giving in to the child's demands. d. Children having temper tantrums should be safely isolated and ignored. e. Parents can learn to anticipate times when tantrums are more likely to occur.

A, B, D, E Temper tantrums are a common response to anger and frustration in toddlers. They occur more often when toddlers are tired, hungry, bored, or excessively stimulated. A nap before fatigue or a snack if mealtime is delayed will be helpful in alleviating the times when tantrums are most likely to occur. Tantrums may include screaming, kicking, throwing things, biting themselves, or banging their head. Effective management of tantrums includes safely isolating and ignoring the child. The child should learn that nothing is gained by having a temper tantrum. Giving in to the child's demands only increases the behavior. PTS: 1 DIF: Cognitive Level: Comprehension/Understanding REF: pp. 119-120 OBJ: Nursing Process: Assessment MSC: Client Needs: Health Promotion and Maintenance

1. The nurse preparing to administer the Denver Developmental Screening Test II (DDST-II) should understand that it assesses which functional areas? (Select all that apply.) a. Personal-functional b. Fine motor c. Intelligence d. Language e. Gross motor

A, B, D, E The four functional areas assessed by this tool are personal-functional, fine motor, language, and gross motor. It is not an intelligence test. PTS: 1 DIF: Cognitive Level: Knowledge/Remembering REF: p. 72 OBJ: Nursing Process: Assessment MSC: Client Needs: Health Promotion and Maintenance

3. The nurse explain to the student that which of the following factors increase a woman's risk for thrombosis? (Select all that apply.) a. Use of stirrups for a prolonged period of time b. Prolonged bedrest during or after labor and delivery c. Adherence to a strict vegetarian diet d. Excessive sweating during labor e. Maternal age greater than 30 years of age

A, B, D, E Use of stirrups for a prolonged period of time, bedrest, excessive sweating (leading to dehydration) all increase the risk of thrombosis. Vegetarian diets are not related. Maternal age >35 increases the risk. PTS: 1 DIF: Cognitive Level: Comprehension/Understanding REF: p. 606 | Box 28.2 OBJ: Integrated Process: Teaching-Learning MSC: Client Needs: Physiologic Integrity

2. What are the priority nursing assessments for a woman receiving tocolytic therapy with terbutaline? (Select all that apply.) a. Fetal heart rate b. Maternal heart rate c. Intake and output d. Maternal blood glucose e. Maternal blood pressure f. Odor of amniotic fluid

A, B, E All assessments are important, but those most relevant to the medication include the fetal heart rate and maternal pulse, which tend to increase, and the maternal blood pressure, which tends to exhibit a wide pulse pressure. The other assessments are important but not related to this medication. PTS: 1 DIF: Cognitive Level: Knowledge/Remembering REF: p. 587 | Table 27.3 OBJ: Nursing Process: Assessment MSC: Client Needs: Physiologic Integrity

2. The nurse knows that which of the following chromosomal abnormalities are structural in nature? (Select all that apply.) a. Part of a chromosome is missing. b. The material within a chromosome is rearranged. c. One or more sets of chromosomes are added. d. An entire single chromosome is added. e. Two chromosomes adhere to each other.

A, B, E Characteristics of structural abnormalities include part of a chromosome missing or added, rearrangement of material within chromosomes, two chromosomes that adhered to each other, and fragility of a specific site on the X chromosome. The addition of a single chromosome (trisomy), the deletion of a single chromosome (monosomy), and one or more added sets of chromosomes (polyploidy) are numerical abnormalities. PTS: 1 DIF: Cognitive Level: Comprehension REF: pp. 175-176 OBJ: Integrated Process: Teaching-Learning MSC: Client Needs: Physiologic Integrity

2. The anterior pituitary gland is responsible for producing which hormones? (Select all that apply.) a. Follicle-stimulating hormone (FSH) b. Luteinizing hormone (LH) c. Gonadotropin-releasing hormone (GnRH) d. Oxytocin e. Prolactin

A, B, E FSH and LH are both produced by the anterior pituitary gland. Both of these hormones assist in the stimulation and maturation of the ovarian follicle. Prolactin is also produced by the anterior pituitary and is required for milk production (lactogenesis) to occur. GnRH is produced by the hypothalamus and stimulates the release of FSH and LH. Oxytocin is produced by the posterior pituitary gland and is responsible for stimulating uterine contractions during birth. PTS: 1 DIF: Cognitive Level: Knowledge/Remembering REF: Table 11.1 OBJ: Integrated Process: Teaching-Learning MSC: Client Needs: Physiologic Integrity Chapter 12: Conception and Prenatal Development McKinney: Evolve Resources for Maternal-Child Nursing, 5th Edition MULTIPLE CHOICE

2. The nurse understands that risk factors for hearing loss include (Select all that apply.) a. structural abnormalities of the ear. b. family history of hearing loss. c. alcohol or drug use by the mother during pregnancy. d. gestational diabetes. e. trauma.

A, B, E Structural abnormalities of the ear, a family history of hearing loss, and trauma are risk factors for hearing loss. Other risk factors include persistent otitis media and developmental delay. The American Academy of Pediatrics suggests that infants who demonstrate hearing loss be eligible for early intervention and specialized hearing and language services. Prenatal alcohol or drug intake and gestational diabetes are not risk factors for hearing loss in the infant. PTS: 1 DIF: Cognitive Level: Knowledge/Remembering REF: p. 88 OBJ: Nursing Process: Assessment MSC: Client Needs: Physiologic Integrity

2. The nurse is caring for a child from a Middle Eastern family. Which interventions should the nurse include in planning care? (Select all that apply.) a. Include the father in the decision making. b. Ask for a dietary consult to maintain religious dietary practices. c. Plan for a male nurse to care for a female patient. d. Ask the housekeeping staff to interpret if needed. e. Allow time for prayer.

A, B, E The man is typically the head of the household in Muslim families. So the father should be included in all decision making. Muslims do not eat pork and do not use alcohol. Many are vegetarians. The dietitian should be consulted for dietary preferences. Compulsory prayer is practiced several times throughout the day. The family should not be interrupted during prayer, and treatments should not be scheduled during this time. Muslim women often prefer a female health care provider because of laws of modesty; therefore, the female patient should not be assigned a male nurse. A housekeeping staff member should not be asked to interpret. When interpreters are used, they should be of the same country and religion, if possible, because of regional differences and hostilities. PTS: 1 DIF: Cognitive Level: Knowledge/Remembering REF: Table 3.1 OBJ: Integrated Process: Culture and Spirituality MSC: Client Needs: Psychosocial Integrity

2. A parent calls the emergency department (ED) reporting a front tooth completely knocked out of an adolescent's mouth while playing soccer. What information should the nurse provide? (Select all that apply.) a. Rinse the tooth in lukewarm tap water. b. Place the tooth in saline, milk, or water. c. Scrub the tooth with a disinfectant. d. Come to the ED within 1 hour. e. Prognosis is best if they are seen within 30 minutes.

A, B, E The parent should be advised to rinse the tooth in lukewarm tap water and to place it in saline, milk, or a commercial tooth preservative. Prognosis is best if the tooth can be re-implanted within 30 minutes. The tooth should not be scrubbed. PTS: 1 DIF: Cognitive Level: Application/Applying REF: p. 160 OBJ: Nursing Process: Implementation MSC: Client Needs: Health Promotion and Maintenance

1. The nurse tells the nursing student that late preterm infants are at increased risk for which of the following problems? (Select all that apply.) a. Problems with thermoregulation b. Cardiac distress c. Hyperbilirubinemia d. Sepsis e. Hyperglycemia

A, C, D Problems with thermoregulation, hyperbilirubinemia, and sepsis are common with late preterm infants. They typically have respiratory distress and hypoglycemia. PTS: 1 DIF: Cognitive Level: Comprehension/Understanding REF: p. 619 OBJ: Integrated Process: Teaching-Learning MSC: Client Needs: Health Promotion and Maintenance

2. The nursing faculty teaches that the placenta produces many hormones necessary for normal pregnancy. These include (Select all that apply.) a. human chorionic gonadotropin (hCG). b. insulin. c. estrogen. d. progesterone. e. testosterone.

A, C, D The placenta produces hCG, estrogen, and progesterone. It does not produce insulin or testosterone. PTS: 1 DIF: Cognitive Level: Comprehension/Understanding REF: p. 209 OBJ: Integrated Process: Teaching-Learning MSC: Client Needs: Physiologic Integrity

1. The nurse is caring for a woman who had infibulation performed on her as a child. Which of the following actions by the perinatal nursing staff are appropriate for this patient? (Select all that apply.) a. Obtaining frequent urinalysis collections b. Providing larger equipment for exams c. Astute assessments for pain during procedures d. Monitoring for infections e. Draping the woman maximally

A, C, D, E Female genital mutilation, cutting, or circumcision, also called infibulation, involves removal of some or all of the external female genitalia. The labia majora are often stitched together over the vaginal and urethral opening as part of this practice. The woman is at risk for many issues including urinary tract and other genital infections and pain. Often the woman will not give any verbal or nonverbal signs of pain so the nurse must be astute in assessing for it. Draping the woman should be done as completely as possible. The equipment for exams must be smaller, such as a pediatric speculum. PTS: 1 DIF: Cognitive Level: Application/Applying REF: p. 246 OBJ: Integrated Process: Culture and Spirituality MSC: Client Needs: Psychosocial Integrity

3. The student nurse learns that maternal risks of systemic lupus erythematosus include (Select all that apply.) a. Premature rupture of membranes (PROM) b. Fetal death resulting in stillbirth c. Hypertension d. Preeclampsia e. Renal complications

A, C, D, E PROM, hypertension, preeclampsia, and renal complications are all maternal risks associated with SLE. Stillbirth and prematurity are fetal risks of SLE. PTS: 1 DIF: Cognitive Level: Comprehension/Understanding REF: p. 563 OBJ: Integrated Process: Teaching-Learning MSC: Client Needs: Physiologic Integrity

1. The labor and delivery nurse must be cognizant of the specific conditions appropriate for labor induction, including which of the following? (Select all that apply.) a. Rupture of membranes at or near term b. Convenience of the woman or her physician c. Chorioamnionitis d. Postterm pregnancy e. Fetal death

A, C, D, E Rupture of membranes at or near term, chorioamnionitis, postterm pregnancy, and fetal death are all appropriate indications for induction of labor. Convenience is not. PTS: 1 DIF: Cognitive Level: Knowledge/Remembering REF: pp. 377-378 OBJ: Nursing Process: Planning MSC: Client Needs: Health Promotion and Maintenance

3. Auditory screening of all newborns within the first month of life is recommended by the American Academy of Pediatrics. Reasons for having this testing performed include (Select all that apply.) a. To prevent or reduce developmental delay b. Reassurance for concerned new parents c. Early identification and treatment d. To help the child communicate better e. To achieve one of the Healthy People 2020 goals

A, C, D, E These are all appropriate reasons for auditory screening of the newborn. Infants who do not pass should be rescreened. If they still do not pass the test, they should have a full audiologic and medical evaluation by 3 months of age. If necessary, the infant should be enrolled in early intervention by 6 months of age. New parents are often anxious about this test and the impending results; however, it is not the reason for the screening to be performed. Auditory screening is usually done before hospital discharge. It is important for the nurse to ensure that the infant receive the appropriate testing and that the test is fully explained to the parents. For infants who are referred for further testing and follow-up, it is important for the nurse to provide further explanation and emotional support. PTS: 1 DIF: Cognitive Level: Knowledge/Remembering REF: p. 476 OBJ: Nursing Process: Diagnosis MSC: Client Needs: Health Promotion and Maintenance

3. A woman reports a sudden gush of fluid from her vagina and is worried about premature rupture of her membranes. What other causes of this does the nurse assess for? (Select all that apply.) a. Urinary incontinence b. Leaking of amniotic fluid c. Loss of mucous plug d. An increase in vaginal discharge e. Bloody show

A, C, D, E Urinary incontinence, loss of the mucous plug (leading to bloody show), and increased vaginal discharge can all be mistaken for PROM. Leaking amniotic fluid is an indication of PROM. PTS: 1 DIF: Cognitive Level: Knowledge/Remembering REF: p. 581 OBJ: Nursing Process: Assessment MSC: Client Needs: Physiologic Integrity

2. Which behaviors by the nurse may indicate professional separation or underinvolvement? (Select all that apply.) a. Avoiding the child or his or her family b. Revealing personal information c. Calling in sick d. Spending less time with a particular child e. Asking to trade assignments

A, C, D, E Whether nurses become too emotionally involved or find themselves at the other end of the spectrum—being underinvolved—they lose effectiveness as objective professional resources. These are all indications of the nurse who is underinvolved in a child's care. Revealing personal information to a patient or his or her family is an indication of overinvolvement. PTS: 1 DIF: Cognitive Level: Knowledge/Remembering REF: Box 4.3 OBJ: Integrated Process: Communication and Documentation MSC: Client Needs: Safe and Effective Care Environment

4. A pregnant woman asks the nursing student what to do about her frequent heartburn. What suggestions can the student make that are appropriate? (Select all that apply.) a. Try chewing gum during the day. b. Take Alka-Seltzer or other antacid. c. Drink a small sip of cream before meals. d. Eat small amounts of dry crackers. e. Wear loose-fitting clothing.

A, C, E Chewing gum, a small sip of cream before meals, and wearing loose clothing all can help relieve heartburn. The patient can take antacids recommended by the provider, but Alka-Seltzer has too much sodium. Dry crackers help with morning sickness. PTS: 1 DIF: Cognitive Level: Comprehension/Understanding REF: p. 230 OBJ: Integrated Process: Teaching-Learning MSC: Client Needs: Physiologic Integrity Chapter 14: Nutrition for Childbearing McKinney: Evolve Resources for Maternal-Child Nursing, 5th Edition MULTIPLE CHOICE

4. The nursing faculty explains to students on the labor and delivery unit that late preterm and term births are very different. What distinguishes the late preterm birth from a term birth? (Select all that apply.) a. Late preterm births are between 34 and 36 completed weeks of pregnancy. b. There is no real difference in mortality between the two types of births. c. Late preterm infants may appear to be full term at delivery. d. A late preterm infant who appears full term is classified full term. e. Late preterm infants need careful assessments of gestational age.

A, C, E Late preterm and term deliveries are very different, with late preterm occurring between 34 and 36 completed weeks of gestation. Mortality for late preterm babies is three times higher than for term babies. Because infant appearance can be deceiving, very careful assessment are needed; the late preterm baby can appear as if he or she is full term. PTS: 1 DIF: Cognitive Level: Comprehension/Understanding REF: p. 582 | Safety Alert Box OBJ: Nursing Process: Assessment MSC: Client Needs: Physiologic Integrity COMPLETION

3. During pregnancy there are a number of changes that occur as a direct result of the presence of the fetus. Which of these adaptations meet these criteria? (Select all that apply.) a. Leukorrhea b. Development of a mucous plug c. Quickening d. Ballottement e. Lightening

A, C, E Leukorrhea is a white or slightly gray vaginal discharge that develops in response to cervical stimulation by estrogen and progesterone. Quickening is the first recognition of fetal movements, or "feeling life." Quickening is often described as a flutter and is felt earlier in multiparous women than in primiparas. Lightening occurs when the fetus begins to descent into the pelvis. This occurs 2 weeks before labor in the nullipara and at the start of labor in the multipara. Mucus fills the cervical canal, creating a plug that acts as a barrier against bacterial invasion during pregnancy. Passive movement of the unengaged fetus is referred to as ballottement. PTS: 1 DIF: Cognitive Level: Comprehension/Understanding REF: p. 222 OBJ: Nursing Process: Assessment MSC: Client Needs: Physiologic Integrity

2. Many women given up smoking during pregnancy to protect the health of the fetus. The majority of women resumed smoking within the first 6 months postpartum. Factors that increase the likelihood of relapse include (Select all that apply.) a. living with a smoker. b. returning to work. c. weight concerns. d. successful breastfeeding. e. failure to breastfeed.

A, C, E Living with a smoker, weight concerns, and failure to breastfeed are all associated with a higher relapse rate after smoking cessation during pregnancy. PTS: 1 DIF: Cognitive Level: Knowledge/Remembering REF: p. 415 OBJ: Nursing Process: Assessment MSC: Client Needs: Health Promotion and Maintenance

2. The nurse plans a teaching session with a toddler's parents on car safety. Which will the nurse teach? (Select all that apply.) a. Secure in a rear-facing, upright car safety seat. b. Place the car safety seat in the rear seat, behind the driver's seat. c. Harness safety straps should fit snugly. d. Place the car safety seat in the front passenger seat equipped with an airbag. e. After the age of 2 years, toddlers can be placed in a forward-facing car seat.

A, C, E Toddlers should be secured in a rear-facing, upright, approved car safety seat. Harness straps should be adjusted to provide a snug fit. After age 2, the child can sit in a forward-facing car seat. The car safety seat should be placed in the middle of the rear seat. Children younger than 13 years should not ride in a front passenger seat that is equipped with an airbag. PTS: 1 DIF: Cognitive Level: Comprehension/Understanding REF: p. 115 OBJ: Nursing Process: Planning MSC: Client Needs: Health Promotion and Maintenance

2. The nurse should model and teach practices used to prevent sudden infant death syndrome. Which of the following do these include? (Select all that apply.) a. Fully supine position for all sleep b. Side-sleeping position as an acceptable alternative c. "Tummy time" for play d. Placing the infant's crib in the parents' room e. A soft mattress

A, D The back to sleep position is now recommended as the only position for every sleep period. Ideally the infant's crib should be placed in the parents' room. Side sleeping is not an acceptable alternative because of the possibility the infant will roll to the prone position. Tummy time helps develop muscles and reduces plagiocephaly. Mattresses in cribs should be firm. PTS: 1 DIF: Cognitive Level: Knowledge/Remembering REF: p. 468 OBJ: Nursing Process: Implementation MSC: Client Needs: Safe and Effective Care Environment

4. A preschool-age child is being admitted for some diagnostic tests and possible surgery. The nurse planning care should use which phrases when explaining procedures to the child? (Select all that apply.) a. Fluids will be given through tubing connected to a tiny tube inserted into your arm. b. After surgery we will be doing dressing changes. c. You will get a shot before surgery. d. The doctor will give you medicine that will help you go into a deep sleep. e. We will take you to surgery on a bed on wheels.

A, D, E A preschool child needs simple concrete explanations that cannot be misinterpreted. An IV should be explained as fluids going into a tube connected to a small tube in your hand; anesthesia can be explained as a medicine that will help you go into a deep sleep (put to sleep should be avoided); and a stretcher can be described as riding on a bed with wheels. The term "dressing changes" is ambiguous and will not be understood by a preschooler. The term "get a shot" should not be used. A preschooler or young child is likely to misinterpret this information. PTS: 1 DIF: Cognitive Level: Comprehension/Understanding REF: Table 4.4 OBJ: Integrated Process: Communication and Documentation MSC: Client Needs: Psychosocial Integrity Chapter 05: Health Promotion for the Developing Child McKinney: Evolve Resources for Maternal-Child Nursing, 5th Edition MULTIPLE CHOICE

1. Which factors contribute to early adolescents engaging in risk-taking behaviors? (Select all that apply.) a. Peer pressure b. A desire to master their environment c. Trying to separate from their parents d. A belief that they are invulnerable e. Impulsivity

A, D, E Peer pressure (including impressing peers) is a factor contributing to adolescent injuries. During early to middle adolescence, children feel that they are exempt from the consequences of risk-taking behaviors; they believe that negative consequences only happen to others. Feelings of invulnerability ("It can't happen to me") are evident in adolescence. Impulsivity places adolescents in unsafe situations. Mastering the environment is the task of young school-age children. Emancipation is a major issue for the older adolescent. The process is accomplished as the teenager gains an education or vocational training. PTS: 1 DIF: Cognitive Level: Knowledge/Remembering REF: p. 154 OBJ: Nursing Process: Assessment MSC: Client Needs: Health Promotion and Maintenance

3. The nurse is teaching a community group about preventing sudden infant death syndrome (SIDS). What information does the nurse provide? (Select all that apply.) a. Placing the baby supine to sleep b. Covering the baby warmly with blankets c. Have the baby sleep upright in the infant carrier d. Provide "tummy time" while awake e. Do not allow smoking in the house

A, D, E Recommendations to prevent SIDS include placing the baby supine in a crib with a well-fitting bottom sheet without covers or toys, providing tummy time during play, and avoiding exposure to environmental hazards such as smoke. The child should not be put to sleep in an infant carrier or covered warmly with blankets. PTS: 1 DIF: Cognitive Level: Knowledge/Remembering REF: p. 96 OBJ: Integrated Process: Teaching-Learning MSC: Client Needs: Safe and Effective Care Environment Chapter 07: Health Promotion During Early Childhood McKinney: Evolve Resources for Maternal-Child Nursing, 5th Edition MULTIPLE CHOICE

5. The nurse's initial action when caring for an infant with a slightly decreased temperature is to a. notify the physician immediately. b. place a cap on the infant's head. c. Keep the infant in the nursery for the next 4 hours. d. Assess for other signs of inaccurate gestational age.

B A cap will prevent further heat loss from the head, and having the mother place the infant skin-to-skin should increase the infant's temperature. Nursing actions are needed first to correct the problem. If the problem persists after interventions, notification may then be necessary. A slightly decreased temperature can be treated in the mother's room. This would be an excellent time for parent teaching on prevention of cold stress. There is no need for another gestational age assessment. PTS: 1 DIF: Cognitive Level: Application/Applying REF: p. 464 OBJ: Nursing Process: Implementation MSC: Client Needs: Physiologic Integrity

8. A postpartum patient is at increased risk for postpartum hemorrhage if she delivers a(n) a. 5-lb, 2-oz infant with outlet forceps. b. 6.5-lb infant after a 2-hour labor. c. 7-lb infant after an 8-hour labor. d. 8-lb infant after a 12-hour labor.

B A rapid (precipitous) labor and delivery may cause exhaustion of the uterine muscle and prevent contraction. The use of forceps may cause lacerations that could lead to bleeding, but that is not as common as hemorrhage after a precipitous labor when they are used only in the outlet. Eight-hour and 12-hour labors are normal in length. PTS: 1 DIF: Cognitive Level: Comprehension/Understanding REF: p. 600 | Box 28.1 OBJ: Nursing Process: Assessment MSC: Client Needs: Physiologic Integrity

9. Many adolescents decide to follow a vegetarian diet during their teen years. The nurse can advise the adolescent and his or her parents that a. this diet will not meet the nutritional requirements of growing teens. b. a vegetarian diet can be healthy for this population. c. an adolescent on a vegetarian diet is less likely to eat high-fat foods. d. a vegetarian diet requires little extra meal planning.

B A vegetarian diet is healthy for this population, and the low-fat aspect of the diet can prevent future cardiovascular problems. Several dietary organizations have suggested that a vegetarian diet, if correctly followed, is healthy for this population. As with any adolescent, nurses need to advise teens who follow a vegetarian eating plan to avoid low-nutrient, high-fat foods. The nurse can assist with planning food choices that will provide sufficient calories and necessary nutrients. The focus is on obtaining enough calories for growth and energy from a variety of fruits and vegetables, whole grains, nuts, and soymilk. PTS: 1 DIF: Cognitive Level: Application/Applying REF: p. 160 OBJ: Integrated Process: Teaching-Learning MSC: Client Needs: Physiologic Integrity 10. Which is assessed with Tanner staging? a. Hormone levels b. Secondary sex characteristics c. Growth hormone secretion d. Hyperthyroidism ANS: B Tanner stages are used to assess staging of secondary sex characteristics at puberty. Hormone levels are assessed by their concentration in the blood. Growth hormone secretion tests are not associated with Tanner staging. Tanner stages are not associated with hyperthyroidism. PTS: 1 DIF: Cognitive Level: Knowledge/Remembering REF: p. 151 OBJ: Nursing Process: Assessment MSC: Client Needs: Health Promotion and Maintenance 11. A nurse is teaching adolescent boys about pubertal changes. The first sign of pubertal change seen with boys is a. testicular enlargement. b. facial hair. c. scrotal enlargement. d. voice deepens. ANS: A The first sign of pubertal changes in boys is testicular enlargement in response to testosterone secretion, which usually occurs in Tanner stage 2. Slight pubic hair is present and the smooth skin texture of the scrotum is somewhat altered. During Tanner stages 4 and 5, facial hair appears at the corners of the upper lip and chin. As testosterone secretion increases, the penis, testes, and scrotum enlarge. During Tanner stages 4 and 5, rising levels of testosterone cause the voice to deepen. PTS: 1 DIF: Cognitive Level: Comprehension/Understanding REF: p. 150 OBJ: Integrated Process: Teaching-Learning MSC: Client Needs: Health Promotion and Maintenance 12. A student nurse learns that according to Erikson, the psychosocial task of adolescence is to develop a. intimacy. b. identity. c. initiative. d. independence. ANS: B Traditional psychosocial theory holds that the developmental crises of adolescence lead to the formation of a sense of identity. Intimacy is the developmental stage for early adulthood. Initiative is the developmental stage for early childhood. Independence is not one of Erikson's developmental stages. PTS: 1 DIF: Cognitive Level: Knowledge/Remembering REF: p. 155 OBJ: Integrated Process: Teaching-Learning MSC: Client Needs: Health Promotion and Maintenance 13. A student nurse learns that according to Piaget, the adolescent is in the fourth stage of cognitive development, or period of what? a. Formal operations b. Concrete operations c. Conventional thought d. Postconventional thought ANS: A Cognitive thinking culminates with capacity for abstract thinking. This stage, the period of formal operations, is Piaget's fourth and last stage. Concrete operations usually develops between ages 7 and 11 years. Conventional and postconventional thought refer to Kohlberg's stages of moral development. PTS: 1 DIF: Cognitive Level: Knowledge/Remembering REF: p. 154 OBJ: Integrated Process: Teaching-Learning MSC: Client Needs: Health Promotion and Maintenance 14. A student nurse asks the faculty why peer relationships become more important during adolescence. Which of the following is the nurse's best response? a. Adolescents dislike their parents. b. Adolescents no longer need parental control. c. They provide adolescents with a feeling of belonging. d. They promote a sense of individuality in adolescents. ANS: C The peer group serves as a strong support to teenagers, providing them with a sense of belonging (versus individuality) and a sense of strength and power. During adolescence, the parent/child relationship changes from one of protection-dependency to one of mutual affection and quality. This does not mean teens do not like their parents who continue to play an important role in their personal and health-related decisions. PTS: 1 DIF: Cognitive Level: Comprehension/Understanding REF: p. 155 OBJ: Integrated Process: Teaching-Learning MSC: Client Needs: Health Promotion and Maintenance 15. A 14-year-old male seems to be always eating, although his weight is appropriate for his height. The parents ask the nurse if they should be concerned about this behavior. Which response by the nurse is best? a. This is normal because of increase in body mass during this time. b. This is abnormal and suggestive of possible future obesity. c. His caloric intake would have to be excessive for him to gain weight. d. He is substituting food for unfilled needs. ANS: A In adolescence, nutritional needs are closely related to the increase in body mass. The peak requirements occur in the years of maximal growth. The caloric and protein requirements are higher than at almost any other time of life. It is not suggestive of possible future obesity or unmet psychosocial needs. It may be true that the teen would need to eat an enormous amount of food in order to gain weight, but that does not give the parents the information they are requesting. PTS: 1 DIF: Cognitive Level: Comprehension/Understanding REF: p. 159 OBJ: Integrated Process: Teaching-Learning MSC: Client Needs: Health Promotion and Maintenance 16. What does the nurse learn that predisposes the adolescent to feel an increased need for sleep? a. An inadequate diet b. Rapid physical growth c. Decreased activity d. Typical lack of ambition ANS: B Rapid physical growth, the tendency toward overexertion, and the overall increased activity of this age contributes to fatigue in this population. It is not due to dietary factors, decreased activity, or lack of ambition. PTS: 1 DIF: Cognitive Level: Knowledge/Remembering REF: p. 161 OBJ: Integrated Process: Teaching-Learning MSC: Client Needs: Health Promotion and Maintenance 17. A nurse wants to volunteer for an organization that helps prevent death in older adolescents. What action by the nurse would have the most impact? a. Volunteer for a suicide hotline. b. Teach firework safety classes. c. Work on a poison control hot line. d. Educate teens on gun safety. ANS: A Of the four causes of death listed, suicide ranks highest, being the second most common cause of death in the 15 to 24 age group. The nurse would make the biggest impact volunteering for a suicide hotline. PTS: 1 DIF: Cognitive Level: Application/Applying REF: p. 163 OBJ: Nursing Process: Implementation MSC: Client Needs: Safe and Effective Care Environment 18. In girls, the initial indication of puberty is a. menarche. b. growth spurt. c. growth of pubic hair. d. breast development. ANS: D In most girls, the initial indication of puberty is the appearance of breast buds, an event known as thelarche. The usual sequence of secondary sexual characteristic development in girls is breast changes, rapid increase in height and weight, growth of pubic hair, appearance of axillary hair, menstruation, and abrupt deceleration of linear growth. PTS: 1 DIF: Cognitive Level: Knowledge/Remembering REF: p. 152 OBJ: Nursing Process: Assessment MSC: Client Needs: Health Promotion and Maintenance MULTIPLE RESPONSE

8. The nurse is assisting a normally active pregnant woman in developing a meal plan. Before she got pregnant, she ate 1800 calories a day. How many calories does she need now? a. 2000 b. 2140 c. 2342 d. 2400

B A woman should increase her daily caloric intake by 340 calories during the second trimester, so this woman needs 2140 daily calories. PTS: 1 DIF: Cognitive Level: Application/Applying REF: Table 14.2 OBJ: Nursing Process: Assessment MSC: Client Needs: Physiologic Integrity

4. A multiparous woman is admitted to the postpartum unit after a rapid labor and birth of a 4000-g infant. Her fundus is boggy, lochia is heavy, and vital signs are unchanged. The nurse has the woman void and massages her fundus, but her fundus remains difficult to find, and the rubra lochia remains heavy. What action should the nurse take next? a. Continue to massage the fundus. b. Notify the provider. c. Recheck vital signs. d. Insert an indwelling urinary catheter.

B After taking these corrective actions, the nurse should contact the provider and anticipate collaborative care measures. Another nurse can assess vital signs. Since the woman just voided, an indwelling catheter is not needed. PTS: 1 DIF: Cognitive Level: Application/Applying REF: p. 601 OBJ: Nursing Process: Implementation MSC: Client Needs: Physiologic Integrity

7. Which child is most likely to be frightened by hospitalization? a. A 4-month-old infant admitted with a diagnosis of bronchiolitis b. A 2-year-old toddler admitted for cystic fibrosis c. A 9-year-old child hospitalized with a fractured femur d. A 15-year-old adolescent admitted for abdominal pain

B All children can be frightened by hospitalization. However, toddlers are most likely to be frightened by hospitalization because their thought processes are egocentric, magical, and illogical. They feel very threatened by unfamiliar people and strange environments. Young infants are not as likely to be as frightened as toddlers by hospitalization because they are not as aware of the environment. The 9-year-old child's cognitive ability is sufficient for the child to understand the reason for hospitalization. The 15-year-old adolescent has the cognitive ability to interpret the reason for hospitalization. PTS: 1 DIF: Cognitive Level: Comprehension/Understanding REF: Table 5.2 OBJ: Nursing Process: Assessment MSC: Client Needs: Health Promotion and Maintenance

4. Which statement is the most appropriate advice to give parents of a 16-year-old who is rebellious? a. "You need to be stricter so that your teen stops trying to test the limits." b. "Try to collaborate to set limits that are perceived as being reasonable." c. "Increasing your teen's involvement with her peers will improve her behavior." d. "Allow your teenager to choose the type of discipline that is used in your home."

B Allowing teenagers to choose between realistic options and offering consistent and structured discipline typically enhances cooperation and decreases rebelliousness. Structure helps adolescents to feel more secure and assists them in the decision-making process. Setting stricter limits typically does not decrease rebelliousness or decrease testing of parental limits. Increasing peer involvement does not typically improve behavior. Allowing teenagers to choose the method of discipline is not realistic and typically does not reduce rebelliousness. PTS: 1 DIF: Cognitive Level: Application/Applying REF: p. 157 OBJ: Integrated Process: Teaching-Learning MSC: Client Needs: Health Promotion and Maintenance

7. An indication for an episiotomy would be a woman who a. has a routine vaginal birth. b. has fetal shoulder dystocia. c. is delivering a preterm infant. d. has a history of rapid deliveries.

B An episiotomy is indicated in the situation where the shoulder of the fetus becomes lodged under the mother's symphysis pubis during birth. The other situations are not indications for an episiotomy. PTS: 1 DIF: Cognitive Level: Knowledge/Remembering REF: p. 387 OBJ: Nursing Process: Assessment MSC: Client Needs: Health Promotion and Maintenance

9. What instructions should be included in the discharge teaching plan to assist the patient in recognizing early signs of complications? a. Palpate the fundus daily to ensure that it is soft. b. Notify the physician of a return to bright red bleeding. c. Report any decrease in the amount of brownish red lochia. d. The passage of clots as large as an orange can be expected.

B An increase in lochia or a return to bright red bleeding after the lochia has become pink indicates a complication. The fundus should stay firm. Large clots after discharge are a sign of complications and should be reported. PTS: 1 DIF: Cognitive Level: Application/Applying REF: p. 602 OBJ: Nursing Process: Implementation MSC: Client Needs: Health Promotion and Maintenance 10. Which woman is at greatest risk for early postpartum hemorrhage? a. A primiparous woman being prepared for an emergency cesarean birth for fetal distress b. A woman with severe preeclampsia on magnesium sulfate whose labor is being induced c. A multiparous woman with an 8-hour labor d. A primigravida in spontaneous labor with preterm twins ANS: B Magnesium sulfate administration during labor poses a risk for PPH. Magnesium acts as a smooth muscle relaxant, thereby contributing to uterine relaxation and atony. The other situations do not post risk factors or causes of early PPH. PTS: 1 DIF: Cognitive Level: Knowledge/Remembering REF: p. 600 | Box 28.1 OBJ: Nursing Process: Planning MSC: Client Needs: Physiologic Integrity 11. When caring for a postpartum woman experiencing hypovolemic shock, the nurse recognizes that the most objective and least invasive assessment of adequate organ perfusion and oxygenation is a. absence of cyanosis in the buccal mucosa. b. cool, dry skin. c. diminished restlessness. d. decreased urinary output. ANS: D Hemorrhage may result in hypovolemic shock. Shock is an emergency situation in which the perfusion of body organs may become severely compromised, and death may occur. The presence of adequate urinary output indicates adequate tissue perfusion. The assessment of the buccal mucosa for cyanosis can be subjective in nature. The presence of cool, pale, clammy skin is an indicative finding associated with hypovolemic shock. Restlessness indicates decreased cerebral perfusion. PTS: 1 DIF: Cognitive Level: Analysis/Analyzing REF: p. 603 OBJ: Nursing Process: Assessment MSC: Client Needs: Physiologic Integrity 12. The nurse should expect medical intervention for subinvolution to include a. oral methylergonovine maleate (Methergine) for 48 hours. b. oxytocin intravenous infusion for 8 hours. c. oral fluids to 3000 mL/day. d. intravenous fluid and blood replacement. ANS: A Methergine provides long-sustained contraction of the uterus and is the usual treatment. Oxytocin and oral fluids are not used for this condition. There is no indication that blood loss has occurred in this situation; if it does blood replacement may be necessary. PTS: 1 DIF: Cognitive Level: Comprehension/Understanding REF: p. 605 OBJ: Nursing Process: Planning MSC: Client Needs: Physiologic Integrity 13. If nonsurgical treatment for late postpartum hemorrhage is ineffective, which surgical procedure is appropriate to correct the cause of this condition? a. Hysterectomy b. Laparoscopy c. Laparotomy d. D&C ANS: D D&C allows examination of the uterine contents and removal of any retained placental fragments or blood clots. Hysterectomy, laparoscopy, and laparotomy are not indicated. PTS: 1 DIF: Cognitive Level: Knowledge/Remembering REF: pp. 602-603 OBJ: Nursing Process: Planning MSC: Client Needs: Physiologic Integrity 14. The mother-baby nurse must be able to recognize what sign of thrombophlebitis? a. Visible varicose veins b. Positive Homans sign c. Local tenderness, heat, and swelling d. Pedal edema in the affected leg ANS: C Tenderness, heat, and swelling are classic signs of thrombophlebitis that appear at the site of the inflammation. Varicose veins may predispose the woman to thrombophlebitis but are not a sign. A positive Homans sign may be caused by a strained muscle or contusion. Edema may be caused by other factors, and the edema with thrombophlebitis may be more extensive. Edema may be more involved than pedal. PTS: 1 DIF: Cognitive Level: Knowledge/Remembering REF: p. 606 OBJ: Nursing Process: Assessment MSC: Client Needs: Physiologic Integrity 15. Which nursing measure is appropriate to prevent thrombophlebitis in the recovery period after a cesarean birth? a. Roll a bath blanket and place it firmly behind the knees. b. Limit oral intake of fluids for the first 24 hours. c. Assist the patient in performing gentle leg exercises. d. Ambulate the patient as soon as her vital signs are stable. ANS: C Leg exercises and passive range of motion promote venous blood flow and prevent venous stasis while the patient is still on bed rest. The blanket behind the knees will cause pressure and decrease venous blood flow. Limiting oral intake will produce hemoconcentration, which may lead to thrombophlebitis. The patient may not have full return of leg movements, and ambulating is contraindicated until she has full motion and sensation. PTS: 1 DIF: Cognitive Level: Application/Applying REF: p. 607 OBJ: Nursing Process: Implementation MSC: Client Needs: Physiologic Integrity 16. One of the first symptoms of puerperal infection to assess for in the postpartum woman is a. fatigue continuing for longer than 1 week. b. pain with voiding. c. profuse vaginal bleeding with ambulation. d. temperature of 38° C (100.4° F) or higher after 24 hours. ANS: D Postpartum or puerperal infection is any clinical infection after childbirth. The definition used in the United States continues to be the presence of a fever of 38° C (100.4° F) or higher on 2 successive days of the first 10 postpartum days, starting 24 hours after birth. Fatigue is a later finding associated with infection. Pain with voiding may indicate a UTI, but it is not typically one of the earlier symptoms of infection. Profuse lochia may be associated with endometritis, but it is not the first symptom associated with infection. PTS: 1 DIF: Cognitive Level: Knowledge/Remembering REF: p. 609 OBJ: Nursing Process: Assessment MSC: Client Needs: Health Promotion and Maintenance 17. The perinatal nurse caring for the postpartum woman understands that late postpartum hemorrhage is most likely caused by a. subinvolution of the uterus. b. defective vascularity of the decidua. c. cervical lacerations. d. coagulation disorders. ANS: A The most common causes of late postpartum hemorrhage are subinvolution and retained placental fragments. PTS: 1 DIF: Cognitive Level: Knowledge/Remembering REF: p. 602 OBJ: Nursing Process: Planning MSC: Client Needs: Physiologic Integrity 18. The patient who is being treated for endometritis is placed in Fowler's position because it a. promotes comfort and rest. b. facilitates drainage of lochia. c. prevents spread of infection to the urinary tract. d. decreases tension on the reproductive organs. ANS: B Lochia and infectious material are eliminated by gravity drainage when the woman is placed in the Fowler's position. PTS: 1 DIF: Cognitive Level: Knowledge/Remembering REF: p. 610 OBJ: Nursing Process: Implementation MSC: Client Needs: Physiologic Integrity 19. Nursing measures that help prevent postpartum urinary tract infection include which of the following? a. Promoting bed rest for 12 hours after delivery b. Discouraging voiding until the sensation of a full bladder is present c. Forcing fluids to at least 3000 mL/day d. Encouraging the intake of orange, grapefruit, or apple juice ANS: C Adequate fluid intake of 2500 to 3000 mL/day prevents urinary stasis, dilutes urine, and flushes out waste products. The woman should be encouraged to ambulate early. With pain medications, trauma to the area, and anesthesia, the sensation of a full bladder may be decreased. The woman needs to be encouraged to void frequently. Juices such as cranberry juice can discourage bacterial growth. PTS: 1 DIF: Cognitive Level: Knowledge/Remembering REF: p. 611 OBJ: Nursing Process: Implementation MSC: Client Needs: Physiologic Integrity 20. Which measure may prevent mastitis in the breastfeeding mother? a. Initiating early and frequent feedings b. Nursing the infant for 5 minutes on each breast c. Wearing a tight-fitting bra d. Applying ice packs before feeding ANS: A Early and frequent feedings prevent stasis of milk, which contributes to engorgement and mastitis. Five minutes does not adequately empty the breast. This will produce stasis of the milk. A firm-fitting bra will support the breast but not prevent mastitis. The breast should not be bound. Warm packs before feeding will increase the flow of milk. PTS: 1 DIF: Cognitive Level: Knowledge/Remembering REF: p. 611 OBJ: Nursing Process: Implementation MSC: Client Needs: Physiologic Integrity 21. A mother with mastitis is concerned about breastfeeding while she has an active infection. The nurse should explain that a. the infant is protected from infection by immunoglobulins in the breast milk. b. the infant is not susceptible to the organisms that cause mastitis. c. the organisms that cause mastitis are not passed to the milk. d. the organisms will be inactivated by gastric acid. ANS: C The organisms are localized in the breast tissue and are not excreted in the breast milk. The mother is just producing the immunoglobulin from this infection, so it is not available for the infant. Because of an immature immune system, infants are susceptible to many infections. However, this infection is in the breast tissue and is not excreted in the breast milk. The organism will not get into the infant's gastrointestinal system. PTS: 1 DIF: Cognitive Level: Comprehension/Understanding REF: p. 611 OBJ: Nursing Process: Implementation MSC: Client Needs: Physiologic Integrity 22. If the nurse suspects a uterine infection in the postpartum patient, she should assess the a. pulse and blood pressure. b. odor of the lochia. c. episiotomy site. d. abdomen for distention. ANS: B An abnormal odor of the lochia indicates infection in the uterus. The pulse may be altered with an infection, but the odor of the lochia will be an earlier sign and more specific. The infection may move to the episiotomy site if proper hygiene is not followed, but this does not demonstrate a uterine infection. The abdomen becomes distended usually because of a decrease of peristalsis, such as after cesarean section. PTS: 1 DIF: Cognitive Level: Application REF: p. 612 | Safety Alert Box OBJ: Nursing Process: Assessment MSC: Client Needs: Physiologic Integrity 23. Which condition is a transient, self-limiting mood disorder that affects new mothers after childbirth? a. Postpartum depression b. Postpartum psychosis c. Postpartum bipolar disorder d. Postpartum blues ANS: D Postpartum blues, or "baby blues," is a transient self-limiting disease that is believed to be related to hormonal fluctuations after childbirth. Postpartum depression is not the normal worries (blues) that many new mothers experience. Many caregivers believe that postpartum depression is underdiagnosed and underreported. Postpartum psychosis is a rare condition that usually surfaces within 3 weeks of delivery. Hospitalization of the woman is usually necessary for treatment of this disorder. Bipolar disorder is one of the two categories of postpartum psychosis, characterized by both manic and depressive episodes. PTS: 1 DIF: Cognitive Level: Knowledge/Remembering REF: p. 613 OBJ: Nursing Process: Assessment MSC: Client Needs: Psychosocial Integrity 24. When a woman is diagnosed with postpartum psychosis, one of the main concerns is that she may a. have outbursts of anger. b. neglect her hygiene. c. harm her infant. d. lose interest in her husband. ANS: C Thoughts of harm to one's self or the infant are among the most serious symptoms of PPD and require immediate assessment and intervention. The other problems can be attributed to postpartum psychosis, but the major concern is harm to the infant. PTS: 1 DIF: Cognitive Level: Knowledge/Remembering REF: p. 614 OBJ: Nursing Process: Assessment MSC: Client Needs: Psychosocial Integrity 25. What risk factor for peripartum depression (PPD) is likely to have the greatest effect on the woman's condition? a. Personal history of depression b. Single-mother status c. Low socioeconomic status d. Unplanned or unwanted pregnancy ANS: A A personal history of depression is a known risk factor for peripartum depression. Being single, from a low socioeconomic status, or having an unplanned or unwanted pregnancy may contribute to depression for some women but are not strong predictors. PTS: 1 DIF: Cognitive Level: Knowledge/Remembering REF: p. 614 OBJ: Nursing Process: Assessment MSC: Client Needs: Psychosocial Integrity 26. The maternity nurse knows that which disorder can be triggered by a birth the woman views as traumatic? a. A phobia b. Panic disorder c. Posttraumatic stress disorder (PTSD) d. Obsessive-compulsive disorder (OCD) ANS: C In PTSD, women perceive childbirth as a traumatic event. They have nightmares and flashbacks about the event, anxiety, and avoidance of reminders of the traumatic event. This will not lead to phobias, panic disorder, or OCD. PTS: 1 DIF: Cognitive Level: Knowledge/Remembering REF: p. 615 OBJ: Nursing Process: Assessment MSC: Client Needs: Psychosocial Integrity 27. To provide adequate postpartum care, the nurse should be aware that peripartum depression (PPD) a. is the "baby blues," plus the woman has a visit with a counselor or psychologist. b. does not affect the father who can then care for the baby. c. is distinguished by pervasive sadness that lasts at least 2 weeks. d. will disappear on its own without outside help. ANS: C PPD is characterized by a persistent depressed state. The woman is unable to feel pleasure or love although she is able to care for her infant. She often experiences generalized fatigue, irritability, little interest in food and sleep disorders. PPD is more serious and persistent than postpartum baby blues. Fathers are often affected. Most women need professional help to get through PPD, including pharmacologic intervention. PTS: 1 DIF: Cognitive Level: Knowledge/Remembering REF: p. 613 OBJ: Nursing Process: Assessment MSC: Client Needs: Psychosocial Integrity 28. What teaching does the nurse provide to help new mothers prevent postpartum depression? a. Stay home and avoid outside activities to ensure adequate rest. b. Be the only caregiver for your baby to facilitate infant attachment. c. Keep feelings of sadness and adjustment to your new role to yourself. d. Realize that this is a common occurrence that affects many women. ANS: D The new mother should understand that postpartum depression is common. Rest is important, but she does not need to confine herself to the house. Others need to help care for the baby so the mother can rest. Women need to be open and discuss their feelings. PTS: 1 DIF: Cognitive Level: Application/Applying REF: p. 613 OBJ: Nursing Process: Implementation MSC: Client Needs: Psychosocial Integrity 29. A provider left an order for a woman to have Methylergonovine 0.2 mg IM. The nurse assesses the woman and finds her vital signs to be: temperature 37.9° C (100.2° F), pulse 90 beats/minute, respirations 18 breaths/minute, and blood pressure 152/90 mm Hg. What action by the nurse is most appropriate? a. Administer acetaminophen first. b. Check policy for administration. c. Give the medication as prescribed. d. Consult with the provider. ANS: B Methylergonovine is contraindicated in women with hypertension. The nurse should check the agency's policy to see at what blood pressure reading this medication should be held. After checking the policy, the nurse can consult the provider if it can't be given. Acetaminophen is not related to this situation. PTS: 1 DIF: Cognitive Level: Application/Applying REF: p. 601 | Drug Guide OBJ: Nursing Process: Implementation MSC: Client Needs: Safe and Effective Care Environment MULTIPLE RESPONSE

2. Approximately how much would a newborn who weighed 7 pounds 6 ounces at birth weigh at 1 year of age? a. 14 3/4 lb b. 22 1/8 lb c. 29 1/2 lb d. Unable to estimate weigh at 1 year

B An infant triples birth weight by 1 year of age. The other calculations are incorrect. PTS: 1 DIF: Cognitive Level: Application/Applying REF: p. 83 OBJ: Nursing Process: Assessment MSC: Client Needs: Health Promotion and Maintenance

6. Which preterm infant should receive gavage feedings instead of a bottle? a. Sometimes gags when a feeding tube is inserted b. Is unable to coordinate sucking and swallowing c. Sucks on a pacifier during gavage feedings d. Has an axillary temperature of 98.4° F, an apical pulse of 149 beats/min, and respirations of 54 breaths/min

B An infant who cannot coordinate sucking, swallowing, and breathing should receive gavage feedings. The other infants are ready for bottle feedings. PTS: 1 DIF: Cognitive Level: Comprehension/Understanding REF: p. 627 OBJ: Nursing Process: Assessment MSC: Client Needs: Physiologic Integrity

6. The nurse working with a pregnant woman explains that a major advantage of nonpharmacologic pain management is that a. more complete pain relief is possible. b. no side effects or risks to the fetus are involved. c. the woman remains fully alert at all times. d. a more rapid labor is likely.

B Because nonpharmacologic pain management does not include analgesics, adjunct drugs, or anesthesia, it is harmless to the mother and the fetus. There is less pain relief with nonpharmacologic pain management during childbirth. The woman's alertness is not altered by medication, but the increase in pain will decrease alertness. Pain management may or may not alter the length of labor. At times when pain is decreased, the mother relaxes and labor progresses at a quicker pace. PTS: 1 DIF: Cognitive Level: Comprehension/Understanding REF: p. 356 OBJ: Integrated Process: Teaching-Learning MSC: Client Needs: Physiologic Integrity

1. The perinatal nurse is giving discharge instructions to a woman, status post suction and curettage secondary to a hydatidiform mole. The woman asks why she must take oral contraceptives for the next 12 months. The best response from the nurse is a. "If you get pregnant within 1 year, the chance of a successful pregnancy is very small. Therefore, if you desire a future pregnancy, it would be better for you to use the most reliable method of contraception available." b. "The major risk to you after a molar pregnancy is a type of cancer that can be diagnosed only by measuring the same hormone that your body produces during pregnancy. If you were to get pregnant, it would make the diagnosis of this cancer more difficult." c. "If you can avoid a pregnancy for the next year, the chance of developing a second molar pregnancy is rare. Therefore, to improve your chance of a successful pregnancy, it is better not to get pregnant at this time." d. "Oral contraceptives are the only form of birth control that will prevent a recurrence of a molar pregnancy."

B Beta-hCG levels will be drawn for 1 year to ensure that the mole is completely gone. There is an increased chance of developing choriocarcinoma after the development of a hydatidiform mole. The goal is to achieve a "zero" hCG level. If the woman were to become pregnant, it may obscure the presence of the potentially carcinogenic cells. Any contraceptive method except an IUD is acceptable. PTS: 1 DIF: Cognitive Level: Comprehension/Understanding REF: p. 528 OBJ: Integrated Process: Teaching-Learning MSC: Client Needs: Physiologic Integrity

1. The nurse knows that a urinary catheter is added to the instrument table if a forceps-assisted birth is anticipated. What is the correct rationale for this intervention? a. Spontaneous release of urine might contaminate the sterile field. b. An empty bladder provides more room in the pelvis. c. A sterile urine specimen is needed preoperatively. d. A Foley catheter prevents the membranes from spontaneously rupturing.

B Catheterization provides room for the application of the forceps blades and limits bladder trauma. Urine is sterile. A clean-catch urinalysis is usually sufficient for preoperative treatment. The membranes must be ruptured and the cervix completely dilated for a forceps-assisted birth. PTS: 1 DIF: Cognitive Level: Comprehension/Understanding REF: p. 385 | Box 17.2 OBJ: Nursing Process: Implementation MSC: Client Needs: Physiologic Integrity

3. The nurse caring for women in labor understands that childbirth pain is different from other types of pain in that it is a. more responsive to pharmacologic management. b. associated with a physiologic process. c. designed to make one withdraw from the stimulus. d. less intense.

B Childbirth pain is part of a normal process, whereas other types of pain usually signify an injury or illness. Childbirth pain is not more or less responsive to medication. The pain with childbirth is a normal process; it is not caused by the type of injury when withdrawal from the stimuli is seen. Childbirth pain is not less intense than other types of pain. PTS: 1 DIF: Cognitive Level: Comprehension/Understanding REF: p. 354 OBJ: Nursing Process: Assessment MSC: Client Needs: Physiologic Integrity

6. A nurse has been teaching a parent of a toddler about effective discipline. Which statement by the parent indicates that goals for teaching have been met? a. "I always include explanations and morals when I am disciplining my toddler." b. "I always try to be immediate and consistent when disciplining the children." c. "I believe that discipline should be done by only one family member." d. "My rule of thumb is no more than one spanking a day."

B Consistent and immediate discipline for toddlers is the most effective approach. Unless disciplined immediately, the toddler will have difficulty connecting the discipline with the behavior. The toddler's cognitive level of development precludes the use of explanations and morals as a part of discipline. Discipline for the toddler should be immediate; therefore the family member caring for the child should provide discipline to the toddler when it is necessary. Discipline is required for unacceptable behavior, and the one-spanking-a-day rule contradicts the concept of a consistent response to inappropriate behavior. In addition, spanking is an inappropriate method of disciplining a child. PTS: 1 DIF: Cognitive Level: Evaluation/Evaluating REF: p. 119 OBJ: Nursing Process: Evaluation MSC: Client Needs: Health Promotion and Maintenance

3. Which statement made by a mother is consistent with a developmental delay? a. "I notice my 9-month-old infant responds consistently to his name." b. "My 12-month-old child does not get herself to a sitting position or pull to stand." c. "I am so happy when my 1 1/2-month-old infant smiles at me." d. "My 5-month-old infant is not rolling over in both directions yet."

B Critical developmental milestones for gross motor development in a 12-month-old include standing briefly without support, getting to a sitting position, and pulling to stand. If a 12-month-old child does not perform these activities, it may be indicative of a developmental delay. An infant who responds to his name at 9 months of age is demonstrating abilities to both hear and interpret sound. A social smile is present by 2 months of age. Rolling over in both directions is not a critical milestone for gross motor development until the child reaches 6 months of age. PTS: 1 DIF: Cognitive Level: Comprehension/Understanding REF: Table 6.1 OBJ: Nursing Process: Assessment MSC: Client Needs: Health Promotion and Maintenance

9. A pregnant woman in the perinatal clinic is a recovering anorexic. She is distressed at the emphasis on weight gain. The nurse explains that the most important reason for evaluating the pattern of weight gain in pregnancy is to a. prevent excessive adipose tissue deposits. b. identify potential nutritional problems or complications of pregnancy. c. assess if this woman has relapsed. d. determine cultural influences on the woman's diet.

B Deviations from the recommended pattern of weight gain may indicate nutritional problems or developing complications. The nurse should assure this patient that monitoring weight gain is a routine part of prenatal care to ensure the baby's well-being. Preventing adipose tissue deposits is not the reason for monitoring weight gain. Determining cultural influences on diet and weight gain is important but not the most important reason. PTS: 1 DIF: Cognitive Level: Comprehension/Understanding REF: p. 255 OBJ: Integrated Process: Teaching-Learning MSC: Client Needs: Health Promotion and Maintenance 10. The nurse is counseling a woman in her third trimester about eating enough protein. If the woman already gets her non-pregnant RDA of protein, how much more does she need in her diet? a. 5 grams/day b. 10 grams/day c. 25 grams/day d. 30 grams/day ANS: C The current RDA for protein in the non-pregnant woman is 46 grams. To reach the recommendation for protein in the second half of pregnancy (71 grams), the patient needs to add 25 more grams of protein to her diet daily. PTS: 1 DIF: Cognitive Level: Application/Applying REF: Table 14.2 OBJ: Nursing Process: Assessment MSC: Client Needs: Physiologic Integrity 11. A pregnant patient would like to know a good food source of calcium other than dairy products. Which answer by the nurse is best? a. Legumes b. Yellow vegetables c. Lean meat d. Whole grains ANS: A Although dairy products contain the greatest amount of calcium, it also is found in legumes, nuts, dried fruits, and some dark green leafy vegetables. Yellow vegetables are rich in vitamin A. Lean meats are rich in protein and phosphorus. Whole grains are rich in zinc and magnesium. PTS: 1 DIF: Cognitive Level: Knowledge REF: p. 258 | Table 14.3 OBJ: Integrated Process: Teaching-Learning MSC: Client Needs: Physiologic Integrity 12. Which pregnant adolescent is most at risk for a nutritional deficit during pregnancy? a. A 15-year-old of normal height and weight b. A 17-year-old who is 10 pounds underweight c. A 16-year-old who is 10 pounds overweight d. A 16-year-old of normal height and weight ANS: B All adolescents are at nutritional risk during pregnancy, but the adolescent who is pregnant and underweight is most at risk, because she is already deficient in nutrition and must now supply the nutritional intake for both herself and her fetus. PTS: 1 DIF: Cognitive Level: Comprehension/Understanding REF: p. 265 OBJ: Nursing Process: Assessment MSC: Client Needs: Health Promotion and Maintenance 13. A patient has the nursing diagnosis: Imbalanced Nutrition: Less Than Body Requirements related to diet choices inadequate to meet nutrient requirements of pregnancy. What goal is most appropriate for this diagnosis? a. Weight change from 135 pounds to 165 pounds at delivery b. Take daily supplements consistently. c. Decrease intake of snack foods. d. Increase intake of complex carbohydrates. ANS: A A weight gain of 30 lb is one indication that the patient has gained a sufficient amount for the nutritional needs of pregnancy. A daily supplement is not the best goal for this patient. It does not meet the basic need of proper nutrition during pregnancy. Decreasing snack foods may be the problem and should be assessed. However, assessing the weight gain is the best method of monitoring nutritional intake for this pregnancy. Increasing the intake of complex carbohydrates is important for this patient, but monitoring the weight gain should be the end goal. PTS: 1 DIF: Cognitive Level: Evaluation/Evaluating REF: p. 256 | Table 14.1 OBJ: Nursing Process: Evaluation MSC: Client Needs: Health Promotion and Maintenance 14. A patient who is in week 28 of gestation is concerned about her weight gain of 1 pound in 1 week. Which response by the nurse is best? a. "You should try to decrease your amount of weight gain for the next 12 weeks." b. "You have gained an appropriate amount for the number of weeks of your pregnancy." c. "You should not gain any more weight until you reach the third trimester." d. "You have not gained enough weight for the number of weeks of your pregnancy." ANS: B At 28 weeks, a weight gain of 1 pound in 1 week is within the recommended range of 0.8 to 1 pound per week. The woman should be reassured that this is normal and healthy. The other responses are inaccurate. PTS: 1 DIF: Cognitive Level: Comprehension/Understanding REF: p. 256 | Table 14.1 OBJ: Integrated Process: Teaching-Learning MSC: Client Needs: Health Promotion and Maintenance 15. In teaching the pregnant adolescent about nutrition, what suggestion by the nurse is best? a. Eliminate common teen snack foods, because they are too high in fat and sodium. b. Work with the teen to include some fast food in a healthy prenatal diet. c. Suggest that she not eat at fast-food restaurants where the foods are of poor nutritional value. d. Realize that most adolescents are unwilling to make dietary changes during pregnancy. ANS: B Adolescents have some special nutritional needs during pregnancy, but they also need to feel that they fit in with their peers. Working with the teen to develop a healthy diet while including some snack and fast foods has the best chance of providing good nutrition. Telling the teen to eliminate certain foods or restaurants is likely not to work. Including the teen will make her more willing to make dietary changes. PTS: 1 DIF: Cognitive Level: Application/Applying REF: p. 265 OBJ: Nursing Process: Implementation MSC: Client Needs: Health Promotion and Maintenance 16. The traditional diet of Asian women includes little meat or dairy products and may be low in calcium and iron. The nurse can help the woman increase her intake of these foods by a. emphasizing the need for increased milk intake during pregnancy. b. suggesting she eat more "hot" foods during pregnancy. c. telling her husband that she must increase her intake of fruits and vegetables for the baby's sake. d. suggesting she eat more tofu, bok choy, and broccoli. ANS: D To increase the intake of calcium and iron in a culturally-appropriate way, the nurse can suggest the woman eat more broccoli and tofu for calcium and to eat more tofu and leafy green vegetables such as bok choy for iron. PTS: 1 DIF: Cognitive Level: Knowledge REF: p. 289 OBJ: Nursing Process: Implementation MSC: Client Needs: Physiologic Integrity 17. A pregnant woman's diet consists almost entirely of whole grain breads and cereals, fruits, and vegetables. The nurse should be most concerned about this woman's intake of which nutrient? a. Calcium b. Protein c. Vitamin B12 d. Folic acid ANS: C This diet is consistent with that followed by a strict vegetarian (vegan). Vegans consume only plant products. Because vitamin B12 is found in foods of animal origin, this diet is deficient in vitamin B12. Depending upon the woman's food choices this diet may be adequate in calcium, protein, and folic acid. PTS: 1 DIF: Cognitive Level: Knowledge/Remembering REF: p. 266 OBJ: Nursing Process: Assessment MSC: Client Needs: Physiologic Integrity 18. Which statement made by a lactating woman leads the nurse to believe that the woman might have lactose intolerance? a. "I always have heartburn after I drink milk." b. "If I drink more than a cup of milk, I get abdominal cramps and bloating." c. "Drinking milk usually makes me break out in hives." d. "Sometimes I notice that I have bad breath after I drink a cup of milk." ANS: B One problem that can interfere with milk consumption is lactose intolerance, which is the inability to digest milk sugar because of a lack of the enzyme lactose in the small intestine. Milk consumption may cause abdominal cramping, bloating, and diarrhea in such people, although many lactose-intolerant individuals can tolerate small amounts of milk without symptoms. The woman with lactose intolerance is more likely to experience bloating and cramping, not heartburn. A woman who breaks out in hives after consuming milk is more likely to have a milk allergy. Bad breath is not a sign of lactose intolerance. PTS: 1 DIF: Cognitive Level: Comprehension/Understanding REF: p. 266 OBJ: Nursing Process: Assessment MSC: Client Needs: Physiologic Integrity 19. To prevent GI upset, patients should be instructed to take iron supplements a. on a full stomach. b. at bedtime. c. after eating a meal. d. with milk. ANS: B Taking iron supplements at bedtime may reduce GI upset. Iron supplements are best absorbed if they are taken when the stomach is empty. Bran, tea, coffee, milk, and eggs may reduce absorption. PTS: 1 DIF: Cognitive Level: Application/Applying REF: p. 261 OBJ: Integrated Process: Teaching-Learning MSC: Client Needs: Physiologic Integrity 20. Which statement by a patient indicates that she understands the role of protein in her pregnancy? a. "Protein will help my baby grow." b. "Eating protein will prevent me from becoming anemic." c. "Eating protein will make my baby have strong teeth after he is born." d. "Eating protein will prevent me from being diabetic." ANS: A Protein is the nutritional element basic to growth. An adequate protein intake is essential to meeting the increasing demands of pregnancy. These demands arise from the rapid growth of the fetus; the enlargement of the uterus, mammary glands, and placenta; the increase in the maternal blood volume; and the formation of amniotic fluid. Iron intake prevents anemia. Calcium intake is needed for fetal bone and tooth development. Eating protein will not prevent diabetes. PTS: 1 DIF: Cognitive Level: Evaluation/Evaluating REF: p. 257 OBJ: Nursing Process: Evaluation MSC: Client Needs: Health Promotion and Maintenance 21. Which nutritional recommendation about fluids is accurate? a. A woman's daily intake should be 8 to 10 cups, and most of it should be water. b. Coffee should be limited to no more than 2 cups, but tea and cocoa can be consumed without worry. c. Of the artificial sweeteners, only aspartame has not been associated with any maternity health concerns. d. Water with fluoride is especially encouraged because it reduces the child's risk of tooth decay. ANS: A Eight to 10 cups is the standard for fluids; however, they should be the right fluids. All beverages containing caffeine, including tea, cocoa, and some soft drinks, should be avoided or should be drunk only in limited amounts. Artificial sweeteners, including aspartame, have no ill effects on the normal mother or fetus. However, mothers with phenylketonuria (PKU) should avoid aspartame. No evidence indicates that prenatal fluoride consumption reduces childhood tooth decay. However, it still helps the mother. PTS: 1 DIF: Cognitive Level: Knowledge/Remembering REF: p. 262 OBJ: Nursing Process: Planning MSC: Client Needs: Physiologic Integrity 22. The nurse explains to the expectant mother that which vitamin or mineral can lead to congenital malformations of the fetus if taken in excess by the mother? a. Zinc b. Vitamin D c. Folic acid d. Vitamin A ANS: D Zinc, vitamin D, and folic acid are vital to good maternity and fetal health and are highly unlikely to be consumed in excess. Vitamin A, taken in excess, causes a number of problems. An analog of vitamin A appears in prescribed acne medications, which must not be taken during pregnancy. PTS: 1 DIF: Cognitive Level: Knowledge/Remembering REF: p. 266 OBJ: Integrated Process: Teaching-Learning MSC: Client Needs: Health Promotion and Maintenance 23. A nurse teaching a prenatal class is discussing nutrition. What foods does the nurse advise pregnant women to avoid? a. Canned white tuna as a preferred choice b. Shark, swordfish, and mackerel c. Treating fish caught in local waterways as the safest d. High levels of mercury in salmon and shrimp ANS: B As a precaution against ingesting too much mercury, the pregnant patient should avoid eating all of these as well as the less common tilefish. Six ounces a week of canned albacore tuna is acceptable. Pregnant women should check with local authorities on the safety of eating locally caught fish, but if no advisories are in effect, eating them is fine. Salmon and shrimp are fine too up to 12 ounces a week. PTS: 1 DIF: Cognitive Level: Comprehension/Understanding REF: p. 263 | Safety Alert Box OBJ: Integrated Process: Teaching-Learning MSC: Client Needs: Health Promotion and Maintenance 24. A pregnant woman is at a picnic and asks a friend of hers, who is a nurse, what foods she can eat. What response by the nurse is best? a. Bologna sandwich b. Hot dog c. Smoked salmon spread d. Cheddar cheese and crackers ANS: D Hard cheeses like cheddar are safe for the pregnant woman to eat. She should not eat lunch meat or hotdogs unless they are heated until steaming. She should also not eat refrigerated smoked seafood. PTS: 1 DIF: Cognitive Level: Comprehension/Understanding REF: p. 263 | Safety Alert Box OBJ: Integrated Process: Teaching-Learning MSC: Client Needs: Physiologic Integrity 25. The woman in her third trimester asks the nurse how fast she will lose weight after giving birth. What information from the nurse is most accurate? a. You will lose about 20 pounds immediately. b. By the end of 2 weeks after birth you will have lost about 21 pounds. c. You can go on a diet after your first postnatal checkup. d. Most women do not lose all the weight they gain during pregnancy. ANS: B The woman can expect to lose 12 pounds immediately after birth and another 9 pounds by the end of the 2nd week, putting her total weight loss at that time around 21 pounds. The woman should wait 3 weeks before going on a diet. Most women lose all but a pound or two after childbirth, but this statement is discouraging to the patient. PTS: 1 DIF: Cognitive Level: Comprehension/Understanding REF: p. 269 OBJ: Integrated Process: Teaching-Learning MSC: Client Needs: Health Promotion and Maintenance MULTIPLE RESPONSE

6. To assess the duration of labor contractions, the nurse determines the time a. from the beginning of one contraction to the beginning of the next. b. from the beginning to the end of each contraction. c. of the strongest intensity of each contraction. d. of uterine relaxation between two contractions.

B Duration of labor contractions is the average length of contractions from beginning to end. Assessing from the beginning of one contraction to the beginning of the next is the frequency. The strongest intensity of each contraction is the strength or intensity. The interval of the contraction phase is the time of uterine relaxation between two contractions. PTS: 1 DIF: Cognitive Level: Knowledge/Remembering REF: p. 314 OBJ: Nursing Process: Assessment MSC: Client Needs: Health Promotion and Maintenance

2. Which behavior suggests appropriate psychosocial development in the adolescent? a. The adolescent seeks validation for socially acceptable behavior from older adults. b. The adolescent is self-absorbed and self-centered and has sudden mood swings. c. Adolescents move from peers and enjoy spending time with family members. d. Conformity with the peer group increases in late adolescence.

B During adolescence, energy is focused within. Adolescents concentrate on themselves in an effort to determine who they are or who they will be. Adolescents are likely to be impulsive and impatient. Parents often describe their teenager as being "self-centered or lazy." The peer group validates acceptable behavior during adolescence. Adolescents move from family and enjoy spending time with peers. Adolescents also spend time alone; they need this time to think and concentrate on themselves. Conformity becomes less important in late adolescence. PTS: 1 DIF: Cognitive Level: Knowledge/Remembering REF: p. 155 OBJ: Nursing Process: Assessment MSC: Client Needs: Health Promotion and Maintenance

2. During which phase of the cycle of violence does the batterer become contrite and remorseful? a. Battering phase b. Honeymoon phase c. Tension-building phase d. Increased drug-taking phase

B During the honeymoon phase, the battered person wants to believe that the battering will never happen again, and the batterer will promise anything to get back into the home. During the battering phase violence actually occurs, and the victim feels powerless. During the tension-building phase, the batterer becomes increasingly hostile, swears, threatens, throws things, and pushes the battered. Often the batterer increases the use of drugs during the tension-building phase; however, this is not an actual phase of the cycle. PTS: 1 DIF: Cognitive Level: Knowledge/Remembering REF: p. 518 | Figure 24.5 OBJ: Nursing Process: Assessment MSC: Client Needs: Psychosocial Integrity

9. Which nursing intervention is necessary before a second trimester transabdominal ultrasound? a. Place the woman NPO for 12 hours. b. Instruct the woman to drink 1 to 2 quarts of water. c. Administer a soapsuds enema. d. Perform an abdominal prep.

B During the second trimester, a full bladder may be needed to displace the intestines and elevate the uterus for better visibility. If indicated, the woman should be instructed to drink several glasses of clear fluid an hour before the time of the examination and to delay urination until the examination is completed. Since she needs to fill her bladder, being NPO is not appropriate. Enemas and abdominal preps are not necessary for this procedure. PTS: 1 DIF: Cognitive Level: Knowledge/Remembering REF: p. 276 OBJ: Nursing Process: Implementation MSC: Client Needs: Physiologic Integrity 10. The major advantage of chorionic villus sampling (CVS) over amniocentesis is that it a. is not an invasive procedure. b. does not require hospitalization. c. has less risk of spontaneous abortion. d. is performed earlier in pregnancy. ANS: D CVS is performed between 10 and 13 weeks of gestation, providing earlier results than amniocentesis, which is normally done during the second and third trimesters, although it can be done as early as 11 weeks if needed. The woman does not need hospitalization for this invasive procedure, and the risk of spontaneous abortion is about the same for both procedures. PTS: 1 DIF: Cognitive Level: Knowledge/Remembering REF: p. 279 OBJ: Nursing Process: Assessment MSC: Client Needs: Physiologic Integrity 11. What is the purpose of amniocentesis for the patient hospitalized at 34 weeks with pregnancy-induced hypertension? a. Identification of abnormal fetal cells b. Detection of metabolic disorders c. Determination of fetal lung maturity d. Identification of sex of the fetus ANS: C During the third trimester, amniocentesis is most often performed to determine fetal lung maturity. In pregnancy-induced hypertension, preterm delivery may be necessary because of changes in placental perfusion. It is not done to identify abnormal fetal cells, detect metabolic disorders, or identify the sex of the fetus. PTS: 1 DIF: Cognitive Level: Knowledge/Remembering REF: p. 279 OBJ: Nursing Process: Assessment MSC: Client Needs: Physiologic Integrity 12. An NST in which two or more fetal heart rate (FHR) accelerations of 15 beats per minute (bpm) or more occur with fetal movement in a 20-minute period is termed a. nonreactive. b. positive. c. negative. d. reactive. ANS: D The NST is reactive (normal) when two or more FHR accelerations of at least 15 bpm (each with a duration of at least 15 seconds) occur in a 20-minute period. A nonreactive result means that the heart rate did not accelerate during fetal movement. Positive and negative are not results given with this test. PTS: 1 DIF: Cognitive Level: Knowledge/Remembering REF: p. 281 OBJ: Nursing Process: Assessment MSC: Client Needs: Physiologic Integrity 13. The purpose of initiating contractions in a CST is to a. determine the degree of fetal activity. b. apply a stressful stimulus to the fetus. c. identifying fetal acceleration patterns. d. increase placental blood flow. ANS: B The CST involves recording the response of the FHR to stress induced by uterine contractions. The NST and biophysical profiles look at fetal movements. The NST looks at fetal heart accelerations with fetal movements. The CST records the fetal response to stress. It does not increase placental blood flow. PTS: 1 DIF: Cognitive Level: Knowledge/Remembering REF: p. 282 OBJ: Nursing Process: Assessment MSC: Client Needs: Physiologic Integrity 14. A pregnant woman states "This test isn't my idea, but my husband insists." Which response by the nurse is most appropriate? a. "Don't worry. Everything will be fine." b. "Why don't you want to have this test?" c. "You're concerned about having this test?" d. "It's your decision." ANS: C The nurse should clarify the statement and assist the patient in exploring her feelings about the test. Stating that everything will be fine is giving false reassurance and belittles the woman's concerns. "Why" questions usually put people on the defensive and are not therapeutic. Of course having the test is the woman's decision, but this closed statement does not encourage the woman to express her feelings. PTS: 1 DIF: Cognitive Level: Application/Applying REF: p. 286 OBJ: Integrated Process: Communication and Documentation MSC: Client Needs: Psychosocial Integrity 15. A nurse is preparing a woman for a nonstress test (NST). What nursing action is most appropriate? a. Position the woman on her left side. b. Seat the woman comfortably in a recliner. c. Have the woman to drink 1 liter of water prior to the test. d. Place conduction gel on the obese woman's abdomen. ANS: B To correctly position the pregnant patient for an NST, the woman usually sits in a reclining chair. Alternatively she can be in a semi-Fowler position with a lateral tilt. This will optimize uterine perfusion and prevent supine hypotension. The woman does not need to drink water. Conduction gel is used in all NST tests. PTS: 1 DIF: Cognitive Level: Application/Applying REF: p. 281 OBJ: Nursing Process: Implementation MSC: Client Needs: Health Promotion and Maintenance 16. Which statement regarding various biochemical assessments used during pregnancy is correct? a. Chorionic villus sampling (CVS) is becoming more popular because it provides early diagnosis. b. Screening for maternal serum alpha-fetoprotein (MSAFP) levels is recommended between 10 and 12 weeks of gestation in order to give parents time to consider options. c. Percutaneous umbilical blood sampling (PUBS) is one of the multiple marker screen tests for Down syndrome. d. MSAFP is a screening tool only; it identifies candidates for more definitive procedures. ANS: D MSAFP is a screening tool, not a diagnostic tool. Further diagnostic testing is indicated after an abnormal MSAFP. CVS does provide a rapid result, but it is declining in popularity because of advances in noninvasive screening techniques. MSAFP screening is recommended for all pregnant women. Screening is recommended between 15 and 20 weeks of gestation. Abnormal findings give parents time to have additional tests done. PTS: 1 DIF: Cognitive Level: Comprehension/Understanding REF: p. 277 OBJ: Integrated Process: Teaching-Learning MSC: Client Needs: Health Promotion and Maintenance 17. A woman is scheduled for an ultrasound and is asking the nurse questions about this test. Which statement by the nurse regarding ultrasonography during pregnancy is most accurate? a. Ultrasonography uses infrared technology to create an image. b. Ultrasonography is only utilized as an adjunct to more invasive tests. c. Ultrasonography is not harmful to the fetus. d. Ultrasonography is not a component of biophysical profile testing. ANS: C Most women look forward to the results of this test, which causes no harm to the fetus. Ultrasonography uses sound waves to create an image. As an adjunct to more invasive tests, ultrasonography can provide visual guidance for increased safety. It can be done as a standalone test. Ultrasonography is a component of biophysical profile testing. PTS: 1 DIF: Cognitive Level: Knowledge/Remembering REF: p. 274 OBJ: Integrated Process: Teaching-Learning MSC: Client Needs: Health Promotion and Maintenance 18. A nurse is teaching a woman how to do "kick counts." What information about this assessment is most appropriate? a. Notify your provider if the baby's movement patterns change. b. Count the number of fetal movements over 2 hours. c. Call the OB triage area if there are fewer than 10 movements/hour. d. Have your partner verify your count at the same time you perform it. ANS: A Since there is no consensus on how the mother should be taught to perform this assessment, it is more important that she become familiar with her baby's movements and patterns and notify the provider about any change from normal. PTS: 1 DIF: Cognitive Level: Comprehension/Understanding REF: p. 285 OBJ: Integrated Process: Teaching-Learning MSC: Client Needs: Physiologic Integrity MULTIPLE RESPONSE

2. A student nurse is trying to assess vital signs on a laboring woman. Which statement by the registered nurse is the best rationale for assessing maternal vital signs between contractions? a. During a contraction, assessing fetal heart rates is the priority. b. Maternal circulating blood volume increases temporarily during contractions. c. Maternal blood flow to the heart is reduced during contractions. d. Vital signs taken during contractions are not accurate.

B During uterine contractions, blood flow to the placenta temporarily stops, causing a relative increase in the mother's blood volume, which in turn temporarily increases blood pressure and slows pulse. It is important to monitor fetal response to contractions, but the question is concerned with the maternal vital signs so assessing the fetal heart rate is not the priority. Vital signs are altered by contractions but are considered accurate for that period of time. However, they do not reflect the woman's baseline. PTS: 1 DIF: Cognitive Level: Comprehension/Understanding REF: p. 289 OBJ: Integrated Process: Teaching-Learning MSC: Client Needs: Health Promotion and Maintenance

8. A nurse uses Erikson's theory to guide nursing practice. What action by a hospitalized 4-year-old child would the nurse evaluate as developmentally appropriate? a. Dressed and fed by the parents b. Independently ask for play materials or other personal needs c. Verbalizes an understanding of the reason for the hospitalization d. Asks for a parent stay in the room at all times

B Erikson identifies initiative as a developmental task for the preschool child. Initiating play activities and asking for play materials or assistance with personal needs demonstrates developmental appropriateness. Parents need to foster appropriate developmental behavior in the 4-year-old child. Dressing and feeding the child do not encourage independent behavior. A 4-year-old child cannot be expected to cognitively understand the reason for hospitalization. Expecting the child to verbalize an understanding for hospitalization is an inappropriate outcome. Parents staying with the child throughout a hospitalization is not a developmental outcome. Although children benefit from parental involvement, parents may not have the support structure to stay in the room with the child at all times. PTS: 1 DIF: Cognitive Level: Evaluation/Evaluating REF: Table 5.2 OBJ: Nursing Process: Evaluation MSC: Client Needs: Health Promotion and Maintenance

6. A student nurse is preparing to administer misoprostol (Cytotec). What action by the student seen by the registered nurse demonstrates adequate knowledge about this medication? a. Assesses maternal blood pressure 30 minutes after administration b. Assesses fetal heart tones prior to administering the medication c. Documents the drug administration in the woman's chart d. Takes and records an apical pulse for 1 minute prior to administration

B Fetal heart tones should be assessed prior to giving cervical ripening agents such as misoprostol. It is not necessary to assess maternal blood pressure afterward or an apical pulse prior to administering the medication. Documentation of all medications is a legal requirement but is not related specifically to this drug. PTS: 1 DIF: Cognitive Level: Evaluation/Evaluating REF: Box 17.2 OBJ: Nursing Process: Evaluation MSC: Client Needs: Physiologic Integrity

1. Which information should the nurse include when preparing a 5-year-old child for a cardiac catheterization? a. A detailed explanation of the procedure b. A description of what the child will feel and see during procedure c. An explanation about the dye that will go directly into his vein d. An assurance to the child that he and the nurse can talk about the procedure when it is over

B For a preschooler, the provision of sensory information about what to expect during the procedure will enhance the child's ability to cope with the events of the procedure and will decrease anxiety. Explaining the procedure in detail is probably more than the 5-year-old child can comprehend, and it will likely produce anxiety. Using the word "dye" with a preschooler can be frightening for the child. The child needs information before the procedure. PTS: 1 DIF: Cognitive Level: Comprehension/Understanding REF: Table 4.3 OBJ: Integrated Process: Teaching-Learning MSC: Client Needs: Health Promotion and Maintenance

2. The nurse learns that the most common cause of pathologic hyperbilirubinemia is which of the following? a. Hepatic disease b. Hemolytic disorders in the newborn c. Postmaturity d. Congenital heart defect

B Hemolytic disorders in the newborn are the most common cause of pathologic jaundice. Hepatic damage and prematurity may be causes of pathologic hyperbilirubinemia, but they are not the most common cause. Congenital heart defect is not a common cause of pathologic hyperbilirubinemia in neonates. PTS: 1 DIF: Cognitive Level: Knowledge/Remembering REF: p. 645 OBJ: Integrated Process: Teaching-Learning MSC: Client Needs: Physiologic Integrity

3. The nurse knows that a measure for preventing late postpartum hemorrhage is to a. administer broad-spectrum antibiotics. b. inspect the placenta after delivery. c. manually remove the placenta. d. pull on the umbilical cord to hasten the delivery of the placenta.

B If a portion of the placenta is missing, the clinician can explore the uterus, locate the missing fragments, and remove the potential cause of late postpartum hemorrhage. Broad-spectrum antibiotics will be given if postpartum infection is suspected. Manual removal of the placenta increases the risk of postpartum hemorrhage. The placenta is usually delivered 5 to 30 minutes after birth of the baby without pulling on the cord. That can cause uterine inversion. PTS: 1 DIF: Cognitive Level: Knowledge/Remembering REF: p. 602 OBJ: Nursing Process: Implementation MSC: Client Needs: Physiologic Integrity

2. Which maternal condition always necessitates delivery by cesarean section? a. Partial abruptio placentae b. Total placenta previa c. Ectopic pregnancy d. Eclampsia

B In total placenta previa, the placenta completely covers the cervical os. The fetus would die if a vaginal delivery occurred. In a partial abruptio placentae, if the mother has stable vital signs and the fetus is alive, a vaginal delivery can be attempted. If the fetus has died, a vaginal delivery is preferred. The most common ectopic pregnancy is a tubal pregnancy, which is usually detected and treated in the first trimester. Labor can be safely induced if the eclampsia is under control. PTS: 1 DIF: Cognitive Level: Knowledge/Remembering REF: p. 531 OBJ: Nursing Process: Planning MSC: Client Needs: Physiologic Integrity

7. What action by the nurse is most important to prevent the kidnapping of newborns from the hospital? a. Instruct the mother not to give her infant to anyone except the one nurse assigned to her that day. b. Question anyone who is seen walking in the hallways carrying an infant. c. Allow no visitors in the maternity area except those who have identification bracelets. d. Restrict the amount of time infants are out of the nursery.

B Infants should be transported in the hallways only in their cribs. It is impossible for one nurse to be on call for one mother and baby for the entire shift, so the parents need to be able to identify the nurses who are working on the unit. Limiting visitors may cut the new family off from vital support. Infants should be with their parents the majority of the time. PTS: 1 DIF: Cognitive Level: Application/Applying REF: p. 469 OBJ: Nursing Process: Implementation MSC: Client Needs: Safe and Effective Care Environment

1. The breastfeeding mother should be taught a safe method to remove the breast from the baby's mouth. Which suggestion by the nurse is most appropriate? a. Slowly remove the breast from the baby's mouth when the infant has fallen asleep and the jaws are relaxed. b. Break the suction by inserting your finger into the corner of the infant's mouth. c. A popping sound occurs when the breast is correctly removed from the infant's mouth. d. Elicit the Moro reflex to wake the baby and remove the breast when the baby cries.

B Inserting a finger into the corner of the baby's mouth between the gums to break the suction avoids trauma to the breast. The infant who is sleeping may lose grasp on the nipple and areola, resulting in "chewing" on the nipple, making it sore. A popping sound indicates improper removal of the breast from the baby's mouth and may cause cracks or fissures in the breast. Most mothers prefer the infant to continue to sleep after the feeding. Gentle wake-up techniques are recommended. PTS: 1 DIF: Cognitive Level: Comprehension/Understanding REF: p. 488 OBJ: Nursing Process: Implementation MSC: Client Needs: Health Promotion and Maintenance

7. A woman has a history of hypertension during pregnancy. What method of intrapartum fetal monitoring does the nurse initiate? a. Continuous auscultation with a fetoscope b. Continuous electronic fetal monitoring c. Intermittent assessment with a Doppler transducer d. Intermittent electronic fetal monitoring for 15 minutes each hour

B Maternal hypertension may reduce placental blood flow through vasospasm of the spiral arteries. Reduced placental perfusion is best assessed with continuous electronic fetal monitoring to identify patterns associated with this condition. It is not practical to provide continuous auscultation with a fetoscope. This fetus needs continuous monitoring because it is at high risk for complications. PTS: 1 DIF: Cognitive Level: Application/Applying REF: p. 333 OBJ: Nursing Process: Implementation MSC: Client Needs: Physiologic Integrity

7. A woman is 16 weeks pregnant with her first baby. She asks how long it will be before she feels the baby move. The best answer is a. "You should have felt the baby move by now." b. "Within the next month, you should start to feel fluttering sensations." c. "The baby is moving, but you can't feel it yet." d. "Some babies are quiet, and you don't feel them move."

B Maternal perception of fetal movement usually begins 17 to 20 weeks after conception, particularly if this is the first pregnancy. "The baby is moving, but you can't feel it yet" is a true statement. The fetus's movements are not strong enough to be felt until 17 to 20 weeks; however, this statement does not answer the concern of the woman. If no movement is felt at the end of 20 weeks, further assessment is needed. PTS: 1 DIF: Cognitive Level: Comprehension/Understanding REF: p. 204 OBJ: Nursing Process: Implementation MSC: Client Needs: Physiologic Integrity

4. An abortion in which the fetus dies but is retained in the uterus is called ________ abortion. a. inevitable b. missed c. incomplete d. threatened

B Missed abortion refers to a dead fetus being retained in the uterus. An inevitable abortion means that the cervix is dilating with the contractions. An incomplete abortion means that not all of the products of conception were expelled. With a threatened abortion the woman has cramping and bleeding but not cervical dilation. PTS: 1 DIF: Cognitive Level: Knowledge/Remembering REF: p. 524 OBJ: Nursing Process: Assessment MSC: Client Needs: Physiologic Integrity

7. What characteristic would most likely be found in a Mexican-American family? a. Stoicism b. Close extended family c. Considering docile children weak d. Very interested in health-promoting lifestyles

B Most Mexican-American families are very close, and it is not unusual for children to be surrounded by parents, siblings, grandparents, and godparents. It is important to respect this cultural characteristic and to see it as a strength, not a weakness. Although stoicism may be present in any family, Mexican-American families tend to be more expressive. Considering docile children weak is a characteristic of Native Americans. Although everyone tends now to embrace more health-promoting lifestyles, they are more prominent in Anglo-Americans. PTS: 1 DIF: Cognitive Level: Knowledge/Remembering REF: p. 41 OBJ: Integrated Process: Culture and Spirituality MSC: Client Needs: Psychosocial Integrity

5. Which statement is true of multifactorial disorders? a. They may not be evident until later in life. b. They are usually present and detectable at birth. c. The disorders are characterized by multiple defects. d. Secondary defects are rarely associated with multifactorial disease.

B Multifactorial disorders result from an interaction between a person's genetic susceptibility and environmental conditions that favor development of the defect. They are characteristically present and detectable at birth. They are usually single isolated defects, although the primary defect may cause secondary defects. Secondary defects can occur with multifactorial disorders. PTS: 1 DIF: Cognitive Level: Knowledge/Remembering REF: p. 176 OBJ: Nursing Process: Assessment MSC: Client Needs: Physiologic Integrity

6. Which statement related to changes in the breasts during pregnancy is the most accurate? a. During the early weeks of pregnancy there is decreased sensitivity. b. Nipples and areolae become more pigmented. c. Montgomery tubercles are no longer visible around the nipples. d. Venous congestion of the breasts is more visible in the multiparous woman.

B Nipples and areolae become more pigmented, and the nipples become more erectile and may express colostrum. Fullness, heightened sensitivity, tingling, and heaviness of the breasts occur in the early weeks of gestation in response to increased levels of estrogen and progesterone. Montgomery tubercles may be seen around the nipples. These sebaceous glands may have a protective role in that they keep the nipples lubricated for breastfeeding. Venous congestion in the breasts is more obvious in primigravidas. PTS: 1 DIF: Cognitive Level: Knowledge/Remembering REF: p. 216 OBJ: Nursing Process: Assessment MSC: Client Needs: Physiologic Integrity

3. Which nursing intervention is an independent (nurse-driven) function of the nurse? a. Administering oral analgesics b. Teaching the woman perineal care c. Requesting diagnostic studies d. Providing wound care to a surgical incision

B Nurses are responsible for various independent functions, including teaching, counseling, and intervening in nonmedical problems. Interventions initiated by the physician and carried out by the nurse are called dependent functions. Administering oral analgesics is a dependent function; it is initiated by a physician or other provider and carried out by the nurse. Requesting diagnostic studies is a dependent function. Providing wound care is a dependent function; it is usually initiated by the physician or other provider through direct orders or protocol. PTS: 1 DIF: Cognitive Level: Comprehension/Understanding REF: Box 2.3 OBJ: Integrated Process: Teaching-Learning MSC: Client Needs: Health Promotion and Maintenance

8. If a woman's menstrual cycle began on June 2 and normally lasts 28 days, ovulation would mostly likely occur on June a. 10 b. 16 c. 21 d. 29

B Ovulation occurs approximately 12 to 14 days after the beginning of the menstrual period in a 28-day cycle. In this woman, ovulation would most likely occur on June 16. June 10 would just be 8 days into the cycle and too early for ovulation. June 21 would be 18 days into the cycle. Ovulation should have already occurred at this point. June 29 would be 27 days into the cycle and almost time for the next period. PTS: 1 DIF: Cognitive Level: Comprehension/Understanding REF: p. 190 OBJ: Nursing Process: Assessment MSC: Client Needs: Physiologic Integrity

4. Which family will most likely have the most difficulty coping with a seriously ill child? a. A single-parent mother who has the support of her parents and siblings b. Parents who have just moved to the area and have not yet found health care providers c. The family of a child who has had multiple hospitalizations related to asthma and has adequate relationships with the nursing staff d. A family in which there is a young child and four older married children who live in the area

B Parents in a new environment will have increased stress related to their lack of a support system. They have no previous experiences in the setting from which to draw confidence. Not only does this family not have friends or relatives to help them, they must find a provider when their child is seriously ill. Although only one parent is available, she has the support of her extended family, which will assist her in adjusting to the crisis. Because this family has had positive experiences in the past, family members can draw from those experiences and feel confident about the setting. This family has an extensive support system that will assist the parents in adjusting to the crisis. PTS: 1 DIF: Cognitive Level: Comprehension/Understanding REF: p. 38 | Box 3.1 OBJ: Nursing Process: Assessment MSC: Client Needs: Psychosocial Integrity

1. The parent of a 14-month-old child is concerned because the child's appetite has decreased. The best response for the nurse to make to the parent is, a. "It is important for your toddler to eat three meals a day and no snacks." b. "It is not unusual for toddlers to eat less due to slower growth." c. "Be sure to increase your child's milk consumption, which will improve nutrition." d. "Give your child a multivitamin daily to increase your toddler's nutrition."

B Physiologically, growth slows and appetite decreases during the toddler period. So the nurse should assure the parent that this is normal behavior. Toddlers need small, frequent meals. Nutritious selection throughout the day, rather than quantity, is more important with this age-group. Milk consumption should not exceed 16 to 24 oz daily. Juice should be limited to 4 to 6 oz per day. Increasing the amount of milk will only further decrease solid food intake. Supplemental vitamins are important for all children, but they do not increase appetite. PTS: 1 DIF: Cognitive Level: Application/Applying REF: p. 116 OBJ: Nursing Process: Implementation MSC: Client Needs: Health Promotion and Maintenance

6. A nurse wants to assess a chronically ill child's feelings regarding a lengthy hospitalization and treatments. What action by the nurse is best? a. Ask direct questions of the child as to feelings. b. Watch the child play on several occasions. c. Discuss the situation with the parents. d. Refer the child to the child life specialist for assessment.

B Play for all children is an activity woven with meaning and purpose. For chronically ill children, play can indicate their state of wellness and response to treatment. It is a way to express joy, fear, anxiety, and disappointments. The nurse can best decipher the child's emotional state by observing this activity. Children often are threatened by direct questions, especially if the questioner is not well known to the child. The nurse may want to discuss the situation with the parents or enlist the help of the child life specialist, but these will not give the nurse the rich data that can be obtained through watching the child play. PTS: 1 DIF: Cognitive Level: Application/Applying REF: p. 73 OBJ: Nursing Process: Assessment MSC: Client Needs: Health Promotion and Maintenance

1. Preconception counseling is critical to the outcome of diabetic pregnancies because poor glycemic control before and during early pregnancy is associated with a. frequent episodes of maternal hypoglycemia. b. congenital anomalies in the fetus. c. polyhydramnios. d. hyperemesis gravidarum.

B Preconception counseling is particularly important because strict metabolic control before conception and in the early weeks of gestation is instrumental in decreasing the risks of congenital anomalies. Frequent episodes of maternal hypoglycemia may occur during the first trimester (not before conception) as a result of hormone changes and the effects on insulin production and usage. Hydramnios occurs about 10 times more often in diabetic pregnancies than in nondiabetic pregnancies. Typically, it is seen in the third trimester of pregnancy. Hyperemesis gravidarum may exacerbate hypoglycemic events as the decreased food intake by the mother and glucose transfer to the fetus contribute to hypoglycemia. PTS: 1 DIF: Cognitive Level: Knowledge/Remembering REF: p. 550 OBJ: Nursing Process: Planning MSC: Client Needs: Physiologic Integrity

1. What is most helpful in preventing premature birth? a. High socioeconomic status b. Adequate prenatal care c. Transitional Assistance to Needy Families d. Women, Infants, and Children nutritional program

B Prenatal care is vital in identifying possible problems. Women from higher economic status are more likely to seek adequate prenatal care, but it is the care that is most helpful. Government programs help with specific needs of the pregnant woman, but adequate care is more important. PTS: 1 DIF: Cognitive Level: Knowledge/Remembering REF: p. 620 OBJ: Nursing Process: Assessment MSC: Client Needs: Health Promotion and Maintenance

6. The student nurse learns that the hormone necessary for milk production is a. estrogen. b. prolactin. c. progesterone. d. lactogen.

B Prolactin, secreted by the anterior pituitary, is a hormone that causes the breasts to produce milk. Estrogen decreases the effectiveness of prolactin and prevents mature breast milk from being produced. Progesterone decreases the effectiveness of prolactin and prevents mature breast milk from being produced. Human placental lactogen decreases the effectiveness of prolactin and prevents mature breast milk from being produced. PTS: 1 DIF: Cognitive Level: Knowledge/Remembering REF: p. 484 OBJ: Integrated Process: Teaching-Learning MSC: Client Needs: Physiologic Integrity

9. In caring for the preterm infant, what complication is thought to be a result of high arterial blood oxygen level? a. Necrotizing enterocolitis (NEC) b. Retinopathy of prematurity (ROP) c. Bronchopulmonary dysplasia (BPD) d. Intraventricular hemorrhage (IVH)

B ROP is thought to occur as a result of high levels of oxygen in the blood. NEC is due to the interference of blood supply to the intestinal mucosa. Necrotic lesions occur at that site. BPD is caused by the use of positive pressure ventilation against the immature lung tissue. IVH is due to rupture of the fragile blood vessels in the ventricles of the brain. It is most often associated with hypoxic injury, increased blood pressure, and fluctuating cerebral blood flow. PTS: 1 DIF: Cognitive Level: Comprehension/Understanding REF: p. 636 OBJ: Nursing Process: Assessment MSC: Client Needs: Physiologic Integrity 10. With regard to eventual discharge of the high-risk newborn or transfer to a different facility, nurses and families should be aware that a. infants will stay in the NICU until they are ready to go home. b. once discharged to home, the high-risk infant should be treated like any healthy term newborn. c. parents of high-risk infants need special support and detailed contact information. d. if a high-risk infant and mother need transfer to a specialized regional center, it is better to wait until after birth and the infant is stabilized. ANS: C High-risk infants can cause profound parental stress and emotional turmoil. Parents need support, special teaching, and quick access to various resources available to help them care for their baby. Parents and their high-risk infant should get to spend a night or two in a predischarge room, where care for the infant is provided away from the NICU. Just because high-risk infants are discharged does not mean they are normal, healthy babies. Follow-up by specialized practitioners is essential. Ideally, the mother and baby are transported with the fetus in utero; this reduces neonatal morbidity and mortality. PTS: 1 DIF: Cognitive Level: Comprehension REF: p. 634 OBJ: Nursing Process: Planning MSC: Client Needs: Psychosocial Integrity 11. Which combination of expressing pain could be demonstrated in a neonate? a. Low-pitched crying, tachycardia, eyelids open wide b. Cry face, flaccid limbs, closed mouth c. High-pitched, shrill cry, withdrawal, change in heart rate d. Cry face, eye squeeze, increase in blood pressure ANS: D Cry face, eye squeeze, and an increase in blood pressure indicate pain. The other manifestations are not those of pain in the neonate. PTS: 1 DIF: Cognitive Level: Knowledge/Remembering REF: p. 624 OBJ: Nursing Process: Assessment MSC: Client Needs: Health Promotion and Maintenance 12. Which is true about newborns classified as small for gestational age (SGA)? a. They weigh less than 2500 g. b. They are born before 38 weeks of gestation. c. Placental malfunction is the only recognized cause of this condition. d. They are below the 10th percentile on gestational growth charts. ANS: D SGA infants are defined as below the 10th percentile in growth when compared with other infants of the same gestational age. SGA is not defined by weight. Infants born before 38 weeks are defined as preterm. There are many causes of SGA babies. PTS: 1 DIF: Cognitive Level: Knowledge/Remembering REF: p. 637 OBJ: Nursing Process: Assessment MSC: Client Needs: Health Promotion and Maintenance 13. A nurse is caring for an SGA newborn. What nursing action is most important? a. Observe for respiratory distress syndrome. b. Observe for and prevent dehydration. c. Promote bonding. d. Prevent hypoglycemia by early and frequent feedings. ANS: D The SGA infant has poor glycogen stores and is subject to hypoglycemia. Respiratory distress syndrome is seen in preterm infants. Dehydration is a concern for all infants and is not specific for SGA infants. Promoting bonding is a concern for all infants and is not specific for SGA infants. PTS: 1 DIF: Cognitive Level: Application/Applying REF: p. 638 OBJ: Nursing Process: Implementation MSC: Client Needs: Physiologic Integrity 14. A nurse is assessing an SGA infant with asymmetric intrauterine growth restriction. What assessment finding correlates with this condition? a. One side of the body appears slightly smaller than the other. b. All body parts appear proportionate. c. The head seems large compared with the rest of the body. d. The extremities are disproportionate to the trunk. ANS: C In asymmetric intrauterine growth restriction, the head is normal in size but appears large because the infant's body is long and thin due to lack of subcutaneous fat. The left and right side growth should be symmetric. With asymmetric intrauterine growth restrictions, the body appears smaller than normal compared to the head. The body parts are out of proportion, with the body looking smaller than expected due to the lack of subcutaneous fat. The body, arms, and legs have lost subcutaneous fat so they will look small compared to the head. PTS: 1 DIF: Cognitive Level: Knowledge/Remembering REF: p. 637 OBJ: Nursing Process: Assessment MSC: Client Needs: Physiologic Integrity 15. Which statement is true about large for gestational age (LGA) infants? a. They weigh more than 3500 g. b. They are above the 80th percentile on gestational growth charts. c. They are prone to hypoglycemia, polycythemia, and birth injuries. d. Postmaturity syndrome and fractured clavicles are the most common complications. ANS: C Hypoglycemia, polycythemia, and birth injuries are common in LGA infants. LGA infants are determined by their weight compared to their age. They are above the 90th percentile on the gestational growth charts. Birth injuries are a problem, but postmaturity syndrome is not an expected complication with LGA infants. PTS: 1 DIF: Cognitive Level: Knowledge/Remembering REF: p. 638 OBJ: Nursing Process: Assessment MSC: Client Needs: Physiologic Integrity 16. Of all the signs seen in infants with respiratory distress syndrome, which sign is especially indicative of the syndrome? a. Pulse more than 160 beats/min b. Circumoral cyanosis c. Grunting d. Substernal retractions ANS: C Grunting increases the pressure inside the alveoli to keep them open when surfactant is insufficient. This is a characteristic and often early sign of RDS. The other assessments are not specific to RDS. PTS: 1 DIF: Cognitive Level: Knowledge/Remembering REF: p. 621 OBJ: Nursing Process: Assessment MSC: Client Needs: Physiologic Integrity 17. While caring for the postterm infant, the nurse recognizes that the fetus may have passed meconium prior to birth as a result of a. hypoxia in utero. b. NEC. c. placental insufficiency. d. rapid use of glycogen stores. ANS: A When labor begins, poor oxygen reserves may cause fetal compromise. The fetus may pass meconium as a result of hypoxia before or during labor, increasing the risk of meconium aspiration. Meconium is not passed as a result of NEC, placental insufficiency, or rapid use of glycogen stores. PTS: 1 DIF: Cognitive Level: Knowledge/Remembering REF: p. 636 OBJ: Nursing Process: Assessment MSC: Client Needs: Physiologic Integrity 18. Which data should alert the nurse that the neonate is postmature? a. Cracked, peeling skin b. Short, chubby arms and legs c. Presence of vernix caseosa d. Presence of lanugo ANS: A Loss of vernix caseosa, which protects the fetal skin in utero, may leave the skin macerated and appearing cracked and peeling. Postmature infants usually have long, thin arms and legs. Vernix caseosa decreases in the postmature infant. Absence of lanugo is common in postmature infants. PTS: 1 DIF: Cognitive Level: Knowledge/Remembering REF: p. 637 OBJ: Nursing Process: Assessment MSC: Client Needs: Health Promotion and Maintenance 19. Because of the premature infant's decreased immune functioning, what nursing diagnosis should the nurse include in a plan of care for a premature infant? a. Delayed growth and development b. Ineffective thermoregulation c. Ineffective infant feeding pattern d. Risk for infection ANS: D The nurse needs to know that decreased immune functioning increases the risk for infection. The other diagnoses are appropriate for the premature infant but not related directly to immune function. PTS: 1 DIF: Cognitive Level: Comprehension/Understanding REF: p. 624 OBJ: Nursing Process: Planning MSC: Client Needs: Physiologic Integrity 20. To maintain optimal thermoregulation for the premature infant, what action by the nurse is most appropriate? a. Bathe the infant once a day. b. Put an undershirt on the infant in the incubator. c. Assess the infant's hydration status. d. Lightly clothe the infant under the radiant warmer. ANS: B Air currents around an unclothed infant will result in heat loss. Bathing causes evaporative heat loss. Assessing hydration will not maintain thermoregulation. Clothing is not worn when the infant is under a radiant warmer. PTS: 1 DIF: Cognitive Level: Application/Applying REF: p. 623 OBJ: Nursing Process: Implementation MSC: Client Needs: Health Promotion and Maintenance 21. A nurse is caring for a preterm baby who weighs 4.8 pounds. What assessment finding indicates the baby is dehydrated? a. Urine output of 3.3 mL/hour b. Urine specific gravity of 1.001 c. Low serum sodium d. Weight gain of 43 g in one day ANS: A This baby weighs 2.18 kg. Dehydration is noted with a urine output of <2 mL/kg/hour. A urine output of 3.3 mL is 1.5 mL/kg/hour and so indicates dehydration. The dilute urine specific gravity indicates overhydration as does the low serum sodium. The weight gain is normal (15 to 20 g/kg/day). PTS: 1 DIF: Cognitive Level: Analysis/Analyzing REF: p. 623 | Safety Alert Box OBJ: Nursing Process: Assessment MSC: Client Needs: Health Promotion and Maintenance 22. The nurse is observing a parent holding a preterm infant. The infant is sneezing, yawning, and extending the arms and legs. What action by the nurse is best? a. Cover the infant with a warmed blanket. b. Encourage the parent to do kangaroo care. c. Encourage the parent to place the infant back in the warmer d. Have the parent fold the infant's arms across the chest. ANS: C These are signs that the preterm infant is overstimulated. The parent should place the infant back in her warmer, and the nurse can turn down the lights and limit noise. The other suggestions will not help decrease stimulation. PTS: 1 DIF: Cognitive Level: Application/Applying REF: p. 626 | Safety Alert Box OBJ: Nursing Process: Implementation MSC: Client Needs: Physiologic Integrity 23. A nurse is caring for a preterm infant who has a weak cry and is irritable. What action by the nurse is best? a. Assess the infant for pain. b. Take the infant's temperature. c. Obtain a bedside glucose reading. d. Reduce stimulation in the environment. ANS: B These are signs of inadequate thermoregulation. The nurse should assess the infant's temperature first. The other actions do not address thermoregulation. PTS: 1 DIF: Cognitive Level: Application/Applying REF: p. 622 | Safety Alert Box OBJ: Nursing Process: Assessment MSC: Client Needs: Physiologic Integrity MULTIPLE RESPONSE

4. Which finding in the urine analysis of a pregnant woman is considered a variation of normal? a. Proteinuria b. Glycosuria c. Bacteria d. Ketonuria

B Small amounts of glucose may indicate "physiologic spilling," which occurs because the filtered load exceeds the renal tubules' ability to absorb them. The presence of protein could indicate kidney disease or preeclampsia. Urinary tract infections are associated with bacteria in the urine. An increase in ketones indicates that the patient is exercising too strenuously or has an inadequate fluid and food intake. PTS: 1 DIF: Cognitive Level: Knowledge/Remembering REF: p. 219 OBJ: Nursing Process: Assessment MSC: Client Needs: Physiologic Integrity

4. A new mother recalls that she should feed her newborn when she exhibits feeding readiness cues rather than waiting until her infant is crying frantically. Based on this information, this woman should feed her infant when she a. waves her arms in the air. b. makes sucking motions. c. has hiccups. d. stretches out her legs straight.

B Sucking motions, rooting, mouthing, and hand-to-mouth motions are examples of feeding-readiness cues. The other observations are not feeding cues. PTS: 1 DIF: Cognitive Level: Comprehension/Understanding REF: p. 485 | Box 23.3 OBJ: Nursing Process: Planning MSC: Client Needs: Health Promotion and Maintenance

4. The nursing students learn that fertilization of the ovum takes place in which part of the fallopian tube? a. Interstitial portion b. Ampulla c. Isthmus d. Infundibulum

B The ampulla is the wider middle part of the tube lateral to the isthmus and is where fertilization occurs. It does not occur in the interstitial portion, isthmus, or infundibulum. PTS: 1 DIF: Cognitive Level: Knowledge/Remembering REF: p. 188 OBJ: Integrated Process: Teaching-Learning MSC: Client Needs: Physiologic Integrity

3. Prior to cesarean birth, the nurse places the indwelling catheter and tubing in which manner? a. Placed on the floor on a sterile drape b. Placed near the head of the table c. Clamped during the cesarean section d. Positioned at the foot of the bed

B The anesthesia clinician must monitor urine output during the surgery so the catheter and tubing are placed near the head of the table. They are not placed on the floor, even on a sterile drape. The catheter is not clamped because urinary output must be monitored continuously. An early sign of hypovolemia is a decreasing urinary output. PTS: 1 DIF: Cognitive Level: Application/Applying REF: p. 393 OBJ: Nursing Process: Implementation MSC: Client Needs: Safe and Effective Care Environment

7. A 17-year-old tells the nurse that he is not having sex because it would make his parents very angry. This response indicates that the adolescent has a developmental lag in which area? a. Cognitive development b. Moral development c. Psychosocial development d. Psychosexual development

B The appropriate moral development for a 17-year-old would include evidence that the teenager has internalized a value system and does not depend on parents to determine right and wrong behaviors. Adolescents who remain concrete thinkers may never advance beyond conformity to please others and avoid punishment. Cognitive development is related to moral development, but it is not the pivotal point in determining right and wrong behaviors. Identity formation is the psychosocial development task. Energy is focused within the adolescent, who exhibits behavior that is self-absorbed and egocentric. Although a task during adolescence is the development of a sexual identity, the teenager's dependence on the parents' sanctioning of right or wrong behavior is more appropriately related to moral development. PTS: 1 DIF: Cognitive Level: Analysis/Analyzing REF: p. 158 OBJ: Nursing Process: Evaluation MSC: Client Needs: Health Promotion and Maintenance

7. The nurse teaching a prenatal class explains that which is the best indicator of true labor? a. Bloody show b. Cervical dilation and effacement c. Fetal descent into the pelvic inlet d. Uterine contractions every 7 minutes

B The conclusive distinction between true and false labor is that contractions of true labor cause progressive change in the cervix. Bloody show can occur before true labor. Fetal descent can occur before true labor. False labor may have contractions that occur this frequently, but it is usually inconsistent. PTS: 1 DIF: Cognitive Level: Comprehension/Understanding REF: p. 298 OBJ: Integrated Process: Teaching-Learning MSC: Client Needs: Health Promotion and Maintenance

2. After giving birth the nurse suggests that the woman place the infant to her breast within 15 minutes. The nurse knows that breastfeeding is effective during the first 30 minutes after birth because this is the a. transition period. b. first period of reactivity. c. organizational stage. d. second period of reactivity.

B The first period of reactivity is the first phase of transition and lasts up to 30 minutes after birth. The infant is highly alert during this phase. The transition period is the phase between intrauterine and extrauterine existence. There is no such phase as the organizational stage. The second period of reactivity occurs roughly between 4 and 8 hours after birth, after a period of prolonged sleep. PTS: 1 DIF: Cognitive Level: Comprehension/Understanding REF: p. 435 OBJ: Nursing Process: Implementation MSC: Client Needs: Health Promotion and Maintenance

2. A new father wants to know what medication was put into his infant's eyes and why it is needed. The nurse explains to the father that the purpose of the ophthalmic ointment is to a. destroy an infectious exudate caused by Staphylococcus that could make the infant blind. b. prevent gonorrheal and chlamydial infection of the infant's eyes potentially acquired from the birth canal. c. prevent potentially harmful exudate from invading the tear ducts of the infant's eyes, leading to dry eyes. d. prevent the infant's eyelids from sticking together and help the infant see.

B The ointment is used to prevent potential gonorrheal and chlamydial infection of the infant's eyes. PTS: 1 DIF: Cognitive Level: Comprehension/Understanding REF: p. 462 OBJ: Integrated Process: Teaching-Learning MSC: Client Needs: Physiologic Integrity

9. If the fundus is palpated on the right side of the abdomen above the expected level, the nurse should suspect that the patient has a. been lying on her right side too long. b. a distended bladder. c. stretched ligaments that are unable to support the uterus. d. a normal involution.

B The presence of a full bladder will displace the uterus. This finding does not signify a problem with positioning or ligaments, nor is it an expected finding. PTS: 1 DIF: Cognitive Level: Knowledge/Remembering REF: p. 403 OBJ: Nursing Process: Assessment MSC: Client Needs: Health Promotion and Maintenance 10. A woman gave birth vaginally to a 9-pound, 12-ounce girl yesterday. Her primary health care provider has written orders for perineal ice packs, use of a sitz bath tid, and a stool softener. What information is most closely correlated with these orders? a. The woman is a gravida 2, para 2. b. The woman had a vacuum-assisted birth. c. The woman received epidural anesthesia. d. The woman has an episiotomy. ANS: D These orders are typical interventions for a woman who has had an episiotomy, lacerations, and hemorrhoids. A multiparous classification is not an indication for these orders. A vacuum-assisted birth may be used in conjunction with an episiotomy, which indicates these interventions, but that is not the only situation in which an episiotomy would be used, so this is not the best answer. Use of epidural anesthesia has no correlation with these orders. PTS: 1 DIF: Cognitive Level: Comprehension/Understanding REF: p. 397 OBJ: Nursing Process: Planning MSC: Client Needs: Health Promotion and Maintenance 11. Rho immune globulin will be ordered postpartum if which situation occurs? a. Mother Rh-, baby Rh+ b. Mother Rh-, baby Rh- c. Mother Rh+, baby Rh+ d. Mother Rh+, baby Rh- ANS: A An Rh- mother delivering an Rh+ baby may develop antibodies to fetal cells that entered her bloodstream when the placenta separated. The Rho immune globulin works to destroy the fetal cells in the maternal circulation before sensitization occurs. The other blood type combinations would not necessitate the use of Rhogam. PTS: 1 DIF: Cognitive Level: Comprehension/Understanding REF: p. 400 OBJ: Nursing Process: Assessment MSC: Client Needs: Physiologic Integrity 12. If rubella vaccine is indicated for a postpartum patient, instructions to the patient should include a. drinking plenty of fluids to prevent fever. b. no specific instructions. c. recommending that she stop breastfeeding for 24 hours after injection. d. explaining the risks of becoming pregnant within 1 month after injection. ANS: D Potential risks to the fetus can occur if pregnancy results within 28 days after rubella vaccine administration. Drinking fluids will not prevent a fever. Small amounts of the vaccine do cross the breast milk, but it is believed that there is no need to discontinue breastfeeding. PTS: 1 DIF: Cognitive Level: Knowledge/Remembering REF: p. 400 OBJ: Nursing Process: Implementation MSC: Client Needs: Health Promotion and Maintenance 13. Which nursing action is most appropriate to correct a boggy uterus that is displaced above and to the right of the umbilicus? a. Notify the provider of an impending hemorrhage. b. Assess the blood pressure and pulse. c. Evaluate the lochia. d. Assist the patient in emptying her bladder. ANS: D Urinary retention can cause overdistention of the urinary bladder, which lifts and displaces the uterus. Nursing actions need to be implemented before notifying the provider. Blood pressure, pulse, and lochia are important to assess, but first the nurse assesses the bladder so corrective action can be taken if needed. PTS: 1 DIF: Cognitive Level: Application/Applying REF: p. 402 OBJ: Nursing Process: Implementation MSC: Client Needs: Health Promotion and Maintenance 14. When caring for a newly delivered woman, the nurse is aware that the best measure to prevent abdominal distention after a cesarean birth is a. rectal suppositories. b. early and frequent ambulation. c. tightening and relaxing abdominal muscles. d. providing carbonated beverages. ANS: B Activity can aid the movement of accumulated gas in the gastrointestinal tract so early, and frequent ambulation is the best option. Rectal suppositories can be helpful after distention occurs but do not prevent it. Tightening and relaxing the abdominal muscles is not related. Carbonated beverages may increase distention. PTS: 1 DIF: Cognitive Level: Knowledge/Remembering REF: p. 406 OBJ: Nursing Process: Planning MSC: Client Needs: Physiologic Integrity 15. What documentation on a woman's chart on postpartum day 14 indicates a normal involution process? a. Moderate bright red lochial flow b. Breasts firm and tender c. Fundus below the symphysis and not palpable d. Episiotomy slightly red and puffy ANS: C The fundus descends 1 cm/day, so by postpartum day 14 it is no longer palpable. The lochia should be changed by this day to serosa. Breasts are not part of the involution process. The episiotomy should not be red or puffy at this stage. PTS: 1 DIF: Cognitive Level: Knowledge/Remembering REF: p. 395 OBJ: Nursing Process: Assessment MSC: Client Needs: Physiologic Integrity 16. To assess fundal contraction 6 hours after cesarean delivery, the nurse should a. palpate forcefully through the abdominal dressing. b. gently palpate, applying the same technique used for vaginal deliveries. c. place hands on both sides of the abdomen and press downward. d. rely on assessment of lochial flow rather than palpating the fundus. ANS: B Assessment of the fundus is the same for both vaginal and cesarean deliveries; however, palpation should be gentle due to increased discomfort caused by the uterine incision. Forceful palpation should never be used. The top of the fundus, not the sides, should be palpated and massaged. The fundus should be palpated and massaged to prevent bleeding. PTS: 1 DIF: Cognitive Level: Knowledge/Remembering REF: p. 403 OBJ: Nursing Process: Assessment MSC: Client Needs: Health Promotion and Maintenance 17. The mother-baby nurse is able to recognize reciprocal attachment behavior. What does this refer to? a. The positive feedback an infant exhibits toward parents during the attachment process b. Behavior during the sensitive period when the infant is in the quiet alert stage c. Unidirectional behavior exhibited by the infant, initiated and enhanced by eye contact d. Behavior by the infant during the sensitive period to elicit feelings of "falling in love" from the parents ANS: A In this definition, reciprocal refers to the feedback from the infant during the attachment process. PTS: 1 DIF: Cognitive Level: Knowledge/Remembering REF: p. 412 | Nursing Quality Alert Box OBJ: Nursing Process: Assessment MSC: Client Needs: Health Promotion and Maintenance 18. The postpartum woman who continually repeats the story of her labor, delivery, and recovery experiences is a. providing others with her knowledge of events. b. making the birth experience "real." c. taking hold of the events leading to her labor and delivery. d. accepting her response to labor and delivery. ANS: B Reliving the birth experience makes the event real and helps the mother realize that the pregnancy is over and that the infant is born and is now a separate individual. She is in the taking-in phase, trying to make the birth experience seem real. This process meets her needs, not those of others. PTS: 1 DIF: Cognitive Level: Knowledge/Remembering REF: p. 413 OBJ: Nursing Process: Assessment MSC: Client Needs: Psychosocial Integrity 19. During which stage of role attainment do the parents become acquainted with their baby and combine parenting activities with cues from the infant? a. Anticipatory b. Formal c. Informal d. Personal ANS: B A major task of the formal stage of role attainment is getting acquainted with the infant. The anticipatory stage begins during the pregnancy when the parents choose a physician and attend childbirth classes. The informal stage begins once the parents have learned appropriate responses to their infant's cues. The personal stage is attained when parents feel a sense of harmony in their role. PTS: 1 DIF: Cognitive Level: Knowledge/Remembering REF: p. 413 OBJ: Nursing Process: Assessment MSC: Client Needs: Psychosocial Integrity 20. A nurse observes a mother on her first postpartum day sitting in bed while her newborn lies awake in the bassinet. What action by the nurse is best? a. Realize that this situation is perfectly acceptable. b. Offer to hand the baby to the woman. c. Hand the baby to the woman. d. Explain "taking in" to the woman. ANS: C During the "taking-in" phase of maternal adaptation, in which the mother may be passive and dependent, the nurse should encourage bonding when the infant is in the quiet alert stage. This is done best by simply giving the baby to the mother. While acceptable, the nurse can still facilitate infant bonding. The woman is dependent and passive at this stage and may have difficulty making a decision so offering her the baby is not the best option. Women learn best in the taking-hold phase. PTS: 1 DIF: Cognitive Level: Application/Applying REF: p. 413 OBJ: Nursing Process: Implementation MSC: Client Needs: Psychosocial Integrity 21. A nurse is observing a family. The mother is holding the baby she delivered less than 24 hours ago. Her husband is watching his wife and asking questions about newborn care. The 4-year-old brother is punching his mother on the back. What action by the nurse is best? a. Report the incident to the social services department. b. Advise the parents that the toddler needs to be reprimanded. c. Report to oncoming staff that the mother is not a good disciplinarian. d. Realize that this is a normal family adjusting to family change. ANS: D The observed behaviors are normal variations of families adjusting to change. The nurse could provide suggestions on managing the adjustments. There is no need to report this one incident. The child does not need to be reprimanded, however; when the family is receptive the nurse could provide anticipatory guidance for this situation and help them problem solve. The nurse should avoid labeling the parents. PTS: 1 DIF: Cognitive Level: Application/Applying REF: p. 416 OBJ: Nursing Process: Implementation MSC: Client Needs: Psychosocial Integrity 22. What is the best way for the nurse to promote and support the maternal-infant bonding process? a. Help the mother identify her positive feelings toward the newborn. b. Encourage the mother to provide all newborn care. c. Assist the family with rooming-in. d. Return the newborn to the nursery during sleep periods. ANS: C Close and frequent interaction between mother and infant, which is facilitated by rooming-in, is important in the bonding process. This is often referred to as the mother-baby care or couplet care. Having the mother express her feelings is important, but it is not the best way to promote bonding. The mother needs time to rest and recuperate; she should not be expected to do all of the care. The mother needs to observe the infant during all stages so she will be aware of what to expect when they go home. PTS: 1 DIF: Cognitive Level: Comprehension/Understanding REF: p. 419 OBJ: Nursing Process: Planning MSC: Client Needs: Psychosocial Integrity 23. During which phase of maternal adjustment will the mother relinquish the baby of her fantasies and accept the real baby? a. Letting go b. Taking hold c. Taking in d. Taking on ANS: A Accepting the real infant and relinquishing the fantasy infant occurs during the letting-go phase of maternal adjustment. During the taking-hold phase the mother assumes responsibility for her own care and shifts her attention to the infant. In the taking-in phase the mother is primarily focused on her own needs. There is no taking-on phase of maternal adjustment. PTS: 1 DIF: Cognitive Level: Knowledge/Remembering REF: p. 419 | Table 20.3 OBJ: Nursing Process: Assessment MSC: Client Needs: Psychosocial Integrity 24. A 25-year-old gravida 1 para 1 who had an emergency cesarean birth 3 days ago is scheduled for discharge. As the nurse prepares her for discharge, she begins to cry. What action should the nurse take first? a. Assess her for pain. b. Point out how lucky she is to have a healthy baby. c. Explain that she is experiencing postpartum blues. d. Allow her time to express her feelings. ANS: D Many women experience transient postpartum blues and need assistance in expressing their feelings. This condition affects 70% to 80% of new mothers. The nurse should allow time for the new mother to express herself. The nurse should not assume she is in pain at this point. Pointing out how lucky she is belittles her feelings. Patient teaching can be done later. PTS: 1 DIF: Cognitive Level: Application/Applying REF: pp. 415-416 OBJ: Nursing Process: Implementation MSC: Client Needs: Psychosocial Integrity 25. A man calls the nurse's station stating that his wife, who delivered 2 days ago, is happy one minute and crying the next. The man says, "She was never like this before the baby was born." What response by the nurse is best? a. Tell him to ignore the mood swings, as they will go away. b. Reassure him that this behavior is normal. c. Advise him to get immediate psychological help for her. d. Instruct him in the signs, symptoms, and duration of postpartum blues. ANS: B Before providing further instructions, inform family members of the fact that postpartum blues are a normal process to allay anxieties and increase receptiveness to learning. Telling him the mood swings will go away is belittling his concerns. Postpartum blues are a normal process that is short lived; no medical intervention is needed. Client teaching is important; however, his anxieties need to be allayed before he will be receptive to teaching. PTS: 1 DIF: Cognitive Level: Application/Applying REF: pp. 415-416 OBJ: Nursing Process: Implementation MSC: Client Needs: Psychosocial Integrity 26. To promote bonding and attachment immediately after delivery, what action by the nurse is most important? a. Allow the mother quiet time with her infant. b. Assist the mother in assuming an en face position with her newborn. c. Teach the mother about the concepts of bonding and attachment. d. Assist the mother in feeding her baby. ANS: B Assisting the mother in assuming an en face position with her newborn will support the bonding process. Quiet time with the infant is helpful but not as important as en face positioning. The mother has just delivered and is more focused on the infant; she will not be receptive to teaching at this time. This is a good time to initiate breastfeeding, but this is not as specific to bonding and attachment as the en face position. PTS: 1 DIF: Cognitive Level: Application/Applying REF: p. 420 OBJ: Nursing Process: Implementation MSC: Client Needs: Health Promotion and Maintenance 27. In providing support to a new mother who must return to full-time employment 6 weeks after a vaginal delivery, which action by the nurse is best? a. Allow her to express her positive and negative feelings freely. b. Reassure her that she'll get used to leaving her baby. c. Discuss child care arrangements with her. d. Allow her to solve the problem on her own. ANS: A Allowing the patient to express feelings will provide positive support in her process of maternal adjustment. Simply reassuring the mother blocks further communication and belittles her feelings. Discussing child care arrangements should wait until she has expressed herself. She should be instrumental in solving the problem; however, allowing her time to express her feelings and talk the problem over will assist her in making this decision. PTS: 1 DIF: Cognitive Level: Application/Application REF: p. 415 OBJ: Nursing Process: Implementation MSC: Client Needs: Psychosocial Integrity 28. A new father states, "I know nothing about babies," but he seems to be interested in learning. What action by the nurse is best? a. Continue to observe his interaction with the newborn. b. Tell him when he does something wrong. c. Show no concern, as he will learn on his own. d. Include him in teaching sessions. ANS: D The nurse must be sensitive to the father's needs and include him whenever possible. As fathers take on care new role, the nurse should praise every attempt even if his early care is awkward. It is important to note the bonding process of the mother and the father, but that does not satisfy the expressed needs of the father. He should be encouraged by pointing out the correct procedures he does. Criticizing him will discourage him. The nurse should be sure to include him in all teaching sessions. PTS: 1 DIF: Cognitive Level: Application/Applying REF: p. 422 OBJ: Nursing Process: Implementation MSC: Client Needs: Health Promotion and Maintenance 29. A 25-year-old multiparous woman gave birth to an infant boy 1 day ago. Today her husband brings a large container of brown seaweed soup to the hospital. When the nurse enters the room, the husband asks for help with warming the soup so that his wife can eat it. The nurse's most appropriate response is to ask the woman a. "Didn't you like your lunch?" b. "Does your doctor know that you are planning to eat that?" c. "What is that anyway?" d. "I'll warm the soup in the microwave for you." ANS: D This statement shows cultural sensitivity to the dietary preferences of the woman and is the most appropriate response. Cultural dietary preferences must be respected. Women may request that family members bring favorite or culturally appropriate foods to the hospital. Asking if the provider knows she is eating this soup is insensitive. PTS: 1 DIF: Cognitive Level: Application/Applying REF: p. 418 OBJ: Integrated Process: Culture and Spirituality MSC: Client Needs: Psychosocial Integrity 30. A postpartum woman is unable to empty her bladder. What intervention would the nurse try last? a. Pouring water from a squeeze bottle over the woman's perineum b. Providing hot tea c. Asking the physician to prescribe analgesics d. Inserting a sterile catheter ANS: D Invasive procedures usually are the last to be tried, especially with so many other simple methods available. Pouring water over the perineum may stimulate voiding. It is easy, noninvasive, and should be tried early on. Hot tea or other fluids ad lib is an easy, noninvasive strategy that should be tried early on. If the woman is anticipating pain from voiding, pain medications may be helpful. Other nonmedical means could be tried first, but medications still come before insertion of a catheter. PTS: 1 DIF: Cognitive Level: Comprehension/Understanding REF: p. 405 OBJ: Nursing Process: Implementation MSC: Client Needs: Physiologic Integrity 31. The nurse caring for the postpartum woman understands that breast engorgement is caused by a. overproduction of colostrum. b. accumulation of milk in the lactiferous ducts and glands. c. hyperplasia of mammary tissue. d. congestion of veins and lymphatics. ANS: D Breast engorgement is caused by the temporary congestion of veins and lymphatics, not overproduction of colostrum, accumulation of milk, or hyperplasia. PTS: 1 DIF: Cognitive Level: Knowledge/Remembering REF: p. 404 OBJ: Nursing Process: Assessment MSC: Client Needs: Physiologic Integrity 32. Which hormone remains elevated in the immediate postpartum period of the breastfeeding woman? a. Estrogen b. Progesterone c. Prolactin d. Human placental lactogen ANS: C Prolactin levels in the blood increase progressively throughout pregnancy. In women who breastfeed, prolactin levels remain elevated into the sixth week after birth. Estrogen and progesterone levels decrease markedly after expulsion of the placenta, reaching their lowest levels 1 week into the postpartum period. Human placental lactogen levels dramatically decrease after expulsion of the placenta. PTS: 1 DIF: Cognitive Level: Knowledge/Remembering REF: p. 400 OBJ: Nursing Process: Assessment MSC: Client Needs: Physiologic Integrity 33. The nurse explains to the nursing student that one mechanism for the diaphoresis and diuresis experienced during the early postpartum period is which of the following? a. Elevated temperature caused by postpartum infection b. Increased basal metabolic rate after giving birth c. Loss of increased blood volume associated with pregnancy d. Increased venous pressure in the lower extremities ANS: C Within 12 hours of birth, women begin to lose the excess tissue fluid that has accumulated during pregnancy. One mechanism for reducing these retained fluids is the profuse diaphoresis that often occurs, especially at night, for the first 2 or 3 days after childbirth. Postpartal diuresis is another mechanism by which the body rids itself of excess fluid. An elevated temperature causes chills and may cause dehydration, not diaphoresis and diuresis. Diaphoresis and diuresis are not caused by an increase in the basal metabolic rate. Postpartal diuresis may be caused by the removal of increased venous pressure in the lower extremities. PTS: 1 DIF: Cognitive Level: Comprehension/Understanding REF: p. 397 OBJ: Integrated Process: Teaching-Learning MSC: Client Needs: Physiologic Integrity 34. Which condition seen in the postpartum period is likely to require careful medical assessment? a. Varicosities of the legs b. Carpal tunnel syndrome c. Periodic numbness and tingling of the fingers d. Headaches ANS: D Headaches in the postpartum period can have a number of causes, some of which deserve medical attention. Varicosities are common. Carpal tunnel syndrome is relieved in childbirth when the compression on the median nerve is lessened. Periodic numbness of the fingers usually disappears after birth unless carrying the baby aggravates the condition. PTS: 1 DIF: Cognitive Level: Knowledge/Remembering REF: p. 399 OBJ: Nursing Process: Evaluation MSC: Client Needs: Physiologic Integrity 35. A nurse has taught a woman how to do Kegel exercises. What statement by the patient shows good understanding? a. "I contract my thighs, buttocks, and abdomen." b. "I do 10 of these exercises every day." c. "I stand while practicing this new exercise routine." d. "I pretend that I am trying to stop the flow of urine midstream." ANS: D The woman can pretend that she is attempting to stop the passing of gas, or the flow of urine midstream. This will replicate the sensation of the muscles drawing upward and inward. Each contraction should be as intense as possible without contracting the abdomen, buttocks, or thighs. Guidelines suggest that these exercises should be done 24 to 100 times per day. Positive results are shown with a minimum of 24 to 45 repetitions per day. The best position to learn Kegel exercises is to lie supine with knees bent. A secondary position is on the hands and knees. PTS: 1 DIF: Cognitive Level: Evaluation/Evaluating REF: p. 407 OBJ: Nursing Process: Evaluation MSC: Client Needs: Health Promotion and Maintenance: Self-Care 36. A nurse is examining a woman 2 months after delivery. The woman has lost 25 pounds. What action by the nurse is best? a. Counsel her on other weight loss measures. b. Ask her for a dietary recall for 3 days. c. Instruct her on exercises for faster loss. d. Explain that her weight loss is affecting her breast milk. ANS: B This woman has lost too much weight for being 8 weeks postpartum. Gradual weight loss is recommended, so the nurse should first assess the woman's eating habits by conducting a nutrition history. From that information the nurse can help the woman plan a safer weight loss plan. She does not need to lose weight faster, so counseling her on weight loss measures or more exercise is not beneficial. Telling her she is harming her baby is not therapeutic and may make her less likely to listen to the nurse. PTS: 1 DIF: Cognitive Level: Application/Applying REF: p. 415 OBJ: Nursing Process: Assessment MSC: Client Needs: Health Promotion and Maintenance 37. A nurse has taught a woman and partner about measures to improve sexuality after childbirth. Which statement by the partner demonstrates a need for further teaching? a. "We will use water-soluble lubricant before intercourse." b. "We can try having sex in the morning when we are rested." c. "Breastfeeding before sex will increase vaginal lubrication." d. "My wife will be more comfortable if she is on top." ANS: C Breastfeeding just prior to intercourse may allow uninterrupted time while the baby sleeps afterward, although it will not increase vaginal lubrication. It also decreases the chance of leaking milk. The other statements show good understanding. PTS: 1 DIF: Cognitive Level: Evaluation/Evaluating REF: p. 409 OBJ: Nursing Process: Evaluation MSC: Client Needs: Psychosocial Integrity MULTIPLE RESPONSE

8. The maternity nurse understands that vascular volume increases 40% to 60% during pregnancy to a. compensate for decreased renal plasma flow. b. provide adequate perfusion of the placenta. c. eliminate metabolic wastes of the mother. d. prevent maternal and fetal dehydration.

B The primary function of increased vascular volume is to transport oxygen and nutrients to the fetus via the placenta. Renal plasma flow increases during pregnancy. Assisting with pulling metabolic wastes from the fetus for maternal excretion is one purpose of the increased vascular volume. However, this answer is not the best because it doesn't explain the overall purpose and only includes one purpose. Prevention of dehydration is not the reason for increased vascular volume. PTS: 1 DIF: Cognitive Level: Knowledge/Remembering REF: p. 216 OBJ: Nursing Process: Assessment MSC: Client Needs: Physiologic Integrity

8. The step of the nursing process in which the nurse determines the appropriate interventions for the identified nursing diagnosis is called a. assessment. b. planning. c. intervention. d. evaluation.

B The third step in the nursing process involves planning care for problems that were identified during assessment. The first step of the nursing process is assessment, during which data are collected. The intervention phase is when the plan of care is carried out. The evaluation phase is determining whether the goals have been met. PTS: 1 DIF: Cognitive Level: Knowledge/Remembering REF: pp. 30-31 OBJ: Nursing Process: Planning MSC: Client Needs: Safe and Effective Care Environment

5. The nurse is assessing a toddler's growth and development. Which statement does the nurse understand about language development in a toddler? a. Language development skills slow during the toddler period. b. The toddler understands more than he or she can express. c. Most of the toddler's speech is not easily understood. d. The toddler's vocabulary contains approximately 600 words.

B The toddler's ability to understand language (receptive language) exceeds the child's ability to speak it (expressive language). Although language development varies in relationship to physical activity, language skills are rapidly accelerating by 15 to 24 months of age. By 2 years of age, 60% to 70% of the toddler's speech is understandable. The toddler's vocabulary contains approximately 300 or more words. PTS: 1 DIF: Cognitive Level: Knowledge/Remembering REF: p. 112 OBJ: Nursing Process: Assessment MSC: Client Needs: Health Promotion and Maintenance

5. The nursing faculty explains to students that the upper uterus is the best place for the fertilized ovum to implant because it is here that the a. placenta attaches most firmly. b. developing baby is best nourished. c. uterine endometrium is softer. d. maternal blood flow is lower.

B The uterine fundus is richly supplied with blood and has the thickest endometrium, both of which promote optimal nourishment of the fetus. If the placenta attaches too deeply, it does not easily detach after birth. Softness is not a concern with implantation; attachment and nourishment are the major concerns. The blood supply is rich in the fundus, which allows for optimal nourishment of the fetus. PTS: 1 DIF: Cognitive Level: Comprehension/Understanding REF: p. 199 OBJ: Integrated Process: Teaching-Learning MSC: Client Needs: Physiologic Integrity

8. Why is continuous electronic fetal monitoring usually used when oxytocin is administered? a. The mother may become hypotensive. b. Uteroplacental exchange may be compromised. c. Maternal fluid volume deficit may occur. d. Fetal chemoreceptors are stimulated.

B The uterus may contract more firmly, and the resting tone may be increased with oxytocin use. This response reduces entrance of freshly oxygenated maternal blood into the intervillous spaces, depleting fetal oxygen reserves. Hypotension is not a common side effect of oxytocin. All laboring women are at risk for fluid volume deficit; oxytocin administration does not increase the risk. Oxytocin affects the uterine muscles. PTS: 1 DIF: Cognitive Level: Comprehension/Understanding REF: p. 333 OBJ: Nursing Process: Planning MSC: Client Needs: Physiologic Integrity

2. A woman in labor at 34 weeks of gestation is hospitalized and treated with intravenous magnesium sulfate for 18 to 20 hours. When the magnesium sulfate is discontinued, which oral drug will probably be prescribed for continuation of the tocolytic effect? a. Ritodrine b. Terbutaline c. Calcium gluconate d. Pitocin

B The woman receiving decreasing doses of magnesium sulfate is often switched to oral terbutaline to maintain tocolysis for 48 hours. The terbutaline will probably be discontinued prior to discharge. Ritodrine is the only drug approved by the FDA for tocolysis; however, it is rarely used because of significant side effects. Calcium gluconate reverses magnesium sulfate toxicity. The drug should be available for complications of magnesium sulfate therapy. Pitocin is used to augment labor, not stop it. PTS: 1 DIF: Cognitive Level: Knowledge/Remembering REF: p. 588 OBJ: Nursing Process: Planning MSC: Client Needs: Physiologic Integrity

6. A neonate has white patches in her mouth that bled when the mother tried wiping them away. What action by the nurse is best? a. Tell the mother to leave the patches alone. b. Assess the mother for a perineal rash. c. Give the infant medicated pacifiers. d. Test the infant for toxoplasmosis.

B These patches are characteristic of maternal infection with candidiasis or yeast. The nurse assesses the mother's perineal area for a rash. Telling the mother to leave the rash alone may be appropriate information but does not get to the bottom of the issue. The nurse should not provide medication without knowing what is being treated. The baby does not have toxoplasmosis. PTS: 1 DIF: Cognitive Level: Application/Applying REF: p. 650 | Table 30.1 OBJ: Nursing Process: Implementation MSC: Client Needs: Physiologic Integrity

8. A premature infant never seems to sleep longer than an hour at a time. Each time a light is turned on, an incubator closes, or people talk near her crib, she wakes up and cries inconsolably until held. The correct nursing diagnosis is ineffective coping related to a. severe immaturity. b. environmental stress. c. physiologic distress. d. behavioral responses.

B This nursing diagnosis is the most appropriate for this infant. Light and sound are known adverse stimuli that add to an already stressed premature infant. The nurse must monitor the environment closely for sources of overstimulation. The other diagnoses do not recognize that fact. PTS: 1 DIF: Cognitive Level: Application/Applying REF: p. 626 OBJ: Nursing Process: Diagnosis MSC: Client Needs: Safe and Effective Care Environment

2. The karyotype of a person is 47, XY, +21. This person is a a. normal male. b. male with Down syndrome. c. normal female. d. female with Turner syndrome.

B This person is male because his sex chromosomes are XY. He has one extra copy of chromosome 21 (for a total of 47 instead of 46), resulting in Down syndrome. A normal male has 46 chromosomes. A normal female has 46 chromosomes and XX for the sex chromosomes. A female with Turner syndrome has 45 chromosomes; the sex chromosomes have just one X. PTS: 1 DIF: Cognitive Level: Knowledge/Remembering REF: p. 175 OBJ: Nursing Process: Assessment MSC: Client Needs: Physiologic Integrity

4. Four women are admitted to Labor and Delivery. Which woman met the goal for a healthy weight gain in pregnancy? a. 17 years old, 5′2″ tall, initial weight 116 pounds, today's weight 120 pounds b. 22 years old, 5′2″ tall, initial weight 230 pounds, today's weight 245 pounds c. 24 years old, 5′3″ tall, initial weight 135, today's weight 182 pounds d. 27 years old, 5′6″ tall, initial weight 112 pounds, today's weight 135 pounds

B This woman was obese at the start of her pregnancy, so a weight gain of 11 to 20 pounds has met the goal (245 − 230 = 15). Adolescents need to gain enough weight to support both their needs and those of the fetus, so they should gain the recommended amount for normal weight women, so this teen should weigh between 127 and 136, so she clearly did not gain enough weight. The woman who weighed a healthy 135 pounds should not weigh more than 170 pounds, so this woman gained more weight than recommended. The woman who was 5′6″ tall was underweight at conception, so she needed to gain 28 to 40 pounds, which would put her minimum acceptable weight at delivery at 140. PTS: 1 DIF: Cognitive Level: Analysis/Analyzing REF: Table 14.1 OBJ: Nursing Process: Assessment MSC: Client Needs: Health Promotion and Maintenance

8. What is the most appropriate response for the nurse to make to the parent of a 3-year-old child found in a bed with the side rails down? a. "You must never leave the child in the room alone with the side rails down." b. "I am very concerned about your child's safety when you leave the side rails down." c. "It is hospital policy that side rails need to be up if the child is in bed." d. "When parents leave side rails down, they might be considered as uncaring."

B To express concern and then choose words that convey a policy without appearing to cast blame on improper behavior is appropriate. Framing the communication in the negative does not facilitate effective communication. Stating a policy to parents conveys the attitude that the hospital has authority over parents in matters concerning their children and may be perceived negatively. It also does not give information as to why the side rails need to be up. This statement conveys blame and judgment to the parent. PTS: 1 DIF: Cognitive Level: Application/Applying REF: Table 4.2 OBJ: Integrated Process: Communication and Documentation MSC: Client Needs: Psychosocial Integrity

4. In terms of the incidence and classification of diabetes, maternity nurses should know that a. type 1 diabetes is most common. b. type 2 diabetes often goes undiagnosed. c. there is only one type of gestational diabetes. d. type 1 diabetes may become type 2 during pregnancy.

B Type 2 often goes undiagnosed, because hyperglycemia develops gradually and often is not severe. Type 2, previously called adult onset diabetes, is the most common. There are 2 subgroups of gestational diabetes. Type GDM A1 is diet-controlled whereas type GDM A2 is controlled by insulin and diet. People do not go back and forth between type 1 and type 2 diabetes. PTS: 1 DIF: Cognitive Level: Knowledge/Remembering REF: p. 550 OBJ: Nursing Process: Assessment MSC: Client Needs: Physiologic Integrity

7. A steady trickle of bright red blood from the vagina in the presence of a firm fundus suggests a. uterine atony. b. lacerations of the genital tract. c. perineal hematoma. d. infection of the uterus.

B Undetected lacerations will bleed slowly and continuously. Bleeding from lacerations is uncontrolled by uterine contraction. The fundus is not firm with uterine atony. A hematoma would be internal. Swelling and discoloration would be noticed, but bright bleeding would not be. With an infection of the uterus there would be an odor to the lochia and systemic symptoms such as fever and malaise. PTS: 1 DIF: Cognitive Level: Knowledge/Remembering REF: p. 602 OBJ: Nursing Process: Assessment MSC: Client Needs: Physiologic Integrity

2. An important consideration for the nurse who is communicating with a 5-year-old child is to a. speak loudly, clearly, and directly. b. use picture or story books, or puppets. c. disguise own feelings, attitudes, and anxiety. d. initiate contact with child when parent is not present.

B Using objects such as a puppet or doll allows the young child an opportunity to evaluate an unfamiliar person (the nurse). This will facilitate communication with a child of this age. Speaking in this manner will tend to increase anxiety in very young children as they may interpret this as being yelled at. The nurse must be honest with the child. Attempts at deception will lead to a lack of trust. Whenever possible, the parent should be present for interactions with young children. PTS: 1 DIF: Cognitive Level: Comprehension/Understanding REF: Table 4.3 OBJ: Integrated Process: Communication and Documentation MSC: Client Needs: Psychosocial Integrity

7. Cardiovascular changes that cause the foramen ovale to close at birth are a direct result of a. increased pressure in the right atrium. b. increased pressure in the left atrium. c. decreased blood flow to the left ventricle. d. changes in the hepatic blood flow.

B With the increase in the blood flow to the left atrium from the lungs, the pressure is increased, and the foramen ovale is functionally closed. The pressure in the right atrium decreases at birth. It is higher during fetal life. Blood flow increases to the left ventricle after birth. The hepatic blood flow changes, but that is not the reason for the closure of the foramen ovale. PTS: 1 DIF: Cognitive Level: Knowledge/Remembering REF: p. 427 OBJ: Nursing Process: Assessment MSC: Client Needs: Physiologic Integrity

5. A student asks how pregnant women can usually tolerate the normal blood loss associated with childbirth. Which response by the nurse is best? "It is because they have a. a higher hematocrit." b. increased blood volume." c. a lower fibrinogen level." d. increased leukocytes."

B Women have a significant increase in blood volume during pregnancy, which allows them to tolerate the normal blood loss seen in delivery. The hematocrit decreases with pregnancy due to the high fluid volume. Fibrinogen levels increase with pregnancy. Leukocyte levels increase during labor, but that is not the reason for the toleration of blood loss. PTS: 1 DIF: Cognitive Level: Comprehension/Understanding REF: p. 290 OBJ: Integrated Process: Teaching-Learning MSC: Client Needs: Physiologic Integrity

2. The nurse explains to parents that which organs are nonfunctional during fetal life? (Select all that apply.) a. Kidneys b. Lungs c. Liver d. Gastrointestinal system e. Adrenal glands

B, C Most of the fetal blood flow bypasses the nonfunctional lungs and liver. The other organs are functional during fetal life. PTS: 1 DIF: Cognitive Level: Comprehension/Understanding REF: p. 426 OBJ: Integrated Process: Teaching-Learning MSC: Client Needs: Physiologic Integrity COMPLETION

3. A preschool aged child is in the clinic for a well-child checkup. Which statement identifies an appropriate level of language development in this child? (Select all that apply.) a. Vocabulary of 300 words b. Relates elaborate tales c. Uses correct grammar in sentences d. Able to pronounce consonants clearly e. Expresses abstract thought

B, C The 4-year-old child is able to use correct grammar in sentence structure and can tell elaborate tales and stories. A vocabulary of 300 words is appropriate for a 2-year-old. The 4-year-old child typically has difficulty in pronouncing consonants. The use of language to express abstract thought is developmentally appropriate for the adolescent. PTS: 1 DIF: Cognitive Level: Knowledge/Remembering REF: p. 62 OBJ: Nursing Process: Assessment MSC: Client Needs: Health Promotion and Maintenance

3. A woman has several relatives who had gestational hypertension and wants to decrease her risk for it. What information does the nurse provide this woman? (Select all that apply.) a. There is no way to reduce risk factors for gestational hypertension. b. Losing weight before you get pregnant will help prevent it. c. Eating a diet high in protein and iron may help prevent it. d. The father contributes no risk factors for hypertension in pregnancy e. Waiting until you are 35 to get pregnant cuts the risk in half.

B, C There are many risk factors for gestational hypertension, including obesity and anemia. The woman can take action to address these factors prior to becoming pregnant. The father's risks include the first baby and having fathered other preeclamptic pregnancies. Maternal age >35 increases the risk. PTS: 1 DIF: Cognitive Level: Comprehension/Understanding REF: p. 536 | Box 25.2 OBJ: Integrated Process: Teaching-Learning MSC: Client Needs: Physiologic Integrity Chapter 26: Concurrent Disorders during Pregnancy McKinney: Evolve Resources for Maternal-Child Nursing, 5th Edition MULTIPLE CHOICE

1. The causes of preterm labor are not fully understood although many factors have been associated with early labor. These include (Select all that apply.) a. Singleton pregnancy b. History of cone biopsy c. Smoking d. Short cervical length e. Higher level of education

B, C, D A history of cone biopsies, smoking, and a short cervical length are all associated with early labor. Singleton pregnancy and higher level of education are not. PTS: 1 DIF: Cognitive Level: Knowledge/Remembering REF: p. 583 | Table 27.2 OBJ: Nursing Process: Assessment MSC: Client Needs: Health Promotion and Maintenance

1. What are modes of heat loss in the newborn? (Select all that apply.) a. Perspiration b. Convection c. Radiation d. Conduction e. Urination

B, C, D Convection, radiation, evaporation, and conduction are the four modes of heat loss in the newborn. Perspiration and urination are not modes of heat loss in newborns. PTS: 1 DIF: Cognitive Level: Knowledge/Remembering REF: p. 427 OBJ: Nursing Process: Assessment MSC: Client Needs: Health Promotion and Maintenance

1. Some nipple conditions make it necessary to provide intervention before birth in the mother who plans to breastfeed. These include (Select all that apply.) a. Everted nipples b. Flat nipples c. Inverted nipples d. Nipples that contract when compressed e. Cracked nipples

B, C, D Flat nipples appear soft, like the areola, and do not stand erect unless stimulated by rolling them between the fingers. Inverted nipples are retracted into the breast tissue. These nipples appear normal; however, they will draw inward when the areola is compressed by the infant's mouth. Dome-shaped devices known as breast shells can be worn during the last weeks of pregnancy and between feedings after birth. The shells are placed inside the bra with the opening over the nipple. The shells exert slight pressure against the areola to help the nipples protrude. The helpfulness of breast shells is debated. A breast pump can be used to draw the nipples out before feedings after delivery. Everted nipples protrude and are normal. No intervention will be required. Cracked, blistered, and bleeding nipples occur after breastfeeding has been initiated and are the result of improper latch. PTS: 1 DIF: Cognitive Level: Knowledge/Remembering REF: pp. 484-485 OBJ: Nursing Process: Assessment MSC: Client Needs: Health Promotion and Maintenance

1. When providing anticipatory guidance to parents regarding disciplining children, the nurse teaches that behavioral consequences fall into which categories? (Select all that apply.) a. Corporal b. Natural c. Logical d. Unrelated e. Behavioral

B, C, D Natural consequences are those that occur spontaneously. For example, a child leaves a toy outside and it is lost. Logical consequences are those that are directly related to the misbehavior. If two children are fighting over a toy, the toy is removed and neither child has it. Unrelated consequences are purposely imposed; for example, the child is late for dinner so he or she is not allowed to watch television. Corporal punishment is not part of this behavioral approach and usually takes the approach of spanking the child. Corporal punishment is highly controversial and is strongly discouraged by the American Academy of Pediatrics. Behavior modification is another disciplinary technique that rewards positive behavior and ignores negative behavior. PTS: 1 DIF: Cognitive Level: Knowledge/Remembering REF: p. 45 OBJ: Integrated Process: Teaching-Learning MSC: Client Needs: Health Promotion and Maintenance

1. A patient at 34 weeks of gestation has reported to the OB triage unit for assessment of oligohydramnios. The nurse assigned to care for this patient is aware that prolonged oligohydramnios may result in (Select all that apply.) a. intrauterine limb amputations. b. clubfoot. c. delayed lung development. d. other fetal abnormalities. e. fetal deformations.

B, C, D Oligohydramnios, an abnormally small volume of amniotic fluid, reduces the cushion surrounding the fetus and may result in deformations such as clubfoot. Prolonged oligohydramnios interferes with fetal lung development because it does not allow normal development of the alveoli. Oligohydramnios may not be the primary fetal problem but rather may be related to other fetal anomalies. This does not lead to intrauterine limb amputations or fetal deformations. PTS: 1 DIF: Cognitive Level: Knowledge/Remembering REF: p. 178 OBJ: Nursing Process: Assessment MSC: Client Needs: Physiologic Integrity

2. What assessment findings indicate to the nurses that a woman's preeclampsia should now be considered severe? (Select all that apply.) a. Urine output 40 mL/hour for the past 2 hours b. Serum creatinine 3.1 mg/dL c. Seeing "sparkly" things in the visual field d. Crackles in both lungs e. Soft, non-tender abdomen

B, C, D Signs of severe preeclampsia include elevated creatinine, seeing sparkles, and pulmonary edema (manifested by crackles). The urine output is above the minimum requirements, and a soft non-tender abdomen is a reassuring sign. PTS: 1 DIF: Cognitive Level: Knowledge/Remembering REF: p. 538 | Table 25.3 OBJ: Nursing Process: Assessment MSC: Client Needs: Physiologic Integrity

4. The school nurse is presenting information on some risks of tattoos. What information should the nurse provide? (Select all that apply.) a. Amateur tattoos are difficult to remove. b. Tattoos pose a risk for bloodborne and skin infections. c. A tattoo may keep you from getting an MRI. d. Tattoo dyes may cause allergic reactions. e. Tattoo parlors are well regulated.

B, C, D Tattoos carry the risk for contracting bloodborne diseases such as hepatitis B and HIV. Infection, allergic reaction to the dye, scarring, or keloid formation can occur. Should an MRI ever be required, it is important to notify the health care professionals, because the dyes can contain iron and other metals. Amateur tattoos are easily removed; however, studio tattoos made with red and green dye are extremely difficult to remove. Very little regulation exists in the tattoo industry; therefore, the cleanliness of each tattoo parlor varies. Teens should be counseled to avoid making an impulsive decision to get a tattoo. PTS: 1 DIF: Cognitive Level: Application/Applying REF: pp. 164-165 OBJ: Integrated Process: Teaching-Learning MSC: Client Needs: Health Promotion and Maintenance

4. A nurse is planning care for a hospitalized toddler in the preoperational thinking stage. Which characteristics should the nurse expect in this stage? (Select all that apply.) a. Concrete thinking b. Egocentrism c. Animism d. Magical thought e. Ability to reason

B, C, D The characteristics of preoperational thinking that occur for the toddler include egocentrism (views everything in relation to self), animism (believes that inert objects are alive), and magical thought (believes that thinking something causes that event). Concrete thinking is seen in school-age children, and ability to reason is seen with adolescents. PTS: 1 DIF: Cognitive Level: Knowledge/Remembering REF: pp. 124-125 OBJ: Nursing Process: Assessment MSC: Client Needs: Health Promotion and Maintenance

2. Newborns whose mothers are substance abusers frequently have what behaviors? (Select all that apply.) a. Circumoral cyanosis b. Decreased amounts of sleep c. Hyperactive Moro (startle) reflex d. Difficulty feeding e. Weak cry

B, C, D The infant exposed to drugs in utero often has poor sleeping patterns, hyperactive reflexes, and uncoordinated sucking and swallowing behavior. They do not have circumoral cyanosis and will have a high-pitched cry. PTS: 1 DIF: Cognitive Level: Knowledge/Remembering REF: p. 654 OBJ: Nursing Process: Assessment MSC: Client Needs: Physiologic Integrity COMPLETION

1. Some infants develop hypoxic-ischemic encephalopathy after asphyxia. Therapeutic hypothermia has been used to improve neurologic outcomes for these infants. Criteria for the use of this modality include (Select all that apply.) a. The infant must be 28 weeks gestation or greater. b. Have evidence of an acute hypoxic event. c. Be in a facility they can initiate treatment within 6 hours. d. The infant must be 36 or more weeks' gestation. e. The treatment must be initiated within the first 12 hours of life.

B, C, D The infant must be at least 36 weeks of gestation to meet the criteria for therapeutic hypothermia. Treatment should be initiated within the first 6 hours of life, ideally at a tertiary care center. The infant must have evidence of perinatal hypoxic-ischemic episodes. PTS: 1 DIF: Cognitive Level: Comprehension/Understanding REF: p. 642 OBJ: Nursing Process: Assessment MSC: Client Needs: Physiologic Integrity

5. The increase in the number of overweight children in this country is addressed in Healthy People 2020. Strategies designed to approach this issue include (Select all that apply.) a. decreased calcium and iron intake. b. increased fiber and whole grain intake. c. decreased use of sugar and sodium. d. increase fruit and vegetable intake. e. decrease the use of solid fats.

B, C, D, E Along with these recommendations, children at risk for being overweight should be screened beginning at age 2 years. Children with a family history of dyslipidemia or early cardiovascular disease development, children whose body mass index percentile exceeds the definition for overweight, and children who have high blood pressure should have a fasting lipid screen. The nurse should instruct parents that calcium and iron intake should be increased as part of this strategy. PTS: 1 DIF: Cognitive Level: Knowledge/Remembering REF: p. 117 OBJ: Nursing Process: Planning MSC: Client Needs: Health Promotion and Maintenance

5. The student nurse learns that which factors place children at risk for malocclusion? (Select all that apply.) a. Sucking the thumb b. Mouth breathing c. Cleft palate d. Early loss of "baby" teeth e. Heredity

B, C, D, E Factors that contribute to malocclusion include mouth breathing, cleft palate, early loss of deciduous teeth, and heredity. Sucking the thumb is not a contributing factor unless it persists beyond 2 to 4 years. PTS: 1 DIF: Cognitive Level: Knowledge/Remembering REF: pp. 136-137 OBJ: Integrated Process: Teaching-Learning MSC: Client Needs: Health Promotion and Maintenance Chapter 09: Health Promotion for the Adolescent McKinney: Evolve Resources for Maternal-Child Nursing, 5th Edition MULTIPLE CHOICE

6. A nurse is planning for a sports pre-participation physical exam day. What goals for this event does the nurse set? (Select all that apply.) a. Comprehensive physical examination b. Assess general health c. Identify limiting conditions d. Provide wellness counseling e. Adhere to insurance requirements

B, C, D, E In a pre-participation sports examination, goals are to identify the teen's general health, identify any condition that would limit participation, provide wellness counseling, and ensure that participants meet insurance guidelines for participation. It is not meant to be a comprehensive physical examination. PTS: 1 DIF: Cognitive Level: Knowledge/Remembering REF: Box 9.1 OBJ: Nursing Process: Planning MSC: Client Needs: Safe and Effective Care Environment

1. Nurses must be aware of the conditions that increase the risk of hemorrhage, one of the most common complications of the puerperium. What are these conditions? (Select all that apply.) a. Primipara b. Rapid or prolonged labor c. Overdistention of the uterus d. Uterine fibroids e. Preeclampsia

B, C, D, E Rapid or prolonged labor, overdistention of the uterus, uterine fibroids, and preeclampsia are all risk factors for postpartum hemorrhage. Being a primipara is not a risk factor. PTS: 1 DIF: Cognitive Level: Knowledge/Remembering REF: p. 401 | Safety Alert Box OBJ: Nursing Process: Assessment MSC: Client Needs: Physiologic Integrity

1. A Labor and Delivery nurse knows that four of the five fetal factors that interact to regulate the heart rate are which of the following? (Select all that apply.) a. Uterine activity b. Autonomic nervous system c. Baroreceptors d. Chemoreceptors e. Adrenal glands

B, C, D, E The sympathetic and parasympathetic branches of the autonomic nervous system are balanced forces that regulate FHR. Sympathetic stimulation increases the heart rate, while parasympathetic responses, through stimulation of the vagus nerve, reduce the FHR and maintain variability. The baroreceptors stimulate the vagus nerve to slow the FHR and decrease the blood pressure. These are located in the carotid arch and major arteries. The chemoreceptors are cells that respond to changes in oxygen, carbon dioxide, and pH. They are found in the medulla oblongata and the aortic and carotid bodies. The adrenal medulla secretes epinephrine and norepinephrine in response to stress, causing an acceleration in FHR. Uterine activity is a maternal factor. PTS: 1 DIF: Cognitive Level: Knowledge/Remembering REF: p. 333 OBJ: Nursing Process: Assessment MSC: Client Needs: Physiologic Integrity

4. A nurse is teaching a parenting group about behavior modification. What information does this nurse include in teaching? (Select all that apply.) a. Food rewards are highly motivating and as such are encouraged. b. Negative behavior from the child should be ignored by parents. c. Undesirable behavior may initially get worse if it is ignored. d. 1 minute per age is the suggested time limit for discipline. e. For younger kids, a behavior chart is a good visual cue.

B, C, E In behavior modification parents ignore "bad" behavior by the child, which initially may get worse as the child tries to recapture the attention it once brought. Younger children respond positively to charts with stickers that show good behavior. Food rewards should not be used as food is an essential necessity plus extra food may contribute to obesity. The time limit refers to the time-out method of discipline. PTS: 1 DIF: Cognitive Level: Comprehension/Understanding REF: p. 45 OBJ: Integrated Process: Teaching-Learning MSC: Client Needs: Health Promotion and Maintenance Chapter 04: Communicating with Children and Families McKinney: Evolve Resources for Maternal-Child Nursing, 5th Edition MULTIPLE CHOICE

3. Which strategies can a nurse teach to parents of a child experiencing uncomplicated school refusal? (Select all that apply.) a. The child should be allowed to stay home until the anxiety about going to school is resolved. b. Parents should be empathetic yet firm in their insistence that the child attend school. c. A modified school attendance may be necessary. d. Parents need to pick the child up at school whenever the child wants to come home. e. Parents need to communicate with the teachers about the situation.

B, C, E In uncomplicated cases of school refusal, the parent needs to return the child to school as soon as possible. If symptoms are severe, a limited period of part-time or modified school attendance may be necessary. For example, part of the day may be spent in the counselor's or school nurse's office, with assignments obtained from the teacher. Parents should be empathetic yet firm and consistent in their insistence that the child attend school. Parents should not pick the child up at school once the child is there or let the child stay home until this issue is resolved. The principal and teacher should be told about the situation so that they can cooperate with the treatment plan. PTS: 1 DIF: Cognitive Level: Comprehension/Understanding REF: p. 143 OBJ: Integrated Process: Teaching-Learning MSC: Client Needs: Health Promotion and Maintenance

2. Approximately 82% of teen pregnancies are unintended. Seventy percent of teens have had sex by their 19th birthday. Factors that contribute to an increased risk for teen pregnancy include which of the following? (Select all that apply.) a. High self-esteem b. Peer pressure c. Limited access to contraception d. Planning sexual activity e. Lack of role models

B, C, E Peer pressure to begin sexual activity is a contributing factor toward teen pregnancy. Limited access to contraceptive devices and lack of accurate information about how to use these devices are also factors. Lack of appropriate role models, desire to alleviate or escape the present situation at home along with feelings of invincibility also contribute to teen pregnancy. Low self-esteem and the consequent inability to set limits on sexual activity place the adolescent at risk for teen pregnancy. Ambivalence toward sexuality and not planning intercourse are more likely to result in teen pregnancy. PTS: 1 DIF: Cognitive Level: Comprehension/Understanding REF: p. 500 OBJ: Nursing Process: Assessment MSC: Client Needs: Psychosocial Integrity Chapter 25: Pregnancy-Related Complications McKinney: Evolve Resources for Maternal-Child Nursing, 5th Edition MULTIPLE CHOICE

1. In planning care for a preschool-age child, the nurse knows that which open body postures encourage positive communication? (Select all that apply.) a. Leaning away from the preschooler b. Frequent eye contact c. Hands on hips d. Conversing at eye level e. Asking the parents to stay in the room

B, D Frequent eye contact and conversing at eye level are both open body postures that encourage positive communication. Leaning away from the child and placing your hands on your hips are both closed body postures that do not facilitate effective communication. Asking the parents to stay in the room while the nurse is talking to the child is helpful but is not an open body posture. PTS: 1 DIF: Cognitive Level: Knowledge/Remembering REF: Table 4.1 OBJ: Integrated Process: Communication and Documentation MSC: Client Needs: Psychosocial Integrity

2. An important nursing factor during the care of the infant in the NICU is assessment for signs of adequate parental attachment. The nurse must observe for signs that bonding is not occurring as expected. These include (Select all that apply.) a. using positive terms to describe the infant. b. showing interest in other infants equal to that of their own. c. naming the infant. d. decreasing the number and length of visits. e. refusing offers to hold and care for the infant.

B, D, E Bonding is not progressing as expected when parents show interest in other babies equal to that of their own, decreasing the number and length of visits, and refusing to hold and help care for the infant. Using positive terms to describe the baby and naming the infant are signs that bonding is occurring. PTS: 1 DIF: Cognitive Level: Knowledge/Remembering REF: p. 631 | Nursing Quality Alert Box OBJ: Nursing Process: Assessment MSC: Client Needs: Psychosocial Integrity Chapter 30: The High-Risk Newborn: Acquired and Congenital Conditions McKinney: Evolve Resources for Maternal-Child Nursing, 5th Edition MULTIPLE CHOICE

3. Parents of a teenager ask the nurse what signs they should look for if their child is in a gang. The nurse should include which signs when answering? (Select all that apply.) a. Plans to try out for the debate team at school b. Skipping classes to go to the mall c. Hanging out with friends they have had since childhood d. Unexplained source of money e. Fear of the police

B, D, E Signs of gang involvement include skipping classes, unexplained sources of money, and fear of the police. Associating with new friends while ignoring old friends is also a sign. A change in attitude toward participating in activities is another sign of gang involvement. Plans to become more involved in school activities and hanging around old friends are not signs. PTS: 1 DIF: Cognitive Level: Comprehension/Understanding REF: Box 9.3 OBJ: Nursing Process: Implementation MSC: Client Needs: Health Promotion and Maintenance

2. The nurse who uses critical thinking understands that the steps of critical thinking include (Select all that apply.) a. therapeutic communication. b. examining biases. c. setting priorities. d. managing data. e. evaluating other factors.

B, D, E The five steps of critical thinking include recognizing assumptions, examining biases, analyzing the need for closure, managing data, and evaluating other factors such as emotions and environmental factors. Therapeutic communication is a skill that nurses must have to carry out the many roles expected in the profession; however, it is not one of the steps of critical thinking. Setting priorities is part of the planning phase of the nursing process. PTS: 1 DIF: Cognitive Level: Knowledge/Remembering REF: p. 27 OBJ: Nursing Process: Planning |Nursing Process: Implementation MSC: Client Needs: Safe and Effective Care Environment

1. The nurse working with pregnant women understands that anorexia and bulimia are associated with which conditions in the newborn? (Select all that apply.) a. Food cravings b. Low birth weight c. Food aversions d. Electrolyte imbalance e. Small for gestational age infants

B, D, E These conditions are associated with electrolyte imbalance, low birth weight, and small for gestational age infants. All women should be asked about eating disorders, and nurses should watch for behaviors that may indicate disordered eating. Some women eat normally during pregnancy for the sake of the fetus, but others continue their previous dysfunctional eating patterns during pregnancy or in the early postpartum period. Food cravings and aversions are normal for most women during pregnancy. Women may have a strong preference or strong dislike for certain foods. They're generally not harmful, and some, like aversion to alcohol, may be beneficial. PTS: 1 DIF: Cognitive Level: Knowledge/Remembering REF: p. 267 OBJ: Nursing Process: Assessment MSC: Client Needs: Health Promotion and Maintenance

3. A new nurse to Labor and Delivery learns about the three categories of fetal heart rate patterns. Which characteristics of the fetal heart belong in Category III? (Select all that apply.) a. Baseline rate of 110 to 160 bpm b. Tachycardia c. Absent baseline variability NOT accompanied by recurrent decelerations d. Variable decelerations with other characteristics such as shoulders or overshoots e. Absent baseline variability with recurrent variable decelerations f. Bradycardia

B, D, E, F These characteristics are all considered non-reassuring or abnormal and belong in Category III. A fetal heart rate of 110 to 160 bpm is considered normal and belongs in Category I. Absent baseline variability not accompanied by recurrent decelerations is a Category II characteristic. PTS: 1 DIF: Cognitive Level: Knowledge/Remembering REF: p. 344 OBJ: Integrated Process: Teaching-Learning MSC: Client Needs: Health Promotion and Maintenance

4. A woman just received an injection of carboprost, 2500 mcg IM. What actions by the nurse take priority? (Select all that apply.) a. Assess for nausea and vomiting b. Assess fetal well-being. c. Administer acetaminophen for headache. d. Monitor urine output. e. Notify the provider immediately.

B, E The usual dose of carboprost is 250 mcg, so this excessive dose could lead to uterine rupture. The nurse monitors the woman for signs of this and continually monitors the fetus for well-being. The provider would be notified and agency policy followed for variance reporting. Nausea, vomiting, and headache are side effects of the usual dose of the drug. This drug is excreted through urine, so monitoring urine output is important but not as critical as checking fetal well-being and notifying the provider. PTS: 1 DIF: Cognitive Level: Analysis/Analyzing REF: p. 601 | Drug Guide Box OBJ: Nursing Process: Implementation MSC: Client Needs: Safe and Effective Care Environment

6. The student nurse is assessing a woman with abruptio placentae. The student reports to the registered nurse "I can't really palpate her abdomen, it's as hard as a board." What action by the nurse is the priority? a. Tell the student to document the findings. b. Have the student teach the woman relaxation techniques. c. Assess the woman's fundal height and vital signs. d. Administer a dose of opioid pain medication.

C A hard, board-like abdomen in this setting is characteristic of concealed hemorrhage. The nurse assesses the woman's fundal height (which will rise with bleeding) and vital signs to detect shock. Documentation occurs after interventions are complete. Relaxation techniques may help the woman cope with the situation, but anxiety is not the reason for the findings. The woman may or may not need pain medication, and if she is going to need surgery, she should not get opioids until consents are signed. PTS: 1 DIF: Cognitive Level: Application/Applying REF: p. 532 | Safety Alert Box OBJ: Nursing Process: Assessment MSC: Client Needs: Safe and Effective Care Environment

9. The nurse present at the delivery is reporting to the nurse who will be caring for the neonate after birth. What information might be included for an infant who had thick meconium in the amniotic fluid? a. The infant had Apgar scores of 6 and 8. b. An IV was started immediately after birth to treat dehydration. c. No meconium was found below the vocal cords when they were examined. d. The parents spent an hour bonding with the baby after birth.

C A laryngoscope is inserted to examine the vocal cords. If no meconium is below the cords, probably no meconium is present in the lower air passages, and the infant will not develop meconium aspiration syndrome. Apgar scores are important but not directly related to meconium. There is no relationship between dehydration and meconium fluid. Bonding is an expected occurrence. PTS: 1 DIF: Cognitive Level: Comprehension/Understanding REF: p. 643 OBJ: Integrated Process: Communication and Documentation MSC: Client Needs: Safe and Effective Care Environment 10. The nurse is teaching the parents of a newborn who is going to receive phototherapy. What other measure does the nurse teach to help reduce the bilirubin? a. Increase the frequency of feedings. b. Increase oral intake of water between feedings. c. How to prepare the newborn for an exchange transfusion d. Wrap the infant in triple blankets to prevent cold stress during phototherapy. ANS: A Frequent feedings prevent hypoglycemia, provide protein to maintain albumin levels in the blood and promote gastrointestinal motility and removal of bilirubin in the stools. More frequent breastfeeding should be encouraged. Avoid offering water between feedings, because the infant may decrease his or her milk intake. Breast milk or formula is more effective at removing bilirubin from the intestines. Exchange transfusions are seldom necessary but may be performed when phototherapy cannot reduce high bilirubin levels quickly enough. Wrapping the infant in blankets will prevent the phototherapy from getting to the skin and being effective. The infant should be uncovered and unclothed. PTS: 1 DIF: Cognitive Level: Application/Applying REF: p. 647 OBJ: Nursing Process: Implementation MSC: Client Needs: Health Promotion and Maintenance 11. A mother with diabetes has done some reading about the effects of the condition on her newborn. Which statement shows a misunderstanding that should be clarified by the nurse? a. "Although my baby is large, some women with diabetes have very small babies because the blood flow through the placenta may not be as good as it should be." b. "My baby will be watched closely for signs of low blood sugar, especially during the early days after birth." c. "The red appearance of my baby's skin is due to an excessive number of red blood cells." d. "My baby's pancreas may not produce enough insulin because the cells became smaller than normal during my pregnancy." ANS: D Infants of diabetic mothers may have hypertrophy of the islets of Langerhans, which may cause them to produce more insulin than they need. The other statements are correct and show good understanding. PTS: 1 DIF: Cognitive Level: Evaluation/Evaluating REF: p. 652 OBJ: Nursing Process: Evaluation MSC: Client Needs: Health Promotion and Maintenance 12. Nursing care of the infant with neonatal abstinence syndrome should include a. Positioning the infant's crib in a quiet corner of the nursery b. Feeding the infant on a 2-hour schedule c. Placing stuffed animals and mobiles in the crib to provide visual stimulation d. Spending extra time holding and rocking the infant ANS: A Placing the crib in a quiet corner helps avoid excessive stimulation of the infant. These infants have an increase calorie needs but poor suck and swallow coordination. Feeding should occur to meet these needs. Stimulation should be kept to a minimum. PTS: 1 DIF: Cognitive Level: Application/Applying REF: p. 655 OBJ: Nursing Process: Implementation MSC: Client Needs: Physiologic Integrity 13. The difference between physiologic and nonphysiologic jaundice is that nonphysiologic jaundice a. usually results in kernicterus. b. appears during the first 24 hours of life. c. results from breakdown of excessive erythrocytes not needed after birth. d. begins on the head and progresses down the body. ANS: B Nonphysiologic jaundice appears during the first 24 hours of life, whereas physiologic jaundice appears after the first 24 hours of life. Pathologic jaundice may lead to kernicterus, but it needs to be stopped before that occurs. Both jaundices are the result of the breakdown of erythrocytes. Pathologic jaundice is due to a pathologic condition, such as Rh incompatibility. PTS: 1 DIF: Cognitive Level: Knowledge/Remembering REF: p. 645 OBJ: Nursing Process: Assessment MSC: Client Needs: Physiologic Integrity 14. The goal of treatment of the infant with phenylketonuria (PKU) is to a. cure cognitive delays. b. prevent central nervous system (CNS) damage. c. prevent gastrointestinal symptoms. d. prevent the renal system damage. ANS: B CNS damage can occur as a result of toxic levels of phenylalanine. No cure exists for cognitive delays should they occur. Digestive problems are a clinical manifestation of PKU, but it is more important to prevent the CNS damage. PKU does not involve renal dysfunction. PTS: 1 DIF: Cognitive Level: Comprehension/Understanding REF: p. 655 OBJ: Nursing Process: Planning MSC: Client Needs: Physiologic Integrity 15. Parents of a newborn with phenylketonuria are anxious to learn about the appropriate treatment for their infant. What topic does the nurse include in the teaching plan? a. Fluid and sodium restrictions b. A phenylalanine-free diet c. Progressive mobility and splinting d. A protein-rich diet ANS: B Phenylketonuria is treated with a special diet that restricts phenylalanine intake. Fluid and sodium restrictions are not included in this plan. Mobility and splinting are not included in the plan. A protein-rich diet is not in the plan. PTS: 1 DIF: Cognitive Level: Comprehension/Understanding REF: p. 655 OBJ: Integrated Process: Teaching-Learning MSC: Client Needs: Physiologic Integrity 16. The nurse is caring for a neonate undergoing phototherapy. What action does the nurse include on the infant's care plan? a. Keep the infant's eyes covered under the light. b. Keep the infant supine at all times. c. Restrict parenteral and oral fluids. d. Dress the infant in only a T-shirt and diaper. ANS: A Retinal damage from phototherapy should be prevented by using eye shields on the infant under the light. To ensure total skin exposure, the infant's position is changed frequently. Special attention to increasing fluid intake ensures that the infant is well hydrated. To ensure total skin exposure, the infant is not dressed. PTS: 1 DIF: Cognitive Level: Application/Applying REF: p. 647 OBJ: Nursing Process: Implementation MSC: Client Needs: Physiologic Integrity 17. An infant with hypocalcemia is receiving an intravenous bolus of calcium. The infant's heart rate changes from 144 beats/minute to 62 beats/minute. What action by the nurse is best? a. Call for a stat EGG. b. Stop the infusion. c. Stimulate the infant. d. Administer magnesium. ANS: B IV calcium can lead to bradycardia. When this infant's heart rate drops to 60 beats/minute, the nurse stops the infusion. A stat ECG is not necessary unless policy requires it or the bradycardia does not resolve. Stimulating the infant will not increase the heart rate. Magnesium infusion will also not increase the heart rate. PTS: 1 DIF: Cognitive Level: Application/Applying REF: p. 653 OBJ: Nursing Process: Implementation MSC: Client Needs: Physiologic Integrity 18. A macrosomic infant is born after a difficult, forceps-assisted delivery. After stabilization, the infant is weighed, and the birth weight is 4550 g (9 pounds, 6 ounces). What action by the nurse is most appropriate? a. Leave the infant in the room with the mother. b. Take the infant immediately to the nursery. c. Perform a gestational age assessment. d. Monitor blood glucose levels frequently. ANS: D This infant is macrosomic (over 4000 g) and is at high risk for hypoglycemia. Blood glucose levels should be monitored frequently, and the infant should be observed closely for signs of hypoglycemia. The infant can stay with the mother, but this is not the best answer since it does not include the close monitoring needed. Regardless of gestational age, this infant is macrosomic. PTS: 1 DIF: Cognitive Level: Application/Applying REF: p. 652 OBJ: Nursing Process: Implementation MSC: Client Needs: Physiologic Integrity 19. A pregnant woman at 37 weeks of gestation has had ruptured membranes for 26 hours. A cesarean section is performed for failure to progress. The fetal heart rate before birth is 180 beats/min with limited variability. At birth, the newborn has Apgar scores of 6 and 7 at 1 and 5 minutes and is noted to be pale and tachypneic. Based on the maternal history, the cause of this newborn's distress is most likely a. hypoglycemia. b. phrenic nerve injury. c. respiratory distress syndrome. d. sepsis. ANS: D The prolonged rupture of membranes and the tachypnea (before and after birth) both suggest sepsis. There is no evidence of phrenic nerve damage or respiratory distress syndrome. Early signs of sepsis may be difficult to distinguish from other problems such as hypoglycemia, but the prolonged rupture of membranes puts this baby at high risk of sepsis. PTS: 1 DIF: Cognitive Level: Analysis/Analyzing REF: p. 650 OBJ: Nursing Process: Assessment MSC: Client Needs: Physiologic Integrity 20. What action by the nurse is the most important action in preventing neonatal infection? a. Good hand hygiene b. Isolation of infected infants c. Separate gown technique d. Standard Precautions ANS: A Virtually all controlled clinical trials have demonstrated that effective handwashing is responsible for the prevention of nosocomial infection in nursery units. The other actions do reduce risk but not nearly to the degree that good hand hygiene does. PTS: 1 DIF: Cognitive Level: Knowledge/Remembering REF: p. 651 OBJ: Nursing Process: Implementation MSC: Client Needs: Safe and Effective Care Environment 21. What action does the nurse add to the plan of care for an infant experiencing symptoms of drug withdrawal? a. Keeping the newborn sedated b. Feeding every 4 to 6 hours to allow extra rest c. Swaddling the infant snugly d. Playing soft music during feeding ANS: C The infant should be wrapped snugly to reduce self-stimulation behaviors and protect the skin from abrasions. The baby is not kept sedated. The infant should be fed in small, frequent amounts and burped well to diminish aspiration and maintain hydration. The infant should not be stimulated (such as with music), because this will increase activity and potentially increase CNS irritability. PTS: 1 DIF: Cognitive Level: Application/Applying REF: p. 655 OBJ: Nursing Process: Implementation MSC: Client Needs: Physiologic Integrity 22. The nursing student learns that transmission of HIV from mother to baby occurs in which fashion? a. From the maternal circulation only in the third trimester b. From the use of unsterile instruments c. Only through the ingestion of amniotic fluid d. Through the ingestion of breast milk from an infected mother ANS: D Postnatal transmission of HIV through breastfeeding may occur. Transplacental transmission can occur at any time during pregnancy. Unsterile instruments are possible sources of transmission but highly unlikely. Transmission of HIV may also occur during birth from blood or secretions. Transmission of HIV from the mother to the infant may occur transplacentally at various gestational ages. This is highly unlikely as most health care facilities must meet sterility standards for all instrumentation. PTS: 1 DIF: Cognitive Level: Comprehension/Understanding REF: p. 649 | Table 30.1 OBJ: Integrated Process: Teaching-Learning MSC: Client Needs: Physiologic Integrity 23. A primigravida has just delivered a healthy infant girl. The nurse is about to administer erythromycin ointment in the infant's eyes when the mother asks, "What is that medicine for?" The nurse responds a. "It is an eye ointment to help your baby see you better." b. "It is to protect your baby from contracting herpes from your vaginal tract." c. "Erythromycin is given to prevent a gonorrheal infection." d. "This medicine will protect your baby's eyes from drying out." ANS: C With the prophylactic use of erythromycin, the incidence of gonococcal conjunctivitis has declined to less than 0.5%. Eye prophylaxis is administered at or shortly after birth to prevent ophthalmia neonatorum. Erythromycin has no bearing on enhancing vision and is not used for herpes infections or lubrication. PTS: 1 DIF: Cognitive Level: Comprehension/Understanding REF: p. 649 | Table 30.1 OBJ: Integrated Process: Teaching-Learning MSC: Client Needs: Physiologic Integrity 24. Near the end of the first week of life, an infant who has not been treated for any infection develops a copper-colored, maculopapular rash on the palms and around the mouth and anus. The newborn is showing signs of a. gonorrhea. b. herpes simplex virus infection. c. congenital syphilis. d. HIV. ANS: C This rash is indicative of congenital syphilis. The lesions may extend over the trunk and extremities. This is not characteristic of gonorrhea, herpes, or HIV. PTS: 1 DIF: Cognitive Level: Knowledge REF: p. 650 | Table 30.1 | p. 654 OBJ: Nursing Process: Assessment MSC: Client Needs: Physiologic Integrity 25. Providing care for the neonate born to a mother who abuses substances can present a challenge for the health care team. Nursing care for this infant requires a multisystem approach. The first step in the provision of this care is a. pharmacologic treatment. b. reduction of environmental stimuli. c. neonatal abstinence syndrome scoring. d. adequate nutrition and maintenance of fluid and electrolyte balance. ANS: C Various scoring systems exist to determine the number, frequency, and severity of behaviors that indicate neonatal abstinence syndrome. The score is helpful in determining the necessity of drug therapy to alleviate withdrawal. Pharmacologic treatment is based on the severity of withdrawal symptoms. Swaddling, holding, and reducing environmental stimuli are essential in providing care to the infant who is experiencing withdrawal. However, the scoring helps provide definitive care. Fluids and electrolyte balance are appropriate for any infant. PTS: 1 DIF: Cognitive Level: Knowledge/Remembering REF: p. 654 OBJ: Nursing Process: Assessment MSC: Client Needs: Physiologic Integrity 26. A woman who has had no prenatal care enters the labor and delivery unit in advanced labor. She has chickenpox. What action by the nurse is best? a. Place the woman in isolation. b. Give the woman immune globulin before delivery. c. Treat the woman with acyclovir. d. Administer antibiotics to the infant after birth. ANS: A Women with varicella infections (chickenpox or shingles) need to be in isolation (airborne and contact per the CDC). There might not be enough time to administer immune globulin to the mother before delivery, but it could be given to the baby. Acyclovir is the drug of choice for treatment, but the staff needs to be protected from this infection through isolation precautions. Antibiotics are not used for this disease. PTS: 1 DIF: Cognitive Level: Application/Applying REF: p. 649 | Table 30.1 OBJ: Nursing Process: Implementation MSC: Client Needs: Safe and Effective Care Environment 27. A woman who has a history of frequent substance abuse is close to delivering. What action by the nurse is best? a. Notify social services of the situation prior to the birth. b. Draw up and label a syringe of naloxone. c. Administer naloxone if the baby shows signs of withdrawal. d. Prepare to administer naloxone to the mother. ANS: B When anticipating the delivery of a baby whose mother is addicted to opioids, the nurse prepares to give the newborn naloxone for respiratory depression. To administer the drug in the fastest way possible, the nurse prepares a syringe with the medication. Then when the baby's weight is known, the nurse discards the excess drug and administers the correct dose to the baby. Social services will need to be involved but not at this point; the medication is the priority. The naloxone may cause signs of withdrawal in the infant. The baby gets the naloxone, not the mother. PTS: 1 DIF: Cognitive Level: Application/Applying REF: p. 641 | p. 642 | Drug Guide OBJ: Nursing Process: Implementation MSC: Client Needs: Physiologic Integrity MULTIPLE RESPONSE

4. The nurse understands that which condition is a contraindication for an amniotomy? a. Dilation less than 3 cm b. Cephalic presentation c. -2 station d. Right occiput posterior position

C A prolapsed cord can occur if the membranes artificially rupture when the presenting part is not engaged. Engagement is assumed at zero station. The other conditions are not contraindications to this procedure. PTS: 1 DIF: Cognitive Level: Knowledge/Remembering REF: p. 376 OBJ: Nursing Process: Assessment MSC: Client Needs: Health Promotion and Maintenance

5. The nurse understands that respirations are initiated at birth as a result of a. an increase in the PO2 and a decrease in PCO2. b. the continued functioning of the foramen ovale. c. chemical, thermal, sensory, and mechanical factors. d. drying off the infant.

C A variety of these factors are responsible for initiation of respirations. The PO2 decreases at birth and the PCO2 increases. The foramen ovale closes at birth. Tactile stimuli aid in initiating respirations but are not the main cause. PTS: 1 DIF: Cognitive Level: Knowledge/Remembering REF: p. 425 OBJ: Nursing Process: Assessment MSC: Client Needs: Physiologic Integrity

3. To increase the absorption of iron in a pregnant woman, the nurse teaches her that iron preparations should be given with a. milk. b. tea. c. orange juice. d. coffee.

C A vitamin C source may increase the absorption of iron. The calcium and phosphorus in milk decrease iron absorption. Tannin in the tea reduces the absorption of iron. Coffee reduces iron absorption. PTS: 1 DIF: Cognitive Level: Knowledge/Remembering REF: Patient-Centered Teaching Box| p. 262 OBJ: Nursing Process: Implementation MSC: Client Needs: Physiologic Integrity

3. An effective technique for communicating with toddlers is to a. have the toddler make up a story from a picture. b. involve the toddler in dramatic play with dress-up clothing. c. use picture books. d. ask the toddler to draw pictures of his fears.

C Activities and procedures should be described as they are about to be done. Use picture books and play for demonstration. Toddlers experience the world through their senses. Most toddlers do not have the vocabulary to make up stories. Dramatic play is associated with older children. Toddlers probably are not capable of drawing or verbally articulating their fears. PTS: 1 DIF: Cognitive Level: Knowledge/Remembering REF: Table 4.3 OBJ: Integrated Process: Communication and Documentation MSC: Client Needs: Health Promotion and Maintenance

9. A patient states, "My breasts are so small, I don't think I will be able to breastfeed." The nurse's best response is a. "It may be difficult, but you should try anyway." b. "You can always supplement with formula." c. "Breast size is not related to the ability to breastfeed." d. "The ability to breastfeed depends on secretion of estrogen and progesterone."

C All women have approximately the same amount of glandular tissue to secrete milk, despite breast size. Saying that nursing will be difficult or that the woman can use formula does not provide the woman with accurate information. Increased estrogen decreases the production of milk. PTS: 1 DIF: Cognitive Level: Comprehension/Understanding REF: p. 192 OBJ: Integrated Process: Teaching-Learning MSC: Client Needs: Physiologic Integrity 10. The function of the cremaster muscle in men is to a. aid in voluntary control of excretion of urine. b. entrap blood in the penis to produce an erection. c. assist with transporting sperm. d. aid in temperature control of the testicles. ANS: D A cremaster muscle is attached to each testicle. Its function is to bring the testicle closer to the body to warm it or allow it to fall away from the body to cool it, thus promoting normal sperm production. It is not involved in urination, causing an erection or assist in transporting sperm. PTS: 1 DIF: Cognitive Level: Knowledge/Remembering REF: p. 193 OBJ: Nursing Process: Assessment MSC: Client Needs: Physiologic Integrity 11. The average man is taller than the average woman at maturity because of a. a longer period of skeletal growth. b. earlier development of secondary sexual characteristics. c. earlier onset of growth spurt. d. starting puberty at an earlier age. ANS: A The man's greater height at maturity is the combined result of beginning the growth spurt at a later age and continuing it for a longer period. Girls develop earlier than boys. Boys' growth spurts start at a later age. Girls start puberty approximately 6 months to 1 year earlier than boys. PTS: 1 DIF: Cognitive Level: Knowledge/Remembering REF: p. 185 OBJ: Nursing Process: Assessment MSC: Client Needs: Health Promotion and Maintenance 12. A student nurse just read that up to 200 million sperm are deposited in the vagina with each ejaculation and asks the faculty why so many are needed. What response by the faculty is most accurate? a. Competition results in fewer genetic defects. b. Sperm are weak and die off quickly. c. Few sperm reach the fallopian tube and ova. d. Most sperm are not the correct shape. ANS: C Although a huge quantity of sperm are released with each ejaculation, very few make it to the fallopian tube where an ovum may be waiting to be fertilized. PTS: 1 DIF: Cognitive Level: Comprehension/Understanding REF: p. 194 OBJ: Integrated Process: Teaching-Learning MSC: Client Needs: Physiologic Integrity 13. The student nurse learns that follicle stimulating hormone is produced in which gland? a. Anterior pituitary b. Posterior pituitary c. Hypothalamus d. Adrenal glands ANS: A Follicle stimulating hormone is produced in the anterior pituitary gland. PTS: 1 DIF: Cognitive Level: Knowledge/Remembering REF: Table 11.1 OBJ: Integrated Process: Teaching-Learning MSC: Client Needs: Physiologic Integrity MULTIPLE RESPONSE

7. The parents of a newborn infant state, "We will probably not have our baby immunized because we are concerned about the risks." What is the nurse's best response? a. "It is your decision to immunize your child or not." b. "You should probably think about this decision." c. "It is far riskier to not immunize your baby." d. "This has to be reported to the health department."

C Although immunizations have been documented to have a negative effect in a small number of cases, an unimmunized infant is at greater risk for development of complications from childhood diseases than from the vaccines. Plus children who get ill from communicable diseases are a threat to those who are immunocompromised. Telling parents they should think about a decision does not give them any information to consider. Of course the parents have the final decision, but the nurse needs to educate them on the risks of that decision. The parents will not be reported to the health department. PTS: 1 DIF: Cognitive Level: Application/Applying REF: p. 90 OBJ: Integrated Process: Teaching-Learning MSC: Client Needs: Health Promotion and Maintenance

9. The nurse-midwife is concerned that a woman's uterine activity is too intense and that her obesity is preventing accurate assessment of the actual intrauterine pressure. On the basis of this information, the nurse should obtain a(n) a. tocotransducer. b. scalp electrode. c. intrauterine pressure catheter. d. Doppler transducer.

C An intrauterine pressure catheter can measure actual intrauterine pressure. The tocotransducer measures the uterine pressure externally; this not be accurate with obesity. A scalp electrode measures the fetal heart rate (FHR). A Doppler auscultates the FHR. PTS: 1 DIF: Cognitive Level: Application/Applying REF: p. 339 OBJ: Nursing Process: Implementation MSC: Client Needs: Physiologic Integrity 10. In which situation is a baseline fetal heart rate of 160 to 170 beats per minute considered a normal finding? a. The fetus is at 28 weeks of gestation. b. The mother has been given an epidural block. c. The mother has a history of fast labors. d. The mother has mild preeclampsia but is not in labor. ANS: A The normal preterm fetus may have a baseline rate slightly higher than the term fetus because of an immature parasympathetic nervous system that does not yet exert a slowing effect on the fetal heart rate (FHR). Any change in the FHR with an epidural is not considered an expected outcome. Fast labors should not alter the FHR normally. Preeclampsia should not cause a normal elevation of the FHR. PTS: 1 DIF: Cognitive Level: Knowledge/Remembering REF: p. 340 OBJ: Nursing Process: Assessment MSC: Client Needs: Health Promotion and Maintenance 11. The nurse assesses the fetal monitor and sees the following strip. What action by the nurse is most appropriate? a. Administer oxygen by nasal cannula. b. Reposition the woman. c. Apply a fetal scalp electrode. d. Record this reassuring pattern. ANS: D This is a reassuring pattern and no intervention is necessary beyond documentation. PTS: 1 DIF: Cognitive Level: Analysis/Analyzing REF: Figure 17.6 OBJ: Nursing Process: Assessment MSC: Client Needs: Health Promotion and Maintenance 12. When the mother's membranes rupture during active labor, the fetal heart rate should be observed for the occurrence of which periodic pattern? a. Increase in baseline variability b. Nonperiodic accelerations c. Early decelerations d. Variable decelerations ANS: D When the membranes rupture, amniotic fluid may carry the umbilical cord to a position where it will be compressed between the maternal pelvis and the fetal presenting part, resulting in a variable deceleration pattern. This is not an expected occurrence after the rupture of membranes. Accelerations are considered reassuring; they are not a concern after rupture of membranes. Early declarations are considered reassuring; they are not a concern after rupture of membranes. PTS: 1 DIF: Cognitive Level: Application/Applying REF: p. 343 OBJ: Nursing Process: Assessment MSC: Client Needs: Health Promotion and Maintenance 13. The fetal heart rate baseline increases 15 beats per minute after vibroacoustic stimulation. What action by the nurse is most appropriate? a. Reassure the family the finding is normal. b. Prepare to assist with obtaining cord blood gases. c. Position the woman on her left side. d. Administer oxygen at 4 L via nasal cannula. ANS: A The fetus with adequate reserve for the stress of labor will usually respond to vibroacoustic stimulation with a temporary increase in the fetal heart rate (FHR) over baseline of 15 bpm for 15 seconds or more. The nurse reassures the family that this finding is normal. The other actions are not warranted. PTS: 1 DIF: Cognitive Level: Application/Applying REF: p. 346 OBJ: Nursing Process: Implementation MSC: Client Needs: Health Promotion and Maintenance 14. The nurse notes a nonreassuring pattern of the fetal heart rate. The mother is already lying on her left side. What nursing action is indicated? a. Lower the head of the bed. b. Place the mother in a Trendelenburg position. c. Change her position to the right side. d. Place a wedge under the left hip. ANS: C Repositioning on the opposite side may relieve compression on the umbilical cord and improve blood flow to the placenta. The other actions are not warranted. PTS: 1 DIF: Cognitive Level: Application/Applying REF: p. 347 OBJ: Nursing Process: Implementation MSC: Client Needs: Health Promotion and Maintenance 15. The nurse notes a pattern of late decelerations on the fetal monitor. The most appropriate action is to a. continue observation of this reassuring pattern. b. notify the physician or nurse-midwife. c. give the woman oxygen by face mask. d. place the woman in a Trendelenburg position. ANS: C Late decelerations are associated with reduced placental perfusion. Giving the laboring woman oxygen increases the oxygen saturation in her blood, making more oxygen available to the fetus. This is not a reassuring pattern; interventions are needed. Nursing interventions should be initiated before notifying the health care provider. The Trendelenburg position will not increase the placental perfusion. PTS: 1 DIF: Cognitive Level: Application/Applying REF: p. 342 | p. 344 | Safety Alert Box OBJ: Nursing Process: Implementation MSC: Client Needs: Health Promotion and Maintenance 16. The nurse explains to the student that increasing the infusion rate of non-additive intravenous fluids can increase fetal oxygenation primarily by a. maintaining normal maternal temperature. b. preventing normal maternal hypoglycemia. c. increasing the oxygen-carrying capacity of the maternal blood. d. expanding maternal blood volume. ANS: D Filling the mother's vascular system makes more blood available to perfuse the placenta and may correct hypotension. Increasing fluid volume may alter the maternal temperature only if she is dehydrated. Most intravenous fluids for laboring women are isotonic and do not improve hypoglycemia. Oxygen-carrying capacity is increased by adding more red blood cells. PTS: 1 DIF: Cognitive Level: Comprehension/Understanding REF: p. 344 | Safety Alert Box OBJ: Integrated Process: Teaching-Learning MSC: Client Needs: Physiologic Integrity 17. Which nursing action is correct when initiating electronic fetal monitoring? a. Lubricate the tocotransducer with an ultrasound gel. b. Inform the patient that she should remain in the semi-Fowler's position. c. Securely apply the tocotransducer with a strap or belt. d. Determine the position of the fetus before attaching the electrode. ANS: C The tocotransducer should fit snugly on the abdomen to monitor uterine activity accurately. The tocotransducer does not need gel to operate appropriately. The patient should be encouraged to move around during labor. The tocotransducer should be placed at the fundal area of the uterus. PTS: 1 DIF: Cognitive Level: Knowledge/Remembering REF: p. 338 | Procedure Box OBJ: Nursing Process: Implementation MSC: Client Needs: Health Promotion and Maintenance 18. Which statement correctly describes the nurse's responsibility related to electronic monitoring? a. Teach the woman and her support person about the monitoring equipment, and discuss any questions they have. b. Report abnormal findings to the physician before initiating corrective actions. c. Inform the support person that the nurse will be responsible for all comfort measures when the electronic equipment is in place. d. Document the frequency, duration, and intensity of contractions measured by the external device. ANS: A Teaching is an essential part of the nurse's role. Corrective actions should be initiated first in order to correct abnormal findings as quickly as possible. The support person should be encouraged to assist with the comfort measures. Electronic monitoring will record the contractions and FHR response. PTS: 1 DIF: Cognitive Level: Comprehension/Understanding REF: p. 347 OBJ: Nursing Process: Implementation MSC: Client Needs: Health Promotion and Maintenance 19. The precepting nurse explains to the newly hired nurse that when using IA for FHR which situation is unit protocol? a. The nurses can be expected to cover only two or three patients when IA is the primary method of fetal assessment. b. The best course is to use the descriptive terms associated with EFM when documenting results. c. If the heartbeat cannot be found immediately, a shift must be made to electronic monitoring. d. Ultrasound can be used to find the fetal heartbeat and reassure the mother if initial difficulty was a factor. ANS: D Locating fetal heartbeats often takes time. Mothers can be reassured verbally and by the ultrasound pictures if that device is used to help locate the heartbeat. When used as the primary method of fetal assessment, auscultation requires a nurse-to-patient ratio of one to one. Documentation should use only terms that can be numerically defined; the usual visual descriptions of EFM are inappropriate. Electronic monitoring is not needed at this point. PTS: 1 DIF: Cognitive Level: Comprehension/Understanding REF: p. 335 OBJ: Integrated Process: Teaching-Learning MSC: Client Needs: Health Promotion and Maintenance 20. The new nurse learns that which condition related to decreased variability is considered benign? a. A periodic fetal sleep state b. Extreme prematurity c. Fetal hypoxemia d. Preexisting neurologic injury ANS: A When the fetus is temporarily in a sleep state there is minimal variability present. Periodic fetal sleep states usually last no longer than 30 minutes. The other conditions would be considered conducive to abnormal variability. PTS: 1 DIF: Cognitive Level: Comprehension/Understanding REF: p. 340 OBJ: Integrated Process: Teaching-Learning MSC: Client Needs: Health Promotion and Maintenance 21. A nurse might be called on to stimulate the fetal scalp a. as part of fetal scalp blood sampling. b. in response to tocolysis. c. in preparation for fetal oxygen saturation monitoring. d. to elicit an acceleration in the FHR. ANS: D The scalp can be stimulated using digital pressure during a vaginal examination, which should cause an increase in FHR. Stimulating the fetal scalp is not part of blood sampling, assessing the response to tocolysis, or in preparation for oxygen saturation monitoring. PTS: 1 DIF: Cognitive Level: Knowledge/Remembering REF: p. 345 OBJ: Nursing Process: Implementation MSC: Client Needs: Health Promotion and Maintenance 22. In the low-risk patient assessments for variability and periodic changes if using the fetal monitor should be done how often? a. Every 15 to 30 minutes b. Every 5 to 15 minutes c. Every 30 to 60 minutes d. Only before and after ambulation ANS: A During the active first stage of labor, FHR should be assessed every 15 to 30 minutes just after a contraction. PTS: 1 DIF: Cognitive Level: Application/Applying REF: p. 385 | Box 17.1 OBJ: Nursing Process: Assessment MSC: Client Needs: Safe and Effective Care Environment 23. A new nurse notes a fetal heart rate pattern of late deceleration with minimal variability in a laboring woman with vaginal bleeding. Which action by the new nurse warrants intervention by the charge nurse? a. Assesses maternal blood pressure b. Assesses for a prolapsed cord c. Prepares to administer terbutaline d. Discontinues oxytocin ANS: B Assessing for a prolapsed cord requires a vaginal examination, which is contraindicated when the woman has active vaginal bleeding. The other actions are appropriate. PTS: 1 DIF: Cognitive Level: Application/Applying REF: p. 344 | Safety Alert Box OBJ: Nursing Process: Implementation MSC: Client Needs: Health Promotion and Maintenance MULTIPLE RESPONSE

2. Family-centered maternity care developed in response to a. demands by physicians for family involvement in childbirth. b. the Sheppard-Towner Act of 1921. c. parental requests that infants be allowed to remain with them rather than in a nursery. d. changes in pharmacologic management of labor.

C As research began to identify the benefits of early extended parent-infant contact, parents began to insist that the infant remain with them. This gradually developed into the practice of rooming-in and finally to family-centered maternity care. Family-centered care was a request by parents, not physicians. The Sheppard-Towner Act of 1921 provided funds for state-managed programs for mothers and children. The changes in pharmacologic management of labor were not a factor in family-centered maternity care. PTS: 1 DIF: Cognitive Level: Knowledge/Remembering REF: p. 2 OBJ: Integrated Process: Teaching-Learning MSC: Client Needs: Psychosocial Integrity

9. After a birth complicated by a shoulder dystocia, what action by the nurse is most appropriate? a. Give supplemental oxygen with a small face mask. b. Encourage the parents to hold the infant. c. Palpate the infant's clavicles. d. Perform a complete newborn assessment.

C Because of the shoulder dystocia, the infant's clavicles may have been fractured. Palpation is a simple assessment to identify crepitus or deformity that requires follow-up. There is no indication for oxygen. The infant needs to be assessed for clavicle fractures before excessive movement. A complete newborn assessment is necessary for all newborns, but assessment of the clavicle is top priority for this infant. PTS: 1 DIF: Cognitive Level: Application/Applying REF: p. 575 OBJ: Nursing Process: Assessment MSC: Client Needs: Physiologic Integrity 10. A laboring patient in the latent phase is experiencing uncoordinated, irregular contractions of low intensity. How should the nurse respond to complaints of constant cramping pain? a. "You are only 2 cm dilated, so you should rest and save your energy for when the contractions get stronger." b. "You must breathe more slowly and deeply so there is greater oxygen supply for your uterus. That will decrease the pain." c. "Let me take off the monitor belts and help you get into a more comfortable position." d. "I have notified the doctor that you are having a lot of discomfort. Let me rub your back and see if that helps." ANS: D Intervention is needed to manage the dysfunctional pattern. Offering support and comfort is important to help the patient cope with the situation. Telling the woman to rest is belittling her complaints. Breathing will not reduce the pain. Fetal monitoring should continue as the woman changes positions. PTS: 1 DIF: Cognitive Level: Application/Applying REF: p. 574 OBJ: Nursing Process: Implementation MSC: Client Needs: Health Promotion and Maintenance 11. Why is adequate hydration important when uterine activity occurs before pregnancy is at term? a. Fluid and electrolyte imbalance can interfere with the activity of the uterine pacemakers. b. Dehydration may contribute to uterine irritability for some women. c. Dehydration decreases circulating blood volume, which leads to uterine ischemia. d. Fluid needs are increased because of increased metabolic activity occurring during contractions. ANS: B Dehydration can contribute to uterine irritability for some women, especially if the woman has an infection. Fluid and electrolyte imbalances are not associated with preterm labor. The woman has an increased blood volume during pregnancy. Fluid needs do not increase due to contractions. PTS: 1 DIF: Cognitive Level: Knowledge/Remembering REF: p. 586 OBJ: Nursing Process: Implementation MSC: Client Needs: Physiologic Integrity 12. In planning for home care of a woman with preterm labor, the nurse needs to address which concern? a. Nursing assessments will be different from those done in the hospital setting. b. Restricted activity and medications will be necessary to prevent recurrence of preterm labor. c. Prolonged bed rest may cause negative physiologic effects. d. Home health care providers will be necessary. ANS: C Prolonged bed rest may cause adverse effects such as weight loss, loss of appetite, muscle wasting, weakness, bone demineralization, decreased cardiac output, risk for thrombophlebitis, alteration in bowel functions, sleep disturbance, and prolonged postpartum recovery. Nursing assessments will differ somewhat from those performed in the acute care setting, but this is not the concern that needs to be addressed. Restricted activity and medication may prevent preterm labor but not in all women. Many, but not all, women will receive home health nurse visits. PTS: 1 DIF: Cognitive Level: Comprehension/Understanding REF: p. 585 OBJ: Nursing Process: Planning MSC: Client Needs: Health Promotion and Maintenance 13. A woman in preterm labor at 30 weeks of gestation receives two 12-mg doses of betamethasone intramuscularly. The purpose of this pharmacologic treatment is to a. stimulate fetal surfactant production. b. reduce maternal and fetal tachycardia associated with ritodrine administration. c. suppress uterine contractions. d. maintain adequate maternal respiratory effort and ventilation during magnesium sulfate therapy. ANS: A Antenatal glucocorticoids given as intramuscular injections to the mother accelerate fetal lung maturity. Inderal would be given to reduce the effects of ritodrine administration. Betamethasone has no effect on uterine contractions. Calcium gluconate would be given to reverse the respiratory depressive effects of magnesium sulfate therapy. PTS: 1 DIF: Cognitive Level: Knowledge/Remembering REF: p. 588 | Drug Guide Box OBJ: Nursing Process: Planning MSC: Client Needs: Physiologic Integrity 14. With regard to the care management of preterm labor, nurses should be aware that a. teaching pregnant women the symptoms probably causes more harm through false alarms. b. Braxton Hicks contractions often signal the onset of preterm labor. c. because preterm labor is likely to be the start of an extended labor, a woman with symptoms can wait several hours before contacting the primary caregiver. d. the diagnosis of preterm labor is based on gestational age, uterine activity, and progressive cervical change. ANS: D Gestational age of 20 to 37 weeks, uterine contractions, and a thinning cervix are all indications of preterm labor. It is essential that nurses teach women how to detect the early symptoms of preterm labor. Braxton Hicks contractions resemble preterm labor contractions, but they are not true labor. Waiting too long to see a health care provider could result in essential medications failing to be administered. PTS: 1 DIF: Cognitive Level: Comprehension/Understanding REF: p. 585 OBJ: Nursing Process: Planning MSC: Client Needs: Safe and Effective Care Environment 15. Which nursing action must be initiated first when evidence of prolapsed cord is found? a. Notify the provider. b. Apply a scalp electrode. c. Prepare the mother for an emergency cesarean delivery. d. Reposition the mother with her hips higher than her head. ANS: D The priority is to relieve pressure on the cord. Changing the maternal position will shift the position of the fetus so that the cord is not compressed. The provider needs to be notified but not until the nurse has taken some corrective action. Trying to relieve pressure on the cord should take priority over increasing fetal monitoring techniques. Emergency cesarean delivery may be necessary if relief of the cord is not accomplished, but attempting to relieve the pressure takes priority. Trying to relieve pressure on the cord should be the first priority. PTS: 1 DIF: Cognitive Level: Application/Applying REF: p. 593 OBJ: Nursing Process: Implementation MSC: Client Needs: Physiologic Integrity 16. A woman who had two previous cesarean births is in active labor, when she suddenly complains of pain between her scapulae. The nurse's priority action is to a. reposition the woman with her hips slightly elevated. b. observe for abnormally high uterine resting tone. c. decrease the rate of nonadditive intravenous fluid. d. notify the provider promptly and prepare the woman for surgery. ANS: D Pain between the scapulae may occur when the uterus ruptures, because blood accumulates under the diaphragm. This is an emergency that requires surgical intervention so the nurse notifies the provider and prepares the woman for surgery. Repositioning the woman with her hips slightly elevated is the treatment for a prolapsed cord. That position in this scenario would cause respiratory difficulties. Since the uterus is no longer able to contract, high resting tones cannot be assessed. However, high resting tones during labor indicate a risk for uterine rupture. The woman is now at high risk for shock. Nonadditive intravenous fluids should be increased. PTS: 1 DIF: Cognitive Level: Application/Applying REF: p. 595 OBJ: Nursing Process: Implementation MSC: Client Needs: Physiologic Integrity 17. Which action should be initiated to limit hypovolemic shock when uterine inversion occurs? a. Administer oxygen at 31 L/min by nasal cannula. b. Administer an oxytocin by intravenous push. c. Monitor fetal heart rate every 5 minutes. d. Increase the intravenous infusion rate. ANS: D Intravenous fluids are necessary to replace the lost blood volume that occurs in uterine inversion. The woman may need blood products as well. Administering oxygen will not prevent hypovolemic shock. Oxytocin should not be given until the uterus is repositioned. A uterine inversion occurs during the third stage of labor. PTS: 1 DIF: Cognitive Level: Application REF: p. 595 OBJ: Nursing Process: Implementation MSC: Client Needs: Physiologic Integrity 18. What factor found in maternal history should alert the nurse to the potential for a prolapsed umbilical cord? a. Oligohydramnios b. Pregnancy at 38 weeks of gestation c. Presenting part at station -3 d. Meconium-stained amniotic fluid ANS: C Because the fetal presenting part is positioned high in the pelvis and is not well applied to the cervix, a prolapsed cord could occur if the membranes rupture. Hydramnios puts the woman at high risk for a prolapsed umbilical cord. A very small fetus, normally preterm, puts the woman at risk for a prolapsed umbilical cord. Meconium-stained amniotic fluid shows that the fetus already has been compromised, but it does not increase the chance of a prolapsed cord. PTS: 1 DIF: Cognitive Level: Knowledge/Remembering REF: p. 593 OBJ: Nursing Process: Assessment MSC: Client Needs: Physiologic Integrity 19. The fetus in a breech presentation is often born by cesarean delivery because a. the buttocks are much larger than the head. b. postpartum hemorrhage is more likely if the woman delivers vaginally. c. internal rotation cannot occur if the fetus is breech. d. compression of the umbilical cord is more likely. ANS: D After the fetal legs and trunk emerge from the woman's vagina, the umbilical cord can be compressed between the maternal pelvis and the fetal head if a delay occurs in the birth of the head. The head is the largest part of a fetus. There is no relationship between breech presentation and postpartum hemorrhage. Internal rotation can occur with a breech. PTS: 1 DIF: Cognitive Level: Knowledge/Remembering REF: p. 593 OBJ: Nursing Process: Planning MSC: Client Needs: Physiologic Integrity 20. An important independent nursing action to promote normal progress in labor is a. assessing the fetus. b. encouraging urination about every 1 to 2 hours. c. allowing the woman to stay in her preferred position. d. regulating intravenous fluids. ANS: B The bladder can reduce room in the woman's pelvis that is needed for fetal descent and can increase her discomfort. Assessment of the fetus is an important task, but will not promote normal progression of labor. Position changes help labor progress and should be encouraged. Maintaining hydration is an important task, but it will not promote normal progression of labor. PTS: 1 DIF: Cognitive Level: Comprehension/Understanding REF: p. 577 OBJ: Nursing Process: Implementation MSC: Client Needs: Health Promotion and Maintenance 21. A woman who is 32 weeks pregnant telephones the nurse at her obstetrician's office and complains of constant backache. She asks what pain reliever is safe for her to take. The best nursing response is a. "Back pain is common at this time during pregnancy due to poor posture." b. "Acetaminophen is acceptable during pregnancy; however, do not take aspirin." c. "You should come into the office and let the doctor check you." d. "Try a warm bath or using a heating pad." ANS: C A prolonged backache is one of the subtle symptoms of preterm labor. Early intervention may prevent preterm birth. The woman should be assessed before trying any home care measures. PTS: 1 DIF: Cognitive Level: Application/Applying REF: p. 583 OBJ: Nursing Process: Implementation MSC: Client Needs: Physiologic Integrity 22. The nurse should suspect uterine rupture if a. fetal tachycardia occurs. b. the woman becomes dyspneic. c. contractions abruptly stop during labor. d. labor progresses unusually quickly. ANS: C A large rupture of the uterus will disrupt its ability to contract. Fetal tachycardia is a sign of hypoxia. Dyspnea and unusually quick labor are not signs of rupture. PTS: 1 DIF: Cognitive Level: Comprehension/Understanding REF: p. 595 OBJ: Nursing Process: Assessment MSC: Client Needs: Physiologic Integrity 23. A student nurse is preparing to administer a dose of betamethasone. What action by the student warrants intervention by the registered nurse? a. Starts a separate IV line to infuse the medication b. Tells the woman her blood glucose will be monitored more often c. Prepares an IM injection choosing a ´ needle d. Listens to the woman's lungs prior to administering the medication ANS: A Betamethasone is given in two IM injections with the appropriate needle. When the student begins to insert a dedicated line for administering it, the nurse intervenes to stop this incorrect action. Since this drug is a steroid, blood glucose readings can rise, so diabetic patients will have more frequent blood sugars. Pulmonary edema is uncommon, but the astute nurse (or student) will listen to lung sounds prior to administration for a baseline. PTS: 1 DIF: Cognitive Level: Application/Applying REF: p. 588 | Drug Guide Box OBJ: Nursing Process: Implementation MSC: Client Needs: Physiologic Integrity 24. An hour after her membranes ruptured, a laboring woman has a temperature of 38.2° C (100.7° F). What action does the nurse perform first? a. Provide cool, wet washcloths for the woman's forehead. b. Assess and document the fetal heart rate. c. Administer acetaminophen orally. d. Encourage the woman to drink clear fluids. ANS: B Fetal tachycardia is associated with maternal fever. While all options are reasonable, the nurse needs to assess fetal well-being first. PTS: 1 DIF: Cognitive Level: Application/Applying REF: p. 580 | Safety Alert Box OBJ: Nursing Process: Assessment MSC: Client Needs: Safe and Effective Care Environment 25. The nursing student observes a laboring woman doing lunges to the left side and asks for an explanation of this activity. What response by the nurse is best? a. It decreases the pain associated with back labor. b. It promotes rotation of the fetal occiput to an anterior position. c. It relieves the cramping associated with a prolonged labor. d. It causes the pelvic inlet to open wider in preparation for birth. ANS: B This action encourages rotation of the fetal head to the anterior position. It does relieve back labor, but this response does not explain why. It does not relieve cramping or open the pelvic inlet. PTS: 1 DIF: Cognitive Level: Comprehension/Understanding REF: p. 575 OBJ: Integrated Process: Teaching-Learning MSC: Client Needs: Health Promotion and Maintenance MULTIPLE RESPONSE

6. At what age is an infant first expected to locate an object hidden from view? a. 4 months of age b. 6 months of age c. 9 months of age d. 20 months of age

C By 9 months of age, an infant will actively search for an object that is out of sight. Four-month-old infants are not cognitively capable of searching out objects hidden from their view. Infants at this developmental level do not pursue hidden objects. Six-month-old infants have not developed the ability to perceive objects as permanent and do not search out objects hidden from their view. Twenty-month-old infants actively pursue objects not in their view and are capable of recalling the location of an object not in their view. They first look for hidden objects around age 9 months. PTS: 1 DIF: Cognitive Level: Knowledge/Remembering REF: p. 87 OBJ: Nursing Process: Assessment MSC: Client Needs: Health Promotion and Maintenance

6. While working with the pregnant woman in her first trimester, the nurse is aware that chorionic villus sampling (CVS) can be performed during pregnancy as early as _____ weeks. a. 4 b. 8 c. 10 d. 12

C CVS is usually performed between 10 and 13 weeks of gestation to diagnose fetal chromosomal, metabolic, or DNA abnormalities. PTS: 1 DIF: Cognitive Level: Knowledge/Remembering REF: p. 278 OBJ: Nursing Process: Assessment MSC: Client Needs: Physiologic Integrity

7. Which comments indicate that the mother of a toddler needs further teaching about dental care? a. "We use well water so I give my toddler fluoride supplements." b. "My toddler brushes his teeth with my help." c. "My child will not need a dental checkup until his permanent teeth come in." d. "I use a small nylon bristle brush for my toddler's teeth."

C Children should first see the dentist 6 months after the first primary tooth erupts and no later than age 30 months. Toddlers need fluoride supplements when they use a water supply that is not fluoridated. Toddlers need supervision with dental care. The parent should finish brushing areas not reached by the child. A small nylon bristle brush works best for cleaning toddlers' teeth. PTS: 1 DIF: Cognitive Level: Evaluation/Evaluating REF: p. 117 OBJ: Nursing Process: Evaluation MSC: Client Needs: Health Promotion and Maintenance

7. The maternity nurse should have a clear understanding of the correct use of a clinical pathway. One characteristic of clinical pathways is that they a. are developed and implemented by nurses. b. are used primarily in the pediatric setting. c. set specific time lines for sequencing interventions. d. are part of the nursing process.

C Clinical pathways are standardized, interdisciplinary plans of care devised for patients with a particular health problem. They are used to identify patient outcomes, specify time lines to achieve those outcomes, direct appropriate interventions and sequencing of interventions, include interventions from a variety of disciplines, promote collaboration, and involve a comprehensive approach to care. They are developed by multiple health care professionals and reflect interdisciplinary care. They can be used in multiple settings and for patients throughout the life span. They are not part of the nursing process but can be used in conjunction with the nursing process to provide care to patients. PTS: 1 DIF: Cognitive Level: Knowledge/Remembering REF: p. 7 OBJ: Nursing Process: Planning MSC: Client Needs: Safe and Effective Care Environment

4. According to Piaget's theory, the period of cognitive development in which the child is able to distinguish between concepts related to fact and fantasy, such as human beings are incapable of flying like birds, is the __________ period of cognitive development. a. sensorimotor b. formal operations c. concrete operations d. preoperational

C Concrete operations is the period of cognitive development in which children's thinking is shifted from egocentric to being able to see another's point of view. They develop the ability to distinguish fact from fantasy. The sensorimotor stage occurs in infancy and is a period of reflexive behavior. During this period, the infant's world becomes more permanent and organized. The stage ends with the infant demonstrating some evidence of reasoning. Formal operations is a period in development in which new ideas are created through previous thoughts. Analytic reason and abstract thought emerge in this period. The preoperational stage is a period of egocentrism in which the child's judgments are illogical and dominated by magical thinking and animism. PTS: 1 DIF: Cognitive Level: Knowledge/Remembering REF: p. 68 OBJ: Nursing Process: Assessment MSC: Client Needs: Health Promotion and Maintenance

2. Frequent developmental assessments are important for which reason? a. Stable developmental periods during infancy provide an opportunity to identify any delays or deficits. b. Infants need stimulation specific to the stage of development. c. Critical periods of development occur during childhood. d. Child development is unpredictable and needs monitoring.

C Critical periods are blocks of time during which children are ready to master specific developmental tasks. The earlier those delays in development are discovered and intervention initiated, the less dramatic their effect will be. Infancy is a dynamic time of development that requires frequent evaluations to assess appropriate developmental progress. Infants in a nurturing environment will develop appropriately and will not necessarily need stimulation specific to their developmental stage. Normal growth and development is orderly and proceeds in a predictable pattern based on each individual's abilities and potentials. PTS: 1 DIF: Cognitive Level: Comprehension/Understanding REF: p. 64 OBJ: Nursing Process: Assessment MSC: Client Needs: Health Promotion and Maintenance

1. A yellow crust has formed over the circumcision site. The mother calls the hotline at the local hospital, 5 days after her son was circumcised. She is very concerned. On which rationale should the nurse base her reply? a. After circumcision, the diaper should be changed frequently and fastened snugly. b. This yellow crust is an early sign of infection. c. The yellow crust should not be removed. d. Discontinue the use of petroleum jelly to the tip of the penis.

C Crust is a normal part of healing and should not be removed. The diaper should be fastened loosely to prevent rubbing or pressure on the incision site. The normal yellowish exudate that forms over the site should be differentiated from the purulent drainage of infection. The only contraindication for petroleum jelly is the use of a PlastiBell. PTS: 1 DIF: Cognitive Level: Comprehension/Understanding REF: p. 472 | Patient-Centered Teaching Box OBJ: Nursing Process: Implementation MSC: Client Needs: Physiologic Integrity

4. The nurse teaches the parents that which of the following is the primary purpose of a transitional object? a. It helps the parents with the guilt they feel when they leave the child. b. It keeps the child quiet at bedtime. c. It is effective in decreasing anxiety in the toddler. d. It decreases negativism and tantrums in the toddler.

C Decreasing anxiety, particularly separation anxiety, is the function of a transitional object; it provides comfort to the toddler in stressful situations and helps make the transition from dependence to autonomy. Decreased parental guilt (distress) is an indirect benefit of a transitional object. A transitional object may be part of a bedtime ritual, but it may not keep the child quiet at bedtime. A transitional object does not significantly affect negativity and tantrums, but it can comfort a child after tantrums. PTS: 1 DIF: Cognitive Level: Knowledge/Remembering REF: p. 128 OBJ: Integrated Process: Teaching-Learning MSC: Client Needs: Health Promotion and Maintenance

7. Transient tachypnea of the newborn (TTN) is thought to occur as a result of a. a lack of surfactant. b. hypoinflation of the lungs. c. delayed absorption of fetal lung fluid. d. a slow vaginal delivery associated with meconium-stained fluid.

C Delayed absorption of fetal lung fluid is thought to be the reason for TTN. Lack of surfactant and hypoinflation of the lungs are not related to TTN. A slow vaginal delivery will help prevent TTN. PTS: 1 DIF: Cognitive Level: Knowledge/Remembering REF: p. 642 OBJ: Nursing Process: Assessment MSC: Client Needs: Physiologic Integrity

1. A postpartum woman overhears the nurse tell the obstetrics clinician that she has a positive Homans sign and asks what it means. The nurse's best response is a. "You have pitting edema in your ankles." b. "You have deep tendon reflexes rated 2+." c. "You have calf pain when I flexed your foot." d. "You have a 'fleshy' odor to your vaginal drainage."

C Discomfort in the calf with sharp dorsiflexion of the foot may indicate a deep vein thrombosis. It does not indicate edema, rate deep tendon reflexes, or describe the odor of lochia. PTS: 1 DIF: Cognitive Level: Comprehension/Understanding REF: p. 404 OBJ: Nursing Process: Implementation MSC: Client Needs: Physiologic Integrity

3. The nursing student is planning to assess a fetal heart rate. The registered nurse reminds the student to get gel. Which method of assessing the fetal heart rate is the student planning on conducting? a. Fetoscope b. Tocodynamometer c. Doppler d. Scalp electrode

C Doppler is the only listed method involving ultrasonic transmission of fetal heart rates; it requires use of a gel. PTS: 1 DIF: Cognitive Level: Knowledge/Remembering REF: p. 338 OBJ: Nursing Process: Assessment MSC: Client Needs: Health Promotion and Maintenance

1. The maternity nurse understands that as the uterus contracts during labor, maternal-fetal exchange of oxygen and waste products a. continues except when placental functions are reduced. b. increases as blood pressure decreases. c. diminishes as the spiral arteries are compressed. d. is not significantly affected.

C During labor contractions, the maternal blood supply to the placenta gradually stops as the spiral arteries supplying the intervillous space are compressed by the contracting uterine muscle. The maternal blood supply to the placenta gradually stops with contractions and the exchange of oxygen and waste products decreases. PTS: 1 DIF: Cognitive Level: Knowledge/Remembering REF: p. 290 OBJ: Nursing Process: Assessment MSC: Client Needs: Physiologic Integrity

8. During a pregnancy group meeting, the nurse teaches patients that the fetal period is best described as one of a. development of basic organ systems. b. resistance of organs to damage from external agents. c. maturation of organ systems. d. development of placental oxygen-carbon dioxide exchange.

C During the fetal period, the body systems grow in size and mature in function to allow independent existence after birth. Basic organ systems are developed during the embryonic period. The organs are always at risk for damage from external sources; however, the older the fetus, the more resistant the organs will be. The greatest risk is when the organs are developing. The placental system is complete by week 12, but that is not the best description of the fetal period. PTS: 1 DIF: Cognitive Level: Comprehension/Understanding REF: p. 204 OBJ: Integrated Process: Teaching-Learning MSC: Client Needs: Physiologic Integrity

3. The parents of a 14-year-old girl are concerned that their adolescent spends too much time looking in the mirror. Which statement is the most appropriate for the nurse to make? a. "Your teenager needs clearer and stricter limits about her behavior." b. "Your teenager needs more responsibility at home." c. "During adolescence this behavior is not unusual." d. "The behavior is abnormal and needs further investigation."

C Egocentric and narcissistic behavior, such as staring at oneself in the mirror, is normal during this period of development. The teenager is seeking a personal identity. Stricter limits are not an appropriate response for a behavior that is part of normal development. More responsibility at home is not an appropriate response for this situation. The behavior is normal and needs no further investigation. PTS: 1 DIF: Cognitive Level: Comprehension/Understanding REF: p. 156 OBJ: Integrated Process: Teaching-Learning MSC: Client Needs: Health Promotion and Maintenance

3. The purpose of the ovum's zona pellucida is to a. make a pathway for more than one sperm to reach the ovum. b. allow the 46 chromosomes from each gamete to merge. c. prevent multiple sperm from fertilizing the ovum. d. stimulate the ovum to begin mitotic cell division.

C Fertilization causes the zona pellucida to change its chemical composition so that multiple sperm cannot fertilize the ovum. Each gamete (sperm and ovum) has only 23 chromosomes. There will be 46 chromosomes when they merge. Mitotic cell division begins when the nuclei of the sperm and ovum unite. PTS: 1 DIF: Cognitive Level: Knowledge/Remembering REF: p. 195 OBJ: Integrated Process: Teaching-Learning MSC: Client Needs: Physiologic Integrity

8. The student nurse learns that which factor ensures that the smallest anterior-posterior diameter of the fetal head enters the pelvis? a. Descent b. Engagement c. Flexion d. Station

C Flexion of the fetal head allows the smallest head diameters to pass through the pelvis. Descent is the moving of the fetus through the birth canal. Engagement occurs when the largest diameter of the fetal presenting part has passed the pelvic inlet. Station is the relationship of the fetal presenting part to the level of the ischial spines. PTS: 1 DIF: Cognitive Level: Knowledge/Remembering REF: p. 299 OBJ: Integrated Process: Teaching-Learning MSC: Client Needs: Health Promotion and Maintenance

8. A pregnant woman has been diagnosed with gestational hypertension and is crying. She asks the nurse if this means she has to take blood pressure medicine for the rest of her life. What answer by the nurse is best? a. "Yes, you will have hypertension for the rest of your life." b. "No, this always goes away after you deliver." c. "Maybe, we have to wait and see at your 6-week postpartum checkup." d. "I don't know. But if you need medicine you should take it."

C Gestational hypertension can last after delivery. If it has not resolved by postpartum week 6, it is considered chronic, and the woman will probably have to take medication. It may or may not resolve, but the nurse should not provide false reassurance or state that he or she does not know without finding more information. Telling the woman to take medicine if she needs it belittles her concerns. PTS: 1 DIF: Cognitive Level: Application/Applying REF: p. 536 | Table 25.1 OBJ: Nursing Process: Implementation MSC: Client Needs: Health Promotion and Maintenance

9. To manage her diabetes appropriately and ensure a good fetal outcome, the pregnant woman with diabetes will need to alter her diet by doing which of the following? a. Eating six small equal meals per day b. Reducing carbohydrates in her diet c. Eating her meals and snacks on a fixed schedule d. Increasing her consumption of protein

C Having a fixed meal schedule will provide the woman and the fetus with a steadier blood sugar level, provide better balance with insulin administration, and help prevent complications. It is more important to have a fixed meal schedule than equal division of food intake or increased protein intake. Approximately 45% of the food eaten should be in the form of carbohydrates. PTS: 1 DIF: Cognitive Level: Comprehension/Understanding REF: p. 556 OBJ: Nursing Process: Planning MSC: Client Needs: Physiologic Integrity 10. A pregnant diabetic woman is in the hospital and her blood glucose reading is 42 mg/dL. What action by the nurse is best? a. Provide her with 15 grams of oral carbohydrate if she can swallow. b. Administer a bolus of rapid-acting insulin. c. Order the woman a meal tray from the cafeteria. d. Notify the provider immediately. ANS: A This woman has hypoglycemia and needs to injest 15 grams of carbohydrate if she is able to swallow. Insulin would make the problem worse. The meal tray is a good idea but not as the first response as it will take too long. The provider should be notified but only after the nurse takes corrective action. PTS: 1 DIF: Cognitive Level: Application/Applying REF: p. 556 OBJ: Nursing Process: Implementation MSC: Client Needs: Physiologic Integrity 11. Nursing intervention for the pregnant diabetic is based on the knowledge that the need for insulin a. increases throughout pregnancy and the postpartum period. b. decreases throughout pregnancy and the postpartum period. c. varies depending on the stage of gestation. d. should not change because the fetus produces its own insulin. ANS: C Insulin needs decrease during the first trimester, when nausea, vomiting, and anorexia are a factor. They increase during the second and third trimesters, when the hormones of pregnancy create insulin resistance in maternal cells. PTS: 1 DIF: Cognitive Level: Knowledge/Remembering REF: p. 553 OBJ: Nursing Process: Implementation MSC: Client Needs: Physiologic Integrity 12. With regard to the association of maternal diabetes and other risk situations affecting mother and fetus, nurses should be aware that a. Diabetic ketoacidosis (DKA) can lead to fetal death at any time during pregnancy. b. Hydramnios rarely occurs in diabetic pregnancies. c. Infections occur about as often and are considered about as serious in diabetic and nondiabetic pregnancies. d. Women should not use insulin pumps during pregnancy. ANS: A Prompt treatment of DKA is necessary to save the fetus and the mother. Hydramnios is a potential complication for the diabetic pregnancy. Infections are more common and more serious in pregnant women with diabetes. Women who were treated with an insulin pump before pregnancy can continue this therapy. PTS: 1 DIF: Cognitive Level: Comprehension REF: p. 556 OBJ: Nursing Process: Planning MSC: Client Needs: Physiologic Integrity 13. What form of heart disease in women of childbearing years usually has a benign effect on pregnancy? a. Cardiomyopathy b. Rheumatic heart disease c. Congenital heart disease d. Mitral valve prolapse ANS: D Mitral valve prolapse is a benign condition that is usually asymptomatic. Cardiomyopathy produces congestive heart failure during pregnancy. Rheumatic heart disease can lead to heart failure during pregnancy. Some congenital heart diseases will produce pulmonary hypertension or endocarditis during pregnancy. PTS: 1 DIF: Cognitive Level: Knowledge/Remembering REF: p. 559 OBJ: Nursing Process: Assessment MSC: Client Needs: Physiologic Integrity 14. When teaching the pregnant woman with class II heart disease, what information should the nurse provide? a. Advise her to gain at least 30 lb. b. Explain the importance of a diet high in calcium. c. Instruct her to avoid strenuous activity. d. Inform her of the need to limit fluid intake. ANS: C Activity may need to be limited so that cardiac demand does not exceed cardiac capacity. Weight gain should be kept at a minimum with heart disease. Iron and folic acid intake is important to prevent anemia. Fluid intake should not be limited during pregnancy. She may also be put on a diuretic. Fluid intake is necessary to prevent fluid deficits. PTS: 1 DIF: Cognitive Level: Comprehension/Understanding REF: p. 559 OBJ: Nursing Process: Implementation MSC: Client Needs: Physiologic Integrity 15. Prophylaxis of subacute bacterial endocarditis (SBE) is given before and after birth when a pregnant woman has a. valvular disease. b. congestive heart disease. c. dysrhythmias. d. postmyocardial infarction. ANS: A Prophylaxis for intrapartum endocarditis and pulmonary infection may be provided for women who have mitral valve prolapse. It is not indicated for congestive heart failure, dysrhythmias, or myocardial infarctions. PTS: 1 DIF: Cognitive Level: Knowledge/Remembering REF: p. 559 OBJ: Nursing Process: Implementation MSC: Client Needs: Physiologic Integrity 16. The nurse understands that postpartum care of the woman with cardiac disease a. is the same as that for any pregnant woman. b. includes rest and monitoring of the effect of activity. c. includes ambulating frequently, alternating with active range of motion. d. includes limiting visits with the infant to once per day. ANS: B After delivery, the woman with cardiac disease should rest, and the nurse monitors her for the effect activity has on her cardiovascular status. Care of the woman with cardiac disease in the postpartum period is tailored to the woman's functional capacity. Although the woman may need help caring for the infant, breastfeeding and infant visits are not contraindicated. PTS: 1 DIF: Cognitive Level: Knowledge/Remembering REF: p. 561 OBJ: Nursing Process: Planning MSC: Client Needs: Physiologic Integrity 17. In caring for a pregnant woman with sickle cell anemia the nurse is aware that signs and symptoms of sickle cell crisis include a. anemia. b. endometritis. c. fever and pain. d. urinary tract infection. ANS: C Women with sickle cell anemia have recurrent attacks (crisis) of fever and pain, most often in the abdomen, joints, and extremities. These attacks are attributed to vascular occlusion when RBCs assume the characteristic sickled shape. Crises are usually triggered by dehydration, hypoxia, or acidosis. Signs of crisis do not include anemia, endometriosis, or UTI. PTS: 1 DIF: Cognitive Level: Knowledge/Remembering REF: p. 562 OBJ: Nursing Process: Assessment MSC: Client Needs: Physiologic Integrity 18. With regard to anemia, nurses should be aware that a. it is the most common medical disorder of pregnancy. b. it can trigger reflex brachycardia. c. the most common form of anemia is caused by folate deficiency. d. thalassemia is a European version of sickle cell anemia. ANS: A Iron deficiency anemia causes 75% of anemias in pregnancy. It is difficult to meet the pregnancy needs for iron through diet alone. It does not cause bradycardia. Thalassemia is a distinct disease from sickle cell anemia. PTS: 1 DIF: Cognitive Level: Knowledge/Remembering REF: p. 561 OBJ: Nursing Process: Planning MSC: Client Needs: Physiologic Integrity 19. For which of the infectious diseases can a woman be immunized? a. Toxoplasmosis b. Rubella c. Cytomegalovirus d. Herpesvirus type 2 ANS: B Rubella is the only infectious disease listed for which a vaccine is available. PTS: 1 DIF: Cognitive Level: Knowledge/Remembering REF: p. 566 OBJ: Nursing Process: Assessment MSC: Client Needs: Health Promotion and Maintenance 20. A woman who delivered her third child yesterday has just learned that her two school-age children have contracted chickenpox. What action by the nurse is best? a. Assess if the woman has had chickenpox or been vaccinated. b. Tell her that the baby has immunity from her and is not susceptible. c. Advise her if she is non-immune, she will get vaccinated at her 2-week postpartum checkup. d. The infant will receive prophylactic acyclovir before discharge. ANS: A The first thing the nurse should do is to determine the woman's susceptibility to this infection. If she is non-immune, she will get her first vaccination prior to discharge. The nurse does not know the baby's immune status without knowing the mother's. Acyclovir is not used to treat chickenpox. PTS: 1 DIF: Cognitive Level: Application/Applying REF: p. 566 OBJ: Nursing Process: Implementation MSC: Client Needs: Safe and Effective Care Environment 21. A woman has a history of drug use and is screened for hepatitis B during the first trimester. What is an appropriate action? a. Provide a low-protein diet. b. Offer the vaccine. c. Discuss the recommendation to bottle-feed her baby. d. Practice respiratory isolation. ANS: B A person who has a history of high-risk behaviors should be offered the hepatitis B vaccine. A low-protein diet will not prevent the infection. The first trimester is too early to discuss feeding methods. Respiratory isolation is not needed for this blood- and body fluid-borne disease. PTS: 1 DIF: Cognitive Level: Application/Applying REF: p. 567 OBJ: Nursing Process: Planning MSC: Client Needs: Health Promotion and Maintenance 22. A woman has tested human immunodeficiency virus (HIV)-positive and has now discovered that she is pregnant. Which statement indicates that she understands the risks of this diagnosis? a. "Even though my test is positive, my baby might not be affected." b. "I know I will need to have an abortion as soon as possible." c. "This pregnancy will probably decrease the chance that I will develop AIDS." d. "My baby is certain to have AIDS and die within the first year of life." ANS: A The rate of perinatal transmission of HIV has decreased with the use of antiretroviral medications during pregnancy. There is no need to have an abortion. The mother may or may not go on to develop AIDS. PTS: 1 DIF: Cognitive Level: Evaluation/Evaluating REF: p. 568 OBJ: Nursing Process: Evaluation MSC: Client Needs: Physiologic Integrity 23. A nurse has taught a pregnant woman about toxoplasmosis. What statement by the patients indicates a need for further instruction? a. "I will be certain to empty the litter boxes regularly." b. "I won't eat raw eggs." c. "I had better wash all of my fruits and vegetables." d. "I need to be cautious when cooking meat." ANS: A The patient should avoid contact with materials that are possibly contaminated with cat feces while pregnant. This includes cat litter boxes, sand boxes, and garden soil. She should wash her hands thoroughly after working with soil or handling animals. The other statements show good understanding. PTS: 1 DIF: Cognitive Level: Evaluation/Evaluating REF: p. 569 OBJ: Nursing Process: Evaluation MSC: Client Needs: Health Promotion and Maintenance 24. A woman who had no prenatal care has just delivered after a brief labor. The baby has rough, dry skin; is large for gestational age; and has an umbilical hernia. What action by the nurse is most appropriate? a. Question the mother about substance abuse. b. Reassess the baby's gestational age. c. Inform the mother her thyroid levels will be checked. d. Perform a bedside blood glucose test on the mother. ANS: C These signs in the newborn are indicative of hypothyroidism. The mother will have thyroid levels checked. Asking about substance abuse, reassessing gestational age, and obtaining a blood glucose reading are all unnecessary. PTS: 1 DIF: Cognitive Level: Application/Applying REF: p. 570 | Table 26.3 OBJ: Nursing Process: Implementation MSC: Client Needs: Physiologic Integrity 25. A woman in the perinatal clinic asks the nurse how her asthma will affect her pregnancy and fetus. What response by the nurse is best? a. Asthma medications cannot be used during pregnancy. b. The only problem is that you will not be able to breastfeed. c. Medications for asthma do not appear to harm the fetus. d. Pregnancy tends to make asthma worse. ANS: C Medications for asthma seem to be well tolerated during pregnancy. Breastfeeding is safe for the newborn. The course of asthma is variable in pregnancy. PTS: 1 DIF: Cognitive Level: Application/Applying REF: p. 570 | Table 26.3 OBJ: Integrated Process: Teaching-Learning MSC: Client Needs: Physiologic Integrity 26. A woman has been admitted to the labor and delivery unit who is HIV positive. She is in active labor. What action by the nurse is most appropriate? a. Prepare to administer IV zidovudine. b. Place the mother on contact precautions. c. Administer oxygen by face mask. d. Notify social services. ANS: A During labor, an IV infusion of zidovudine is administered. The woman does not need contact precautions; standard precautions suffice. The woman does not need oxygen because of her HIV status. There is no reason to notify social services. PTS: 1 DIF: Cognitive Level: Application/Applying REF: p. 568 | Box 26.3 OBJ: Nursing Process: Implementation MSC: Client Needs: Physiologic Integrity MULTIPLE RESPONSE

3. Families who deal effectively with stress exhibit which behavior pattern? a. Focus on family problems b. Feel weakened by stress c. Expect that some stress is normal d. Feel guilty when stress exists

C Healthy families recognize that some stress is normal in all families. Healthy families focus on family strengths rather than on the problems and know that stress is temporary and may be positive. If families are dealing effectively with stress, then weakening of the family unit should not occur. Because some stress is normal in all families, feeling guilty is not reasonable. Guilt only immobilizes the family and does not lead to resolution of the stress. PTS: 1 DIF: Cognitive Level: Comprehension/Understanding REF: p. 38 OBJ: Nursing Process: Assessment MSC: Client Needs: Psychosocial Integrity

8. The nurse administers vitamin K to the newborn for what reason? a. Most mothers have a diet deficient in vitamin K, which results in the infant's being deficient. b. Vitamin K prevents the synthesis of prothrombin in the liver and must be given by injection. c. Bacteria that synthesize vitamin K are not present in the newborn's intestinal tract. d. The supply of vitamin K is inadequate for at least 3 to 4 months, and the newborn must be supplemented.

C In order to promote clotting, vitamin K is necessary. However, the bacteria that synthesize vitamin K are not present in the newborn's intestinal tract, so the nurse administers it via injection. The maternal diet has no bearing on the amount of vitamin K found in the newborn. It is not involved in the synthesis of prothrombin. By day 8, normal newborns are able to produce their own vitamin K. PTS: 1 DIF: Cognitive Level: Knowledge/Remembering REF: p. 462 OBJ: Nursing Process: Implementation MSC: Client Needs: Physiologic Integrity

3. Which setting for childbirth allows the least amount of parent-infant contact? a. Labor/delivery/recovery/postpartum room b. Birth center c. Traditional hospital birth d. Home birth

C In the traditional hospital setting, the mother may see the infant for only short feeding periods, and the infant is cared for in a separate nursery. The labor/delivery/recovery/postpartum room setting allows increased parent-infant contact. Birth centers are set up to allow an increase in parent-infant contact. Home births allow an increase in parent-infant contact. PTS: 1 DIF: Cognitive Level: Knowledge/Remembering REF: p. 2 OBJ: Nursing Process: Planning MSC: Client Needs: Health Promotion and Maintenance

1. How can a woman avoid exposing her fetus to teratogens? a. Update her immunizations during the first trimester of her pregnancy. b. Use saunas and hot tubs during the winter months only. c. Use only class A drugs during her pregnancy. d. Use alcoholic beverages only in the first and third trimesters of pregnancy.

C In well-controlled studies, class A drugs have no demonstrated fetal risk. Immunizations, such as rubella, are contraindicated in pregnancy. Maternal hyperthermia is an important teratogen. Alcohol is an environmental substance known to be teratogenic. PTS: 1 DIF: Cognitive Level: Knowledge/Remembering REF: p. 177 OBJ: Nursing Process: Planning MSC: Client Needs: Health Promotion and Maintenance

6. Postpartal overdistention of the bladder and urinary retention can lead to which complication? a. Postpartum hemorrhage and eclampsia b. Fever and increased blood pressure c. Postpartum hemorrhage and urinary tract infection d. Urinary tract infection and uterine rupture

C Incomplete emptying and overdistention of the bladder can lead to urinary tract infection. Overdistention of the bladder displaces the uterus and prevents contraction of the uterine muscle. There is no correlation between bladder distention and eclampsia, blood pressure, or fever. The risk of uterine rupture decreases after the birth. PTS: 1 DIF: Cognitive Level: Comprehension/Understanding REF: p. 398 OBJ: Nursing Process: Assessment MSC: Client Needs: Physiologic Integrity

5. Which 16-year-old female is most likely to experience secondary amenorrhea? a. A girl who is 5 ft 2 in, 130 lb b. A girl who is 5 ft 9 in, 150 lb c. A girl who is 5 ft 7 in, 96 lb d. A girl who is 5 ft 4 in, 120 lb

C Low body fat is a risk factor for secondary amenorrhea. The girl who is 5 ft 7 inches tall and only weighs 96 pounds has less body fat that the other girls and a higher likelihood of secondary amenorrhea. PTS: 1 DIF: Cognitive Level: Analysis/Analyzing REF: p. 185 OBJ: Nursing Process: Assessment MSC: Client Needs: Physiologic Integrity

5. If the patient's white blood cell (WBC) count is 25,000/mm3 on her second postpartum day, the nurse should a. tell the physician immediately. b. have the laboratory draw blood for reanalysis. c. recognize that this is an acceptable range at this point. d. begin antibiotic therapy immediately.

C Marked leukocytosis occurs with WBC counts increasing to as high as 30,000/mm3 during labor and the immediate postpartum period. The WBC falls to normal within 6 days postpartum. No action is necessary. PTS: 1 DIF: Cognitive Level: Knowledge/Remembering REF: p. 397 OBJ: Nursing Process: Implementation MSC: Client Needs: Health Promotion and Maintenance

8. A pregnant patient asks when the dark line on her abdomen (linea nigra) will go away. The nurse knows the pigmentation will decrease after delivery because of a. increased estrogen. b. increased progesterone. c. decreased melanocyte-stimulating hormone. d. decreased human placental lactogen.

C Melanocyte-stimulating hormone increases during pregnancy and is responsible for changes in skin pigmentation; the amount decreases after delivery. The linea nigra will eventually fade away for most women. Estrogen and progesterone levels decrease after delivery. Human placental lactogen production continues to aid in lactation. However, it does not affect pigmentation. PTS: 1 DIF: Cognitive Level: Knowledge/Remembering REF: p. 399 OBJ: Nursing Process: Assessment MSC: Client Needs: Physiologic Integrity

7. What is the primary role of practicing nurses in the research process? a. Designing research studies b. Collecting data for other researchers c. Identifying researchable problems d. Seeking funding to support research studies

C Nursing generates and answers its own questions based on evidence within its unique subject area. Nurses of all educational levels are in a position to find researchable questions based on problems seen in their practice area. Designing research studies is generally left to nurses with advanced degrees. Collecting data may be part of a nurse's daily activity, but not all nurses will have this opportunity. Seeking funding goes along with designing and implementing research studies. PTS: 1 DIF: Cognitive Level: Comprehension/Understanding REF: p. 25 OBJ: Integrated Process: Teaching-Learning MSC: Client Needs: Safe and Effective Care Environment

6. A woman who is older than 35 years may have difficulty achieving pregnancy, because a. personal risk behaviors influence fertility. b. she has used contraceptives for an extended time. c. her ovaries may be affected by the aging process. d. prepregnancy medical attention is lacking.

C Once the mature woman decides to conceive, a delay in becoming pregnant may occur because of the normal aging of the ovaries. The older adult participates in fewer risk behaviors than the younger adult. The problem is the age of the ovaries, not the past use of contraceptives. Prepregnancy medical care is available and encouraged. PTS: 1 DIF: Cognitive Level: Knowledge/Remembering REF: p. 505 OBJ: Nursing Process: Assessment MSC: Client Needs: Physiologic Integrity

4. As a result of changes in health care delivery and funding, a current trend seen in the pediatric setting is a. increased hospitalization of children. b. decreased number of children living in poverty. c. an increase in ambulatory care. d. decreased use of managed care.

C One effect of managed care has been that pediatric health care delivery has shifted dramatically from the acute care setting to the ambulatory setting in order to provide more cost-efficient care. The number of hospital beds being used has decreased as more care is given in outpatient settings and in the home. The number of children living in poverty has increased over the past decade. One of the biggest changes in health care has been the growth of managed care. PTS: 1 DIF: Cognitive Level: Knowledge/Remembering REF: p. 5 OBJ: Nursing Process: Planning MSC: Client Needs: Safe and Effective Care Environment

7. Which is the most appropriate question to ask when interviewing an adolescent to encourage conversation? a. "Are you in school?" b. "Are you doing well in school?" c. "How is school going for you?" d. "How do your parents feel about your grades?"

C Open-ended questions encourage communication. Questions with "yes" or "no" answers do not encourage conversation. Questions that can be interpreted as judgmental do not enhance communication. Asking adolescents about their parents' feelings may block communication. PTS: 1 DIF: Cognitive Level: Comprehension/Understanding REF: Table 4.3 OBJ: Integrated Process: Communication and Documentation MSC: Client Needs: Psychosocial Integrity

5. Which patient situation presents the greatest risk for the occurrence of hypotonic dysfunction during labor? a. A primigravida who is 17 years old b. A 22-year-old multiparous woman with ruptured membranes c. A multiparous woman at 39 weeks of gestation who is expecting twins d. A primigravida woman who has requested no analgesia during her labor

C Overdistention of the uterus in a multiple pregnancy is associated with hypotonic dysfunction because the stretched uterine muscle contracts poorly. A young primigravida usually will have good muscle tone in the uterus. This prevents hypotonic dysfunction. There is no indication that this woman's uterus is overdistended, which is the main cause of hypotonic dysfunction. A primigravida usually will have good uterine muscle tone, and there is no indication of an overdistended uterus. PTS: 1 DIF: Cognitive Level: Comprehension/Understanding REF: p. 578 OBJ: Nursing Process: Assessment MSC: Client Needs: Physiologic Integrity

7. Overstimulation may cause increased oxygen use in a preterm infant. Which nursing intervention helps to avoid this problem? a. Group all care activities together to provide long periods of rest. b. While giving your report to the next nurse, stand in front of the incubator and talk softly about how the infant responds to stimulation. c. Teach the parents signs of overstimulation, such as turning the face away or stiffening and extending the extremities and fingers. d. Keep charts on top of the incubator so the nurses can write on them there.

C Parents should be taught these signs of overstimulation so they will learn to adapt their care to the needs of their infant. This may understimulate the infant during those long periods and overtire the infant during the procedures. Talking in front of the incubator could overstimulate the baby. Placing objects on top of the incubator or using it as a writing surface increases the noise inside. PTS: 1 DIF: Cognitive Level: Application/Applying REF: p. 629 OBJ: Nursing Process: Implementation MSC: Client Needs: Health Promotion and Maintenance

4. A preterm infant is on a respirator with intravenous lines and much equipment around her when her parents come to visit for the first time. What action by the nurse is most important? a. Suggest that the parents visit for only a short time to reduce their anxieties. b. Reassure the parents that the baby is progressing well. c. Encourage the parents to touch her. d. Discuss the care they will give her when she goes home.

C Physical contact with the infant is important to establish early bonding. The nurse as the support person and teacher is responsible for shaping the environment and making the care giving responsive to the needs of both the parents and the infant. The nurse should encourage the parents to touch their baby and show them how to do so safely. Bonding needs to occur, and this can be fostered by encouraging the parents to spend time with the infant. It is important to keep the parents informed about the infant's progression, but the nurse needs to be honest with the explanations. Discussing home care needs to wait until the parents are ready and discharge is closer with known needs. PTS: 1 DIF: Cognitive Level: Application/Applying REF: p. 631 OBJ: Nursing Process: Implementation MSC: Client Needs: Psychosocial Integrity

4. A first-time dad is concerned that his 3-day-old daughter's skin looks "yellow." In the nurse's explanation of physiologic jaundice, what fact should be included? a. Physiologic jaundice occurs during the first 24 hours of life. b. Physiologic jaundice is caused by blood incompatibilities between the mother and infant blood types. c. The bilirubin levels of physiologic jaundice peak between the second and fourth days of life. d. This condition is also known as "breast milk jaundice."

C Physiologic jaundice becomes visible when the serum bilirubin reaches a level of 5 mg/dL or greater, which occurs when the baby is approximately 3 days old. This finding is within normal limits for the newborn. Pathologic jaundice occurs during the first 24 hours of life. Pathologic jaundice is caused by blood incompatibilities, causing excessive destruction of erythrocytes, and must be investigated. Breast milk jaundice occurs in one third of breastfed infants at 2 weeks and is caused by an insufficient intake of fluids. PTS: 1 DIF: Cognitive Level: Comprehension/Understanding REF: p. 433 OBJ: Integrated Process: Teaching-Learning MSC: Client Needs: Physiologic Integrity

8. The fastest growing group of homeless people is a. men and women preparing for retirement. b. migrant workers. c. single women and their children. d. intravenous (IV) substance abusers.

C Pregnancy and birth, especially for a teenager, are important contributing factors for becoming homeless. People preparing for retirement, migrant workers, and IV substance abusers are not among the fastest growing groups of homeless people. PTS: 1 DIF: Cognitive Level: Knowledge/Remembering REF: p. 14 OBJ: Nursing Process: Assessment MSC: Client Needs: Physiologic Integrity

5. A nurse in labor and delivery learns about metabolic changes that occur throughout pregnancy in diabetes. What information does the nurse know? a. Insulin crosses the placenta to the fetus only in the first trimester, after which the fetus secretes its own. b. Women with insulin-dependent diabetes are prone to hyperglycemia during the first trimester, because they are consuming more sugar. c. During the second and third trimesters, pregnancy exerts a diabetogenic effect that ensures an abundant supply of glucose for the fetus. d. Maternal insulin requirements steadily decline during pregnancy.

C Pregnant women develop increased insulin resistance during the second and third trimesters. Insulin never crosses the placenta; the fetus starts making its own around the tenth week. As a result of normal metabolic changes during pregnancy, insulin-dependent women are prone to hypoglycemia. Maternal insulin requirements may double or quadruple by the end of pregnancy. PTS: 1 DIF: Cognitive Level: Knowledge/Remembering REF: p. 549 OBJ: Integrated Process: Teaching-Learning MSC: Client Needs: Physiologic Integrity

9. A nurse wishes to work to reduce infant mortality in the United States. Which activity would this nurse most likely participate in? a. Creating pamphlets in several different languages using an interpreter. b. Assisting women to enroll in Medicaid by their third trimester. c. Volunteering to provide prenatal care at community centers. d. Working as an intake counselor at a women's shelter.

C Prenatal care is vital to reducing infant mortality and medical costs. This nurse would most likely participate in community service providing prenatal care outreach activities in community centers, particularly in low-income areas. Pamphlets in other languages, enrolling in Medicaid, and working at a women's shelter all might impact infant mortality, but the greatest effect would be from assisting women to get consistent prenatal care. PTS: 1 DIF: Cognitive Level: Application/Applying REF: p. 14 OBJ: Nursing Process: Implementation MSC: Client Needs: Health Promotion and Maintenance 10. The intrapartum woman sees no need for a routine admission fetal monitoring strip. If she continues to refuse, what is the first action the nurse should take? a. Consult the family of the woman. b. Notify the provider of the situation. c. Document the woman's refusal in the nurse's notes. d. Make a referral to the hospital ethics committee. ANS: B Patients must be allowed to make choices voluntarily without undue influence or coercion from others. The physician, especially if unaware of the patient's decision, should be notified immediately. Both professionals can work to ensure the mother understands the rationale for the action and the possible consequences of refusal. The woman herself is the decision maker, unless incapacitated. Documentation should occur but is not the first action. This situation does not rise to the level of an ethical issue so there is no reason to call the ethics committee. PTS: 1 DIF: Cognitive Level: Application/Applying REF: p. 18 OBJ: Nursing Process: Implementation MSC: Client Needs: Safe and Effective Care Environment 11. Which statement is true regarding the "quality assurance" or "incident" report? a. The report assures the legal department that no problem exists. b. Reports are a permanent part of the patient's chart. c. The nurse's notes should contain, "Incident report filed, and copy placed in chart." d. This report is a form of documentation of an event that may result in legal action. ANS: D An incident report is used when something occurs that might result in legal action, such as a patient fall or medication error. It warns the legal department that there may be a problem in a particular patient's care. Incident reports are not part of the patient's chart; thus the nurses' notes should not contain any reference to them. PTS: 1 DIF: Cognitive Level: Knowledge/Remembering REF: p. 18 OBJ: Integrated Process: Communication and Documentation MSC: Client Needs: Safe and Effective Care Environment 12. Elective abortion is considered an ethical issue because a. abortion law is unclear about a woman's constitutional rights. b. the Supreme Court ruled that life begins at conception. c. a conflict exists between the rights of the woman and the rights of the fetus. d. it requires third-party consent. ANS: C Elective abortion is an ethical dilemma because two opposing courses of action are available. The belief that induced abortion is a private choice is in conflict with the belief that elective pregnancy termination is taking a life. Abortion laws are clear concerning a woman's constitutional rights. The Supreme Court has not ruled on when life begins. Abortion does not require third-party consent. PTS: 1 DIF: Cognitive Level: Knowledge/Remembering REF: p. 11 OBJ: Integrated Process: Teaching-Learning MSC: Client Needs: Safe and Effective Care Environment 13. Which woman would be most likely to seek prenatal care? a. A 15-year-old who tells her friends, "I don't believe I'm pregnant." b. A 20-year-old who is in her first pregnancy and has access to a free prenatal clinic. c. A 28-year-old who is in her second pregnancy and abuses drugs and alcohol. d. A 30-year-old who is in her fifth pregnancy and delivered her last infant at home. ANS: B The patient who acknowledges the pregnancy early, has access to health care, and has no reason to avoid health care is most likely to seek prenatal care. Being in denial about the pregnancy increases the risk of not seeking care. This patient is also 15, and other social factors may discourage her from seeking care as well. Women who abuse substances are less likely to receive prenatal care. Some women see pregnancy and delivery as a natural occurrence and do not seek health care. PTS: 1 DIF: Cognitive Level: Comprehension/Understanding REF: p. 14 OBJ: Nursing Process: Assessment MSC: Client Needs: Health Promotion and Maintenance 14. A woman who delivered her baby 6 hours ago complains of headache and dizziness. The nurse administers an analgesic but does not perform any assessments. The woman then has a tonic-clonic seizure, falls out of bed, and fractures her femur. How would the actions of the nurse be interpreted in relation to standards of care? a. Negligent: the nurse failed to assess the woman for possible complications b. Negligent: because the nurse medicated the woman c. Not negligent: the woman had signed a waiver concerning the use of side rails d. Not negligent: the woman did not inform the nurse of her symptoms as soon as they occurred ANS: A There are four elements to malpractice, which is negligence in the performance of professional duties: duty, breach of duty, damage, and proximate cause. The nurse was negligent because she or he did not perform any assessments, which is the first step of the nursing process and is a standard of care. By not assessing the patient, the nurse did not meet established standards of care, and thus is guilty of professional negligence, or malpractice. PTS: 1 DIF: Cognitive Level: Knowledge/Remembering REF: p. 16 OBJ: Nursing Process: Evaluation MSC: Client Needs: Safe and Effective Care Environment 15. Which patient situation fails to meet the first requirement of informed consent? a. The patient does not understand the physician's explanations. b. The physician gives the patient only a partial list of possible side effects and complications. c. The patient is confused and disoriented. d. The patient signs a consent form because her husband tells her to. ANS: C The first requirement of informed consent is that the patient must be competent to make decisions about health care. Full disclosure of information is an important element of the consent, but first the patient has to be competent to sign. Understanding is an important element of the consent, but first the patient has to be competent to sign. Voluntary consent is an important element of the consent, but first the patient has to be competent to sign. PTS: 1 DIF: Cognitive Level: Knowledge/Remembering REF: p. 17 OBJ: Nursing Process: Assessment MSC: Client Needs: Safe and Effective Care Environment 16. Which situation reflects a potential ethical dilemma for the nurse? a. A nurse administers analgesics to a patient with cancer as often as the provider's order allows. b. A neonatal nurse provides nourishment and care to a newborn who has a defect that is incompatible with life. c. A labor nurse, whose religion opposes abortion, is asked to assist with an elective abortion. d. A postpartum nurse provides information about adoption to a new mother who feels she cannot adequately care for her infant. ANS: C A dilemma exists in this situation because the nurse is being asked to assist with a procedure that she or he believes is morally wrong. The other situations do not contain elements of conflict for the nurse. PTS: 1 DIF: Cognitive Level: Comprehension/Understanding REF: p. 11 OBJ: Nursing Process: Assessment MSC: Client Needs: Safe and Effective Care Environment 17. When planning a parenting class, the nurse should explain that the leading cause of death in children 1 to 4 years of age in the United States is a. premature birth. b. congenital anomalies. c. accidental death. d. respiratory tract illness. ANS: C Although the rates have dropped, unintentional injury (accidents) are still the leading cause of death for children aged 1 to 19. The other options contribute to morbidity and mortality in children but are not the leading cause. PTS: 1 DIF: Cognitive Level: Knowledge/Remembering REF: p. 10 | Table 1.3 OBJ: Integrated Process: Teaching-Learning MSC: Client Needs: Safe and Effective Care Environment 18. A nurse is floated to a different unit. The nurse does not know how to perform a treatment that has been prescribed for one of his or her assigned patients. What should the nurse's first action be? a. Delay the treatment until another nurse can do it. b. Make the child's parents aware of the situation. c. Inform the nursing supervisor of the problem. d. Arrange to have the child transferred to another unit. ANS: C Nurses who work outside their usual areas of expertise must assess their own skills and avoid performing tasks or taking on responsibilities in areas in which they are not competent. This nurse should inform the supervisor of the situation. The nurse could endanger the child by delaying the intervention until another nurse is available. Telling the child's parents would most likely increase their anxiety and will not resolve the difficulty. Transfer to another unit delays needed treatment and would create unnecessary disruption for the child and family. PTS: 1 DIF: Cognitive Level: Application/Applying REF: p. 19 OBJ: Nursing Process: Implementation MSC: Client Needs: Safe and Effective Care Environment 19. The mother of a 5-year-old female inpatient on the pediatric unit asks the nurse if she could provide information regarding the recommended amount of television viewing time for her daughter. The nurse responds that the appropriate amount of time a child should be watching television is a. 1 to 2 hours per day. b. 2 to 3 hours per day. c. 3 to 4 hours per day. d. 4 hours or more. ANS: A The American Academy of Pediatrics (2013) encourages parents to monitor their children's media exposure and limit their children's screen time (TV, computer, video games) to no more than 1 to 2 hours per day. The other options all contain more screen time than is recommended. PTS: 1 DIF: Cognitive Level: Comprehension/Understanding REF: p. 15 OBJ: Integrated Process: Teaching-Learning MSC: Client Needs: Health Promotion and Maintenance 20. Family-centered care (FCC) describes safe, quality care that recognizes and adapts to both the physical and psychosocial needs of the family. Which nursing practice coincides with the principles of FCC? a. The newborn is returned to the nursery at night so that the mother can receive adequate rest before discharge. b. The father is encouraged to go home after the baby is delivered. c. All patients are routinely placed on the fetal monitor. d. The nurse's assignment includes both mom and baby and increases the nurse's responsibility for education. ANS: D Family-centered care increases the responsibilities of nurses. In addition to the physical care provided, nurses assume a major role in teaching, counseling, and supporting families. The other options do not provide family-centered care because they increase family separation or use technology routinely, which may not be needed. PTS: 1 DIF: Cognitive Level: Comprehension/Understanding REF: p. 2 OBJ: Integrated Process: Caring MSC: Client Needs: Health Promotion and Maintenance 21. Which statement related to nursing care of the child at home is most correct? a. The technology-dependent infant can safely be cared for at home. b. Home care increases readmissions to the hospital for a child with chronic conditions. c. There is increased stress for the family when a sick child is being cared for at home. d. The family of the child with a chronic condition is likely to be separated from their support system if the child is cared for at home. ANS: A Greater numbers of technology-dependent infants and children are now cared for at home. The numbers include those needing ventilator assistance, total parenteral nutrition, IV medications, apnea monitoring, and other device-assisted nursing care. Optimal home care can reduce the rate of readmission to the hospital for children with chronic conditions. Consumers often prefer home care because of the decreased stress on the family when the patient is able to remain at home. When the child is cared for at home the family is less likely to be separated from their support system because of the need for hospitalization. PTS: 1 DIF: Cognitive Level: Comprehension/Understanding REF: p. 7 OBJ: Nursing Process: Planning MSC: Client Needs: Safe and Effective Care Environment 22. Maternity nursing care that is based on knowledge gained through research is known as a. nurse-sensitive indicators. b. evidence-based practice. c. case management. d. outcomes management. ANS: B Evidence-based practice is based on knowledge gained from research and clinical trials. Nurse-sensitive indicators are patient care outcomes particularly dependent on the quality and quantity of nursing care provided. Case management is a practice model that uses a systematic approach to identify specific patients, determine eligibility for care, and arrange access to services. The determination to lower health care costs while maintaining the quality of care has led to a clinical practice model known as outcomes management. PTS: 1 DIF: Cognitive Level: Comprehension/Understanding REF: p. 6 OBJ: Integrated Process: Teaching-Learning MSC: Client Needs: Safe and Effective Care Environment 23. The level of practice a reasonably prudent nurse provides is called a. the standard of care. b. risk management. c. a sentinel event. d. failure to rescue. ANS: A Guidelines for standards of care are published by various professional nursing organizations. The standard of care for neonatal nurses is set by the Association of Women's Health, Obstetric, and Neonatal Nurses (AWHONN). The Society of Pediatric Nurses is the primary specialty organization that sets standards for the pediatric nurse. Risk management identifies risks and establishes preventive practices, but it does not define the standard of care. Sentinel events and failure to rescue can be caused by not practicing up to standards of care, but they do not define it. PTS: 1 DIF: Cognitive Level: Knowledge/Remembering REF: p. 16 OBJ: Nursing Process: Implementation MSC: Client Needs: Safe and Effective Care Environment MULTIPLE RESPONSE

1. Childbirth preparation can be considered successful if the outcome is described as follows: a. Labor and delivery were pain-free. b. The woman's partner participated eagerly. c. The woman rehearsed labor and practiced skills to master pain. d. Only nonpharmacologic methods for pain control were used.

C Preparation allows the woman to rehearse for labor and to learn new skills to cope with the pain of labor and the expected behavioral changes. Childbirth preparation does not guarantee a pain-free labor. A woman should be prepared for pain and anesthesia/analgesia realistically. The partner's role and participation level should be established by the couple. Women will not always achieve their desired level of pain control by using nonpharmacologic methods alone. PTS: 1 DIF: Cognitive Level: Evaluation/Evaluating REF: p. 356 OBJ: Nursing Process: Evaluation MSC: Client Needs: Psychosocial Integrity

9. What is an appropriate preoperative teaching plan for a school-age child? a. Begin preoperative teaching the morning of surgery. b. Schedule a tour of the hospital a few weeks before surgery. c. Show the child books and pictures 4 days before surgery. d. Limit teaching to 5 minutes and use simple terminology.

C Preparatory material can be introduced to the school-age child several days (1 to 5) in advance of the event. Books, pictures, charts, and videos are appropriate. Preoperative teaching a few hours before surgery is more appropriate for the preschool child. Preoperative materials should be introduced 1 to 5 days in advance for school-age children. Preparation too far in advance of the procedure can be forgotten or cause undue anxiety for an extended period of time. A very short, simple explanation of the surgery is appropriate for a younger child such as a toddler. PTS: 1 DIF: Cognitive Level: Comprehension/Understanding REF: Table 4.3 OBJ: Nursing Process: Planning MSC: Client Needs: Health Promotion and Maintenance 10. When a child broke her favorite doll during a hospitalization, her primary nurse bought the child a new doll and gave it to her the next day. What is the best interpretation of the nurse's behavior? a. The nurse is displaying signs of overinvolvement. b. The nurse is a kind and generous person. c. The nurse feels a special closeness to the child. d. The nurse wants to make the child happy. ANS: A Buying gifts for individual children is a warning sign of overinvolvement. Nurses are kind and generous people, but buying gifts for individual children is unprofessional. Nurses may feel closer to some patients and families. This does not make giving gifts to children or families acceptable from a professional standpoint. Replacing lost items is not the nurse's responsibility. Becoming overly involved with a child can inhibit a healthy relationship. PTS: 1 DIF: Cognitive Level: Knowledge/Remembering REF: Box 4.2 OBJ: Nursing Process: Assessment MSC: Client Needs: Psychosocial Integrity 11. When meeting a toddler for the first time, the nurse initiates contact by a. calling the toddler by name and picking the toddler up. b. asking the toddler for his or her first name. c. kneeling in front of the toddler and speaking softly to the child. d. telling the toddler that you are his or her nurse today. ANS: C More positive interactions occur when the toddler perceives the meeting in a nonthreatening way. Placing yourself at the toddler's level and speaking softly can be less threatening for the child. Picking a toddler up at an initial meeting is a threatening action and will more likely result in a negative response from the child. Toddlers are unlikely to respond to direct questions at a first meeting. Telling the toddler you are the nurse is not likely to facilitate or encourage cooperation. The toddler perceives you as a stranger and will find the action threatening. PTS: 1 DIF: Cognitive Level: Application/Applying REF: p. 48 OBJ: Nursing Process: Implementation MSC: Client Needs: Psychosocial Integrity 12. An 8-year-old girl asks the nurse how the blood pressure apparatus works. The most appropriate nursing action is to a. ask why the child wants to know. b. determine why the child is so anxious. c. explain in simple terms how it works. d. tell the child he or she will see how it works as it is used. ANS: C School-age children require explanations and reasons for everything. They are interested in the functional aspect of all procedures, objects, and activities. It is appropriate for the nurse to explain how equipment works and what will happen to the child. The nurse should respond positively for requests for information about procedures and health information. By not responding, the nurse may be limiting communication with the child. The child is not exhibiting anxiety, just requesting clarification of what will be occurring. The nurse must explain how the blood pressure cuff works so that the child can then observe during the procedure. PTS: 1 DIF: Cognitive Level: Comprehension/Understanding REF: Table 4.3 OBJ: Nursing Process: Implementation MSC: Client Needs: Health Promotion and Maintenance 13. A positive, supportive communication technique that is effective from birth throughout adulthood is a. listening. b. physical proximity. c. environment. d. touch. ANS: D Touch can convey warmth, comfort, reassurance, security, caring, and support. In infancy, messages of security and comfort are conveyed when they are being held. Toddlers and preschoolers find it soothing and comforting to be held and rocked. School-aged children and adolescents appreciate receiving a hug or pat on the back (with permission). Listening is an essential component of the communication process. By practicing active listening skills, nurses can be effective listeners. Listening is a component of verbal communication. Individuals have different comfort zones for physical distance. The nurse should be aware of these differences and move cautiously when meeting new children and families. It is important to create a supportive and friendly environment for children including the use of child-sized furniture, posters, developmentally appropriate toys, and art displayed at a child's eye level. PTS: 1 DIF: Cognitive Level: Comprehension/Understanding REF: p. 48 OBJ: Integrated Process: Communication and Documentation MSC: Client Needs: Health Promotion and Maintenance 14. A nurse is caring for a child who does not speak English. The parents are able to understand and speak only limited English. What action by the nurse is best? a. Allow the patient's 12-year-old sister to interpret. b. See if there is another family member who can interpret. c. Use a professionally trained interpreter for this family. d. Use the Internet to translate written information in the native language. ANS: C A professional interpreter is the best option in this situation. They are trained in medical interpreting and do not allow cultural influences into their work. A child should never be asked to interpret; the child may be too young to understand sophisticated concepts involved in the discussion and the information from the patient may be misconstrued and disturbing to the child. An adult family member may have to do temporarily in an emergency, but the best option is a professional interpreter. PTS: 1 DIF: Cognitive Level: Knowledge/Remembering REF: p. 53 OBJ: Integrated Process: Communication and Documentation MSC: Client Needs: Psychosocial Integrity MULTIPLE RESPONSE

7. Which factor is known to increase the risk of gestational diabetes mellitus? a. Underweight before pregnancy b. Maternal age younger than 25 years c. Previous birth of large infant d. Previous diagnosis of type 2 diabetes mellitus

C Previous birth of a large infant suggests gestational diabetes mellitus. Obesity (BMI of 30 or greater) creates a higher risk for gestational diabetes. A woman younger than 25 generally is not at risk for gestational diabetes mellitus. The person with type 2 diabetes mellitus already is a diabetic and will continue to be so during and after pregnancy. PTS: 1 DIF: Cognitive Level: Knowledge/Remembering REF: p. 552 OBJ: Nursing Process: Assessment MSC: Client Needs: Physiologic Integrity

5. Which type of formula is not diluted before being administered to an infant? a. Powdered b. Concentrated c. Ready-to-use d. Modified cow's milk

C Ready-to-use formula can be poured directly from the can into baby's bottle and is good (but expensive) when a proper water supply is not available. Powdered and concentrated formulas should be well mixed to dissolve the powder and make it uniform. Cow's milk is more difficult for the infant to digest and is not recommended, even if it is diluted. PTS: 1 DIF: Cognitive Level: Knowledge/Remembering REF: p. 496 OBJ: Nursing Process: Implementation MSC: Client Needs: Physiologic Integrity

4. Of adolescents who become pregnant, what percentage have had a previous birth? a. 10% b. 15% c. 17% d. 35%

C Seventeen percent of pregnant adolescents have had one or more previous births. PTS: 1 DIF: Cognitive Level: Knowledge/Remembering REF: p. 500 OBJ: Nursing Process: Assessment MSC: Client Needs: Health Promotion and Maintenance

1. A pregnant woman who abuses cocaine admits to exchanging sex for her drug habit. This behavior puts her at a greater risk for which of the following? a. Depression of the central nervous system b. Hypotension and vasodilation c. Sexually transmitted diseases d. Postmature birth

C Sex acts exchanged for drugs place the woman at increased risk for sexually transmitted diseases because of multiple partners and lack of protection. Cocaine is a central nervous system stimulant. Cocaine causes hypertension and vasoconstriction. Premature delivery of the infant is one of the most common problems associated with cocaine use during pregnancy. PTS: 1 DIF: Cognitive Level: Knowledge/Remembering REF: p. 508 | Table 24.1 OBJ: Nursing Process: Assessment MSC: Client Needs: Health Promotion and Maintenance

5. Which statement by the nurse is most appropriate to a 15-year-old whose friend has mentioned suicide? a. "Tell your friend to come to the clinic immediately." b. "You need to gather details about your friend's suicide plan." c. "Your friend's threat needs to be taken seriously and he needs immediate help." d. "If your friend mentions suicide again get your friend some help."

C Suicide is the second most common cause of death among American adolescents and young adults aged 15 to 24. A suicide threat from an adolescent serves as a dramatic message to others and should be taken seriously. Adolescents at risk should be targeted for supportive guidance and counseling before a crisis occurs. Instructing a 15-year-old to tell a friend to come to the clinic immediately provides the teen with limited information and does not address the concern. The teen should not be responsible for getting more information from the friend. Waiting until the teen discusses suicide a second time may be too late. PTS: 1 DIF: Cognitive Level: Application/Applying REF: p. 163 OBJ: Integrated Process: Caring MSC: Client Needs: Psychosocial Integrity

3. Decreased surfactant production in the preterm lung is a problem because surfactant a. causes increased permeability of the alveoli. b. provides transportation for oxygen to enter the blood supply. c. keeps the alveoli open during expiration. d. dilates the bronchioles, decreasing airway resistance.

C Surfactant prevents the alveoli from collapsing each time the infant exhales, thus reducing the work of breathing. It does not cause increased permeability, provide transportation of oxygen or dilate the bronchioles. PTS: 1 DIF: Cognitive Level: Knowledge/Remembering REF: p. 635 OBJ: Nursing Process: Assessment MSC: Client Needs: Physiologic Integrity

1. The nurse is explaining Tanner staging to an adolescent and mother. Which statement best describes Tanner staging? a. Predictable stages of puberty that are based on chronologic age b. Staging of puberty based on the initiation of menarche and nocturnal emissions c. Predictable stages of puberty based on primary and secondary sexual characteristics d. Staging of puberty based on the initiation of primary sexual characteristics

C Tanner sexual-maturing ratings are based on the development of stages of primary and secondary sexual characteristics. Tanner stages are not based on chronologic age. The age at which an adolescent enters puberty is variable. The puberty stage in girls begins with breast development. Puberty stage in boys begins with genital enlargement. Primary sexual characteristics are not the basis of Tanner staging. PTS: 1 DIF: Cognitive Level: Comprehension/Understanding REF: p. 151 OBJ: Nursing Process: Assessment MSC: Client Needs: Health Promotion and Maintenance

8. Which statement is the most accurate about moral development in the 9-year-old school-age child? a. Right and wrong are based on physical consequences of behavior. b. The child obeys parents because of fear of punishment. c. The school-age child conforms to rules to please others. d. Parents are the determiners of right and wrong for the school-age child.

C The 7- to 12-year-old child bases right and wrong on a good-boy or good-girl orientation in which the child conforms to rules to please others and avoid disapproval. Children 4 to 7 years of age base right and wrong on consequences. Consequences are the most important consideration for the child between 4 and 7 years of age. Parents determine right and wrong for the child younger than 4 years of age. PTS: 1 DIF: Cognitive Level: Comprehension/Understanding REF: p. 135 OBJ: Nursing Process: Assessment MSC: Client Needs: Health Promotion and Maintenance

9. A patient at 24 weeks of gestation says she has a glass of wine with dinner every evening. The nurse will counsel her to eliminate all alcohol intake. What is the best rationale provided by the nurse? a. A daily consumption of alcohol indicates a risk for alcoholism. b. She will be at risk for abusing other substances as well. c. The fetus is placed at risk for altered brain growth. d. The fetus is at risk for multiple organ anomalies.

C The brain grows most rapidly in the third trimester and is most vulnerable to alcohol exposure during this time. A risk for alcoholism is not the major risk for the infant. Multiple organ anomalies are not a major concern. PTS: 1 DIF: Cognitive Level: Knowledge/Remembering REF: p. 508 | Table 24.1 OBJ: Nursing Process: Implementation MSC: Client Needs: Physiologic Integrity 10. Which of these substances can lead to miscarriage, preterm labor, placental separation (abruption), and stillbirth? a. Heroin b. Alcohol c. PCP d. Cocaine ANS: D Cocaine is a powerful CNS stimulant. Effects on pregnancy associated with cocaine use include abruptio placentae, preterm labor, precipitous birth, and stillbirth. Heroin is an opiate. Its use in pregnancy is associated with preeclampsia, intrauterine growth restriction, miscarriage, premature rupture of membranes, infections, breech presentation, and preterm labor. The most serious effect of alcohol use in pregnancy is FAS. The major concerns regarding PCP use in pregnant women are its association with polydrug abuse and the neurobehavioral effects on the neonate. PTS: 1 DIF: Cognitive Level: Comprehension REF: p. 507 | Table 24.1 OBJ: Nursing Process: Assessment MSC: Client Needs: Psychosocial Integrity 11. When helping the mother, father, and other family members actualize the loss of the infant, nurses should a. use the words lost or gone rather than dead or died. b. make sure the family understands that it is important to name the baby. c. if the parents choose to visit the baby, apply lotion to the baby and wrap the infant in a pretty blanket. d. set a firm time for ending the visit with the baby so that the parents know when to let go. ANS: C Presenting the baby in a nice way stimulates the parents' senses and provides pleasant memories of their baby. Nurses must use dead and died to assist the bereaved in accepting reality. Although naming the baby can be helpful, it is important not to create the sense that parents have to name the baby. In fact, some cultural taboos and religious rules prohibit the naming of an infant who has died. Parents need different time periods with their baby to say goodbye. Nurses need to be careful not to rush the process. PTS: 1 DIF: Cognitive Level: Comprehension/Understanding REF: p. 514 OBJ: Nursing Process: Planning MSC: Client Needs: Psychosocial Integrity 12. A woman has delivered twins. The first twin was stillborn, and the second is in the intensive care nursery and is recovering quickly from respiratory distress. The woman is crying softly and says, "I wish my baby could have lived." What is the most therapeutic response? a. "Don't be sad. At least you have one healthy baby." b. "How soon do you plan to have another baby?" c. "I have a friend who lost a twin and she's doing just fine now " d. "I am so sorry about your loss. Would you like to talk about it?" ANS: D The nurse should recognize the woman's grief and its significance and allow her to express her feelings. The other three responses belittle the woman's feelings. PTS: 1 DIF: Cognitive Level: Application/Applying REF: p. 514 OBJ: Integrated Process: Communication and Documentation MSC: Client Needs: Psychosocial Integrity 13. Which of the following is an appropriate nursing measure when a baby has an unexpected anomaly? a. Remove the baby from the delivery area immediately. b. Tell the parents that the baby has to go to the nursery immediately. c. Inform the parents immediately that something is wrong. d. Explain the defect, and show the baby to the parents as soon as possible. ANS: D Parents experience less anxiety when they are told about the defect as early as possible and are allowed to touch and hold the baby. The parents should be both informed and able to touch and hold the baby as soon as possible. PTS: 1 DIF: Cognitive Level: Application/Applying REF: p. 511 OBJ: Nursing Process: Implementation MSC: Client Needs: Psychosocial Integrity 14. A woman who is 6 months pregnant has sought medical attention saying she fell down the stairs. What scenario would cause an emergency department nurse to suspect that the woman has been battered? a. The woman and her partner are having an argument that is loud and hostile. b. The woman has injuries on various parts of her body in different stages of healing. c. Examination reveals a fractured arm and fresh bruises. Her husband asks her about her pain. d. She loudly complains about having several injuries. ANS: B The battered woman often has multiple injuries in various stages of healing. Arguing may or may not be sign of battering; many times the batterer will be attentive and refuse to leave the woman's side. A battered woman often has a flat affect or avoids eye contact and is vague about how the injuries occurred. PTS: 1 DIF: Cognitive Level: Analysis/Analyzing REF: p. 519 | Critical to Remember Box OBJ: Nursing Process: Assessment MSC: Client Needs: Psychosocial Integrity 15. Which of the following items are inconsistent with the nurse's knowledge of symptoms of fetal alcohol syndrome? a. Respiratory conditions b. Impaired growth c. CNS abnormality d. Facial abnormalities ANS: A Respiratory difficulties are not a category of conditions that are related to FAS. Abnormalities related to FAS include impaired growth (intrauterine growth restriction), CNS abnormalities, and a constellation of typical facial features. PTS: 1 DIF: Cognitive Level: Knowledge/Remembering REF: p. 508 | Table 24.1 OBJ: Nursing Process: Assessment MSC: Client Needs: Psychosocial Integrity 16. When the nurse is alone with a battered patient, the patient seems extremely anxious and says, "It was all my fault. The house was so messy when he got home and I know he hates that." The best response by the nurse is a. "No one deserves to be hurt. It's not your fault. How can I help you?" b. "What else do you do that makes him angry enough to hurt you?" c. "He will never find out what we talk about. Don't worry. We're here to help you." d. "You have to remember that he is frustrated and angry so he takes it out on you." ANS: A The nurse should stress that the patient is not at fault and offer to help. Asking what else the woman does to make the partner angry or reminding her that he is frustrated is placing blame on the woman. Telling her "don't worry" is giving false reassurance. PTS: 1 DIF: Cognitive Level: Application/Applying REF: p. 519 | Critical to Remember Box OBJ: Nursing Process: Implementation MSC: Client Needs: Psychosocial Integrity 17. In helping bereaved parents cope and move on, nurses should keep in mind that a. a perinatal or parental grief support group is more likely to be helpful if the needs of the parents are matched with the focus of the group. b. when pictures of the infant are taken for keepsakes, no close-ups should be taken of any congenital anomalies. c. no significant differences exist in grieving individuals from various cultures, ethnic groups, and religions. d. calling the hospital clergy for emergency baptism is always appropriate. ANS: A The nurse should try when possible to match the recommended support resources to the parents. For example, a religious-based group may not work for nonreligious parents. Close-up pictures of the baby must be taken as the infant was, congenital anomalies and all. Although death and grieving are events shared by all people, mourning rituals, traditions, and taboos vary by culture, ethnicity, and religion. Differences must be respected. Parents may or may not want the newborn to be baptized; the nurse must assess the family for their religious wishes and facilitate them. PTS: 1 DIF: Cognitive Level: Comprehension/Understanding REF: p. 512 OBJ: Nursing Process: Planning MSC: Client Needs: Psychosocial Integrity 18. A common effect of both smoking and cocaine use on the pregnant woman is a. vasoconstriction. b. increased appetite. c. inactivates fetal hemoglobin. d. euphoria. ANS: A Both smoking and cocaine use cause vasoconstriction, which results in impaired placental blood flow to the fetus. Both smoking and cocaine use decrease the appetite. Smoking inactivates fetal hemoglobin. Euphoria can be seen with cocaine use. PTS: 1 DIF: Cognitive Level: Knowledge/Remembering REF: p. 507 | Table 24.1 OBJ: Nursing Process: Assessment MSC: Client Needs: Physiologic Integrity 19. What information about caffeine in pregnancy does the nurse provide the prenatal class with? a. It stays in your body twice as long as when you are not pregnant. b. It causes vasoconstriction, which could keep the fetus from growing. c. Caffeine depresses your heart's ability to pump and function. d. Severe diuresis can leave you at risk for dehydration during pregnancy. ANS: B Caffeine is a vasoconstrictor. Its half-life is 3 times as long in the pregnant woman. It stimulates cardiac function. It does cause mild but not severe diuresis. PTS: 1 DIF: Cognitive Level: Comprehension/Understanding REF: p. 507 | Table 24.1 OBJ: Integrated Process: Teaching-Learning MSC: Client Needs: Health Promotion and Maintenance 20. The student nurse learns that the most important reason marijuana should not be used during pregnancy is which of the following? a. Unknown effects, more research is needed b. Causes a higher rate of spontaneous abortions c. Leads to multiple organ dysfunction in the newborn d. Responsible for severe cognitive deficits ANS: A Marijuana's effects on the fetus are largely unknown. More research is needed in this area. PTS: 1 DIF: Cognitive Level: Comprehension/Understanding REF: p. 507 | Table 24.1 OBJ: Integrated Process: Teaching-Learning MSC: Client Needs: Health Promotion and Maintenance 21. A nurse is interviewing a pregnant woman in the clinic. She seems hostile and answers many questions with "Whatever" and "I don't really know." At her last appointment she was late and disheveled. What action by the nurse is best? a. Ask the woman if this pregnancy was planned or is wanted. b. Call social services to come evaluate the situation. c. Ask the woman about drug use, including over the counter. d. Encourage the woman to be more forthright with answers. ANS: C This woman is displaying some signs of substance abuse. In a non-judgmental manner, the nurse should ask about all drugs and medications she is using. The questions will appear less confrontative if the nurse begins by asking about over-the-counter medications first. Asking if the pregnancy is planned or wanted is conveying disapproval to the woman for her choices and behavior. Social services may or may not need to be involved, but the nurse needs to assess the woman more completely first. Encouraging the woman to be more forthright implies that she is being dishonest and will not gain more cooperation. PTS: 1 DIF: Cognitive Level: Application/Applying REF: p. 509 | Critical to Remember Box OBJ: Nursing Process: Assessment MSC: Client Needs: Psychosocial Integrity MULTIPLE RESPONSE

9. A 35-year-old woman has an amniocentesis performed to find out whether her baby has a chromosome defect. Which statement by this patient indicates that she understands her situation? a. "The doctor will tell me if I should have an abortion when the test results come back." b. "I know support groups exist for parents who have a baby with birth defects, but we have plenty of insurance to cover what we need." c. "When all the lab results come back, my husband and I will make a decision about the pregnancy." d. "My mother must not find out about all this testing. If she does, she will think I'm having an abortion."

C The final decision about genetic testing and the future of the pregnancy lies with the patient. She will involve only those people whom she chooses in her decisions. The final decision about the future of the pregnancy lies with the patient only. Support groups are extremely important for parents of a baby with a defect. Insurance will help cover expenses, but the defect also takes a toll on the emotional, physical, and social aspects of the parents' lives. The nurse should ensure the woman understands that her care is confidential. PTS: 1 DIF: Cognitive Level: Evaluation/Evaluating REF: p. 180 OBJ: Nursing Process: Evaluation MSC: Client Needs: Health Promotion and Maintenance 10. Which question by the nurse will most likely promote sharing of sensitive information during a genetic counseling interview? a. "How many people in your family are mentally retarded or handicapped?" b. "What kinds of defects or diseases seem to run in the family?" c. "Did you know that you can always have an abortion if the fetus is abnormal?" d. "Are there any family members who have learning or developmental problems?" ANS: D The nurse should probe gently by using lay-oriented terminology rather than direct questions or statements. PTS: 1 DIF: Cognitive Level: Application REF: p. 180 OBJ: Nursing Process: Implementation MSC: Client Needs: Health Promotion and Maintenance 11. A maternal-newborn nurse is caring for a mother who just delivered a baby born with Down syndrome. What nursing diagnosis is the most essential in caring for the mother of this infant? a. Disturbed body image b. Interrupted family processes c. Anxiety d. Risk for injury ANS: B This mother likely will experience a disruption in the family process related to the birth of a baby with an inherited disorder. Family disruption is common, and the strain of having a child with a serious birth defect may lead to divorce. Siblings may feel neglected because the child with a disorder requires more of their parents' time and attention. Women commonly experience body image disturbances in the postpartum period, but this is unrelated to giving birth to a child with Down syndrome. The mother likely will have a mix of emotions that may include anxiety, guilt, and denial, but this is not the most essential nursing diagnosis for this family. Risk for injury is not applicable. PTS: 1 DIF: Cognitive Level: Application/Applying REF: p. 181 OBJ: Nursing Process: Diagnosis MSC: Client Needs: Psychosocial Integrity 12. A couple has been counseled for genetic anomalies. They ask the nurse, "What is karyotyping?" Which of the following is the nurse's best response? a. "Karyotyping will reveal if the baby's lungs are mature." b. "Karyotyping will reveal if your baby will develop normally." c. "Karyotyping will provide information about the number and structure of the chromosomes." d. "Karyotyping will detect any physical deformities the baby has." ANS: C Karyotyping provides genetic information, such as gender and chromosomal structure. Karyotyping is completed by photographing or using computer imaging to arrange chromosomes in pairs from largest to smallest. The karyotype can then be analyzed. Karyotyping does not determine lung maturity or if the baby is developing normally. Although karyotyping can detect genetic anomalies, not all such anomalies display obvious physical deformities. The term deformities is a nondescriptive word. Furthermore, physical anomalies may be present that are not detected by genetic studies (e.g., cardiac malformations). PTS: 1 DIF: Cognitive Level: Comprehension/Understanding REF: p. 172 OBJ: Integrated Process: Teaching-Learning MSC: Client Needs: Physiologic Integrity 13. In practical terms regarding genetic health care, nurses should be aware that a. genetic disorders equally affect people of all socioeconomic backgrounds, races, and ethnic groups. b. genetic health care is more concerned with populations than individuals. c. the most important related nursing function is providing emotional support to the family during counseling. d. taking genetic histories is usually only done at large universities and medical centers. ANS: C Nurses should be prepared to help with a variety of stress reactions from a couple facing the possibility of a genetic disorder. Although anyone may have a genetic disorder, certain disorders appear more often in certain ethnic and racial groups. Genetic health care is highly individualized, because treatments are based on the phenotypic responses of the individual. Individual nurses at any facility can take a genetic history and provide basic genetic information, although larger facilities may have better support services. PTS: 1 DIF: Cognitive Level: Knowledge/Remembering REF: p. 179 OBJ: Nursing Process: Planning MSC: Client Needs: Psychosocial Integrity 14. The nurse is working in an OB/GYN office and commonly obtains patient histories and performs initial assessments. Which woman is likely to be referred for genetic counseling after her first visit? a. A pregnant woman who will be 40 years or older when her infant is born b. A woman whose partner is 41 years of age c. A patient who carries a Y-linked disorder d. An anxious woman with a normal quadruple screening result ANS: B A genetics referral should be made if the woman's (male) partner is over the age of 40 at conception. Other reasons for referral include pregnant women who will be 35 or older at the time of birth or abnormal quadruple (or other) screening results. Women do not carry Y chromosomes. PTS: 1 DIF: Cognitive Level: Knowledge/Remembering REF: Box 10.4 OBJ: Nursing Process: Implementation MSC: Client Needs: Physiologic Integrity 15. A nurse is seeing a pregnant woman who has had genetic testing on her unborn fetus and has been given the results. The nurse notes the results confirm that the husband could not be the father. What action by the nurse is best? a. Do not discuss this information with the mother. b. Inform the mother genetic testing does not establish paternity. c. Call the husband immediately to break the news. d. Be available and offer support as the mother absorbs the news. ANS: D Genetic testing can reveal paternity; hopefully the couple was informed that this can occur before the testing was done. The nurse should offer support to the woman as she tries to absorb the news and determine what to do next. Refusing to discuss the information may leave the woman feeling abandoned and does not address her emotional needs. The nurse should not call the husband. PTS: 1 DIF: Cognitive Level: Application/Applying REF: Box 10.5 OBJ: Integrated Process: Caring MSC: Client Needs: Psychosocial Integrity 16. A nurse is creating a pedigree for a couple whose son has Tay-Sachs disease. What information from the pedigree would the nurse most likely find? a. Parental consanguinity b. Disease has skipped a generation. c. Only men have had this disorder. d. Only women have had this disorder. ANS: A Parental consanguinity increases the risk for autosomal recessive disorders such as Tay-Sachs disease. The pedigree would not show the disease skipping generations. Males and females are equally affected by this disorder. PTS: 1 DIF: Cognitive Level: Analysis/Analyzing REF: Box 10.1 OBJ: Nursing Process: Assessment MSC: Client Needs: Physiologic Integrity MULTIPLE RESPONSE

6. It is important for the nurse to understand that the levator ani is a(n) a. imaginary line that divides the true and false pelvis. b. basin-shaped structure at the lower end of the spine. c. collection of three pairs of muscles. d. division of the fallopian tube.

C The levator ani is a collection of three pairs of muscles that support internal pelvic structures and resist increases in intra-abdominal pressure. The linea terminalis is the imaginary line that divides the false pelvis from the true pelvis. The basin-shaped structure at the lower end of the spine is the bony pelvis. The fallopian tube divisions are the interstitial portion, isthmus, ampulla, and infundibulum. PTS: 1 DIF: Cognitive Level: Knowledge/Remembering REF: p. 189 OBJ: Nursing Process: Assessment MSC: Client Needs: Physiologic Integrity

3. When instructing parents on the correct use of a bulb syringe it is important include what information? a. Avoid suctioning the nares. b. Insert the compressed bulb into the center of the mouth. c. Suction the mouth first. d. Remove the bulb syringe from the crib when finished.

C The mouth should be suctioned first to prevent the infant from inhaling pharyngeal secretions by gasping as the nares are suctioned. The nasal passages should be suctioned one nostril at a time. The mouth should always be suctioned first. After compression of the bulb it should be inserted into one side of the mouth. If it is inserted into the center of the mouth, the gag reflex is likely to be initiated. The bulb syringe should remain in the crib so that it is easily accessible if needed again. PTS: 1 DIF: Cognitive Level: Application/Applying REF: p. 463 OBJ: Integrated Process: Teaching-Learning MSC: Client Needs: Health Promotion and Maintenance

7. In describing the size and shape of the nonpregnant uterus to a patient, the nurse would say it is approximately the size and shape of a a. cantaloupe. b. grapefruit. c. pear. d. large orange.

C The nonpregnant uterus is approximately 7.5 × 5.0 × 2.5 cm, which is close to the size and shape of a pear. PTS: 1 DIF: Cognitive Level: Knowledge/Remembering REF: p. 186 OBJ: Integrated Process: Teaching-Learning MSC: Client Needs: Physiologic Integrity

8. What is the first step in assisting the breastfeeding mother? a. Provide instruction on the composition of breast milk. b. Discuss the hormonal changes that trigger the milk ejection reflex. c. Assess the woman's knowledge of and feelings toward breastfeeding. d. Help her obtain a comfortable position and place the infant to the breast.

C The nurse should first assess the woman's knowledge and feedings toward breastfeeding to determine her teaching needs. Assessment should occur before instruction on positions and placing the infant to the breast. Education on hormonal changes and composition of breast milk also comes after assessment. PTS: 1 DIF: Cognitive Level: Knowledge/Remembering REF: p. 483 OBJ: Nursing Process: Assessment MSC: Client Needs: Health Promotion and Maintenance

7. The nurse's role in diagnostic testing is to provide a. advice to the couple. b. assistance with decision making. c. information about the tests. d. reassurance about fetal safety.

C The nurse should provide the couple with all necessary information about a procedure so that the couple can make an informed decision. The nurse's role is to inform, not to advise the couple. Decision making should always lie with the couple involved. Ensuring fetal safety is not possible with all of the diagnostic testing. To offer this is to give false reassurance to the parents. PTS: 1 DIF: Cognitive Level: Knowledge/Remembering REF: p. 286 OBJ: Nursing Process: Implementation MSC: Client Needs: Physiologic Integrity

2. When addressing the questions of a newly pregnant woman, the nurse can explain that the certified nurse-midwife is qualified to perform a. regional anesthesia. b. cesarean deliveries. c. vaginal deliveries. d. internal versions.

C The nurse-midwife is qualified to deliver infants vaginally in uncomplicated pregnancies. The other procedures must be performed by a physician or other medical provider. PTS: 1 DIF: Cognitive Level: Knowledge/Remembering REF: p. 26 OBJ: Integrated Process: Teaching-Learning MSC: Client Needs: Safe and Effective Care Environment

7. A student nurse in the perinatal clinic sees the term "pica" on a woman's chart and asks the registered nurse what this means. What definition is most accurate? a. Intolerance of milk products b. Iron deficiency anemia c. Ingestion of nonfood substances d. Episodes of anorexia and vomiting

C The practice of eating substances not normally thought of as food is called pica. Clay or dirt and solid laundry starch are the substances most commonly ingested. It is not intolerance of milk products, iron deficiency anemia, or episodes of anorexia and vomiting. PTS: 1 DIF: Cognitive Level: Comprehension/Understanding REF: p. 267 OBJ: Integrated Process: Teaching-Learning MSC: Client Needs: Physiologic Integrity

9. What results from the adaptation of the fetus to the size and shape of the pelvis? a. Lightening b. Lie c. Molding d. Presentation

C The sutures and fontanels allow the bones of the fetal head to move slightly, changing the shape of the fetal head so it can adapt to the size and shape of the pelvis. Lightening is the descent of the fetus toward the pelvic inlet before labor. Lie is the relationship of the long axis of the fetus to the long axis of the mother. Presentation is the fetal part that first enters the pelvic inlet. PTS: 1 DIF: Cognitive Level: Knowledge/Remembering REF: p. 293 OBJ: Nursing Process: Assessment MSC: Client Needs: Physiologic Integrity 10. The nurse assesses a patient whose cervix is dilated to 5 cm. What phase of labor does the nurse recognize the woman to be in? a. Latent phase b. Active phase c. Second stage d. Third stage ANS: B The active phase of labor is characterized by cervical dilation of 4 to 6 cm. The latent phase is from the beginning of true labor until 3 cm of cervical dilation. The second stage of labor begins when the cervix is completely dilated until the birth of the baby. The third stage of labor is from the birth of the baby until the expulsion of the placenta. PTS: 1 DIF: Cognitive Level: Knowledge/Remembering REF: p. 299 OBJ: Nursing Process: Assessment MSC: Client Needs: Health Promotion and Maintenance 11. To teach and support the woman in labor, the nurse explains that the strongest part of a labor contraction is the a. increment. b. acme. c. decrement. d. interval. ANS: B The acme is the peak or period of greatest strength during the middle of a contraction cycle. The increment is the beginning of the contractions until it reaches the peak. The decrement occurs after the peak until the contraction ends. The interval is the period between the end of the contraction and the beginning of the next. PTS: 1 DIF: Cognitive Level: Comprehension/Understanding REF: p. 289 OBJ: Integrated Process: Teaching-Learning MSC: Client Needs: Health Promotion and Maintenance 12. What assessment finding does the nurse expect in a woman with cervical dilation and effacement? a. Bloody show b. False labor c. Lightening d. Bladder distention ANS: A As the cervix begins to soften, dilate, and efface, expulsion of the mucous plug that sealed the cervix during pregnancy occurs. This causes rupture of small cervical capillaries, leading to bloody show. Cervical dilation and effacement do not occur with false labor. Lightening is the descent of the fetus toward the pelvic inlet before labor. Bladder distention occurs when the bladder is not emptied frequently. It may slow down the decent of the fetus during labor. PTS: 1 DIF: Cognitive Level: Knowledge/Remembering REF: Table 16.2 OBJ: Nursing Process: Assessment MSC: Client Needs: Health Promotion and Maintenance 13. The nurse is caring for a woman whose fetus has a breech presentation. What complication does the nurse prepare to assist with? a. Umbilical cord compression b. More rapid labor c. A high risk of infection d. Maternal perineal trauma ANS: A The umbilical cord can be compressed between the fetal body and the maternal pelvis when the body has been born but the head remains within the pelvis. Breech presentation is not associated with a more rapid labor. There is no higher risk of infection or perineal trauma with a breech birth. PTS: 1 DIF: Cognitive Level: Comprehension/Understanding REF: p. 295 OBJ: Nursing Process: Planning MSC: Client Needs: Health Promotion and Maintenance 14. The primary difference between the labor of a nullipara and that of a multipara is the a. amount of cervical dilation. b. total duration of labor. c. level of pain experienced. d. sequence of labor mechanisms. ANS: B Multiparas usually labor more quickly than nulliparas, making the total duration of their labor shorter. Cervical dilation and the sequence of labor mechanisms is the same for all labors. Level of pain is individual to the woman, not to the number of labors she has experienced. PTS: 1 DIF: Cognitive Level: Knowledge/Remembering REF: Table 16.1 OBJ: Nursing Process: Assessment MSC: Client Needs: Health Promotion and Maintenance 15. Which maternal factor may inhibit fetal descent and require further nursing interventions? a. Decreased peristalsis b. A full bladder c. Reduction in internal uterine size d. Rupture of membranes ANS: B A full bladder may inhibit fetal descent because it occupies space in the pelvis needed by the fetal presenting part. Peristalsis does not influence fetal descent. Contractions will reduce the internal uterine size in order to assist fetal descent. Rupture of membranes will assist in the fetal descent. PTS: 1 DIF: Cognitive Level: Knowledge/Remembering REF: p. 326 OBJ: Nursing Process: Assessment MSC: Client Needs: Health Promotion and Maintenance 16. Leopold's maneuvers are used by practitioners to determine a. the best location to assess the fetal heart rate (FHR). b. cervical dilation and effacement. c. whether the fetus is in the posterior position. d. if the woman needs an amniotomy. ANS: A Leopold's maneuvers are often performed before assessing the FHR. These maneuvers help identify the best location to obtain the FHR. Dilation and effacement are best determined by vaginal examination. Assessment of fetal position is more accurate with vaginal examination. Leopold's maneuvers are not used to determine if the woman needs an amniotomy. PTS: 1 DIF: Cognitive Level: Knowledge/Remembering REF: p. 305 OBJ: Nursing Process: Assessment MSC: Client Needs: Health Promotion and Maintenance 17. Which comfort measure should the nurse use to assist the laboring woman? a. Keep the room lights lit so that the patient and her coach can see everything. b. Offer warm, wet cloths to use on the patient's face and neck. c. Palpate her bladder every 15 minutes to assess for distention. d. Recommend frequent position changes. ANS: D Frequent maternal position changes reduce the discomfort from constant pressure and promote fetal descent. Soft, indirect lighting is more soothing than irritating bright lights. Women in labor become hot and perspire. Cool cloths are much better. A full bladder intensifies labor pain. The bladder should be emptied every 2 hours. PTS: 1 DIF: Cognitive Level: Comprehension/Understanding REF: p. 316 OBJ: Nursing Process: Implementation MSC: Client Needs: Physiologic Integrity 18. Which assessment finding could indicate hemorrhage in the postpartum patient? a. Firm fundus at the midline b. Saturation of one perineal pad in the hour after birth c. Elevated blood pressure d. Elevated pulse rate ANS: D An increasing pulse rate is an early sign of excessive blood loss. A firm fundus indicates that the uterus is contracting and compressing the open blood vessels at the placental site. Saturation of one pad within the first hour is the maximum normal amount of lochial flow. If the blood volume were diminishing, the blood pressure would decrease. However, this is a later finding. PTS: 1 DIF: Cognitive Level: Comprehension/Understanding REF: p. 326 OBJ: Nursing Process: Assessment MSC: Client Needs: Physiologic Integrity 19. What nursing intervention is the priority when caring for a laboring woman? a. Helping the woman find ways to manage the pain b. Eliminating the pain associated with labor c. Sharing personal experiences regarding labor and delivery d. Providing the woman food to restore her energy ANS: A Helping a woman manage the pain is an essential part of nursing care, because pain is an expected part of normal labor and cannot be fully relieved. Labor pain cannot be fully relieved. Sharing experiences can sometimes be appropriate, but managing the pain is the priority. Some women may want food during labor, and some may not, but this is not the priority. PTS: 1 DIF: Cognitive Level: Application REF: p. 288 OBJ: Nursing Process: Planning MSC: Client Needs: Health Promotion and Maintenance 20. A woman at 40 weeks of gestation calls the OB triage nurse to report a trickle of fluid from her vagina. What action by the nurse is most appropriate? a. Instruct the woman to come to the hospital. b. Ask her to time her contractions. c. Tell her if she saturates two pads in an hour to come to the hospital. d. Reassure her that she has plenty of time before delivery. ANS: A A trickle of fluid from the vagina may indicate rupture of the membranes requiring evaluation for infection or cord compression. Timing the contractions, waiting until she saturates two pads in an hour, and telling her there is plenty of time before she delivers are inappropriate actions and could lead to complications. PTS: 1 DIF: Cognitive Level: Application/Applying REF: p. 305 | Patient-Centered Teaching Box OBJ: Nursing Process: Implementation MSC: Client Needs: Health Promotion and Maintenance 21. The nurse is answering phone calls in the OB triage area. Which patient should the nurse advise to come to the hospital soonest after labor begins? a. Gravida 2 para 1 who lives 10 minutes away b. Gravida 1 para 0 who lives 40 minutes away c. Gravida 3 para 2 whose longest previous labor was 4 hours d. Gravida 2 para 1 whose first labor lasted 16 hours ANS: C Multiparous women usually have shorter labors than do nulliparous women. The woman described in option c is multiparous with a history of rapid labors, increasing the likelihood that her infant might be born in uncontrolled circumstances. The other women probably have more time. PTS: 1 DIF: Cognitive Level: Analysis/Analyzing REF: p. 305 OBJ: Nursing Process: Implementation MSC: Client Needs: Safe and Effective Care Environment 22. A primigravida at 39 weeks of gestation is observed for 2 hours in the intrapartum unit. The fetal heart rate has been normal. Contractions are 5 to 9 minutes apart, 20 to 30 seconds in duration, and of mild intensity. Cervical dilation is 1 to 2 cm and unchanged from admission. Membranes are intact. What action by the nurse is most appropriate? a. Prepare the woman for a cesarean birth. b. Admit the woman for extended observation. c. Discharge the woman with a sedative so she can rest. d. Provide discharge teaching on signs of true labor. ANS: D The situation describes a woman with normal assessments who is probably in false labor and will probably not deliver rapidly once true labor begins. There is no need to prepare her for a cesarean birth, admit her, or send her home with sedation. PTS: 1 DIF: Cognitive Level: Application/Applying REF: p. 298 OBJ: Nursing Process: Implementation MSC: Client Needs: Health Promotion and Maintenance 23. The nurse auscultates the fetal heart rate (FHR) and determines a rate of 152. Which nursing intervention is most appropriate? a. Document the findings in the chart. b. Reassess the FHR every 5 minutes. c. Report the FHR to the provider or nurse-midwife immediately. d. Apply oxygen and turn the mother on her left side. ANS: A The FHR is within the expected range; no further action is necessary at this point other than documenting the findings in the chart. PTS: 1 DIF: Cognitive Level: Application/Applying REF: Table 16.2 OBJ: Nursing Process: Implementation MSC: Client Needs: Physiologic Integrity 24. A laboring woman is lying in the supine position. The most appropriate nursing action is to a. ask her to turn to one side. b. elevate her feet and legs. c. take her blood pressure. d. let her stay in a position of comfort. ANS: A The woman's supine position may cause the heavy uterus to compress her inferior vena cava, reducing blood return to her heart and reducing placental blood flow. This problem is relieved by having her turn onto her side. The other actions will not prevent this from happening. PTS: 1 DIF: Cognitive Level: Application/Applying REF: p. 315 OBJ: Nursing Process: Implementation MSC: Client Needs: Physiologic Integrity 25. What finding should the nurse recognize as being associated with fetal compromise? a. Active fetal movements b. Contractions lasting 90 seconds c. FHR in the 140s d. Meconium-stained amniotic fluid ANS: D When fetal oxygen is compromised, relaxation of the rectal sphincter allows passage of meconium into the amniotic fluid. Active fetal movement is an expected occurrence. The fetus should be able to tolerate contractions lasting 90 seconds if the resting phase is sufficient to allow for a return of adequate blood flow. Expected FHR range is from 110 to 160. PTS: 1 DIF: Cognitive Level: Knowledge REF: p. 315 | Safety Alert Box OBJ: Nursing Process: Assessment MSC: Client Needs: Health Promotion and Maintenance 26. During the active phase of labor, the FHR of a low-risk patient should be assessed every a. 10 to 15 minutes. b. 15 to 30 minutes. c. 30 to 45 minutes. d. 1 hour. ANS: B For the fetus at low risk for complications, guidelines for frequency of assessments are at least every 15 to 30 minutes during the active phase of labor. PTS: 1 DIF: Cognitive Level: Knowledge/Remembering REF: p. 310 | Table 16.2 OBJ: Nursing Process: Implementation MSC: Client Needs: Health Promotion and Maintenance 27. Which nursing assessment indicates that a woman who is in second-stage labor is almost ready to give birth? a. The fetal head is felt at 0 station during vaginal examination. b. Bloody mucous discharge increases. c. The vulva bulges and encircles the fetal head. d. The membranes rupture during a contraction. ANS: C A bulging vulva that encircles the fetal head describes crowning, which occurs shortly before birth. Birth of the head occurs when the station is +4. A 0 station indicates engagement. Bloody show occurs throughout the labor process and is not an indication of an imminent birth. Rupture of membranes can occur at any time during the labor process and does not indicate an imminent birth. PTS: 1 DIF: Cognitive Level: Knowledge/Remembering REF: p. 300 OBJ: Nursing Process: Assessment MSC: Client Needs: Health Promotion and Maintenance 28. During labor, a vaginal examination should be performed only when necessary because of the risk of a. fetal injury. b. discomfort. c. infection. d. perineal trauma. ANS: C Vaginal examinations increase the risk of infection by carrying vaginal microorganisms upward toward the uterus. Properly performed vaginal examinations should not cause fetal injury. Vaginal examinations may be uncomfortable for some women in labor, but that is not the main reason for limiting them. A properly performed vaginal examination should not cause perineal trauma. PTS: 1 DIF: Cognitive Level: Knowledge/Remembering REF: p. 313 OBJ: Nursing Process: Implementation MSC: Client Needs: Safe and Effective Care Environment 29. A 25-year-old primigravida is in the first stage of labor. She and her husband have been holding hands and breathing together through each contraction. Suddenly the woman pushes her husband's hand away and shouts, "Don't touch me!" What action by the nurse is most appropriate? a. Reassure the husband this is normal in the transition phase. b. Ask the woman if she needs some pain medication. c. Call the anesthesia provider for an epidural block. d. Ask the husband to leave the room for a few minutes. ANS: A The transition phase of labor is often associated with an abrupt change in behavior, including increased anxiety and irritability. The woman may or may not need pain medication or an epidural, but the husband should be reassured. There is no need for the husband to be asked to leave. PTS: 1 DIF: Cognitive Level: Application/Applying REF: p. 299 OBJ: Nursing Process: Implementation MSC: Client Needs: Health Promotion and Maintenance 30. At 1 minute after birth, the nurse assesses the newborn to assign an Apgar score. The apical heart rate is 110 bpm, and the infant is crying vigorously with the limbs flexed. The infant's trunk is pink, but the hands and feet are blue. What is the Apgar score for this infant? a. 7 b. 8 c. 9 d. 10 ANS: C The baby received 2 points for each of the categories except color. Since the infant's hands and feet were blue this category is given a grade of 1. PTS: 1 DIF: Cognitive Level: Application/Applying REF: p. 326 | Table 16.3 OBJ: Nursing Process: Assessment MSC: Client Needs: Health Promotion and Maintenance 31. Thirty minutes after birth, the nurse assesses a woman's fundus as soft and boggy. What action by the nurse takes priority? a. Take the blood pressure. b. Massage the fundus. c. Notify the provider or nurse-midwife. d. Place the woman in the Trendelenburg position. ANS: B The nurse's first response should be to massage the fundus to stimulate contraction of the uterus to compress open blood vessels at the placental site, limiting blood loss. Blood pressure is important but not the priority. Notification should occur after all nursing measures have been attempted with no favorable results. The Trendelenburg position is contraindicated for this woman at this point. This position does not allow for appropriate vaginal drainage of lochia. The lochia remaining in the uterus would clot and produce further bleeding. PTS: 1 DIF: Cognitive Level: Application/Applying REF: p. 327 | Table 16.4 OBJ: Nursing Process: Implementation MSC: Client Needs: Physiologic Integrity 32. Thirty minutes after giving birth a woman's uterus feels boggy to the nurse. The nurse massages the fundus without change. What action does the nurse take next? a. Notify the provider or nurse-midwife immediately. b. Assess the woman for a full bladder. c. Prepare to administer oxytocin. d. Take a full set of vital signs. ANS: B After massaging the uterus, without result, the nurse should assess the woman to see if a full bladder is contributing to the uterine atony. The woman can then be catheterized to empty the bladder. None of the other actions is needed. PTS: 1 DIF: Cognitive Level: Application/Applying REF: p. 327 | Table 16.4 OBJ: Nursing Process: Implementation MSC: Client Needs: Safe and Effective Care Environment 33. The nurse thoroughly dries the infant immediately after birth primarily to a. stimulate crying and lung expansion. b. remove maternal blood from the skin surface. c. reduce heat loss from evaporation. d. increase blood supply to the hands and feet. ANS: C Infants are wet with amniotic fluid and blood at birth, which accelerates evaporative heat loss. Drying the infant with help maintain a normal temperature. Rubbing the infant does stimulate crying, but it is not the main reason for drying the infant. Removing maternal blood is also not the main reason for this action. It does not increase blood supply to the hands and feet. PTS: 1 DIF: Cognitive Level: Comprehension/Understanding REF: p. 325 OBJ: Nursing Process: Implementation MSC: Client Needs: Physiologic Integrity 34. The nurse notes that a woman who has given birth 1 hour ago is touching her infant with the fingertips and talking to him softly in high-pitched tones. On the basis of this observation, the nurse should a. document this evidence of normal early maternal-infant attachment behavior. b. observe for other signs that the mother may not be accepting of the infant. c. request a social service consult for psychosocial support. d. determine whether the mother is too fatigued to interact normally with her infant. ANS: A These are signs of normal attachment behavior; no other assessment or intervention is necessary at this point. PTS: 1 DIF: Cognitive Level: Comprehension REF: p. 329 OBJ: Nursing Process: Implementation MSC: Client Needs: Psychosocial Integrity 35. When assessing the fetus using Leopold maneuvers, the nurse feels a round, firm, movable fetal part in the fundal portion of the uterus and a long, smooth surface in the mother's right side close to midline. What is the likely position of the fetus? a. ROA b. LSP c. RSA d. LOA ANS: C Fetal position is denoted with a three-letter abbreviation. The first letter indicates the presenting part in either the right or left side of the maternal pelvis. The second letter indicates the anatomic presenting part of the fetus. The third letter stands for the location of the presenting part in relation to the anterior, posterior, or transverse portion of the maternal pelvis. This fetus is positioned anteriorly in the right side of the maternal pelvis with the sacrum as the presenting part. RSA is the correct three-letter abbreviation to indicate this fetal position. ROA denotes a fetus that is positioned anteriorly in the right side of the maternal pelvis with the occiput as the presenting part. LSP describes a fetus that is positioned posteriorly in the left side of the pelvis with the sacrum as the presenting part. A fetus that is LOA would be positioned anteriorly in the left side of the pelvis with the occiput as the presenting part. PTS: 1 DIF: Cognitive Level: Comprehension/Understanding REF: pp. 295-296 OBJ: Nursing Process: Assessment MSC: Client Needs: Health Promotion and Maintenance 36. The registered nurse tells the nursing student that which stage of labor varies most in length? a. First b. Second c. Third d. Fourth ANS: A The first stage is much longer than the second and third stages combined. In a first pregnancy, the first stage of labor can take up to 20 hours. However, there is great variability in length of time depending on many factors, including parity. PTS: 1 DIF: Cognitive Level: Comprehension/Understanding REF: p. 302 | Table 16.1 OBJ: Nursing Process: Planning MSC: Client Needs: Health Promotion and Maintenance 37. A pregnant woman is at 38 weeks of gestation. She wants to know if any signs indicate "labor is getting closer to starting." The nurse informs the woman that which of the following is a sign that labor may begin soon? a. Weight gain of 1.5 to 2 kg (3 to 4 lb) b. Increase in fundal height c. Urinary retention d. Surge of energy ANS: D Women speak of having a burst of energy before labor. The woman may lose 0.5 to 1.5 kg, the result of water loss caused by electrolyte shifts, which in turn are caused by changes in the estrogen and progesterone levels. When the fetus descends into the true pelvis (called lightening), the fundal height may decrease. Urinary frequency may return before labor. PTS: 1 DIF: Cognitive Level: Comprehension/Understanding REF: p. 298 OBJ: Nursing Process: Planning MSC: Client Needs: Health Promotion and Maintenance 38. At hand-off report the off-going nurse states that the patient demonstrated clonus on her last assessment. What action by the on-coming nurse takes priority? a. Repeat the woman's vital signs. b. Institute seizure precautions. c. Prepare for cesarean delivery. d. Assess for pain. ANS: B Clonus (repeated tapping when the foot is dorsiflexed) is usually associated with pregnancy-induced hypertension and may precede a seizure. The nurse should place the woman on seizure precautions. Vital signs should be assessed frequently, but this is not the priority. There is no need at this point for cesarean delivery, although that is possible. Assessing for pain is important but does not take priority over a safety measure. PTS: 1 DIF: Cognitive Level: Application/Applying REF: Table 16.2 OBJ: Nursing Process: Implementation MSC: Client Needs: Safe and Effective Care Environment 39. The labor and delivery nurse is evaluating a newly admitted woman's lab and notes a hemoglobin of 9.1 mg/dL and hematocrit of 31%. What action by the nurse takes priority? a. Document the findings on the woman's chart. b. Notify the provider or nurse-midwife immediately. c. Assess for response to blood loss during and after birth. d. Place the patient on bedrest during labor. ANS: C The normal values for a woman about to deliver are 10.5 mg/dL and 33%. Values lower than this indicate the maternal reserves may not be adequate for the normal blood loss in delivery. The nurse should use astute assessments during and after delivery to assess her response to the loss of blood. Documentation and notification should occur, but that is not the priority action. There is no indication that the woman should be restricted to bed during labor. PTS: 1 DIF: Cognitive Level: Analysis/Analyzing REF: Table 16.2 OBJ: Nursing Process: Implementation MSC: Client Needs: Safe and Effective Care Environment MULTIPLE RESPONSE

5. To evaluate the woman's learning about performing infant care, the nurse should a. demonstrate infant care procedures. b. allow the woman to verbalize the procedure. c. observe the woman as she performs the procedure. d. routinely assess the infant for cleanliness.

C The woman's ability to perform the procedure correctly under the nurse's supervision is the best method of evaluation. Demonstration is an excellent teaching method but not an evaluation method. During verbalization of the procedure, the nurse may not pick up on techniques that are incorrect. It is not the best tool for evaluation. Observing the infant for cleanliness does not ensure the proper procedure is carried out. The nurse may miss seeing unsafe techniques being used. PTS: 1 DIF: Cognitive Level: Evaluation/Evaluating REF: p. 31 OBJ: Nursing Process: Evaluation MSC: Client Needs: Health Promotion and Maintenance

5. Which suggestion is appropriate for the pregnant woman who is experiencing nausea and vomiting? a. Eat only three meals a day so the stomach is empty between meals. b. Drink plenty of fluids with each meal. c. Eat dry crackers or toast before arising in the morning. d. Drink coffee or orange juice immediately on arising in the morning.

C This will assist with the symptoms of morning sickness. It is also important for the woman to arise slowly. Instruct the woman to eat five to six small meals rather than three full meals per day. Nausea is more intense when the stomach is empty. Fluids should be taken separately from meals. Fluids overstretch the stomach and may precipitate vomiting. Coffee and orange juice stimulate acid formation in the stomach. It is best to suggest eating dry carbohydrates when rising in the morning. PTS: 1 DIF: Cognitive Level: Knowledge/Remembering REF: p. 230 OBJ: Nursing Process: Implementation MSC: Client Needs: Physiologic Integrity

6. The parent of a child who has had numerous hospitalizations asks the nurse for advice because the child has been having behavior problems at home and in school. In discussing effective discipline, what is an essential component? a. All children display some degree of acting out, and this behavior is normal. b. The child is manipulative and should have firmer limits set on her behavior. c. Positive reinforcement and encouragement should be used to promote cooperation and the desired behaviors. d. Underlying reasons for rules should be given, and the child should be allowed to decide which rules should be followed.

C Using positive reinforcement and encouragement to promote cooperation and desired behaviors is one of the three essential components of effective discipline. Behavior problems should not be disregarded as normal. It would be incorrect to assume the child is being manipulative and should have firmer limits set on her behaviors. Providing the underlying reasons for rules and giving the child a choice concerning which rules to follow constitute a component of permissive parenting and are not considered an essential component of effective discipline. PTS: 1 DIF: Cognitive Level: Comprehension/Understanding REF: p. 44 OBJ: Integrated Process: Teaching-Learning MSC: Client Needs: Psychosocial Integrity

8. While reviewing the dietary-intake documentation of a 7-year-old Asian patient with a fractured femur, the nurse notes that the patient consistently refuses to eat the food on his tray. What assumption is most likely accurate? a. The child is a picky eater. b. The child needs less food because of bed rest. c. The child may have culturally related food preferences. d. The child is probably eating between meals and spoiling his appetite.

C When cultural differences are noted, food preferences should always be obtained. A child will often refuse to eat unfamiliar foods. Although the child may be a picky eater, the key point is that there are cultural differences that need to be considered. The foods he is being served may seem strange to a child. Nutrition plays an important role in healing. Although the energy the child expends has decreased while on bed rest, he or she has increased needs for good nutrition. Although the nurse should determine whether the child is eating food the family has brought from home, the more important point is to determine whether there are culturally related food preferences. PTS: 1 DIF: Cognitive Level: Comprehension/Understanding REF: p. 38 OBJ: Integrated Process: Culture and Spirituality MSC: Client Needs: Psychosocial Integrity

3. What is a major barrier to health care for teen mothers? a. The hospital/clinic is within walking distance of the girl's home. b. The institution is open days, evenings, and Saturdays by special arrangement. c. The teen must be prepared to see a different nurse or doctor or both at every visit. d. The health care workers have a positive attitude.

C Whenever possible, the teen should be scheduled to see the same nurses and practitioners for continuity of care. If the hospital/clinic were within walking distance of the girl's home, it would prevent the teen from missing appointments because of transportation problems. If the institution were open days, evenings, and Saturdays by special arrangement, this availability would be helpful for teens who work, go to school, or have other time-of-day restrictions. Scheduling conflicts are a major barrier to health care. A negative attitude is unfortunate, because it discourages families who would benefit most from consistent prenatal care. PTS: 1 DIF: Cognitive Level: Knowledge/Remembering REF: p. 503 OBJ: Nursing Process: Assessment MSC: Client Needs: Psychosocial Integrity

7. A woman is having her first child. She has been in labor for 15 hours. Two hours ago, her vaginal examination revealed the cervix to be dilated to 5 cm and 100% effaced, and the presenting part was at station 0. Five minutes ago, her vaginal examination indicated that there had been no change. What abnormal labor pattern is associated with this description? a. Prolonged latent phase b. Protracted active phase c. Secondary arrest d. Protracted descent

C With a secondary arrest of the active phase, the progress of labor has stopped. This patient has not had any anticipated cervical change, indicating an arrest of labor. Dilation at 5 cm is past the latent phase. This does not describe a "protracted" labor. PTS: 1 DIF: Cognitive Level: Knowledge/Remembering REF: p. 573 OBJ: Nursing Process: Assessment MSC: Client Needs: Health Promotion and Maintenance

9. The student nurse asks why gloves are needed when handling a newborn because the newborn "hasn't been exposed to anything." What response by the nurse is best? a. It is part of standard precautions. b. It is hospital policy. c. Amniotic fluid and maternal blood pose risks to us. d. We are protecting the infant from our bacteria.

C With the possibility of transmission of viruses such as HBV and HIV through maternal blood and amniotic fluid, the newborn must be considered a potential contamination source until proved otherwise. As part of standard precautions, nurses should wear gloves when handling the newborn until blood and amniotic fluid are removed by bathing. While this may be policy and is part of standard precautions, simply stating these facts does not convey any detailed information. The nurses are not protecting the infant from themselves. PTS: 1 DIF: Cognitive Level: Comprehension/Understanding REF: p. 462 OBJ: Integrated Process: Teaching-Learning MSC: Client Needs: Safe and Effective Care Environment 10. With regard to lab tests and diagnostic tests in the hospital after birth, nurses should be aware that a. all states test for phenylketonuria (PKU), hypothyroidism, cystic fibrosis, and sickle cell diseases. b. federal law prohibits newborn genetic testing without parental consent. c. if genetic screening is done before the infant is 24 hours old, it should be repeated at age 1 to 2 weeks. d. hearing screening is now mandated by federal law. ANS: C If testing is done prior to 24 hours of age, genetic screening should be repeated when the infant is 1 to 2 weeks old. States all test for PKU and hypothyroidism, but other genetic defects are not universally covered. Federal law mandates newborn genetic screening; however, parents can decline testing. A waiver should be signed and a notation made in the infant's medical record. Federal law does not mandate screening for hearing problems; however, the majority of states have enacted legislation mandating newborn hearing screening. In the United States the majority (95%) of infants is screened for hearing loss prior to discharge from the hospital. PTS: 1 DIF: Cognitive Level: Knowledge/Remembering REF: p. 476 OBJ: Nursing Process: Planning MSC: Client Needs: Physiologic Integrity 11. Nurses can help parents deal with the issue and fact of circumcision if they explain a. the pros and cons of the procedure during the prenatal period. b. that the American Academy of Pediatrics (AAP) recommends that all newborn males be routinely circumcised. c. that circumcision is rarely painful and that any discomfort can be managed without medication. d. that the infant will likely be alert and hungry shortly after the procedure. ANS: A Parents need to make an informed choice regarding newborn circumcision based on the most current evidence and recommendations. Health care providers and nurses who care for childbearing families should provide factual, unbiased information regarding circumcision and give parents opportunities to discuss the risks and benefits of the procedure. The AAP and other professional organizations note the benefits but stop short of recommendation for routine circumcision. Circumcision is painful and must be managed with environmental, nonpharmacologic, and pharmacologic measures. Infants may or may not be alert and hungry after the procedure. PTS: 1 DIF: Cognitive Level: Comprehension/Understanding REF: p. 471 OBJ: Integrated Process: Teaching-Learning MSC: Client Needs: Health Promotion and Maintenance 12. A nurse is responsible for teaching new parents about the hygienic care of their newborn. What information does the nurse include? a. Avoid washing the head for at least 1 week to prevent heat loss. b. Sponge bathe only until the cord has fallen off. c. Cleanse the ears and nose with cotton-tipped swabs, such as Q-tips. d. Water temperature should be at least 38° C. ANS: D The ideal temperature of the bath water should be at least 38° C, or 100.4° F. The head can be washed. Tub baths may be initiated from birth. Ensure that the infant is fully immersed. Q-tips should not be used, because they may cause injury. A corner of a moistened washcloth should be twisted into shape so that it can be used to cleanse the ears and nose. PTS: 1 DIF: Cognitive Level: Application/Applying REF: p. 467 OBJ: Integrated Process: Teaching-Learning MSC: Client Needs: Physiologic Integrity 13. An unfortunate but essential role of the nurse is protecting the infant from abduction. Which statement regarding the profile of a potential abductor is the most accurate? a. Male gender b. A young woman who has had a previous pregnancy loss c. A middle-aged woman past childbearing age d. A female with a number of children of her own ANS: B The woman is usually of childbearing age and may have had a previous pregnancy loss or has been unable to have a child of her own. She may want an infant to solidify the relationship with her husband or boyfriend and may have pretended to be pregnant. The women are usually familiar with the facility and its routines. PTS: 1 DIF: Cognitive Level: Knowledge/Remembering REF: p. 469 OBJ: Nursing Process: Assessment MSC: Client Needs: Safe and Effective Care Environment 14. When the nurse is in the process of health teaching it is very important that he or she consider the family's cultural beliefs regarding child care. One of these beliefs includes that a. Arab women are anxious to breastfeed while still in the hospital. b. it is important to complement Asian parents about their new baby. c. women from India tie a black thread around the infant's waist. d. in the Korean culture the patient's mother is the primary caregiver of the infant. ANS: C Women from India may tie a black thread around the infant's wrist, ankle, or waist to ward off evil spirits. This thread should not be removed by the nurse. Arab women are hesitant to breastfeed in the birth facility and wish to wait until they are home and their milk comes in. Asian parents may be uneasy when caregivers are too complementary about the baby or casually touch the infant's head. In the Korean culture, the husband's mother is the primary caregiver for the infant and the mother during the early weeks. PTS: 1 DIF: Cognitive Level: Knowledge/Remembering REF: p. 475 OBJ: Integrated Process: Culture and Spirituality MSC: Client Needs: Psychosocial Integrity 15. Nursing follow-up care often includes home visits for the new mother and her infant. Which information related to home visits is correct? a. Ideally the visit is scheduled between 24 and 72 hours after discharge. b. Home visits are available in all areas. c. Visits are completed within a 30-minute time frame. d. Blood draws are not a part of the home visit. ANS: A The home visit is ideally scheduled during the first 24 to 72 hours after discharge. This timing allows early assessment and intervention for problems with feedings, jaundice, newborn adaptation, and maternal-infant interaction. Because home visits are expensive, they are not available in all geographic areas. Visits are usually 60 to 90 minutes in length to allow enough time for assessment and teaching. When jaundice is found, the nurse can discuss the implications and check the transcutaneous bilirubin level or draw blood for testing. PTS: 1 DIF: Cognitive Level: Knowledge/Remembering REF: p. 477 OBJ: Nursing Process: Planning MSC: Client Needs: Health Promotion and Maintenance 16. A nurse is observing a student nurse apply erythromycin ophthalmic ointment. What action by the student requires the nurse to intervene? a. Applies ointment in thin ribbon b. Applies ointment from outer canthus to inner canthus. c. Holds the tube horizontally while applying ointment d. Wipes excess ointment away after 1 minute. ANS: B The ointment should be applied from inner to outer canthus. When the student does this incorrectly, the nurse should intervene. The other actions are appropriate. PTS: 1 DIF: Cognitive Level: Application/Applying REF: p. 463 | Drug Guide OBJ: Nursing Process: Implementation MSC: Client Needs: Physiologic Integrity 17. A student nurse is preparing an injection of vitamin K (aquaMEPHYTON). What action by the student shows good understanding of this procedure? a. Draws up 1.5 mg of solution b. Protects solution from light c. Finds landmark for subQ injection d. Administers directly after circumcision ANS: B The solution of vitamin K is light-sensitive, so it should be protected from light. The dose is 0.5 to 1 mg. It is given IM and should be administered prior to a circumcision. PTS: 1 DIF: Cognitive Level: Application/Applying REF: p. 463 | Drug Guide OBJ: Nursing Process: Implementation MSC: Client Needs: Physiologic Integrity MULTIPLE RESPONSE

6. A primigravida at 40 weeks of gestation is having uterine contractions every 1.5 to 2 minutes and says that they are very painful. Her cervix is dilated 2 cm and has not changed in 3 hours. The woman is crying and wants an epidural. What is the likely status of this woman's labor? a. She is exhibiting hypotonic uterine dysfunction. b. She is experiencing a normal latent stage. c. She is exhibiting hypertonic uterine dysfunction. d. She is experiencing pelvic dystocia.

C Women who experience hypertonic uterine dysfunction, or primary dysfunctional labor, often are anxious first-time mothers who are having painful and frequent contractions that are ineffective at causing cervical dilation or effacement to progress. With hypotonic uterine dysfunction, the woman initially makes normal progress into the active stage of labor and then the contractions become weak and inefficient or stop altogether. This is not a normal latent stage. Pelvic dystocia can occur whenever contractures of the pelvic diameters reduce the capacity of the bony pelvis, including the inlet, midpelvis, outlet, or any combination of these planes. PTS: 1 DIF: Cognitive Level: Knowledge/Remembering REF: p. 575 OBJ: Nursing Process: Assessment MSC: Client Needs: Health Promotion and Maintenance

2. The nurse is caring for a laboring patient who develops a fever after she has had her epidural initiated. What actions by the nurse are appropriate? (Select all that apply.) a. Palpate the woman's bladder distention. b. Assess the woman's blood pressure. c. Observe the woman for shivering. d. Check the skin for color and warmth. e. Prepare to assist with a blood patch.

C, D Heat dissipation is reduced as a result of decreased hyperventilation, sweating, and activity after the onset of pain relief. Vasodilation redistributes heat from the core to the periphery of the body, where it is lost to the environment. Assessing the skin will demonstrate findings consistent with vasodilation. Shivering often occurs with sympathetic blockade accompanied by a dissociation between warm and cold sensations. In essence, the body believes that the temperature is lower than it actually is and turns up the "thermostat." Bladder distention is an anticipated effect of having an epidural. A woman's bladder fills quickly because of the large quantity of IV solution, yet her sensation to void is reduced. Maternal hypotension is an expected side effect of epidural initiation. The nurse should assess the bladder and blood pressure, but these actions are not related to the fever. A blood patch procedure is not warranted for this patient. PTS: 1 DIF: Cognitive Level: Application REF: p. 364 OBJ: Nursing Process: Implementation MSC: Client Needs: Physiologic Integrity COMPLETION

4. When caring for a pregnant woman with suspected cardiomyopathy, the nurse must be alert for signs and symptoms of cardiac decompensation, which include (Select all that apply.) a. A regular heart rate b. Hypertension c. Shortness of breath d. Weakness e. Crackles in the lung bases

C, D, E Some symptoms of cardiomyopathy include shortness of breath, weakness, and crackles in the lung bases. A regular heart rate may or may not be present. Hypertension is not a typical finding. PTS: 1 DIF: Cognitive Level: Knowledge/Remembering REF: p. 557 | Safety Alert Box OBJ: Nursing Process: Assessment MSC: Client Needs: Physiologic Integrity Chapter 27: The Woman with an Intrapartum Complication McKinney: Evolve Resources for Maternal-Child Nursing, 5th Edition MULTIPLE CHOICE

3. A student nurse has been studying Healthy People 2020. What information about this initiative does the student understand? (Select all that apply.) a. It is a new agenda for health care and research priorities. b. None of the priorities in this document pertains to pregnant women or children. c. Objectives are aimed at keeping people healthy with a good quality of life. d. Ensuring that 77.9% of women receive prenatal care in the first trimester is one goal. e. Increasing to 100% the proportion of people with health insurance.

C, D, E The Healthy People 2020 initiative is an update of previous versions and is the nation's blueprint for health care and research priorities. Many of its objectives pertain to pregnant women and children. The objectives include improving health and quality of life, ensuring that 77.9% of pregnant women receive prenatal care in the first trimester, and increasing the number of people with health insurance to 100%. PTS: 1 DIF: Cognitive Level: Knowledge/Remembering REF: p. 5 OBJ: Integrated Process: Teaching-Learning MSC: Client Needs: Health Promotion and Maintenance Chapter 02: The Nurse's Role in Maternity, Women's Health, and Pediatric Nursing McKinney: Evolve Resources for Maternal-Child Nursing, 5th Edition MULTIPLE CHOICE

5. A placenta previa in which the placental edge just reaches the internal os is called a. total. b. partial. c. complete. d. marginal.

D A placenta previa that does not cover any part of the cervix is termed marginal. With a total placenta previa the placenta completely covers the os. With a partial previa the lower border of the placenta is within 3 cm of the internal cervical os but does not completely cover the os. A complete previa is termed total. The placenta completely covers the internal cervical os. PTS: 1 DIF: Cognitive Level: Knowledge/Remembering REF: p. 529 OBJ: Nursing Process: Assessment MSC: Client Needs: Physiologic Integrity

3. The nurse is planning to teach parents of a 15-month-old child. Which is the priority concern the nurse should address? a. Toilet training guidelines b. Guidelines for weaning children from bottles c. Instructions on preschool readiness d. Instructions on a home safety assessment

D Accidents are the major cause of death in children, including deaths caused by ingestion of poisonous materials. Home and environmental safety assessments are priorities in this age-group because of toddlers' increased motor skills and independence, which puts them at greater risk in an unsafe environment. Although it is appropriate to give parents of a 15-month-old child toilet training guidelines, the child is not usually ready for toilet training, so it is not the priority teaching intervention. Parents of a 15-month-old child should have been advised to begin weaning from the breast or bottle at 6 to 12 months of age. Educating a parent about preschool readiness is important and can occur later in the parents' educational process. The priority teaching intervention for the parents of a 15-month-old child is the importance of a safe environment. PTS: 1 DIF: Cognitive Level: Application/Applying REF: p. 119 OBJ: Nursing Process: Planning MSC: Client Needs: Health Promotion and Maintenance

8. The nurse teaches a pregnant woman that which diagnostic test evaluates the effect of fetal movement on fetal heart activity? a. Contraction stress test (CST) b. Sonography c. Biophysical profile d. Nonstress test (NST)

D An NST evaluates the ability of the fetal heart to accelerate either spontaneously or in association with fetal movement. CST evaluates the fetal reaction to contractions. Sonographic examinations visualize the fetus and are done for various other reasons. The biophysical profile evaluates fetal status using many variables. PTS: 1 DIF: Cognitive Level: Knowledge/Remembering REF: p. 281 OBJ: Integrated Process: Teaching-Learning MSC: Client Needs: Physiologic Integrity

8. The mother of a 9-month-old infant is concerned because the infant cries when approached by an unknown shopper at the grocery store. What is the best response for the nurse to make to the mother? a. "You could consider leaving the infant with other people so he can adjust." b. "You might consider taking her to the doctor because she may be ill." c. "Have you noticed whether the baby is teething?" d. "This is a sign of stranger anxiety and demonstrates healthy attachment."

D An infant who manifests stranger anxiety is showing a normal sign of healthy attachment. This behavior peaks at 7 to 9 months and is developmentally appropriate. The mother leaving the child more often will not change this developmental response to new strangers. The child does not need to see a doctor, and teething is unrelated. PTS: 1 DIF: Cognitive Level: Comprehension/Understanding REF: Table 6.1 OBJ: Integrated Process: Teaching-Learning MSC: Client Needs: Health Promotion and Maintenance

5. Which behavior is most likely to encourage open communication? a. Avoiding eye contact b. Folding arms across chest c. Standing with head bowed d. Soft stance with arms loose at the side

D An open body stance and positioning such as loose arms at the side invite communication and interaction. Avoiding eye contact, folding the arms across the chest, and standing with the head bowed, are closed body postures and do not facilitate communication. PTS: 1 DIF: Cognitive Level: Knowledge/Remembering REF: Table 4.1 OBJ: Integrated Process: Communication and Documentation MSC: Client Needs: Psychosocial Integrity

3. A pregnant woman has come to the emergency department with complaints of nasal congestion and epistaxis. What action by the nurse is best? a. Refer the patient to an ear, nose, and throat specialist. b. Explain that nasal stuffiness and nosebleeds are caused by a decrease in progesterone. c. Attach the woman to a cardiac monitor, and draw blood for hemoglobin and hematocrit. d. Teach that the increased blood supply to the mucous membranes and can result in congestion and nosebleeds.

D As capillaries become engorged, the upper respiratory tract is affected by the subsequent edema and hyperemia, which causes these conditions, seen commonly during pregnancy. No referral is needed. The patient does not need to be attached to a cardiac monitor or have lab drawn. The patient should be taught that estrogen causes these changes, not progesterone. PTS: 1 DIF: Cognitive Level: Application/Applying REF: p. 218 OBJ: Integrated Process: Teaching-Learning MSC: Client Needs: Physiologic Integrity

4. In providing and teaching cord care, what is an important principle? a. Cord care is done only to control bleeding. b. Alcohol is the only agent used for cord care. c. It takes a minimum of 24 days for the cord to separate. d. The process of keeping the cord dry will decrease bacterial growth.

D Bacterial growth increases in a moist environment, so keeping the umbilical cord dry impedes bacterial growth. Cord care is to prevent infection and add in the drying of the cord. No agents are necessary to facilitate drying of the cord. The cord will fall off within 10 to 14 days. PTS: 1 DIF: Cognitive Level: Comprehension/Understanding REF: p. 468 OBJ: Integrated Process: Teaching-Learning MSC: Client Needs: Health Promotion and Maintenance

1. Which actions by the nurse may prevent infections in the labor and delivery area? a. Vaginal examinations every hour while the woman is in active labor b. Use of clean techniques for all procedures c. Cleaning secretions from the vaginal area by using back-to-front motion d. Keeping underpads and linens as dry as possible

D Bacterial growth prefers a moist, warm environment. Vaginal examinations should be limited to decrease transmission of vaginal organisms into the uterine cavity. Use an aseptic technique if membranes are not ruptured; use a sterile technique if membranes are ruptured. Vaginal drainage should be removed with a front-to-back motion to decrease fecal contamination. PTS: 1 DIF: Cognitive Level: Application/Applying REF: p. 579 OBJ: Nursing Process: Implementation MSC: Client Needs: Safe and Effective Care Environment

2. While the nurse assesses the vital signs of a pregnant woman in her third trimester, the patient complains of feeling faint, dizzy, and agitated. Which nursing intervention is appropriate? a. Have the patient stand up and retake her blood pressure. b. Have the patient sit down and hold her arm in a dependent position. c. Have the patient lie supine for 5 minutes and recheck her blood pressure on both arms. d. Have the patient turn to her left side and recheck her blood pressure in 5 minutes.

D Blood pressure is affected by positions during pregnancy. The supine position may cause occlusion of the vena cava and descending aorta. Turning the pregnant woman to a lateral recumbent position alleviates pressure on the blood vessels and quickly corrects supine hypotension. Pressures are significantly higher when the patient is standing. This option causes an increase in systolic and diastolic pressures. The arm should be supported at the same level of the heart. The supine position may cause occlusion of the vena cava and descending aorta, creating hypotension. PTS: 1 DIF: Cognitive Level: Application/Applying REF: p. 216 OBJ: Nursing Process: Implementation MSC: Client Needs: Physiologic Integrity

1. The infant of a mother with diabetes is hypoglycemic. What type of feeding should be instituted first? a. Glucose water in a bottle b. D5W intravenously c. Formula via nasogastric tube d. Breast milk

D Breast milk is metabolized more slowly and provides longer normal glucose levels. Breast milk is best for nearly all babies. High levels of dextrose correct the hypoglycemia but will stimulate the production of more insulin. Oral feedings are tried first; intravenous lines should be a later choice if the hypoglycemia continues. Formula does provide longer normal glucose levels but would be administered via bottle, not by tube feeding unless the baby is unable to take oral feedings. PTS: 1 DIF: Cognitive Level: Knowledge/Remembering REF: p. 652 OBJ: Nursing Process: Implementation MSC: Client Needs: Physiologic Integrity

2. The nurse observes that when an 8-year-old enters the playroom, the child often causes disruption by taking toys from other children. The nurse's best approach for this behavior is to a. ban the child from the playroom until the child learns to control behavior. b. explain to the children in the playroom that this child is very ill and should be allowed to have the toys. c. approach the child in his or her room and ask, "Would you like it if the other children took your toys from you?" d. approach the child in his room and state, "I am concerned that you are taking the other children's toys. It upsets them and me."

D By the nurse's using "I" rather than the "you" message, the child can focus on the behavior. The child and the nurse can begin to explore why the behavior occurs. Banning the child from the playroom will not solve the problem. The problem is his behavior, not the place where he exhibits it. Illness is not a reason for a child to be undisciplined. When the child recovers, the parents will have to deal with a child who is undisciplined and unruly. Children should not be made to feel guilty and to have their self-esteem attacked. PTS: 1 DIF: Cognitive Level: Application/Applying REF: p. 44 OBJ: Integrated Process: Communication and Documentation MSC: Client Needs: Health Promotion and Maintenance

2. The major source of nutrients in the diet of a pregnant woman should be composed of a. simple sugars. b. fats. c. fiber. d. complex carbohydrates.

D Complex carbohydrates supply the pregnant woman with vitamins, minerals, and fiber. The most common simple carbohydrate is table sugar, which is a source of energy but does not provide any nutrients. Fats provide 9 kcal in each gram, in contrast to carbohydrates and proteins, which provide only 4 kcal in each gram. However, fat is not a good source of nutrients. Fiber is supplied mainly by the complex carbohydrates. PTS: 1 DIF: Cognitive Level: Knowledge/Remembering REF: p. 257 OBJ: Nursing Process: Assessment MSC: Client Needs: Physiologic Integrity

6. When teaching parents about mandatory newborn screening, it is important for the nurse to explain that the main purpose is to a. keep the state records updated. b. allow accurate statistical information. c. document the number of births. d. recognize and treat newborn disorders early.

D Early treatment of disorders will prevent morbidity associated with inborn errors of metabolism or other genetic conditions. Keeping and updating records are not the reasons for the testing. PTS: 1 DIF: Cognitive Level: Knowledge/Remembering REF: p. 476 OBJ: Integrated Process: Teaching-Learning MSC: Client Needs: Physiologic Integrity

4. To adequately care for patients, the nurse understands that labor contractions facilitate cervical dilation by a. contracting the lower uterine segment. b. enlarging the internal size of the uterus. c. promoting blood flow to the cervix. d. pulling the cervix over the fetus and amniotic sac.

D Effective uterine contractions pull the cervix upward at the same time that the fetus and amniotic sac are pushed downward. The contractions are stronger at the fundus. The internal size becomes smaller with the contractions; this helps to push the fetus down. Blood flow decreases to the uterus during a contraction. PTS: 1 DIF: Cognitive Level: Knowledge/Remembering REF: p. 290 OBJ: Nursing Process: Assessment MSC: Client Needs: Health Promotion and Maintenance

7. Which statement should a nurse make when telling a couple about the prenatal diagnosis of genetic disorders? a. Diagnosis can be obtained promptly through most hospital laboratories. b. Common disorders can quickly be diagnosed through blood tests. c. A comprehensive evaluation will result in an accurate diagnosis. d. Diagnosis may be slow and could be inconclusive.

D Even the best efforts at diagnosis do not always yield the information needed to counsel patients. They may require many visits over several weeks. Some tests must be sent to a special laboratory, which requires a longer waiting period for results. There is no quick blood test available at this time to diagnose genetic disorders. Despite a comprehensive evaluation, a diagnosis may never be established. PTS: 1 DIF: Cognitive Level: Comprehension/Understanding REF: p. 178 OBJ: Nursing Process: Implementation MSC: Client Needs: Health Promotion and Maintenance

7. A school nurse is conducting a class on safety for a group of school-age children. Which statement indicates that the children may need further teaching? a. "My sister and I know two different ways to get out of the house." b. "I can dial 911 if there is a fire or a burglar in the house." c. "If we have a fire, we have to meet at the neighbor's house." d. "If there is a fire I will go back for my cat Fluffy because she will be scared."

D Fire safety is important at any age, but for this age group children should know two different ways out of the house, how to call 911, and where the family will meet outside the house. Children should be taught never to return to a burning house, not even for a pet. PTS: 1 DIF: Cognitive Level: Evaluation/Evaluating REF: p. 140 OBJ: Nursing Process: Evaluation MSC: Client Needs: Health Promotion and Maintenance

3. Which maternal event is abnormal in the early postpartum period? a. Diuresis and diaphoresis b. Flatulence and constipation c. Extreme hunger and thirst d. Lochial color changes from rubra to alba

D For the first 3 days after childbirth, lochia is mostly red and is termed rubra. Lochia serosa follows, and then at about 11 days, the discharge becomes clear, colorless, or white. The body rids itself of increased plasma volume after birth. Urine output of 3000 mL/day is common for the first few days after delivery and is facilitated by hormonal changes in the mother. Bowel tone remains sluggish for days after birth, leading to flatulence and constipation. The new mother is hungry and thirsty because of energy used in labor and thirsty because of fluid restrictions during labor. PTS: 1 DIF: Cognitive Level: Knowledge/Remembering REF: p. 396 | Table 20.1 OBJ: Nursing Process: Assessment MSC: Client Needs: Health Promotion and Maintenance

8. What is the best response a nurse can make to a 15-year-old girl who has verbalized a desire to have a baby? a. "Have you talked with your parents about this?" b. "Do you have plans to continue school?" c. "Will you be able to support the baby?" d. "Can you tell me how your life will be if you have an infant?"

D Having the teenager describe how the infant will affect her life will allow the teen to think more realistically. Her description will allow the nurse to assess the teen's perception and reality orientation. Asking the teenager whether she has talked to her parents is not particularly helpful to the teen or the nurse and may terminate the communication. A direct question about continuing school will not facilitate communication. Open-ended questions encourage communication. Asking the teenager about how she will support the child will not facilitate communication. Open-ended questions encourage communication. PTS: 1 DIF: Cognitive Level: Application/Applying REF: p. 166 OBJ: Nursing Process: Implementation MSC: Client Needs: Health Promotion and Maintenance

5. A nurse is participating in a neonatal resuscitation. What action by the nurse takes priority? a. Suction the mouth and nose. b. Stimulate the infant by rubbing the back. c. Perform the Apgar test. d. Place the infant in a preheated warmer.

D In a resuscitation situation, the nurse places the newborn in a preheated warmer immediately to reduce cold stress. Next position the infant in a "sniffing" position. Suctioning is the third step. Drying the infant is fourth, although if more than one health care provider is present, drying can occur simultaneously with the other actions. PTS: 1 DIF: Cognitive Level: Application/Applying REF: p. 643 OBJ: Nursing Process: Implementation MSC: Client Needs: Physiologic Integrity

5. When providing labor support, the nurse knows that which fetal position might cause the laboring woman more back discomfort? a. Right occiput anterior b. Left occiput anterior c. Right occiput transverse d. Left occiput posterior

D In the left occiput posterior position, each contraction pushes the fetal head against the mother's sacrum, which results in intense back discomfort. The other fetal positions do not cause more back discomfort. PTS: 1 DIF: Cognitive Level: Knowledge/Remembering REF: p. 358 OBJ: Nursing Process: Assessment MSC: Client Needs: Health Promotion and Maintenance

6. The standard of care for obstetrics dictates that an internal version might be used to manipulate the a. fetus from a breech to a cephalic presentation before labor begins. b. fetus from a transverse lie to a longitudinal lie before cesarean birth. c. second twin from an oblique lie to a transverse lie before labor begins. d. second twin from a transverse lie to a breech presentation during vaginal birth.

D Internal version is used only during vaginal birth to manipulate the second twin into a presentation that allows it to be born vaginally. For internal version to occur, the cervix needs to be completely dilated; the cervix is not dilated before labor begins. An internal version would not be done in the case of a cesarean birth. PTS: 1 DIF: Cognitive Level: Knowledge/Remembering REF: p. 383 OBJ: Nursing Process: Assessment MSC: Client Needs: Physiologic Integrity

6. The nurse is talking to a 7-year-old boy during a well-child clinic visit. The boy states "I am a Power Ranger, so don't make me angry!" What action by the nurse is best? a. Ask the child about other friends he might play with. b. Find out why the child thinks he is a Power Ranger. c. Ask the parents if he has any opposite sex friends. d. Conduct further developmental screening on the child.

D Magical thinking is developmentally appropriate for the preschooler not a 7-year-old. The nurse should assess this child's development further. Asking about other friends or special powers will not provide information related to development. A 7-year-old does not typically have opposite sex playmates. PTS: 1 DIF: Cognitive Level: Application/Applying REF: p. 134 OBJ: Nursing Process: Assessment MSC: Client Needs: Health Promotion and Maintenance

3. How can the nurse help the mother who is breastfeeding and has engorged breasts? a. Suggest that she switch to bottled formula just for today. b. Assist her in removing her bra, making her more comfortable. c. Apply heat to her breasts between feeding and cold to the breasts just before feedings. d. Instruct and assist the mother to massage her breasts.

D Massage of the breasts causes release of oxytocin and increases the speed of milk release. Engorgement is more likely to increase if breastfeeding is delayed or infrequent. A well-fitting bra should be worn both day and night to support the breasts. Cold applications are used between feedings to reduce edema and pain. Heat is applied just before feedings to increase vasodilation. PTS: 1 DIF: Cognitive Level: Comprehension/Understanding REF: p. 492 OBJ: Nursing Process: Implementation MSC: Client Needs: Physiologic Integrity

6. In most states, adolescents who are not emancipated minors must have the permission of their parents before a. treatment for drug abuse. b. treatment for sexually transmitted diseases (STDs). c. accessing birth control. d. surgery.

D Minors are not considered capable of giving informed consent, so a surgical procedure would require consent of the parent or guardian. Exceptions exist for obtaining treatment for drug abuse or STDs or for getting birth control in most states. PTS: 1 DIF: Cognitive Level: Knowledge/Remembering REF: p. 17 OBJ: Nursing Process: Planning MSC: Client Needs: Safe and Effective Care Environment

9. A woman received 50 mcg of fentanyl intravenously 1 hour before delivery. What drug should the nurse have readily available? a. Promethazine (Phenergan) b. Nalbuphine (Nubain) c. Butorphanol (Stadol) d. Naloxone (Narcan)

D Naloxone reverses narcotic-induced respiratory depression, which may occur with administration of narcotic analgesia. Phenergan is normally given for nausea. Nubain and Stadol are analgesics that can be given to women in labor. PTS: 1 DIF: Cognitive Level: Knowledge/Remembering REF: p. 372 | Table 18.2 OBJ: Nursing Process: Planning MSC: Client Needs: Physiologic Integrity 10. The nerve block used in labor that provides anesthesia to the lower vagina and perineum is called a(n) a. epidural. b. pudendal. c. local. d. spinal block. ANS: B A pudendal block anesthetizes the lower vagina and perineum to provide anesthesia for an episiotomy and use of low forceps if needed. An epidural provides anesthesia for the uterus, perineum, and legs. A local provides anesthesia for the perineum at the site of the episiotomy. A spinal block provides anesthesia for the uterus, perineum, and down the legs. PTS: 1 DIF: Cognitive Level: Knowledge/Remembering REF: p. 372 | Table 18.2 OBJ: Nursing Process: Assessment MSC: Client Needs: Physiologic Integrity 11. A laboring woman has been given an injection of epidural anesthesia. Which assessment by the nurse takes priority? a. Urinary output b. Contraction pattern c. Maternal blood pressure d. Intravenous infusion rate ANS: C Epidural anesthesia may produce maternal hypotension due to vasodilation so the priority assessment by the nurse is maternal blood pressure. The other assessments are important for this woman but are not directly related to the anesthetic injection. PTS: 1 DIF: Cognitive Level: Application/Applying REF: p. 372 | Table 18.2 OBJ: Nursing Process: Implementation MSC: Client Needs: Physiologic Integrity 12. Which statement is true about the physiologic effects of pain in labor? a. It usually results in a more rapid labor. b. It is considered to be a normal occurrence. c. It may result in decreased placental perfusion. d. It has no effect on the outcome of labor. ANS: C When experiencing excessive pain, the woman may react with a stress response that diverts blood flow from the uterus and the fetus. Excessive pain may prolong the labor due to increased anxiety in the woman. Pain is considered normal for labor, however; this statement does not explain the physiologic effects. Pain may affect the outcome of the labor depending on the cause and the effect on the woman. PTS: 1 DIF: Cognitive Level: Knowledge/Remembering REF: p. 354 OBJ: Nursing Process: Assessment MSC: Client Needs: Physiologic Integrity 13. Which woman will most likely have increased anxiety and tension during her labor? a. Gravida 1 who did not attend prepared childbirth classes b. Gravida 2 who refused any medication c. Gravida 2 who delivered a stillborn baby last year d. Gravida 3 who has two children younger than 3 years ANS: C If a previous pregnancy had a poor outcome, the woman will probably be more anxious during labor and delivery. The woman is not prepared for labor and will have increased anxiety during labor. However, the woman with a poor previous outcome is more likely to experience more anxiety, and good teaching by the nurse will diminish some of the anxiety. A gravida 2 has previous experience and can anticipate what to expect. By refusing any medication, she is taking control over her situation and will have less anxiety. This gravida 3 has previous experience and is aware of what to expect. PTS: 1 DIF: Cognitive Level: Comprehension/Understanding REF: p. 356 OBJ: Nursing Process: Assessment MSC: Client Needs: Psychosocial Integrity 14. Which method of pain management does the nurse plan for a gravida 3 para 2 admitted at 8-cm cervical dilation? a. Epidural anesthesia b. Narcotics c. Spinal block d. Breathing and relaxation techniques ANS: D Nonpharmacologic methods of pain management may be the best option for a woman in advanced labor. There is probably not enough remaining time to administer epidural anesthesia or spinal anesthesia. A narcotic given at this time may reach its peak about the time of birth and result in respiratory depression in the newborn. PTS: 1 DIF: Cognitive Level: Application/Applying REF: p. 357 OBJ: Nursing Process: Planning MSC: Client Needs: Health Promotion and Maintenance 15. The laboring woman who imagines her body opening to let the baby out is using a mental technique called a. dissociation. b. effleurage. c. imagery. d. distraction. ANS: C Imagery is a technique of visualizing images that will assist the woman in coping with labor. Dissociation helps the woman learn to relax all muscles except those that are working. Effleurage is self-massage. Distraction can be used in the early latent phase by having the woman involved in another activity. PTS: 1 DIF: Cognitive Level: Knowledge/Remembering REF: p. 359 OBJ: Nursing Process: Assessment MSC: Client Needs: Psychosocial Integrity 16. The registered nurse explains to the student that when giving a narcotic to a laboring woman, the nurse should inject the medication at the beginning of a contraction so that a. full benefit of the medication is received during that contraction. b. less medication will be transferred to the fetus. c. the medication will be rapidly circulated. d. the maternal vital signs will not be adversely affected. ANS: B Injecting at the beginning of a contraction, when blood flow to the placenta is normally reduced, limits transfer to the fetus. The full benefit will be received by the woman; however, it will decrease the amount reaching the fetus. It will not increase the circulation of the medication. It will not alter the vital signs any more than giving it at another time. PTS: 1 DIF: Cognitive Level: Comprehension/Understanding REF: p. 367 OBJ: Nursing Process: Implementation MSC: Client Needs: Physiologic Integrity 17. The method of anesthesia in labor considered the safest for the fetus is the a. pudendal block. b. epidural block. c. spinal (subarachnoid) block. d. local infiltration. ANS: D Local infiltration of the perineum rarely has any adverse effects on either the mother or the fetus. The fetus can be affected by maternal side effects of the other types of anesthesia. PTS: 1 DIF: Cognitive Level: Knowledge/Remembering REF: p. 362 OBJ: Nursing Process: Assessment MSC: Client Needs: Health Promotion and Maintenance 18. A woman received an epidural anesthetic and now her blood pressure is 88/64 mm Hg. What action by the nurse takes priority? a. Turn the woman to the left side. b. Place a wedge under the woman's right hip. c. Call the provider or nurse-anesthetist immediately. d. Decrease the intravenous infusion rate. ANS: B If hypotension occurs after administration of an epidural, turn the patient to the left lateral side-lying position, and infuse intravenous crystalloids. These actions will improve placental blood flow. Oxygen administration is also recommended, but placing the patient on the left side takes priority. The providers should be notified after corrective actions have occurred. PTS: 1 DIF: Cognitive Level: Application/Applying REF: p. 373 | Table 18.2 OBJ: Nursing Process: Implementation MSC: Client Needs: Safe and Effective Care Environment 19. The priority nursing intervention for the patient who has received an epidural narcotic is a. monitoring respiratory rate hourly. b. administering analgesics as needed. c. monitoring blood pressure every 4 hours. d. assessing the level of anesthesia. ANS: A The possibility of respiratory depression exists for up to 24 hours after administration of an epidural narcotic. The nurse should monitor the woman's respiratory rate hourly during this time frame. Epidural narcotic should be enough pain relief that further medication is not necessary. Administering any other narcotic may cause an overdose. The patient's blood pressure needs to be monitored. However, that is not the major concern with this medication. The epidural narcotic should provide pain relief but not anesthesia. PTS: 1 DIF: Cognitive Level: Comprehension/Understanding REF: p. 367 OBJ: Nursing Process: Implementation MSC: Client Needs: Physiologic Integrity 20. One of the greatest risks to the mother during administration of general anesthesia is a. respiratory depression. b. uterine relaxation. c. inadequate muscle relaxation. d. aspiration of stomach contents. ANS: D Aspiration of acidic gastric contents and possible airway obstruction is a potentially fatal complication of general anesthesia. Respirations can be altered during general anesthesia, and the anesthesiologist will take precautions to maintain proper oxygenation. Uterine relaxation can occur with some anesthesia, but this can be monitored and prevented. Inadequate muscle relaxation can be altered. This is not the greatest risk for the mother. PTS: 1 DIF: Cognitive Level: Knowledge/Remembering REF: p. 367 OBJ: Nursing Process: Assessment MSC: Client Needs: Physiologic Integrity 21. The student nurse is working with a laboring woman. What action by the student requires the registered nurse to intervene? a. Placing the woman in a supine position b. Assisting the woman to a sitting position c. Turning the woman to a side-lying position d. Providing safety while the woman labors while standing ANS: A The supine position allows the heavy uterus to compress the inferior vena cava and can reduce placental blood flow, compromising fetal oxygen supply. The nurse should intervene to position the woman in any of the other positions, which are all appropriate for labor if no contraindications exist. PTS: 1 DIF: Cognitive Level: Application/Applying REF: p. 363 OBJ: Nursing Process: Implementation MSC: Client Needs: Health Promotion and Maintenance 22. A woman had spinal anesthesia for delivery. Now she complains of a pounding headache rated 7/10. What action by the nurse is most appropriate? a. Prepare to assist with a blood patch procedure. b. Give the woman IV opioid pain medications. c. Increase the rate of her nonadditive IV fluids. d. Place a cool cloth on her forehead and dim the room lights. ANS: A The subarachnoid block may cause a postspinal headache due to loss of cerebrospinal fluid from the puncture in the dura. When blood is injected into the epidural space in the area of the dural puncture ("blood patch"), it forms a seal over the hole to stop leaking of cerebrospinal fluid. The spinal anesthesia makes further narcotic administration inadvisable at this time. Increasing IV fluid rate is not needed for headache. A cool cloth and dim lights may be very comforting but will not eliminate this severe headache. PTS: 1 DIF: Cognitive Level: Application/Applying REF: p. 373 | Table 18.2 OBJ: Nursing Process: Implementation MSC: Client Needs: Physiologic Integrity 23. The nurse teaching a childbirth preparation class teaches the participants that the first type of breathing technique used in labor is called a. slow-paced. b. modified-paced. c. patterned-paced. d. pant-blow. ANS: A Breathing for the first stage of labor consists of a cleansing breath and various breathing techniques known as paced breathing. The first type used in labor is the slow-paced. Modified-paced breathing is used when the slow-paced breathing is no longer effective Patterned-paced breathing is used later in the labor and has the woman focusing on a pattern of breathing. Pant-blow breathing can be used to prevent pushing before the cervix is completely dilated. PTS: 1 DIF: Cognitive Level: Comprehension/Understanding REF: p. 359 OBJ: Integrated Process: Teaching-Learning MSC: Client Needs: Health Promotion and Maintenance 24. When instructing the woman in early labor, the nurse teaches her that an important aspect of proper breathing technique is a. breathing no more than three times the normal rate. b. beginning and ending with a cleansing breath. c. holding the breath no longer than 10 seconds. d. adhering exactly to the techniques as they were taught. ANS: B The cleansing breath helps the woman clear her mind to focus on relaxing and signals the coach that the contraction is beginning or ending. It is important to prevent hyperventilation; however, the cleansing breaths are the most important aspect of the breathing techniques. The woman should hold her breath for no more than 6 to 8 seconds. The woman needs to be flexible and change her breathing techniques as needed to keep her comfortable. PTS: 1 DIF: Cognitive Level: Comprehension/Understanding REF: p. 359 OBJ: Integrated Process: Teaching-Learning MSC: Client Needs: Psychosocial Integrity 25. Which patient is most likely to experience pain during labor? a. Gravida 2 who has not attended childbirth preparation classes b. Gravida 2 who is anxious because her last labor was difficult c. Gravida 1 whose fetus is in a breech presentation d. Gravida 3 who is using Lamaze breathing techniques ANS: B Anxiety affects a woman's perception of pain. Tension during labor causes tightening of abdominal muscles, impeding contractions and increasing pain by stimulation of nerve endings. The gravida 2 has previous experience, and this will decrease anxiety. This woman will have more pain than if the infant is in vertex. Also, there is an increased likelihood that she will have a cesarean section and not go through labor. The gravida 3 has previous experience and has prepared herself for the labor. PTS: 1 DIF: Cognitive Level: Comprehension/Understanding REF: p. 356 OBJ: Nursing Process: Assessment MSC: Client Needs: Psychosocial Integrity 26. Which type of cutaneous stimulation involves massage of the abdomen? a. Thermal stimulation b. Imagery c. Mental stimulation d. Effleurage ANS: D Effleurage is massage usually performed on the abdomen during contractions. Thermal stimulation is the use of warmth to provide comfort, such as showers and baths. Imagery involves the woman creating a relaxing mental scene and dissociating herself from the painful aspects of labor. Mental stimulation occupies the woman's mind and competes with pain stimuli. PTS: 1 DIF: Cognitive Level: Knowledge/Remembering REF: p. 357 OBJ: Nursing Process: Implementation MSC: Client Needs: Physiologic Integrity 27. A woman is experiencing most of her labor pain in her back. What action by the nurse is best? a. Positioning the woman lying supine with head slightly elevated b. Showing the support person how to apply firm pressure to the sacrum c. Assisting the woman to sit upright with the legs straight d. Massaging her upper back during a contraction ANS: B Firm pressure against the sacrum may be helpful in relieving the discomfort associated with back labor. The nurse can provide this action, but including the support person (if desired) is beneficial. The woman should not lie on her back. Sitting up with legs straight would put more pressure onto the lower back area. The massage should be in the lower back where the pain is located. PTS: 1 DIF: Cognitive Level: Application/Applying REF: p. 358 OBJ: Nursing Process: Implementation MSC: Client Needs: Physiologic Integrity 28. Which technique could the support person use when the laboring woman appears to be losing control? a. Have the nurse take over the role of support. b. Tell the woman that she is causing stress to her baby and herself. c. Wait for the contraction to end and discuss the problem with her. d. Make eye contact with the woman and breathe along with her. ANS: D Making eye contact and breathing along with the laboring woman to help pace her breathing will assist her in remaining calm. The woman already has a trusting relationship with the support person so they should stay in that position if possible. Telling the woman she is stressing herself and the baby is very uncaring and will not be helpful. A woman who has lost control will not be able to engage in a productive discussion. PTS: 1 DIF: Cognitive Level: Comprehension/Understanding REF: p. 371 OBJ: Nursing Process: Implementation MSC: Client Needs: Psychosocial Integrity 29. A nurse admits a woman to the labor and delivery unit who has a history of IV drug abuse. In planning care for this patient, the nurse explains to the student that which pain control plan is contraindicated for this woman? a. Epidural anesthesia b. Bolus administration of butorphanol (Stadol) c. Promethazine (Phenergan) for opioid-induced nausea d. Naloxone (Narcan) if needed for respiratory depression ANS: B Women who are opiate-dependent should not receive analgesics having mixed agonist and antagonist actions (butorphanol and nalbuphine). Epidural anesthesia not using these drugs is appropriate as are promethazine and naloxone if needed. PTS: 1 DIF: Cognitive Level: Knowledge/Remembering REF: Table 18.2 OBJ: Nursing Process: Planning MSC: Client Needs: Physiologic Integrity 30. A woman has received an epidural block. What action by the nurse takes priority? a. Instruct her to call for help when getting out of bed. b. Assess the woman for a post-procedure headache. c. Determine type and time of last oral intake. d. Administer metoclopramide within the first hour. ANS: A Due to variable leg strength and sensation with an epidural block, the woman who is able to get out of bed needs to call for assistance for safety. Post-procedure headaches are associated with subarachnoid blocks. Oral intake and pro-motility agents are important for the woman having general anesthesia. PTS: 1 DIF: Cognitive Level: Application/Applying REF: Table 18.2 OBJ: Nursing Process: Implementation MSC: Client Needs: Safe and Effective Care Environment 31. What statement by the woman after a childbirth education class demonstrates that she needs more information? a. "I'm having a pudendal block so control my labor pain." b. "I may get a headache after a subarachnoid block." c. "I don't want IV opioids as they may cause breathing problems." d. "Some anesthetic agents may cause itching but it can be treated." ANS: A A pudendal block numbs the lower vagina and perineum for vaginal birth. There is no relief of labor pain because it is done just before birth. This woman needs further education. The other statements are all accurate. PTS: 1 DIF: Cognitive Level: Evaluation/Evaluating REF: Table 18.2 OBJ: Integrated Process: Teaching-Learning MSC: Client Needs: Physiologic Integrity 32. A woman had an epidural place an hour ago and is now complaining of severe itching. What action by the nurse is most appropriate? a. Discontinue the epidural infusion at once. b. Notify the anesthesia provider. c. Prepare to administer diphenhydramine (Benedryl). d. Prepare to administer promethazine (Phenergan). ANS: C Pruritis (itching) is a common side effect of epidural medications. The nurse should be prepared to administer diphenhydramine. There is no need to discontinue the epidural infusion or notify the anesthesia provider. Promethazine is used for nausea. PTS: 1 DIF: Cognitive Level: Application/Applying REF: Table 18.1 OBJ: Nursing Process: Implementation MSC: Client Needs: Physiologic Integrity MULTIPLE RESPONSE

6. Which strategy is most likely to encourage a child to express feelings about the hospital experience? a. Avoiding periods of silence b. Asking yes/no questions c. Sharing personal experiences d. Using open-ended questions

D Open-ended questions encourage conversation. Periods of silence can serve to facilitate communication, but this is not the most effective means of getting the child to communicate. Yes/no questions are closed ended and do not encourage conversation. Talking about yourself shifts the focus of the conversation away from the child. PTS: 1 DIF: Cognitive Level: Knowledge/Remembering REF: p. 50 OBJ: Integrated Process: Communication and Documentation MSC: Client Needs: Psychosocial Integrity

9. Which goal is most appropriate for demonstrating effective parenting? a. The parents will demonstrate correct bathing by discharge. b. The mother will make an appointment with the lactation specialist prior to discharge. c. The parents will place the baby in the proper position for sleeping and napping by 2300 on postpartum day 1. d. The parents will demonstrate effective parenting by discharge.

D Outcomes and goals are not the same. Goals are broad and not measurable and so must be linked to more measurable outcome criteria. Demonstrating effective parenting is one such goal. The other options are measurable outcome indicators that help determine if the goal has been met. PTS: 1 DIF: Cognitive Level: Evaluation/Evaluating REF: p. 31 OBJ: Nursing Process: Planning MSC: Client Needs: Safe and Effective Care Environment 10. Which nursing intervention is correctly written? a. Encourage turning, coughing, and deep breathing. b. Force fluids as necessary. c. Assist to ambulate for 10 minutes at 8 AM, 2 PM, and 6 PM. d. Observe interaction with infant. ANS: C This intervention is the most specific and details what should be done, for how long, and when. The other interventions are too vague. PTS: 1 DIF: Cognitive Level: Comprehension/Understanding REF: p. 31 OBJ: Nursing Process: Planning MSC: Client Needs: Safe and Effective Care Environment 11. What part of the nursing process includes the collection of data on vital signs, allergies, sleep patterns, and feeding behaviors? a. Assessment b. Planning c. Intervention d. Evaluation ANS: A Assessment includes gathering baseline data. Planning is based on baseline data and physical assessment. Implementation is the initiation and completion of nursing interventions. Evaluation is the last step in the nursing process and involves determining whether the goals were met. PTS: 1 DIF: Cognitive Level: Knowledge/Remembering REF: p. 29 OBJ: Nursing Process: Assessment MSC: Client Needs: Safe and Effective Care Environment 12. The nurse who coordinates and manages a patient's care with other members of the health care team is functioning in which role? a. Teacher b. Collaborator c. Researcher d. Advocate ANS: B The nurse collaborates with other members of the health care team, often coordinating and managing the patient's care. Care is improved by this interdisciplinary approach as nurses work together with dietitians, social workers, physicians, and others. Education is an essential role of today's nurse. The nurse functions as a teacher during prenatal care, during maternity care, and when teaching parents of children regarding normal growth and development. Nurses contribute to their profession's knowledge base by systematically investigating theoretic for practice issues and nursing. A nursing advocate is one who speaks on behalf of another. As the health professional who is closest to the patient, the nurse is in an ideal position to humanize care and to intercede on the patient's behalf. PTS: 1 DIF: Cognitive Level: Knowledge/Remembering REF: p. 25 OBJ: Nursing Process: Planning MSC: Client Needs: Safe and Effective Care Environment 13. Which statement about alternative and complementary therapies is true? a. Replace conventional Western modalities of treatment b. Are used by only a small number of American adults c. Allow for more patient autonomy but also may carry risks d. Focus primarily on the disease an individual is experiencing ANS: C Being able to choose alternative and complementary health products and practices does allow for patient autonomy, but the major concern is risk as patients may not disclose their use or substances may interact with other medications the patient is taking. Alternative and complementary therapies are part of an integrative approach to health care for most people, although some may choose only these types of therapies. An increasing number of American adults are seeking alternative and complementary health care options. Alternative healing modalities offer a holistic approach to health, focusing on the whole person and not just the disease. PTS: 1 DIF: Cognitive Level: Comprehension/Understanding REF: p. 31 OBJ: Integrated Process: Culture and Spirituality MSC: Client Needs: Physiologic Integrity 14. Which step in the nursing process identifies the basis or cause of the patient's problem? a. Intervention b. Expected outcome c. Nursing diagnosis d. Evaluation ANS: C A nursing diagnosis states the problem and its cause ("related to"). Interventions are actions taken to meet the problem. Expected outcome is a statement of how the goal will be measured. Evaluation determines whether the goal has been met. PTS: 1 DIF: Cognitive Level: Knowledge/Remembering REF: pp. 30-31 OBJ: Nursing Process: Planning MSC: Client Needs: Safe and Effective Care Environment MULTIPLE RESPONSE

8. Glucose metabolism is profoundly affected during pregnancy because a. pancreatic function in the islets of Langerhans is affected by pregnancy. b. the pregnant woman uses glucose at a more rapid rate than the nonpregnant woman. c. the pregnant woman increases her dietary intake significantly. d. placental hormones are antagonistic to insulin, resulting in insulin resistance.

D Placental hormones, estrogen, progesterone, and human placental lactogen (HPL) create insulin resistance. Insulin also is broken down more quickly by the enzyme placental insulinase. Pancreatic functioning is not affected by pregnancy. The glucose requirements differ because of the growing fetus. The pregnant woman should increase her intake by 200 calories a day. PTS: 1 DIF: Cognitive Level: Knowledge/Remembering REF: p. 553 OBJ: Nursing Process: Assessment MSC: Client Needs: Physiologic Integrity

2. Compared to the term infant, the preterm infant has a. few blood vessels visible though the skin. b. more subcutaneous fat. c. well-developed flexor muscles. d. greater surface area in proportion to weight.

D Preterm infants have greater surface area in proportion to their weight. They often have visible blood vessels because their skin is thin and they have less fat. More fat and well- developed flexor muscles are characteristic of a more mature infant. PTS: 1 DIF: Cognitive Level: Knowledge/Remembering REF: p. 623 OBJ: Nursing Process: Assessment MSC: Client Needs: Physiologic Integrity

6. When planning care for adolescents, the nurse should a. teach parents first, and they, in turn, will teach the teenager. b. provide information for long-term health needs. c. provide explanations for treatment and procedures to the parents only. d. give information privately to adolescents on specific problems that they identify.

D Problems that teenagers identify and are interested in are typically the problems that they are the most willing to address. Confidentiality is important to adolescents. Adolescents prefer to confer privately (without parents) with the nurse and health care provider. Teenagers are socially and cognitively at the developmental stage where the health care provider can teach them. Teenagers are more interested in immediate health care needs than in long-term needs. Teenagers are at the developmental level that allows them to receive explanations about health care directly from the nurse. PTS: 1 DIF: Cognitive Level: Application/Applying REF: p. 159 OBJ: Nursing Process: Planning MSC: Client Needs: Health Promotion and Maintenance

6. What situation is most conducive to learning? a. A teacher who speaks very little Spanish is teaching a class of Latino students. b. A class is composed of students of various ages and educational backgrounds. c. An auditorium is being used as a classroom for 300 students. d. An Asian nurse provides nutritional information to a group of pregnant Asian women.

D Teaching is a vital function of the professional nurse. A patient's language and culture influence the learning process; thus a situation that is most conducive to learning is one in which the teacher has knowledge and understanding of the patient's language and cultural beliefs. The ability to understand the language in which teaching is done determines how much the patient learns. Patients for whom English is not their primary language may not understand idioms, nuances, slang terms, informal usage of words, or medical words. The teacher should be fluent in the language of the student. Developmental levels and educational levels influence how a person learns best. In order for the teacher to best present information, the class should be composed of the same levels. A large class is not conducive to learning. It does not allow for questions, and the teacher is not able to see the nonverbal cues from the students to ensure understanding. PTS: 1 DIF: Cognitive Level: Application/Applying REF: p. 25 OBJ: Nursing Process: Planning MSC: Client Needs: Psychosocial Integrity

1. Which statement made by a mother of a school-age boy indicates a need for further teaching? a. "My child is playing soccer on a team this year." b. "He is always active with his friends playing games." c. "I limit his television watching to about 2 hours a day." d. "I am glad his coach emphasizes winning and discipline in today's society."

D Team sports are important for the development of sportsmanship and teamwork and for exercise and refinement of motor skills. A coach who emphasizes winning and strict discipline is not appropriate for children in this age-group. Team sports such as soccer are appropriate for exercise and refinement of motor skills. Limiting television to 2 hours a day is an appropriate restriction. School-age children should be encouraged to participate in physical activities. PTS: 1 DIF: Cognitive Level: Evaluation/Evaluating REF: p. 133 OBJ: Nursing Process: Evaluation MSC: Client Needs: Health Promotion and Maintenance

1. Which factor significantly contributed to the shift from home births to hospital births in the early 20th century? a. Puerperal sepsis was identified as a risk factor in labor and delivery. b. Forceps were developed to facilitate difficult births. c. The importance of early parental-infant contact was identified. d. Technologic developments became available to physicians.

D Technologic developments were available to physicians, not lay midwives. So in-hospital births increased in order to take advantage of these advancements. Puerperal sepsis has been a known problem for generations. In the late 19th century, Semmelweis discovered how it could be prevented with improved hygienic practices. The development of forceps is an example of a technology advance made in the early 20th century but is not the only reason birthplaces moved. Unlike home births, early hospital births hindered bonding between parents and their infants. PTS: 1 DIF: Cognitive Level: Knowledge/Remembering REF: p. 1 OBJ: Integrated Process: Teaching-Learning MSC: Client Needs: Safe and Effective Care Environment

6. The student nurse learns that some of the embryo's intestines remain within the umbilical cord during the embryonic period because the a. umbilical cord is much larger at this time than it will be at the end of pregnancy. b. intestines begin their development within the umbilical cord. c. nutrient content of the blood is higher in this location. d. abdomen is too small to contain all the organs while they are developing.

D The abdominal contents grow more rapidly than the abdominal cavity, so part of their development takes place in the umbilical cord. By 10 weeks, the abdomen is large enough to contain them. The intestines remain within the umbilical cord only until approximately week 10. Intestines begin their development within the umbilical cord but only because the liver and kidneys occupy most of the abdominal cavity. All the intestines are within the abdominal cavity around week 10. PTS: 1 DIF: Cognitive Level: Knowledge/Remembering REF: Table 12.2 OBJ: Integrated Process: Teaching-Learning MSC: Client Needs: Physiologic Integrity

4. When is the best time to determine gestational age based on biparietal diameter through ultrasound? a. First trimester only b. Second trimester only c. Any time d. Second half of pregnancy

D The biparietal diameter is used to determine gestational age during the second half of pregnancy. PTS: 1 DIF: Cognitive Level: Knowledge/Remembering REF: p. 276 OBJ: Nursing Process: Assessment MSC: Client Needs: Health Promotion and Maintenance

4. What is the most important consideration for effectively communicating with a child? a. The child's chronologic age b. The parent-child interaction c. The child's receptiveness d. The child's developmental level

D The child's developmental level is the basis for selecting the terminology and structure of the message most likely to be understood by the child. The child's age may not correspond with the child's developmental level; therefore it is not the most important consideration for communicating with children. Parent-child interaction is useful in planning communication with children, but it is not the primary factor in establishing effective communication. The child's receptiveness is a consideration in evaluating the effectiveness of communication. PTS: 1 DIF: Cognitive Level: Comprehension/Understanding REF: p. 54 OBJ: Nursing Process: Assessment MSC: Client Needs: Health Promotion and Maintenance

1. A nursing student is helping the nursery nurses with morning vital signs. A baby born 10 hours ago via cesarean section is found to have moist lung sounds. What is the best interpretation of these data? a. The nurse should notify the pediatrician stat for this emergency situation. b. The neonate must have aspirated surfactant. c. If this baby was born vaginally, it could indicate a pneumothorax. d. The lungs of a baby delivered by cesarean section may sound moist for 24 hours after birth.

D The condition will resolve itself within a few hours. For this common condition of newborns, surfactant acts to keep the expanded alveoli partially open between respirations. In vaginal births, absorption of remaining lung fluid is accelerated by the process of labor and delivery. Remaining lung fluid will move into interstitial spaces and be absorbed by the circulatory and lymphatic systems. There is no need to notify the pediatrician. Surfactant is produced by the lungs, so aspiration is not a concern. Pneumothorax is also not a concern. PTS: 1 DIF: Cognitive Level: Comprehension/Understanding REF: p. 425 OBJ: Nursing Process: Assessment MSC: Client Needs: Health Promotion and Maintenance

5. A pregnant woman's diet may not meet her need for folate. The nurse teaches the woman to take how much folate as a supplement each day? a. 100 to 200 mcg b. 200 to 400 mcg c. 400 to 600 mcg d. 400 to 800 mcg

D The current recommendation for folate (folic acid) is 400 to 800 mcg (0.4 to 0.8 mg) per day. PTS: 1 DIF: Cognitive Level: Knowledge/Remembering REF: p. 260 OBJ: Integrated Process: Teaching-Learning MSC: Client Needs: Physiologic Integrity

9. A primiparous woman wants to begin breastfeeding as soon as possible. The nurse can facilitate the infant's correct latch-on by helping the woman hold the infant a. with his arms folded together over his chest. b. curled up in a fetal position. c. with his head cupped in her hand. d. with his head and body in alignment.

D The infant's head and body should be in correct alignment with the mother and the breast during latch-on and feeding. The other positions do not facilitate nursing. PTS: 1 DIF: Cognitive Level: Knowledge/Remembering REF: p. 486 OBJ: Nursing Process: Implementation MSC: Client Needs: Health Promotion and Maintenance 10. A postpartum woman telephones about her 4-day-old infant. She is not scheduled for a weight check until the infant is 10 days old, and she is worried about whether breastfeeding is going well. Effective breastfeeding is indicated by the newborn who a. sleeps for 6 hours at a time between feedings. b. has at least one breast milk stool every 24 hours. c. gains 1 to 2 ounces per week. d. has at least six to eight wet diapers per day. ANS: D After day 4, when the mother's milk comes in, the infant should have six to eight wet diapers every 24 hours. Typically infants sleep 2 to 4 hours between feedings, depending on whether they are being fed on a 2- to 3-hour schedule or cluster-fed. The infant should have a minimum of three bowel movements in a 24-hour period. The mother will not know what her child weighs until the appointment and so the nurse needs to provide her with other indicators of successful breastfeeding. PTS: 1 DIF: Cognitive Level: Comprehension/Understanding REF: p. 489 OBJ: Integrated Process: Teaching-Learning MSC: Client Needs: Health Promotion and Maintenance 11. To prevent breast engorgement, the new breastfeeding mother should be instructed to a. apply cold packs to the breast before feeding. b. breastfeed frequently and for adequate lengths of time. c. limit her intake of fluids for the first few days. d. feed her infant no more than every 4 hours. ANS: B Engorgement occurs when the breasts are not adequately emptied at each feeding or if feedings are not frequent enough. Warm packs should be applied to the breast before feedings. Fluid intake should not be limited with a breastfeeding mother; that will decrease the amount of breast milk produced. Breast milk moves through the stomach within 1.5 to 2 hours, so waiting 4 hours to feed is too long. Frequent feedings are important to empty the breast and to establish lactation. PTS: 1 DIF: Cognitive Level: Knowledge/Remembering REF: p. 491 OBJ: Nursing Process: Implementation MSC: Client Needs: Physiologic Integrity 12. The difference between the aseptic and terminal methods of sterilization is that the a. aseptic method does not require boiling of the bottles. b. terminal method requires boiling water to be added to the formula. c. aseptic method requires a longer preparation time. d. terminal method sterilizes the prepared formula at the same time it sterilizes the equipment. ANS: D In the terminal sterilization method, the formula is prepared in the bottles, which are loosely capped, and then the bottles are placed in the sterilizer, where they are boiled for 25 minutes. With the aseptic method, the bottles are boiled separate from the formula. This process takes about 5 minutes. PTS: 1 DIF: Cognitive Level: Knowledge/Remembering REF: p. 496 OBJ: Nursing Process: Implementation MSC: Client Needs: Safe and Effective Care Environment 13. How many ounces will a formula-fed infant who is on a 4-hour feeding schedule need to consume at each feeding to meet daily caloric needs? a. 0.5 to 1 b. 1 to 2 c. 2 to 3 d. 4 ANS: C The newborn requires approximately 2 to 3 ounces per feeding within 1 week after birth. PTS: 1 DIF: Cognitive Level: Knowledge/Remembering REF: p. 496 OBJ: Nursing Process: Implementation MSC: Client Needs: Physiologic Integrity 14. A new mother is concerned because her 1-day-old newborn is taking only 1 ounce at each feeding. The nurse should explain that the a. infant does not require as much formula in the first few days of life. b. infant's stomach capacity is small at birth but will expand within a few days. c. infant tires easily during the first few days but will gradually take more formula. d. infant is probably having difficulty adjusting to the formula. ANS: B The infant's stomach capacity at birth is 10 to 20 mL and increases to 60 to 90 mL by the end of the first week. One ounce is 30 mL. The infant's requirements are the same, but the stomach capacity needs to increase before taking in adequate amounts. The infant's sleep patterns do change, but the infant should be awake enough to feed. There are other symptoms that occur if there is a formula intolerance. PTS: 1 DIF: Cognitive Level: Comprehension/Understanding REF: p. 488 OBJ: Nursing Process: Implementation MSC: Client Needs: Physiologic Integrity 15. As the nurse assists a new mother with breastfeeding, she asks, "If formula is prepared to meet the nutritional needs of the newborn, what is in breast milk that makes it better?" The nurse's best response is that it contains a. more calories. b. essential amino acids. c. important immunoglobulins. d. more calcium. ANS: C Breast milk contains immunoglobulins that protect the newborn against infection. The calorie count of formula and breast milk is about the same. All of the essential amino acids are in both formula and breast milk. The concentrations may differ. Calcium levels are higher in formula than breast milk. This higher level can cause an excessively high renal solute load if the formula is not diluted properly. PTS: 1 DIF: Cognitive Level: Knowledge/Remembering REF: p. 480 OBJ: Integrated Process: Teaching-Learning MSC: Client Needs: Physiologic Integrity 16. When responding to the question "Will I produce enough milk for my baby as she grows and needs more milk at each feeding?" the nurse should explain that a. the breast milk will gradually become richer to supply additional calories. b. as the infant requires more milk, feedings can be supplemented with cow's milk. c. early addition of baby food will meet the infant's needs. d. the mother's milk supply will increase as the infant demands more at each feeding. ANS: D The amount of milk produced depends on the amount of stimulation of the breast. Increased demand with more frequent and longer breastfeeding sessions results in more milk available for the infant. Mature breast milk will stay the same. The amounts will increase as the infant feeds for longer times. Supplementation will decrease the amount of stimulation of the breast and decrease the milk production. Solids should not be added until about 4 to 6 months, when the infant's immune system is more mature. This will decrease the chance of allergy formations. PTS: 1 DIF: Cognitive Level: Comprehension/Understanding REF: p. 484 OBJ: Integrated Process: Teaching-Learning MSC: Client Needs: Physiologic Integrity 17. In order to prevent nipple trauma, the nurse should teach the new mother to a. limit the feeding time to less than 5 minutes. b. position the infant so the nipple is far back in the mouth. c. assess the nipples before each feeding. d. wash the nipples daily with mild soap and water. ANS: B If the infant's mouth does not cover as much of the areola as possible, the pressure during sucking will be applied to the nipple, causing trauma to the area. Stimulating the breast for less than 5 minutes will not produce the extra milk the infant may need. Assessing the nipples for trauma is important, but it will not prevent sore nipples. Soap can be drying to the nipples and should be avoided during breastfeeding. PTS: 1 DIF: Cognitive Level: Application REF: p. 487 OBJ: Nursing Process: Implementation MSC: Client Needs: Health Promotion and Maintenance 18. A breastfeeding mother who was discharged yesterday calls to ask about a tender, hard area on her right breast. The nurse's first response should be a. "Try massaging the area and apply heat, as this is probably a plugged duct." b. "Stop breastfeeding because you probably have an infection." c. "Notify your doctor so he can start you on antibiotics." d. "This is a normal response in breastfeeding mothers." ANS: A A plugged lactiferous duct results in localized edema, tenderness, and a palpable hard area. Massage of the area followed by heat will cause the duct to open. This does not indicate an infection or a need for antibiotics. This is a normal deviation but requires intervention to prevent further complications. PTS: 1 DIF: Cognitive Level: Comprehension/Understanding REF: p. 493 OBJ: Integrated Process: Teaching-Learning MSC: Client Needs: Physiologic Integrity 19. An important aspect about storage of breast milk is that it a. can be frozen for up to 2 months. b. should be stored only in glass bottles. c. can be thawed and refrozen. d. can be kept refrigerated for 48 hours. ANS: D If used within 48 hours after being refrigerated, breast milk will maintain its full nutritional value. PTS: 1 DIF: Cognitive Level: Knowledge REF: p. 496 OBJ: Nursing Process: Assessment MSC: Client Needs: Safe and Effective Care Environment 20. The nurse should explain to new parents that the most serious consequence of propping an infant's bottle is a. dental caries. b. aspiration. c. ear infections. d. colic. ANS: B Propping the bottle increases the likelihood of choking and aspiration if regurgitation occurs. Dental caries become a problem when milk stays on the gums for a long period of time. This may cause a buildup of bacteria that will alter the growing teeth buds. However, this is not the most serious consequence. Ear infections can occur when the warm formula runs into the ear and bacterial growth occurs. However, this is not the most serious consequence. Colic can occur, but it is not the most serious consequence. PTS: 1 DIF: Cognitive Level: Comprehension/Understanding REF: p. 497 OBJ: Integrated Process: Teaching-Learning MSC: Client Needs: Physiologic Integrity 21. Parents have been asked by the neonatologist to provide breast milk for their newborn son, who was born prematurely at 32 weeks of gestation. The nurse who instructs them about pumping, storing, and transporting the milk needs to assess their knowledge of lactation. What statement is valid? a. A premature infant more easily digests breast milk than formula. b. A glass of wine just before pumping will help reduce stress and anxiety. c. The mother should only pump as much as the infant can drink. d. The mother should pump every 2 to 3 hours, including during the night. ANS: A Human milk is the ideal food for preterm infants, with benefits that are unique in addition to those received by term, healthy infants. Greater physiologic stability occurs with breastfeeding compared with formula feeding. Consumption of alcohol during lactation is approached with caution. Excessive amounts can have serious effects on the infant and can adversely affect the mother's milk ejection reflex. It is generally taught that lactating mothers avoid it. To establish an optimal milk supply, the mother should be instructed to pump 8 to 10 times a day for 10 to 15 minutes on each breast. The mother should be instructed to pump 8 to 10 times a day for 10 to 15 minutes on each breast. PTS: 1 DIF: Cognitive Level: Knowledge/Remembering REF: p. 480 OBJ: Integrated Process: Teaching-Learning MSC: Client Needs: Health Promotion and Maintenance 22. A new mother asks if she should feed her newborn colostrum, because it is not "real milk." The nurse's best answer is that a. colostrum is high in antibodies, protein, vitamins, and minerals. b. colostrum is lower in calories than milk and should be supplemented by formula. c. giving colostrum helps the mother learn how to breastfeed. d. colostrum is unnecessary for newborns. ANS: A Colostrum is important because it has high levels of the nutrients needed by the neonate and helps protect against infection. Supplementation is not necessary. It will decrease stimulation to the breast and decrease the production of milk. It is important for the mother to feel comfortable in this role before discharge, but the importance of the colostrum to the infant is top priority. Colostrum provides immunities and enzymes necessary to clean the gastrointestinal system, among other things. PTS: 1 DIF: Cognitive Level: Knowledge/Remembering REF: p. 480 OBJ: Nursing Process: Assessment MSC: Client Needs: Physiologic Integrity 23. What information about iron supplementation should the nurse teach a new mother? a. Start iron supplementation shortly after birth if the infant is breastfeeding exclusively. b. Iron-fortified formula will meet the infant's iron requirements. c. Iron supplements must be given when the infant begins teething. d. Infants need a multivitamin with iron every day. ANS: B Iron-fortified formula will meet the infant's initial iron requirements. Solid foods added to the diet maintain iron needs as formula intake decreases. Term infants who are exclusively breastfed have adequate iron stored until they are age 6 months. Iron supplements are not necessary for adequate teething. Vitamins and minerals are added to processed formulas and cereals. It should not be necessary for the child to receive a multivitamin with iron unless the infant is at risk for undernutrition. PTS: 1 DIF: Cognitive Level: Knowledge/Remembering REF: p. 481 OBJ: Integrated Process: Teaching-Learning MSC: Client Needs: Health Promotion and Maintenance 24. A new mother wants to be sure that she is meeting her daughter's needs while feeding her commercially prepared infant formula. The nurse determines that the mother meets her child's needs when she a. adds rice cereal to her formula at 2 weeks of age to ensure adequate nutrition. b. warms the bottles using a microwave oven. c. burps her infant during and after the feeding as needed. d. refrigerates any leftover formula for the next feeding. ANS: C Most infants swallow air when fed from a bottle and should be given a chance to burp several times during a feeding and after the feeding. Solid food should not be introduced to the infant for at least 4 to 6 months after birth. A microwave should never be used to warm any food to be given to an infant. The heat is not distributed evenly, which may pose a risk of burning the infant. Any formula left in the bottle after the feeding should be discarded, because the infant's saliva has mixed with it. PTS: 1 DIF: Cognitive Level: Evaluation/Evaluating REF: p. 496 OBJ: Nursing Process: Evaluation MSC: Client Needs: Health Promotion and Maintenance 25. According to the recommendations of the American Academy of Pediatrics (AAP) on infant nutrition a. Infants should be given only human milk for the first 6 months of life. b. Infants fed on formula should be started on solid food sooner than breastfed infants. c. If infants are weaned from breast milk before 12 months, they should receive cow's milk, not formula. d. After 6 months, mothers should shift from breast milk to cow's milk. ANS: A Breastfeeding/human milk should also be the sole source of milk for the second 6 months. Infants start on solids when they are ready, usually at 6 months, whether they start on formula or breast milk. If infants are weaned from breast milk before 12 months, they should receive iron-fortified formula, not cow's milk. Breastfeeding/human milk should also be the sole source of milk for the second 6 months. PTS: 1 DIF: Cognitive Level: Knowledge/Remembering REF: p. 482 | Box 23.2 OBJ: Nursing Process: Planning MSC: Client Needs: Physiologic Integrity 26. The Baby Friendly Hospital Initiative was founded to encourage institutions to offer optimal levels of care for lactating mothers. Which is of the following is inconsistent with the nurse's knowledge about the "Ten Steps to Successful Breastfeeding for Hospitals"? a. Give newborns no food or drink other than breast milk. b. Have a written breastfeeding policy that is communicated to all staff. c. Help mothers initiate breastfeeding within one half-hour of birth. d. Give artificial pacifiers as necessary. ANS: D No artificial pacifiers should be given to breastfeeding infants. The other statements are consistent with the "Ten Steps to Successful Breastfeeding for Hospitals." PTS: 1 DIF: Cognitive Level: Knowledge/Remembering REF: p. 482 | Box 23.2 OBJ: Nursing Process: Implementation MSC: Client Needs: Physiologic Integrity 27. The best reason for recommending formula over breastfeeding is that a. the mother has a medical condition or is taking drugs that could be passed along to the infant via breast milk. b. the mother lacks confidence in her ability to breastfeed. c. other family members or care providers also need to feed the baby. d. the mother sees bottle-feeding as more convenient. ANS: A Breastfeeding is contraindicated when mothers have certain viruses, are undergoing chemotherapy, or are using/abusing drugs. Some women lack confidence in their ability to produce breast milk of adequate quantity or quality. The key to encouraging these mothers to breastfeed is anticipatory guidance beginning as early as possible in pregnancy. A major barrier for many women is the influence of family and friends. She may view formula feeding as a way to ensure that the father and other family members can participate. Each encounter with the family is an opportunity for the nurse to educate, dispel myths, and clarify information regarding the benefits of breastfeeding. Many women see bottle-feeding as more convenient and less embarrassing than breastfeeding. They may also see breastfeeding as incompatible with an active social life. There may be modesty issues related to feeding the infant in public. Although concerning, these are not legitimate reasons to formula-feed an infant. Often this decision is made without complete information regarding the benefits of breastfeeding. PTS: 1 DIF: Cognitive Level: Knowledge/Remembering REF: p. 492 OBJ: Nursing Process: Planning MSC: Client Needs: Physiologic Integrity 28. The nurse providing couplet care should understand that nipple confusion results when a. breastfeeding babies receive supplementary bottle feedings. b. the baby is weaned too abruptly. c. pacifiers are used before breastfeeding is established. d. twins are breastfed together. ANS: A Nipple confusion can result when babies go back and forth between bottles and breasts, especially before breastfeeding is established in 3 to 4 weeks, because the two require different skills. Abrupt weaning can be distressing to mother and/or baby but should not lead to nipple confusion. Pacifiers used before breastfeeding is established can be disruptive, but this does not lead to nipple confusion. Breastfeeding twins require some logistic adaptations, but this should not lead to nipple confusion. PTS: 1 DIF: Cognitive Level: Comprehension/Understanding REF: p. 490 OBJ: Nursing Process: Planning MSC: Client Needs: Health Promotion and Maintenance 29. The mother who is pumping for an occasional bottle would be most suited for which type of breast pump? a. Manual or hand pump b. Hospital-grade pump c. Electric self-cycling double pumps d. Smaller electric or battery-operated pump ANS: A These are the least expensive and can be the most appropriate choice for mothers pumping for the occasional bottle. Full-service electric or hospital-grade pumps most closely duplicate the sucking action of the breastfeeding infant. These are used when mother and baby (preterm or sick) are separated for long periods. Self-cycling pumps are easy to use, efficient, and designed for working mothers. Smaller pumps operated with a battery are typically used when pumping occasionally. PTS: 1 DIF: Cognitive Level: Knowledge/Remembering REF: p. 495 OBJ: Nursing Process: Implementation MSC: Client Needs: Health Promotion and Maintenance 30. The nurse notes a new mother is waiting until her newborn begins crying prior to breastfeeding her. What response by the nurse is best? a. Praise the mother for her efforts to nurse. b. Teach the mother signs of hunger in the newborn. c. Inform the mother she is inhibiting bonding. d. Demonstrate calming methods prior to feeding. ANS: B Crying is a late sign of hunger in the newborn. The nurse should teach the mother other signs of hunger so the baby will be more ready to eat when the mother attempts to feed the baby. Of course the nurse should praise all attempts at breastfeeding, but this is not the best response. Telling the mother she is inhibiting bonding will discourage her. The nurse should also demonstrate calming methods, but the goal is to feed the infant when he or she displays early signs of hunger. PTS: 1 DIF: Cognitive Level: Application/Applying REF: p. 483 | Box 23.3 OBJ: Nursing Process: Implementation MSC: Client Needs: Health Promotion and Maintenance 31. A woman wants to breastfeed, but her nipples are inverted and she is concerned it won't be possible. What does the nurse teach the woman about this condition? a. A woman with inverted nipples rarely is successful at breastfeeding. b. You can use a breast pump just prior to feeding to evert the nipples. c. Massage the breasts prior to feeding to allow milk let-down. d. Try changing the infant's position during feedings. ANS: B A breast pump can be used just prior to feeding. As soon as the suction everts the nipple, the woman needs to place the baby to the breast. Women with inverted nipples can breastfeed. Massage and changing the infant's position will not affect the inverted nipples. PTS: 1 DIF: Cognitive Level: Application/Applying REF: p. 485 OBJ: Nursing Process: Implementation MSC: Client Needs: Physiologic Integrity 32. The nurse notices an infant has dimpling of the cheeks when breastfeeding. What action by the nurse is best? a. Tell the mother this is a sign of adequate feeding. b. Have the mother remove the baby from the breast and try again. c. Make a referral for a lactation consultation. d. Instruct the mother to feed for at least 15 minutes. ANS: B Dimpling the cheeks is a sign of an infant-derived problem during breastfeeding. The nurse should have the mother gently remove the baby from the breast, reposition the infant if needed, and try to get the baby to latch on correctly so she can try again. The nurse may need to call for a lactation consultant, but all mother-baby or labor and delivery nurses should be able to provide basic assistance first. This is not a sign of adequate feeding so the nurse should not encourage her to keep going for another 15 minutes. PTS: 1 DIF: Cognitive Level: Application/Applying REF: p. 490 | Safety Alert Box OBJ: Nursing Process: Implementation MSC: Client Needs: Health Promotion and Maintenance MULTIPLE RESPONSE

9. Which statement concerning physiologic factors is true? a. The infant has a slower metabolic rate than an adult. b. An infant has an inability to digest protein and lactase. c. Infants have a slower circulatory response than adults do. d. The infant's kidneys are less efficient in concentrating urine than an adult's kidneys.

D The infant's kidneys are not as effective at concentrating urine compared with an adult's because of immaturity of the renal system and slower glomerular filtration rates. This puts the infant at greater risk for fluid and electrolyte imbalance. Infants do not have slower metabolic rates, inabilities to digest protein and lactase, or a slower circulatory response compared to adults. PTS: 1 DIF: Cognitive Level: Knowledge/Remembering REF: p. 86 OBJ: Integrated Process: Teaching-Learning MSC: Client Needs: Physiologic Integrity 10. Which is a priority in counseling parents of a 6-month-old infant? a. Increasing food intake for secondary growth spurt b. Encouraging the infant to smile c. Securing a developmentally safe environment for the infant d. Teaching strategies to teach infants to sit up ANS: C Safety is a primary concern as an infant becomes increasingly mobile. The infant's appetite and growth velocity decrease in the second half of infancy. Although a social smile should be present by 6 months of age, encouraging this is not of higher priority than ensuring environmental safety. Unless the infant has a neuromuscular deficit, strategies for teaching a normally developing infant to sit up are not necessary. PTS: 1 DIF: Cognitive Level: Knowledge/Remembering REF: p. 96 OBJ: Integrated Process: Teaching-Learning MSC: Client Needs: Health Promotion and Maintenance 11. A mother of a 2-month-old infant tells the nurse, "My child doesn't sleep as much as his older brother did at the same age." What is the best response for the nurse? a. "Have you tried to feed the baby more often or play more before bedtime?" b. "Infant sleep patterns vary widely, some infants sleep only 2 to 3 hours at a time." c. "Keep a record of your baby's eating, waking, sleeping, and elimination patterns and to come back to discuss them." d. "This infant is difficult. It is important for you to identify what is bothering the baby." ANS: B Newborn infants may sleep as much as 17 to 20 hours per day. Sleep patterns vary widely, with some infants sleeping only 2 to 3 hours at a time. Infants typically do not need more caloric intake to improve sleep behaviors. Stimulating activities before bedtime may keep the baby awake. There is no need for the mother to keep behavior records. Just because an infant may not sleep as much as a sibling did does not justify labeling the child as being difficult. Identifying an infant as difficult without identifying helpful actions is not a therapeutic response for a parent concerned about sleep. PTS: 1 DIF: Cognitive Level: Application/Applying REF: p. 96 OBJ: Nursing Process: Implementation MSC: Client Needs: Health Promotion and Maintenance 12. The mother of a 10-month-old infant tells the nurse that her infant "really likes cow's milk." What is the nurse's best response to this mother? a. "Milk is a nutritious choice at this time." b. "Children should not get cow's milk until 1 year of age." c. "Limit cow's milk to one bedtime bottle." d. "Mix cereal with cow's milk and feed it in a bottle." ANS: B It is best to wait until the infant is at least 1 year old before giving him cow's milk because of the risk of allergies and intestinal problems. Cow's milk protein intolerance is the most common food allergy during infancy. Although milk is a good source of calcium and protein for children after the first year of life, it is not the best source of nutrients for children younger than 1 year old. Bedtime bottles of formula or milk are contraindicated because of their high sugar content, which leads to dental decay in primary teeth. Food and milk or formula should not be mixed in a bottle. PTS: 1 DIF: Cognitive Level: Application/Applying REF: p. 90 OBJ: Integrated Process: Teaching-Learning MSC: Client Needs: Physiologic Integrity 13. The mother of a 10-month-old infant asks the nurse about weaning her child. What assessment by the nurse indicates the child is not ready to be weaned? a. Frequently throwing the bottle down b. Takes very little formula from bottle c. Constantly chewing on the bottle nipple d. Appears to be sucking consistently when given a bottle ANS: D Consistent sucking is a sign that the child is not ready to be weaned. Throwing the bottle down, taking more fluids from a cup than the bottle, and chewing on the nipple all indicate readiness for weaning. PTS: 1 DIF: Cognitive Level: Knowledge/Remembering REF: p. 91 OBJ: Nursing Process: Assessment MSC: Client Needs: Health Promotion and Maintenance 14. A nurse is modeling play time with a 6-month-old infant. Which activity is appropriate? a. Pat-a-cake, peek-a-boo b. Ball rolling, hide-and-seek game c. Bright rattles and tactile toys d. Push-and-pull toys ANS: A Six-month-old children enjoy playing pat-a-cake and peek-a-boo. Nine-month-old infants enjoy rolling a ball and playing hide-and-seek games. Four-month-old infants enjoy bright rattles and tactile toys. Twelve-month-old infants enjoy playing with push-and-pull toys. PTS: 1 DIF: Cognitive Level: Knowledge/Remembering REF: p. 99 OBJ: Nursing Process: Implementation MSC: Client Needs: Health Promotion and Maintenance 15. Parents tell the nurse their 5-month-old has started sitting up without support. What teaching does the nurse plan to provide the parents? a. Providing solid foods safely b. Encouraging cruising and walking c. Providing cow's milk d. Proper sock and shoe selection ANS: A Sitting up is a sign the child is ready to begin solid foods. The nurse should teach the parents how to provide them safely and how to introduce them. The other topics are not related to sitting up. PTS: 1 DIF: Cognitive Level: Application/Applying REF: Box 6.3 OBJ: Integrated Process: Teaching-Learning MSC: Client Needs: Health Promotion and Maintenance 16. A nurse is teaching a parent group about dental hygiene for their babies. What information does the nurse provide? a. Babies don't need dental care until they are three. b. Start brushing teeth when all of them have come in. c. Children are ready for dental care when they can hold a toothbrush. d. Start with the first tooth using a cotton swab and water to wipe the teeth. ANS: D An infant's teeth need to be cleaned as soon as they erupt. Cleaning the teeth with cotton swabs or a face cloth is appropriate. Waiting until all the baby teeth are in is inappropriate and prolongs cleaning until 2 years of age. Being able to hold a toothbrush is not necessary as the parents should clean the teeth. PTS: 1 DIF: Cognitive Level: Application/Applying REF: p. 93 | Table 6.1 OBJ: Integrated Process: Teaching-Learning MSC: Client Needs: Physiologic Integrity 17. A nurse observes that a 3-month-old infant will hold a rattle if it is put in the hands, but the baby will not voluntarily grasp it. What action by the nurse is most appropriate? a. Provide anticipatory guidance. b. Document the findings in the chart. c. Refer the family to a neurologist. d. Perform a developmental screening. ANS: B This child is displaying normal age-appropriate behavior. The nurse should document the findings, but no other action is necessary. The nurse should always provide appropriate anticipatory guidance, but this answer is too vague to be the best response. PTS: 1 DIF: Cognitive Level: Application/Applying REF: Table 6.1 OBJ: Nursing Process: Implementation MSC: Client Needs: Health Promotion and Maintenance 18. In terms of fine motor development, what should the 7-month-old infant be able to do? a. Transfer objects from one hand to the other. b. Use thumb and index finger in crude pincer grasp. c. Hold crayon and make a mark on paper. d. Release cubes into a cup. ANS: A By age 7 months, infants can transfer objects from one hand to the other, crossing the midline. The crude pincer grasp is apparent at approximately age 9 months. The child can scribble spontaneously at age 15 months. At age 12 months, the child can release cubes into a cup. PTS: 1 DIF: Cognitive Level: Knowledge/Remembering REF: Table 6.1 OBJ: Nursing Process: Assessment MSC: Client Needs: Health Promotion and Maintenance 19. In terms of gross motor development, what would the nurse expect a 5-month-old infant to do? a. Roll from abdomen to back. b. Roll from back to abdomen. c. Sit erect without support. d. Move from prone to sitting position. ANS: A Rolling from abdomen to back is developmentally appropriate for a 5-month-old infant. The ability to roll from back to abdomen usually occurs at 6 months old. Sitting erect without support is a developmental milestone usually achieved by 8 months. The 10-month-old infant can usually move from a prone to a sitting position. PTS: 1 DIF: Cognitive Level: Knowledge/Remembering REF: p. 93 | Table 6.1 OBJ: Nursing Process: Assessment MSC: Client Needs: Health Promotion and Maintenance 20. According to Piaget, the 6-month-old infant is in what stage of the sensorimotor phase? a. Use of reflexes b. Primary circular reactions c. Secondary circular reactions d. Coordination of secondary schemata ANS: C Infants are usually in the secondary circular reaction stage from age 4 months to 8 months. This stage is characterized by a continuation of the primary circular reaction because of the response that results. Shaking is performed to hear the noise of the rattle, not just for shaking. The use of reflexes is primarily during the first month of life. Primary circular reaction stage marks the replacement of reflexes with voluntary acts. The infant is in this stage from age 1 month to 4 months. The fourth sensorimotor stage is coordination of secondary schemata. This is a transitional stage in which increasing motor skills enable greater exploration of the environment. PTS: 1 DIF: Cognitive Level: Knowledge/Remembering REF: p. 87 OBJ: Nursing Process: Assessment MSC: Client Needs: Health Promotion and Maintenance 21. A mother tells the nurse that she is discontinuing breastfeeding her 5-month-old infant. What response by the nurse is best? a. "That's OK. formula is just as good for a 5-month-old." b. "Be sure to use an iron-fortified formula instead." c. "The baby will need immunizations earlier now." d. "Be sure to monitor how many diapers the baby wets." ANS: B For children younger than 1 year, the American Academy of Pediatrics recommends the use of breast milk. If breastfeeding has been discontinued, then iron-fortified commercial formula should be used. There is no need to provide immunizations on a different schedule or specific reason for monitoring wet diapers. PTS: 1 DIF: Cognitive Level: Application/Applying REF: p. 90 OBJ: Integrated Process: Teaching-Learning MSC: Client Needs: Health Promotion and Maintenance 22. The parent of a 2-week-old infant asks the nurse whether the baby needs fluoride supplements, since mom is exclusively breastfeeding the baby. What response by the nurse is best? a. "Yes, the baby needs to begin taking them now." b. "Is your water fluoridated?" c. "She may need to begin taking them at age 6 months." d. "You can use infant cereal mixed with fluoridated water instead." ANS: C Fluoride supplementation is recommended by the American Academy of Pediatrics beginning at age 6 months if the child is not drinking adequate amounts of fluoridated water. Asking if the water is fluoridated and advising to mix water and cereal are not the best responses since the child is only 2 months old. PTS: 1 DIF: Cognitive Level: Application/Applying REF: p. 94 OBJ: Integrated Process: Teaching-Learning MSC: Client Needs: Health Promotion and Maintenance 23. A nurse is making a home visit on a new mother with an infant. What action by the mother requires the nurse to intervene? a. Cooks while holding and cuddling infant to provide comfort b. Keeps hand on infant while reaching for supplies on changing table c. Shows the nurse the water heater setting that is on 110° F (43.3° C) d. Places baby to sleep in crib with no blankets, toys, or other objects ANS: A Burns are a leading cause of injury in children. The mother should not be holding the baby while cooking, so the nurse must intervene at this point. The other actions all provide safety. PTS: 1 DIF: Cognitive Level: Application/Applying REF: p. 97 OBJ: Nursing Process: Implementation MSC: Client Needs: Safe and Effective Care Environment MULTIPLE RESPONSE

8. What is the most dangerous effect on the fetus of a mother who smokes cigarettes while pregnant? a. Genetic changes and anomalies b. Extensive central nervous system damage c. Fetal addiction to the substance inhaled d. Intrauterine growth restriction

D The major consequences of smoking tobacco during pregnancy are low-birth-weight infants, prematurity, and increased perinatal loss. Cigarettes normally will not cause genetic changes or extensive central nervous system damage. Addiction is not a normal concern with the neonate. PTS: 1 DIF: Cognitive Level: Knowledge/Remembering REF: p. 508 | Table 24.1 OBJ: Nursing Process: Assessment MSC: Client Needs: Health Promotion and Maintenance

4. While teaching an early pregnancy class, the nurse explains that the morula is a a. fertilized ovum before mitosis begins. b. flattened disk-shaped layer of cells within a fluid-filled sphere. c. double layer of cells that becomes the placenta. d. solid ball composed of the first cells formed after fertilization.

D The morula is so named because it resembles a mulberry. It is a solid ball of 12 to 16 cells that develops after fertilization. The fertilized ovum is called the zygote. This is the embryonic disk. It will develop into the baby. The placenta is formed from two layers of cells: the trophoblast, which is the other portion of the fertilized ovum, and the decidua, which is the portion of the uterus where implantation occurs. PTS: 1 DIF: Cognitive Level: Comprehension/Understanding REF: p. 198 OBJ: Integrated Process: Teaching-Learning MSC: Client Needs: Physiologic Integrity

2. A nurse is assessing an 8-year-old child. Which finding leads the nurse to conduct further assessment? a. Understands that his or her point of view is not the only one b. Enjoys telling riddles and silly jokes c. Demonstrates the principle of object conservation d. Engages in fantasy and magical thinking

D The preschool-age child engages in fantasy and magical thinking. The school-age child moves away from this type of thinking and becomes more skeptical and logical. Belief in Santa Claus or the Easter Bunny ends in this period of development. If the child demonstrated this type of thinking, the nurse would need to follow up with more developmental screening. School-age children enter the stage of concrete operations. They learn that their point of view is not the only one. The school-age child has a sense of humor. The child's increased language mastery and increased logic allow for appreciation of plays on words, jokes, and incongruities. The school-age child understands that properties of objects do not change when their order, form, or appearance does (object conservation). PTS: 1 DIF: Cognitive Level: Analysis/Analyzing REF: p. 134 OBJ: Nursing Process: Assessment MSC: Client Needs: Health Promotion and Maintenance

1. Which pregnant woman should have the least weight gain during pregnancy? a. Woman pregnant with twins b. Woman in early adolescence c. Woman shorter than 62 inches or 157 cm d. Woman who was obese before pregnancy

D The recommended weight gain for overweight or obese women is 11 to 20 pounds. This will provide sufficient nutrients for the fetus. Overweight and obese women should be advised to lose weight prior to conception in order to achieve the best pregnancy outcomes. A higher weight gain in twin gestations may help prevent low birth weights. Adolescents need to gain weight toward the higher acceptable range, which will provide for their own growth as well as for fetal growth. In the past women of short stature were advised to restrict their weight gain; however, evidence to support these guidelines has not been found. PTS: 1 DIF: Cognitive Level: Knowledge/Remembering REF: p. 255 OBJ: Nursing Process: Assessment MSC: Client Needs: Health Promotion and Maintenance

9. The nurse practicing in a labor setting knows that the woman most at risk for a uterine rupture is a gravida a. 3 who has had two low-segment transverse cesarean births. b. 2 who has had a low-segment vertical incision for delivery of a 10-pound infant. c. 5 who has had two vaginal births and two cesarean births. d. 4 who has had four cesarean births.

D The risk of uterine rupture increases as the number of prior uterine incisions increases. More than two previous cesarean births places the woman at increased risk for uterine rupture. The other women are not high-risk candidates. PTS: 1 DIF: Cognitive Level: Knowledge/Remembering REF: p. 388 OBJ: Nursing Process: Assessment MSC: Client Needs: Physiologic Integrity 10. The nurse caring for a woman in labor understands that the primary risk associated with an amniotomy is a. maternal infection. b. maternal hemorrhage. c. prolapse of the umbilical cord. d. separation of the placenta. ANS: C When the membranes are ruptured, the umbilical cord may come downward with the flow of amniotic fluid and become trapped in front of the presenting part. Infection is a risk of amniotomy but not the primary concern. Maternal hemorrhage is not associated with amniotomy. Separation of the placenta may occur if the uterus is overdistended before the amniotomy, but it is not the major concern. PTS: 1 DIF: Cognitive Level: Comprehension/Understanding REF: p. 376 OBJ: Nursing Process: Assessment MSC: Client Needs: Physiologic Integrity 11. Before the physician performs an external version, the nurse should expect an order for a a. tocolytic drug. b. contraction stress test (CST). c. local anesthetic. d. indwelling catheter. ANS: A A tocolytic drug will relax the uterus before and during version, making manipulation easier. A contraction stress test, local anesthetics, and indwelling catheters are not needed. PTS: 1 DIF: Cognitive Level: Knowledge/Remembering REF: p. 383 OBJ: Nursing Process: Planning MSC: Client Needs: Health Promotion and Maintenance 12. A maternal indication for the use of vacuum extraction is a. a wide pelvic outlet. b. maternal exhaustion. c. a history of rapid deliveries. d. failure to progress past 0 station. ANS: B A mother who is exhausted will be unable to assist with the expulsion of the fetus and is a candidate for vacuum extraction. With a wide pelvic outlet and rapid delivery, vacuum extraction is not necessary. A station of 0 is too high for a vacuum extraction. PTS: 1 DIF: Cognitive Level: Knowledge/Remembering REF: p. 384 OBJ: Nursing Process: Assessment MSC: Client Needs: Health Promotion and Maintenance 13. After an amniotomy, which action by the nurse takes priority? a. Assess the color of the amniotic fluid. b. Change the patient's gown. c. Estimate the amount of amniotic fluid. d. Assess the fetal heart rate. ANS: D The fetal heart rate must be assessed immediately after the rupture of the membranes to determine whether cord prolapse or compression has occurred. Assessing the amniotic fluid color, changing the patient's gown, and estimating the amount of amniotic fluid lost are all appropriate interventions but not the priority. PTS: 1 DIF: Cognitive Level: Application/Applying REF: p. 376 OBJ: Nursing Process: Implementation MSC: Client Needs: Safe and Effective Care Environment 14. For which patient should the oxytocin (Pitocin) infusion be discontinued immediately? a. A woman in active labor with contractions every 31 minutes lasting 60 seconds each b. A woman in transition with contractions every 2 minutes lasting 90 seconds each c. A woman in active labor with contractions every 2 to 3 minutes lasting 70 to 80 seconds each d. A woman in early labor with contractions every 5 minutes lasting 40 seconds each ANS: B This woman's contraction pattern represents hyperstimulation, and inadequate resting time occurs between contractions to allow placental perfusion. The other women can continue to use the oxytocin at this point. PTS: 1 DIF: Cognitive Level: Analysis/Analyzing REF: p. 378 OBJ: Nursing Process: Implementation MSC: Client Needs: Physiologic Integrity 15. The priority nursing care associated with an oxytocin (Pitocin) infusion is a. measuring urinary output. b. increasing infusion rate every 30 minutes. c. monitoring uterine response. d. evaluating cervical dilation. ANS: C Because of the risk of hyperstimulation, which could result in decreased placental perfusion and uterine rupture, the nurse's priority intervention is monitoring uterine response. Monitoring urinary output and cervical dilation is appropriate but not the priority. The infusion rate may be increased but only after proper assessment determines that it is appropriate. PTS: 1 DIF: Cognitive Level: Application/Applying REF: p. 380 OBJ: Nursing Process: Implementation MSC: Client Needs: Safe and Effective Care Environment 16. Which event indicates a complication of an external version? a. Maternal pulse rate of 100 bpm b. Fetal bradycardia persisting 10 minutes after the version c. Fetus returning to the original position d. Increased maternal anxiety after the version ANS: B Fetal bradycardia after a version may indicate that the umbilical cord has become compressed, and the fetus is having hypoxia. There are few risks to the woman during an external version. The fetus may return to the original position, but this is not a complication of the version. Anxiety may occur before the version but should decrease after the procedure is completed. PTS: 1 DIF: Cognitive Level: Knowledge/Remembering REF: p. 383 OBJ: Nursing Process: Assessment MSC: Client Needs: Physiologic Integrity 17. Immediately after the forceps-assisted birth of an infant, which action by the nurse is next? a. Assess the infant for signs of trauma. b. Give the infant prophylactic antibiotics. c. Apply a cold pack to the infant's scalp. d. Measure the circumference of the infant's head. ANS: A Forceps delivery can result in local irritation, bruising, or lacerations of the fetal scalp. Prophylactic antibiotics are not necessary with a forceps delivery. This would put the infant at risk for cold stress and would be contraindicated. Measuring the circumference of the head is part of the initial nursing assessment. PTS: 1 DIF: Cognitive Level: Application/Applying REF: p. 386 OBJ: Nursing Process: Implementation MSC: Client Needs: Physiologic Integrity 18. When preparing a woman for a cesarean birth, the nurse's care should include a. injection of narcotic preoperative medications. b. full perineal shave preparation. c. straight catheterization to empty the bladder. d. administration of an oral antacid. ANS: D General anesthesia may be needed unexpectedly for cesarean birth. An oral antacid neutralizes gastric acid and reduces potential lung injury if the woman vomits and aspirates gastric contents during anesthesia. A narcotic at this point would put the fetus at high risk for respiratory distress. Perineal preparation is not necessary for a cesarean section. Some agencies will do an abdominal prep just before the surgery. The catheterization should be indwelling in order to keep the bladder small during the surgery. PTS: 1 DIF: Cognitive Level: Knowledge/Remembering REF: p. 389 OBJ: Nursing Process: Implementation MSC: Client Needs: Physiologic Integrity 19. Surgical, medical, or mechanical methods may be used for labor induction. Which technique is considered a mechanical method of induction? a. Amniotomy b. Intravenous Pitocin c. Transcervical catheter d. Vaginal insertion of prostaglandins ANS: C Placement of a balloon-tipped Foley catheter into the cervix is a mechanical method of induction. Other methods to expand and gradually dilate the cervix include Laminaria tents, Dilapan, and Lamicel. Amniotomy is a surgical method of augmentation and induction. Intravenous Pitocin is a medical method of induction. Insertion of prostaglandins is a medical method of induction. PTS: 1 DIF: Cognitive Level: Knowledge/Remembering REF: p. 378 OBJ: Nursing Process: Implementation MSC: Client Needs: Physiologic Integrity 20. What is an appropriate response to a woman's comment that she is worried about having a cesarean birth? a. "Don't worry. Everything will be okay." b. "What are your feelings about having a cesarean birth?" c. "I know you're worried, but this is a routine procedure." d. "Patients commonly worry about surgery." ANS: B Allowing the patient to express her feelings is the most appropriate nursing response. The nurse should never provide the patient with false reassurance or disregard or belittle her feelings, which is what the other options do. PTS: 1 DIF: Cognitive Level: Application/Applying REF: p. 391 | Nursing Care Plan OBJ: Integrated Process: Communication and Documentation MSC: Client Needs: Psychosocial Integrity 21. While assisting with a vacuum extraction birth, what should the nurse immediately report to the provider? a. Persistent fetal bradycardia below 100 bpm b. Maternal pulse rate of 100 bpm c. Maternal blood pressure of 120/70 mm Hg d. Decrease in intensity of uterine contractions ANS: A Fetal bradycardia may indicate fetal distress and may require immediate intervention. Maternal pulse rate may increase due to the pushing process. This blood pressure is within expected norms for this stage of labor. The birth is imminent at this point. PTS: 1 DIF: Cognitive Level: Application/Applying REF: p. 384 OBJ: Nursing Process: Implementation MSC: Client Needs: Physiologic Integrity 22. To monitor for potential hemorrhage in the woman who has just had a cesarean birth, what action by the recovery room nurse is most appropriate? a. Maintain an intravenous infusion at 100 mL/hr. b. Assess the abdominal dressings for drainage. c. Assess the uterus for firmness every 15 minutes. d. Monitor her urinary output. ANS: C Maintaining contraction of the uterus is important in controlling bleeding from the placental site. Maintaining proper fluid balance will not control hemorrhage. Assessing abdominal dressings is an important assessment, but hemorrhage will first be noted vaginally. Urinary output typically drops in hemorrhage, but this is a later finding. PTS: 1 DIF: Cognitive Level: Application/Applying REF: p. 393 OBJ: Nursing Process: Implementation MSC: Client Needs: Physiologic Integrity 23. A new graduate nurse is preparing to hang oxytocin for a woman to augment her labor. What action by the new nurse warrants intervention from the preceptor? a. Adds oxytocin to the IV as a piggyback b. Programs the IV pump for a primary infusion c. Assesses FHR and uterine activity prior to starting the infusion d. Attaches the infusion line to the proximal port ANS: B Oxytocin is run as a secondary infusion on a pump. When the new nurse programs it as the primary infusion, the preceptor should intervene. Oxytocin should be added to the most proximal IV port. FHR and uterine activity should be assessed prior to starting the infusion. PTS: 1 DIF: Cognitive Level: Application/Applying REF: p. 379 OBJ: Nursing Process: Implementation MSC: Client Needs: Physiologic Integrity 24. A woman is receiving oxytocin to augment labor. The nurse notes that the Montevideo units are measured at 560 and the fetus is showing late decelerations. What action by the nurse takes priority? a. Notify the charge nurse of the situation. b. Document the findings in the chart. c. Increase the rate of oxytocin slowly. d. Stop the oxytocin infusion. ANS: D This woman is showing signs of uterine tachysystole. The nurse first turns the oxytocin infusion off. Notifying the charge nurse and documentation are important but not the priority. The nurse would not increase the rate of the infusion. PTS: 1 DIF: Cognitive Level: Application/Applying REF: p. 382 | Safety Alert Box OBJ: Nursing Process: Implementation MSC: Client Needs: Safe and Effective Care Environment 25. A woman is going to have a vaccum extraction delivery. What nursing intervention is most important to prevent complications? a. Empty the woman's bladder. b. Apply cold packs to the perineum. c. Assess vital signs after the procedure. d. Monitor the woman's temperature. ANS: A The nurse should empty the woman's bladder prior to vacuum extraction delivery. Ice packs can help with pain and prevent or limit the size of hematomas, but that is not the most important safety measure. Assessing vital signs will not prevent complications from occurring but will help identify them when they occur. PTS: 1 DIF: Cognitive Level: Application/Applying REF: p. 386 OBJ: Nursing Process: Implementation MSC: Client Needs: Safe and Effective Care Environment MULTIPLE RESPONSE

7. A postpartum patient asks, "Will these stretch marks go away?" The nurse's best response is a. "They will fade and be gone by your 6-week checkup." b. "No, unfortunately they will never fade away." c. "Yes, eventually they will totally disappear." d. "They will fade to silvery lines but won't disappear completely."

D The stretch marks will fade to silvery lines but will not disappear completely. PTS: 1 DIF: Cognitive Level: Comprehension/Understanding REF: p. 399 OBJ: Integrated Process: Teaching-Learning MSC: Client Needs: Physiologic Integrity

6. The nurse teaches a pregnant woman that one danger in using nonfood supplementation of nutrients is a. increased absorption of all vitamins. b. development of pregnancy-induced hypertension (PIH). c. increased caloric intake. d. toxic effects on the fetus.

D The use of supplements in addition to food may increase the intake of some nutrients to doses much higher than the recommended amounts. Overdoses of some nutrients have been shown to cause fetal defects. Supplements do not have better absorption than natural vitamins and minerals. There is no relationship between supplements and PIH. Supplements do not contain significant calories. PTS: 1 DIF: Cognitive Level: Knowledge/Remembering REF: p. 261 OBJ: Integrated Process: Teaching-Learning MSC: Client Needs: Physiologic Integrity

4. Four hours after delivery of a healthy neonate of an insulin-dependent diabetic woman, the baby appears jittery, irritable, and has a high-pitched cry. Which nursing action has top priority? a. Start an intravenous line with D5W. b. Notify the clinician stat. c. Document the event in the nurses' notes. d. Test for blood glucose level.

D These are signs of hypoglycemia in the newborn. The nurse should test the infant's blood glucose level and then feed the infant if it is low. It is not common practice to give intravenous glucose to a newborn prior to feeding. Feeding the infant is preferable because the formula or breast milk will last longer. The provider needs to be notified after corrective action has been taken. Documentation should occur but is not the priority. PTS: 1 DIF: Cognitive Level: Application/Applying REF: p. 652 OBJ: Nursing Process: Implementation MSC: Client Needs: Safe and Effective Care Environment

1. Which statement by a postpartum woman indicates that teaching about thrombus formation has been effective? a. "I'll stay in bed for the first 3 days after my baby is born." b. "I'll keep my legs elevated with pillows." c. "I'll sit in my rocking chair most of the time." d. "I'll put my support stockings on every morning before rising."

D Venous congestion begins as soon as the woman stands up. The stockings should be applied before she rises from the bed in the morning. As soon as possible, the woman should ambulate frequently. The mother should avoid knee pillows because they increase pressure on the popliteal space. Sitting in a chair with legs in a dependent position causes pooling of blood in the lower extremities. PTS: 1 DIF: Cognitive Level: Evaluation/Evaluating REF: p. 607 OBJ: Nursing Process: Evaluation MSC: Client Needs: Health Promotion and Maintenance

5. The Women, Infants, and Children (WIC) program provides a. well-child examinations for infants and children living at the poverty level. b. immunizations for high-risk infants and children. c. screening for infants with developmental disorders. d. supplemental food supplies to low-income pregnant or breastfeeding women.

D WIC is a federal program that provides supplemental food supplies to low-income women who are pregnant or breastfeeding and to their children until age 5 years. Medicaid's Early and Periodic Screening, Diagnosis, and Treatment Program provides for well-child examinations and for treatment of any medical problems diagnosed during such checkups. Children in the WIC program are often referred for immunizations, but that is not the primary focus of the program. Public Law 99-457 is part of the Individuals with Disabilities Education Act that provides financial incentives to states to establish comprehensive early intervention services for infants and toddlers with, or at risk for, developmental disabilities. PTS: 1 DIF: Cognitive Level: Comprehension REF: p. 8 OBJ: Integrated Process: Teaching-Learning MSC: Client Needs: Health Promotion and Maintenance


Related study sets

APES First Semester Exam Study Guide

View Set

energy forms and transformations test answers (usa test prep. 1.)

View Set

Exam 3: Chapter 10 Cell Reproduction Study Guide

View Set

Security + Topic 2B: Explain threat intelligence sources; Lesson 2: Explaining threat actors and threat intelligence

View Set